Download as pdf or txt
Download as pdf or txt
You are on page 1of 492

C.

BY: HOSSAM HAMDY MATERNITY RNPEDIA – ‫موسوعه التمريض‬

) (

Over
500 Q +

COllECtED BY/ HOSSAM HAMDY


OwNER RNPEDIA
1
‫‪C.BY: HOSSAM HAMDY‬‬ ‫‪MATERNITY‬‬ ‫موسوعه التمريض – ‪RNPEDIA‬‬

‫مقدمه عن االمتحان‬
‫طبقا للقانون رقم ‪ 153‬لسنة ‪ ،2019‬بشأن تعديل القانون رقم ‪ 415‬لسنة ‪ ،1954‬والذي نص في المادة رقم ‪ 3‬منه‪ ،‬على أنه‬ ‫•‬
‫يُشترط للقيد ألول مرة في سجالت وزارة الصحة‪ ،‬أن يجتاز الطبيب االمتحان القومي للتأهيل لمزاولة المهنة‪ ،‬بإشراف هيئة‬
‫التدريب اإللزامي سواء كان خالل دراسته أو بعدها‪ ،‬وأن يتم القيد لمدة ‪ 5‬سنوات قابلة للتجديد لمدد مماثلة‪.‬‬
‫•‬
‫وفقا ً لقانون رقم ‪ 12‬لسنة ‪ 2022‬بشأن إنشاء المجلس الصحي المصري‪ ،‬يتم حاليا ً اإلشراف وعقد اإلمتحان تبعا للمجلس الصحي‬
‫المصري طبقا ً للقانون‪ ،‬وذلك بالتعاون مع وزارة الصحة والسكان‪ ،‬لحين صدور الالئحة التنفيذية للمجلس الصحي المصري‪.‬‬

‫تم إنشاء اإلمتحان القومي لمزاولة مهنه )‪ (EMLE‬للتأكد من أن خريجي كليات القطاع الطبى في مصر مؤهلون بالفعل لبدء‬ ‫•‬
‫ممارسة الطب وللتأكد من حصولهم على التدريب المناسب قبل الحصول على ترخيص لممارسة المهنة في مصر والتأكد من‬
‫استيفاء خريجي منشآت التعليم الصحية المعتمدة للحد االدنى من معايير مزاولة المهنة ‪.‬‬
‫لن يتمكن الطالب من الحصول على ترخيص ممارسة المهنة في مصر حتى يجتاز اإلمتحان‪ ،‬واإلمتحان موحد على مستوي‬ ‫•‬
‫الجمهورية مما يضمن المساواة والعدالة وتكافئ الفرص‪.‬‬

‫شروط التسجيل لإلمتحان‪:‬‬


‫أن يكون الطالب حصل على درجة البكالوريوس من أحد الجامعات المصرية الحكومية أو الخاصة المعتمدة لدى المجلس‬ ‫•‬
‫األعلى للجامعات أو مايعادلها من الجامعات األجنبية بشرط معادلتها من المجلس االعلى للجامعات‪.‬‬
‫أن يكون الطالب قضى على األقل ‪ 6‬شهور كاملة من سنة اإلمتياز‪.‬‬ ‫•‬

‫تسجيل البيانات يتم من خالل شئون الطالب بالجامعة او مسئول اإلمتياز والتدريب اإللزامى على أن يتم إرسال جميع البيانات‬ ‫•‬
‫معتمدة من عميد الكلية ‪.‬‬

‫تسجيل البيانات بالنسبة للحاصلين على درجة البكالوريوس واإلمتياز من الخارج يكون بالمجلس األعلى للمستشفيات الجامعية‬ ‫•‬
‫بعد معادلة الشهادات من المجلس األعلى للجامعات‪.‬‬

‫قواعد التسجيل‪:‬‬
‫يتم التسجيل لكل دور منفصل عن االخر وال يعتد بالتسجيالت السابقة فى حالة عدم إجتياز اإلمتحان أو عدم دخوله‪.‬‬ ‫•‬
‫ترسل البيانات مرة واحدة ويتم مراجعة البيانات من قبل الطالب وشئون الطالب بالكلية بشكل جيد قبل إرسالها وفى حالة‬ ‫•‬
‫إرسال اى من البيانات المطلوبة بشكل خاطىء ‪،‬يتم إعادة إرسال البيانات فى دور اإلمتحان القادم حيث يتم استبعاد البيانات من‬ ‫•‬
‫الدور الحالى‪.‬‬
‫فى حالة وجود وافدين دارسين بفترة اإلمتياز فى أحد الجامعات يتم التأكد من قبل شئون الطالب على وجود معادلة درجة‬ ‫•‬
‫البكالوريوس من المجلس األعلى للجامعات قبل إرسال بيناتهم ‪ ،‬على أن تتحمل الكلية مسئولية ذلك حال مخالفته‪.‬‬
‫يتم ارسال البينات المطلوبة للطالب متضمنة المعلومات التالية‪:‬‬ ‫•‬
‫‪-‬اإلسم رباعى باللغة العربية طبقا لبطاقة الرقم القومى‪.‬‬
‫‪-‬اإلسم رباعى باللغة اإلنجليزية‪.‬‬
‫‪-‬الرقم القومى (‪ 14‬رقم) ‪ ،‬رقم الباسبور بالنسبة للوافدين‪.‬‬
‫‪-‬اسم الجامعة‪.‬‬
‫‪-‬المجموع الكلى‪.‬‬
‫‪-‬اإليميل (يفضل استخدام االيميل العادى وليس الجامعى)‪.‬‬
‫‪-‬رقم الهاتف المحمول‪.‬‬

‫‪2‬‬
‫‪C.BY: HOSSAM HAMDY‬‬ ‫‪MATERNITY‬‬ ‫موسوعه التمريض – ‪RNPEDIA‬‬

‫أنواع اإلمتحانات‪:‬‬
‫اإلمتحان مكون من ‪ 100‬سؤال (‪ )MCQ‬من خمس تخصصات مختلفة (الباطنة – االداره – األطفال – النسا ) ‪ ،‬ويحتوى‬
‫اإلمتحان على ثالثة أنواع من اإلمتحانات كالتالى‪:‬‬
‫• اإلمتحان التجريبى (‪ :)MOCKUP EXAM‬إمتحان يكون متاح للطالب لمعرفة شكل وأسلوب األسئلة وكيفية التعامل مع‬
‫الموقع اإللكترونى ويكون اإلمتحان متاح طول اليوم يستطيع الطالب الدخول والتجربة فى اى وقت فى خالل اليوم كما يكون متاح‬
‫التجربة أكثر من مرة لإلمتحان الواحد‪.‬‬
‫• إمتحان المحاكاة (‪ :)SIMULATION EXAM‬إمتحان يكون متاح للطالب لمعرفة شكل وأسلوب األسئلة وكيفية التعامل مع‬
‫الموقع اإللكترونى فى ظل الوقت المحدد لإلمتحان وبقواعد اإلمتحان وأنظمة المراقبة المختلفة‪.‬‬
‫• اإلمتحان الفعلى (‪ :)REAL EXAM‬اإلمتحان األساسى الذى يتم محاسبة الطالب على الدرجة الحاصل عليها من حيث نجاحه‬
‫او رسوبه للحصول على مزاولة المهنة ويضم الدور الواحد ‪ 3‬إمتحانات فعلية يستطيع الطالب أن يجتاز أحدهما للنجاح علما بأنه يتم‬
‫محاسبة الطالب على درجة أخر امتحان‪.‬‬

‫قواعد وتعليمات دخول اإلمتحان‪:‬‬


‫الدخول من خالل جهاز البتوب )‪ (laptop‬او جهاز كمبيوتر مزود بكاميرا‪(USP).‬‬ ‫•‬
‫التأكد من جودة وسالمة الكاميرا بالجهاز المستخدم وانها مفعلة أثناء الدخول‪.‬‬ ‫•‬
‫الجلوس منفردا ً بمكان منعزل عن االخرين أثناء األمتحان لعدم إلغاء اإلمتحان‪.‬‬ ‫•‬
‫يفضل دخول اإلمتحان من خالل الموقع اإللكترونى قبل اإلمتحان بحد أدنى ‪ 20‬دقيقة‪.‬‬ ‫•‬
‫عدم فتح اى متصفحات او مواقع اواستخدام اى اجهزة أخرى أثناء أداء اإلمتحان وفى حالة مخالفة ذلك يتم إلغاء اإلمتحان‪.‬‬ ‫•‬
‫•‬

‫جميع االساله الموجوده مبنيه علي نظام ‪ NCLEX‬باالجابات الصحيحه ‪% 100‬‬


‫مع ذكر التفسير لالجابات كلها‬

‫لو حابب تتابع باقي السلسله خش صفحه الفيسبوك دي‬


‫‪https://www.facebook.com/EgyRNpedia/‬‬
‫اما بقي لو عاوز تشرفنا في قناه التلليجرام فاضغط هنا‬
‫‪https://t.me/RNpedia‬‬
‫اما بقي لو حابب تتابعني او تتواصل معايا شخصيا خش هنا‬
‫‪https://www.facebook.com/DrHosamHamdy‬‬

‫دعواتكم وبالتوفيق ان شاء للا‬

‫‪3‬‬
C.BY: HOSSAM HAMDY MATERNITY RNPEDIA – ‫موسوعه التمريض‬

QUESTION BANK OF MATERNITY NURINSG PAGE

Maternity Nursing NU.1 (Quiz #1: 75 Questions) 5

Maternity Nursing NU.2 (Quiz #1: 75 Questions) 67


Maternity Nursing NU.3 (Quiz #1: 75 Questions) 132

Maternity Nursing NU.4 (Quiz #1: 75 Questions) 194

Maternity Nursing NU.5 (Quiz #1: 75 Questions) 261

Maternity Nursing NU.6 (Quiz #1: 25 Questions) 329

Maternity Nursing NU.7 (Quiz #1: 25 Questions) 353

Maternity Nursing NU.8 (Quiz #1: 25 Questions) 375


Maternity Nursing NU.9 (Quiz #1: 25 Questions) 400

Maternity Nursing NU.10 (Quiz #1: 25 Questions) 421

Maternity Nursing NU.11 (Quiz #1: 25 Questions) 447

Maternity Nursing NU.12 (Quiz #1: 25 Questions) 474

TOTAL QUESTION = 545

TOTAL PAGES = 492

4
C.BY: HOSSAM HAMDY MATERNITY RNPEDIA – ‫موسوعه التمريض‬

Maternity Nursing NU.1 (Quiz #1: 75 Questions)

1. Question

A postpartum patient was in labor for 30 hours and had ruptured membranes
for 24 hours. For which of the following would the nurse be alert?

A. Endometritis

B. Endometriosis

C. Salpingitis

D. Pelvic thrombophlebitis

Answer: A. Endometritis

Endometritis is an infection of the uterine lining and can occur after


prolonged rupture of membranes. Symptoms include swelling of the
abdomen, abnormal vaginal bleeding or discharge, fever, discomfort with
bowel movement, and pain in the lower abdomen or pelvic region.

Option B: Endometriosis does not occur after a strong labor and prolonged
rupture of membranes. It is a painful disorder in which tissue similar to the
tissue that normally lines the inside of the uterus grows outside of the uterus.

Option C: Salpingitis is a tubal infection and could occur if endometritis is


not treated. It is an inflammation of the fallopian tubes caused by bacterial
infection.

Option D: Pelvic thrombophlebitis involves a clot formation, but it is not a


complication of prolonged rupture of membranes. It is an extremely rare
condition that occurs after delivery when an infected blood clot, or
thrombus, causes inflammation in the pelvic vein.
2. Question

5
C.BY: HOSSAM HAMDY MATERNITY RNPEDIA – ‫موسوعه التمريض‬

A client at 36 weeks gestation is scheduled for a routine ultrasound prior to


amniocentesis. After teaching the client about the purpose of the ultrasound,
which of the following client statements would indicate to the nurse in
charge that the client needs further instruction?

A. The ultrasound will help to locate the placenta.

B. The ultrasound identifies blood flow through the umbilical cord.

C. The test will determine where to insert the needle.

D. The ultrasound locates a pool of amniotic fluid.

Answer: B. The ultrasound identifies blood flow through the umbilical


cord.

Before amniocentesis, a routine ultrasound is valuable in locating the


placenta, locating a pool of amniotic fluid, and showing the physician where
to insert the needle. Color Doppler imaging ultrasonography identifies blood
flow through the umbilical cord. A routine ultrasound does not accomplish
this.

Option A: As early as 10 weeks, the placenta can be detected by an


ultrasound. The normal placenta is discoid with uniform echogenicity and
rounded margins. It is usually located along the anterior or posterior uterine
walls, extending into the lateral walls.

Option C: Ultrasound is done before and during amniocentesis to ensure


that the needle can safely pass through the walls of the abdomen and womb.

Option D: The sample of amniotic fluid is removed through a fine needle


inserted into the uterus through the abdomen, under ultrasound guidance.
3. Question

While the postpartum client is receiving heparin for thrombophlebitis, which


of the following drugs would the nurse expect to administer if the client
develops complications related to heparin therapy?

A. Calcium gluconate

B. Protamine sulfate
6
C.BY: HOSSAM HAMDY MATERNITY RNPEDIA – ‫موسوعه التمريض‬

C. Methylergonovine (Methergine)

D. Nitrofurantoin (Macrodantin)

Answer: B. Protamine sulfate

Protamine sulfate is a heparin antagonist given intravenously to counteract


bleeding complications caused by heparin overdose.

Option A: Calcium gluconate is the calcium salt of gluconic acid, an


intravenous medication used to treat conditions arising from calcium
deficiencies such as hypocalcemic tetany and hypocalcemia.

Option C: Methylergonovine is used to prevent or treat bleeding from the


uterus that can happen after childbirth or an abortion.

Option D: Nitrofurantoin is used to treat urinary tract infections. It is an


antibiotic that works by killing bacteria that cause infection.

4. Question

When caring for a 3-day-old neonate who is receiving phototherapy to treat


jaundice, the nurse in charge would expect to do which of the following?

A. Turn the neonate every 6 hours

B. Encourage the mother to discontinue breastfeeding.

C. Notify the physician if the skin becomes bronze in color.

D. Check the vital signs every 2 to 4 hours.

Answer: D. Check the vital signs every 2 to 4 hours

While caring for an infant receiving phototherapy for treatment of jaundice,


vital signs are checked every 2 to 4 hours because hyperthermia can occur
due to the phototherapy lights.

7
C.BY: HOSSAM HAMDY MATERNITY RNPEDIA – ‫موسوعه التمريض‬

Option A: Only one study reported the significance drop in serum bilirubin
and shorter duration of phototherapy in the supine group. Keeping the
jaundiced newborn in the supine position throughout phototherapy is as
effective as turning them periodically based on appraised studies.

Option B: The baby may be breastfed without interruption during


phototherapy. Jaundice in breastfed babies is not a reason to stop
breastfeeding as long as a baby is feeding well, gaining weight, and
otherwise growing.

Option C: Bronze baby syndrome is a rare complication seen in neonates


with hyperbilirubinemia who are being treated with phototherapy. Affected
neonates develop gray-brown skin, serum, and urine within a week of
initiation of phototherapy.
5. Question

A primigravida in active labor is about 9 days post-term. The client desires a


bilateral pudendal block anesthesia before delivery. After the nurse explains
this type of anesthesia to the client, which of the following locations
identified by the client as the area of relief would indicate to the nurse that
the teaching was effective?

A. Back

B. Abdomen

C. Fundus

D. Perineum

Answer: D. Perineum

A bilateral pudendal block is used for vaginal deliveries to relieve pain


primarily in the perineum and vagina. Pudendal block anesthesia is adequate
for episiotomy and its repair.

Option A: A spinal anesthetic is given into the middle of the lower back and
local anesthetic is injected through the needle into the fluid that surrounds
the spinal cord. It numbs the nerves that supply the abdomen, hips, bottom,
and legs.
8
C.BY: HOSSAM HAMDY MATERNITY RNPEDIA – ‫موسوعه التمريض‬

Option B: General or regional anesthesia can be appropriate for patients


undergoing abdominal surgery. Balanced anesthesia with inhalational
anesthetics, opioids, and neuromuscular blockers are used in general
anesthesia for abdominal surgical procedures.

Option C: Spinal anesthesia is one of the most preferred anesthetic methods


during Cesarean section since it provides easy and rapid induction, effective
sensory and motor block, and has little effect on the fetus.
6. Question

The nurse is caring for a primigravida at about 2 months and 1-week


gestation. After explaining self-care measures for common discomforts of
pregnancy, the nurse determines that the client understands the instructions
when she says:

A. “Nausea and vomiting can be decreased if I eat a few crackers before


rising.”

B. “If I start to leak colostrum, I should cleanse my nipples with soap and
water.”

C. “If I have a vaginal discharge, I should wear nylon underwear.”

D. “Leg cramps can be alleviated if I put an ice pack on the area.”

Answer: A. “Nausea and vomiting can be decreased if I eat a few


crackers before arising”

Eating dry crackers before arising can assist in decreasing the common
discomfort of nausea and vomiting. Avoiding strong food odors and eating a
high-protein snack before bedtime can also help.

Option B: Colostrum is a milky fluid that’s released by mammals that have


recently given birth before breast milk production begins.

Option C: Cotton underwear is breathable and absorbent, which can help


prevent yeast infections.

Option D: A heating pad or hot pack may help relieve tight muscles in leg
cramps.

9
C.BY: HOSSAM HAMDY MATERNITY RNPEDIA – ‫موسوعه التمريض‬

7. Question

Forty-eight hours after delivery, the nurse in charge plans discharge teaching
for the client about infant care. By this time, the nurse expects that the phase
of postpartum psychological adaptation that the client would be in would be
termed which of the following?

A. Taking in

B. Letting go

C. Taking hold

D. Resolution

Answer: C. Taking hold

Beginning after completion of the taking-in phase, the taking-hold phase


lasts about 10 days. During this phase, the client is concerned with her need
to resume control of all facets of her life in a competent manner. At this
time, she is ready to learn self-care and infant care skills.

Option A: The taking-in phase usually sets 1 to 2 days after delivery. The
woman prefers to talk about her experiences during labor and birth and also
her pregnancy. The taking-in phase provides time for the woman to regain
her physical strength and organize her rambling thoughts about her new role.

Option B: During the letting go phase, the woman finally accepts her new
role and gives up her old roles like being a childless woman or just mother of
one child.

Option D: The resolution phase or ending phase is the final stage of the
nurse-client relationship. After the client’s problems or issues are addressed,
the relationship needs to be completed before it can be terminated.
8. Question

A pregnant client is diagnosed with partial placenta previa. In explaining the


diagnosis, the nurse tells the client that the usual treatment for partial
placenta previa is which of the following?
10
C.BY: HOSSAM HAMDY MATERNITY RNPEDIA – ‫موسوعه التمريض‬

A. Activity limited to bed rest.

B. Platelet infusion.

C. Immediate cesarean delivery.

D. Labor induction with oxytocin.

Answer: A. Activity limited to bed rest

Treatment of partial placenta previa includes bed rest, hydration, and careful
monitoring of the client’s bleeding.

Option B: The greatest risk of placenta previa is hemorrhage. Bleeding often


occurs as the lower part of the uterus thins during the third trimester of
pregnancy in preparation for labor. This may require blood transfusion
during Cesarean section.

Option C: In general, there is a higher Cesarean rate associated with


placental edge-to-cervical os distances of less than 2 cm.

Option D: Labor induction is the stimulation of uterine contractions during


pregnancy before labor begins on its own to achieve a vaginal birth. It is not
an option for placenta previa.
9. Question

The nurse plans to instruct the postpartum client about methods to prevent
breast engorgement. Which of the following measures would the nurse
include in the teaching plan?

A. Feeding the neonate a maximum of 5 minutes per side on the first day.

B. Wearing a supportive brassiere with nipple shields.

C. Breastfeeding the neonate at frequent intervals.

D. Decreasing fluid intake for the first 24 to 48 hours.

Answer: C. Breastfeeding the neonate at frequent intervals


11
C.BY: HOSSAM HAMDY MATERNITY RNPEDIA – ‫موسوعه التمريض‬

Prevention of breast engorgement is key. The best technique is to empty the


breast regularly while feeding. Engorgement is less likely when the mother
and neonate are together, as in single-room maternity care continuous
rooming-in, because nursing can be done conveniently to meet the neonate’s
and mother’s needs.

Option A: A newborn feeds every 2 to 3 hours. They should be breastfed 8-


12 times per day for about the first month. Frequent feedings help stimulate
your milk production during the first few weeks.

Option B: A nipple shield is usually meant to be used for a short time. When
using a shield, help the baby to latch on by himself with a wide-open mouth.
This will help the baby learn to breastfeed without a shield.

Option D: Breastfeeding women are recommended to increase fluid intake


by 800 ml/day during the first 6 months postpartum.
10. Question

When the nurse on duty accidentally bumps the bassinet, the neonate throws
out its arms, hands open, and begins to cry. The nurse interprets this reaction
as indicative of which of the following reflexes?

A. Startle reflex

B. Babinski reflex

C. Grasping reflex

D. Tonic neck reflex

Answer: A. Startle reflex

The Moro, or startle, reflex occurs when the neonate responds to stimuli by
extending the arms, hands open, and then moving the arms in an embracing
motion. The Moro reflex, present at birth, disappears at about age 3 months.

Option B: Babinski reflex occurs after the sole of the foot has been firmly
stroked. The big toe then moves upward or toward the top surface of the
foot. The other toe fan out.

12
C.BY: HOSSAM HAMDY MATERNITY RNPEDIA – ‫موسوعه التمريض‬

Option C: Palmar grasp reflex appears around 16 weeks of gestation and


can be elicited in preterm infants as young as 25 weeks of postconceptional
age. To elicit the reflex, the infant is laid in a symmetrical supine and the
examiner strokes the palm of the infant with his or her index finger. The
response to this stimulus comprises two phases: finger closing and clinging.

Option D: When a baby’s head is turned to one side, the arm on that side
stretches out and the opposite arm bends up at the elbow. The tonic neck
reflex lasts until the infant is about 5 to 7 months old.
11. Question

A primigravida client at 25 weeks gestation visits the clinic and tells the
nurse that her lower back aches when she arrives home from work. The
nurse should suggest that the client perform:

A. Tailor sitting

B. Leg lifting

C. Shoulder circling

D. Squatting exercises

Answer: A. Tailor sitting

Tailor sitting is an excellent exercise that helps to strengthen the client’s


back muscles and also prepares the client for the process of labor. The client
should be encouraged to rest periodically during the day and avoid standing
or sitting in one position for a long time.

Option B: The leg raise is a great way to strengthen the abdominal muscles.
It targets the lower abdominal muscles and hip muscles.

Option C: This exercise can warm up the shoulders, specifically the muscles
in the rotator cuff.

Option D: During pregnancy, squats are an excellent resistance exercise to


maintain strength and range of motion in the hips, glutes, core, and pelvic
floor muscles.
12. Question
13
C.BY: HOSSAM HAMDY MATERNITY RNPEDIA – ‫موسوعه التمريض‬

Which of the following would the nurse in charge do first after observing a
2-cm circle of bright red bleeding on the diaper of a neonate who just had a
circumcision?

A. Notify the neonate’s pediatrician immediately.

B. Check the diaper and circumcision again in 30 minutes.

C. Secure the diaper tightly to apply pressure on the site.

D. Apply gentle pressure to the site with a sterile gauze pad.

Answer: D. Apply gentle pressure to the site with a sterile gauze pad

If bleeding occurs after circumcision, the nurse should first apply gentle
pressure on the area with sterile gauze. Bleeding is not common but requires
attention when it occurs.

Option A: Immediate nursing intervention and assessment should be done


first before notifying the physician.

Option B: 30 minutes is a long time to reassess. Addressing the bleeding


immediately may save it from getting worse.

Option C: Tightening the diaper may elicit a case of diaper rash. Applying
direct pressure on the bleeding site is more effective.
13. Question

Which of the following would the nurse most likely expect to find when
assessing a pregnant client with abruption placenta?

A. Excessive vaginal bleeding

B. Rigid, board-like abdomen

C. Tetanic uterine contractions

D. Premature rupture of membranes

14
C.BY: HOSSAM HAMDY MATERNITY RNPEDIA – ‫موسوعه التمريض‬

Answer: B. Rigid, board-like abdomen

The most common assessment finding in a client with abruption placenta is a


rigid or boardlike abdomen. Pain, usually reported as a sharp stabbing
sensation high in the uterine fundus with the initial separation, also is
common.

Option A: It’s possible for the blood to become trapped inside the uterus, so
even with a severe placental abruption, there might be no visible bleeding.

Option C: Uterine contractions are a common finding with placental


abruption. Contractions progress as the abruption expands, and uterine
hypertonus may be noted. Contractions are painful and palpable.

Option D: Increased frequency of placental abruption was found in patients


with early rupture of membranes. The incidence was 50% and 44% when
rupture of the membranes occurred before 20 weeks or between 20-24 weeks
of pregnancy, respectively.
14. Question

While the client is in active labor with twins and the cervix is 5 cm dilated,
the nurse observes contractions occurring at a rate of every 7 to 8 minutes in
a 30-minute period. Which of the following would be the nurse’s most
appropriate action?

A. Note the fetal heart rate patterns.

B. Notify the physician immediately.

C. Administer oxygen at 6 liters by mask.

D. Have the client pant-blow during the contractions.

Answer: B. Notify the physician immediately.

The nurse should contact the physician immediately because the client is
most likely experiencing hypotonic uterine contractions. These contractions
tend to be painful but ineffective. The usual treatment is oxytocin
augmentation unless cephalopelvic disproportion exists.
15
C.BY: HOSSAM HAMDY MATERNITY RNPEDIA – ‫موسوعه التمريض‬

Option A: A baby’s heart rate during labor should be between 110 and 160
beats per minute, but it may fluctuate above and below this rate for a variety
of reasons.

Option C: Maternal oxygen is often given to laboring women to improve


fetal metabolic status or in an attempt to alleviate non-reassuring fetal heart
rate patterns. However, there are only two randomized trials investigating the
use of maternal oxygen supplementation in laboring women. These studies
did not find that supplementation is likely to benefit the fetus and may even
be harmful.

Option D: During active labor, breathing should be as slow as possible, but


it can be sped up as the intensity of the contraction increases. As the
contraction peaks and the breathing rate increases, switch to light breathing
both in and out the mouth, about one breath per second.
15. Question

A client tells the nurse, “I think my baby likes to hear me talk to him.” When
discussing neonates and stimulation with sound, which of the following
would the nurse include as a means to elicit the best response?

A. High-pitched speech with tonal variations.

B. Low-pitched speech with a sameness of tone.

C. Cooing sounds rather than words.

D. Repeated stimulation with loud sounds.

Answer: A. High-pitched speech with tonal variations

Providing stimulation and speaking to neonates is important. Some


authorities believe that speech is the most important type of sensory
stimulation for a neonate. Neonates respond best to speech with tonal
variations and a high-pitched voice. A neonate can hear all sound louder than
about 55 decibels.

Option B: Low pitched speech is less effective for neonates because they
can hear all sounds louder than about 55 decibels.

16
C.BY: HOSSAM HAMDY MATERNITY RNPEDIA – ‫موسوعه التمريض‬

Option C: At about two months, the infant may start cooing and repeating
vowel sounds. Imitate his cooing while also adding simple words and
phrases over the first four to six months.

Option D: A baby’s hearing is very sensitive and can be easily damaged by


loud sounds. It is recommended to keep sounds around the infant quieter
than 60 decibels.
16. Question

A 31-year-old multipara is admitted to the birthing room after initial


examination reveals her cervix to be at 8 cm, completely effaced (100 %),
and at 0 station. What phase of labor is she in?

A. Active phase

B. Latent phase

C. Expulsive phase

D. Transitional phase

Answer: D. Transitional phase

The transitional phase of labor extends from 8 to 10 cm; it is the shortest but
most difficult and intense for the patient.

Option A: The active phase extends from 4 to 7 cm; it is moderate for the
patient.

Option B: The latent phase extends from 0 to 3 cm; it is mild in nature.

Option C: The expulsive phase begins immediately after the birth and ends
with separation and expulsion of the placenta.
17. Question

A pregnant patient asks the nurse if she can take castor oil for her
constipation. How should the nurse respond?

17
C.BY: HOSSAM HAMDY MATERNITY RNPEDIA – ‫موسوعه التمريض‬

A. “Yes, it produces no adverse effect.”

B. “No, it can initiate premature uterine contractions.”

C. “No, it can promote sodium retention.”

D. “No, it can lead to increased absorption of fat-soluble vitamins.”

Answer: B. “No, it can initiate premature uterine contractions.”

Castor oil can initiate premature uterine contractions in pregnant women. It


also can produce other adverse effects, but it does not promote sodium
retention.

Option A: Castor oil is a harsh stimulant laxative that relieves constipation


by forced bowel movements. Side effects may include nausea, stimulation of
uterine activity, meconium-stained fluid, and amniotic fluid embolism.

Option C: There is no evidence that suggests that castor oil can promote
sodium retention.

Option D: Castor oil is not known to increase absorption of fat-soluble


vitamins, although laxatives, in general, may decrease absorption if intestinal
motility is increased.
18. Question

A patient in her 14th week of pregnancy has presented with abdominal


cramping and vaginal bleeding for the past 8 hours. She has passed several
clots. What is the primary nursing diagnosis for this patient?

A. Knowledge deficit

B. Fluid volume deficit

C. Anticipatory grieving

D. Pain

Answer: B. Fluid volume deficit


18
C.BY: HOSSAM HAMDY MATERNITY RNPEDIA – ‫موسوعه التمريض‬

If bleeding and clots are excessive, this patient may become hypovolemic.
Pad count should be instituted. Blood volume expands during pregnancy,
and a considerable portion of the weight of a pregnant woman is retained
water.

Option A: Knowledge deficit is an appropriate nursing diagnosis because


the woman might not have any knowledge on how to manage her symptoms.
However, this is not a priority diagnosis.

Option C: Anticipatory grieving is the name given to the tumultuous set of


feelings and reactions that occur when someone is expecting the death of a
loved one.

Option D: Pain may be felt due to abdominal cramping accompanied by


bleeding. This is not a cause of alarm since true labor pain includes strong
and regular contractions and lower back pain.
19. Question

Immediately after delivery, the nurse-midwife assesses the neonate’s head


for signs of molding. Which factors determine the type of molding?

A. Fetal body flexion or extension

B. Maternal age, body frame, and weight

C. Maternal and paternal ethnic backgrounds

D. Maternal parity and gravidity

Answer: A. Fetal body flexion or extension

Fetal attitude—the overall degree of body flexion or extension—determines


the type of molding in the head of a neonate.

Option B: When a baby is born in a cephalic position, pressure on the head


in the birth canal may mold the head into an oblong shape. The mother’s age,
body frame, and weight do not affect the pressure.

Option C: There is research that indicates that infant head molding, the
application of pressure or bindings to cranial bones to alter their shapes, is

19
C.BY: HOSSAM HAMDY MATERNITY RNPEDIA – ‫موسوعه التمريض‬

prevalent among various Caribbean, Latino, European, African American,


Asian, and Native American groups.

Option D: Infants born by primiparous women showed significantly higher


degrees of molding of the head than those born by multiparous women.
20. Question

For a patient in active labor, the nurse-midwife plans to use an internal


electronic fetal monitoring (EFM) device. What must occur before the
internal EFM can be applied?

A. The membranes must rupture.

B. The fetus must be at 0 station.

C. The cervix must be dilated fully.

D. The patient must receive anesthesia.

Answer: A. The membranes must rupture.

Internal fetal heart rate monitoring uses an electronic transducer connected


directly to the fetal skin. A wire electrode is attached to the fetal scalp or
other body parts through the cervical opening and is connected to the
monitor. Internal EFM can be applied only after the patient’s membranes
have ruptured when the fetus is at least at the -1 station, and when the cervix
is dilated at least 2 cm.

Option B: An electrode will be attached to the part of the infant’s body that
is closest to the cervical opening.

Option C: Dilation of at least 2 cm is adequate enough to insert the


electrode through the cervical opening.

Option D: Although the patient may receive anesthesia, it is not required


before application of an internal EFM device.
21. Question

20
C.BY: HOSSAM HAMDY MATERNITY RNPEDIA – ‫موسوعه التمريض‬

A primigravida patient is admitted to the labor delivery area. Assessment


reveals that she is in the early part of the first stage of labor. Her pain is
likely to be most intense:

A. Around the pelvic girdle

B. Around the pelvic girdle and in the upper arms

C. Around the pelvic girdle and at the perineum

D. At the perineum

Answer: A. Around the pelvic girdle

During most of the first stage of labor, pain centers around the pelvic girdle.
During the late part of this stage and the early part of the second stage, pain
spreads to the upper legs and perineum. The pain of early labor is referred to
T10-T12 dermatomes such that the pain is felt in the lower abdomen,
sacrum, and back. This pain is dull in character and is not always sensitive to
opioid drugs.

Option B: Upper arm pain is not common during any stage of labor. With
each uterine contraction, pressure is transmitted to the cervix causing
stretching and distension and activating excitatory nociceptive afferents.

Option C: Pain arises due to afferents that innervate the vaginal surface of
the cervix, perineum, and vagina and occurs as a result of stretching,
distention, ischemia, and injury of the pelvic floor, perineum, and vagina.

Option D: During the late part of the second stage and childbirth, intense
pain occurs at the perineum. Somatic pain occurs closer to delivery, is sharp
in character and easily localized to the vagina, perineum, and rectum. It
radiates to the adjacent dermatomes T10 and L1 and compared to visceral
pain, is more resistant to opioid drugs.
22. Question

A female adult patient is taking a progestin-only oral contraceptive or mini


pill. Progestin use may increase the patient’s risk for:

A. Endometriosis

21
C.BY: HOSSAM HAMDY MATERNITY RNPEDIA – ‫موسوعه التمريض‬

B. Female hypogonadism

C. Premenstrual syndrome

D. Tubal or ectopic pregnancy

Answer: D. Tubal or ectopic pregnancy

Women taking the mini pill have a higher incidence of tubal and ectopic
pregnancies, possibly because progestin slows ovum transport through the
fallopian tubes.

Option A: Progestins are widely regarded as effective treatments for the


symptoms of endometriosis despite not all being indicated for the treatment
of the disease. It is not yet fully understood how progestins relieve the
symptoms of endometriosis, but they probably work by suppressing the
growth of endometrial implants in some way, causing them to gradually
waste away.

Option B: Hypogonadism is a condition in which the male testes or the


female ovaries produce little or no sex hormones. Treatment may involve
estrogen and progesterone pills or skin patches, GnRH injections, or HCG
injections.

Option C: Premenstrual syndrome is defined as the recurrence of


psychological and physical symptoms in the luteal phase, which remit in the
follicular phase of the menstrual cycle. The rationale for the use of
progesterone and progestogens in the management of premenstrual
syndrome is based on the unsubstantiated premise that progesterone
deficiency is the cause.
23. Question

A patient with pregnancy-induced hypertension probably exhibits which of


the following symptoms?

A. Proteinuria, headaches, vaginal bleeding

B. Headaches, double vision, vaginal bleeding

C. Proteinuria, headaches, double vision

22
C.BY: HOSSAM HAMDY MATERNITY RNPEDIA – ‫موسوعه التمريض‬

D. Proteinuria, double vision, uterine contractions

Answer: C. Proteinuria, headaches, double vision

A patient with pregnancy-induced hypertension complains of a headache,


double vision, and sudden weight gain. A urine specimen reveals proteinuria.

Option A: Pre-eclampsia increases the risk for placental abruption, a


condition in which the placenta separates from the inner wall of the uterus
before delivery. Severe abruption can cause heavy bleeding, which can be
life-threatening for both the baby and the mother.

Option B: Any hypertensive disorder of pregnancy can result in


preeclampsia. It occurs in up to 35% of women with gestational hypertension
and up to 25% of those with chronic hypertension. The underlying
pathophysiology that upholds this transition to, or superposition of,
preeclampsia is not well understood; however, it is thought to be related to a
mechanism of reduced placental perfusion inducing the systemic vascular
endothelial dysfunction.

Option D: Symptoms of preeclampsia may include visual disturbances,


typically scintillations and scotomata, presumed to be due to cerebral
vasospasm. The woman may describe new-onset headache that is frontal,
throbbing, or similar to a migraine headache, and gastrointestinal complaints
of sudden, new-onset, constant epigastric pain that may be moderate to
severe in intensity and due to hepatic swelling and inflammation, with
stretch of the liver capsule.
24. Question

Because cervical effacement and dilation are not progressing in a patient in


labor, the doctor orders I.V. administration of oxytocin (Pitocin). Why
should the nurse monitor the patient’s fluid intake and output closely during
oxytocin administration?

A. Oxytocin causes water intoxication.

B. Oxytocin causes excessive thirst.

C. Oxytocin is toxic to the kidneys.

23
C.BY: HOSSAM HAMDY MATERNITY RNPEDIA – ‫موسوعه التمريض‬

D. Oxytocin has a diuretic effect.

Answer: A. Oxytocin causes water intoxication.

The nurse should monitor fluid intake and output because prolonged
oxytocin infusion may cause severe water intoxication, leading to seizures,
coma, and death. In addition, oxytocin may cause water intoxication via an
antidiuretic hormone-like activity when administered in excessive doses with
electrolyte-free solution.

Option B: Excessive thirst results from the work of labor and limited oral
fluid intake—not oxytocin.

Option C: Oxytocin, when given in rapid bolus, produces marked but short-
lived hypotension and tachycardia. Sometimes, this abrupt and severe
hemodynamic depression may need to be distinguished from placental
abruption, myocardial infarction, or a pulmonary embolism in patients
undergoing delivery.

Option D: Oxytocin is known to possess antidiuretic properties. It can


function physiologically as an antidiuretic hormone, mimicking the short-
term action of vasopressin on water permeability, albeit with somewhat
lower potency.
25. Question

Five hours after birth, a neonate is transferred to the nursery, where the nurse
intervenes to prevent hypothermia. What is a common source of radiant heat
loss?

A. Low room humidity

B. Cold weight scale

C. Cool incubator walls

D. Cool room temperature

Answer: C. Cools incubator walls

24
C.BY: HOSSAM HAMDY MATERNITY RNPEDIA – ‫موسوعه التمريض‬

A common source of radiant heat loss includes cool incubator walls and
windows. Radiant heat loss constitutes the transfer of heat from an infant’s
warm skin, via infrared electromagnetic waves, to the cooler surrounding
walls that absorb heat.

Option A: Low room humidity promotes evaporative heat loss. Evaporative


heat loss occurs through the skin and the respiratory system. The driving
force behind evaporation is the vapor pressure difference between the body
surface and the environment.

Option B: When the skin directly contacts a cooler object, such as a cold
weight scale, conductive heat loss may occur. Heat loss can occur by
conduction of heat from the skin to the layer of still air around the body.

Option D: A cool room temperature may lead to convective heat loss.


Convective heat loss is the transfer of heat from a body to moving molecules
such as air or liquid.
26. Question

After administering bethanechol to a patient with urine retention, the nurse in


charge monitors the patient for adverse effects. Which is most likely to
occur?

A. Decreased peristalsis

B. Increase heart rate

C. Dry mucous membranes

D. Nausea and Vomiting

Answer: D. Nausea and Vomiting

Bethanechol will increase GI motility, which may cause nausea, belching,


vomiting, intestinal cramps, and diarrhea. Bethanechol directly stimulates
cholinergic receptors in the parasympathetic nervous system while
stimulating the ganglia to a lesser extent.

Option A: Peristalsis is increased rather than decreased. Stimulation of


muscarinic receptors in the GI tract restores peristalsis, increases motility,
and increases the resting lower esophageal sphincter pressure.
25
C.BY: HOSSAM HAMDY MATERNITY RNPEDIA – ‫موسوعه التمريض‬

Option B: With high doses of bethanechol, cardiovascular responses may


include vasodilation, decreased cardiac rate, and decreased the force of
cardiac contraction, which may cause hypotension.

Option C: Salivation or sweating may gently increase because of its


cholinergic effects.
27. Question

The nurse in charge is caring for a patient who is in the first stage of labor.
What is the shortest but most difficult part of this stage?

A. Active phase

B. Complete phase

C. Latent phase

D. Transitional phase

Answer: D. Transitional phase

The transitional phase, which lasts 1 to 3 hours, is the shortest but most
difficult part of the first stage of labor. This phase is characterized by intense
uterine contractions that occur every 1 ½ to 2 minutes and last 45 to 90
seconds.

Option A: The active phase lasts 4 ½ to 6 hours; it is characterized by


contractions that start out moderately intense, grow stronger, and last about
60 seconds.

Option B: The complete phase occurs during the second, not first, stage of
labor.

Option C: The latent phase lasts 5 to 8 hours and is marked by mild, short,
irregular contractions.
28. Question

26
C.BY: HOSSAM HAMDY MATERNITY RNPEDIA – ‫موسوعه التمريض‬

After 3 days of breastfeeding, a postpartum patient reports nipple soreness.


To relieve her discomfort, the nurse should suggest that she:

A. Apply warm compresses to her nipples just before feeding.

B. Lubricate her nipples with expressed milk before feeding.

C. Dry her nipples with a soft towel after feeding.

D. Apply soap directly to her nipples, and then rinse.

Answer: B. Lubricate her nipples with expressed milk before feeding

Measures that help relieve nipple soreness in a breastfeeding patient include


lubricating the nipples with a few drops of expressed milk before feedings,
applying ice compresses just before feeding, letting the nipples air dry after
feedings, and avoiding the use of soap on the nipples.

Option A: Cold compresses are applied instead of warm because it reduces


swelling and pain. Use a piece of fabric between the skin and the cold
compress. Never apply an ice pack directly to the skin.

Option C: Air drying prevents the clothing from sticking to and irritating the
breast.

Option D: Soap removes the nipples’ natural lubricants and will dry them
out.
29. Question

The nurse is developing a teaching plan for a patient who is 8 weeks


pregnant. The nurse should tell the patient that she can expect to feel the
fetus move at which time?

A. Between 10 and 12 weeks’ gestation

B. Between 16 and 20 weeks’ gestation.

C. Between 21 and 23 weeks’ gestation.

D. Between 24 and 26 weeks’ gestation.

27
C.BY: HOSSAM HAMDY MATERNITY RNPEDIA – ‫موسوعه التمريض‬

Answer: B. Between 16 and 20 weeks’ gestation.

A pregnant woman usually can detect fetal movement (quickening) between


16 and 20 weeks’ gestation.

Option A: Before 16 weeks, the fetus is not developed enough for the
woman to detect movement.

Option C: After 20 weeks, the fetus continues to gain weight steadily, the
lungs start to produce surfactant, the brain is grossly formed, and
myelination of the spinal cord begins.

Option D: After 24 weeks, the fetus might be able to respond to familiar


sounds such as its mother’s voice, with movement. It is spending most of its
sleep time in rapid eye movement (REM).
30. Question

Normal lochial findings in the first 24 hours post-delivery include:

A. Bright red blood

B. Large clots or tissue fragments

C. A foul odor

D. The complete absence of lochia

Answer: A. Bright red blood

Lochia should never contain large clots, tissue fragments, or membranes. A


foul odor may signal infection, as may absence of lochia.

Option B: The blood clots in the lochia should get smaller and happen less
often as the bleeding gets less over the first few days.

Option C: Lochia with offensive odor may indicate infection.

Option D: Complete absence of lochia might be a sign of infection.

28
C.BY: HOSSAM HAMDY MATERNITY RNPEDIA – ‫موسوعه التمريض‬

31. Question

Accompanied by her husband, a patient seeks admission to the labor and


delivery area. The client states that she is in labor and says she attended the
hospital clinic for prenatal care. Which question should the nurse ask her
first?

A. “Do you have any chronic illness?”

B. “Do you have any allergies?”

C. “What is your expected due date?”

D. “Who will be with you during labor?”

Answer: C. “What is your expected due date?”

When obtaining the history of a patient who may be in labor, the nurse’s
highest priority is to determine her current status, particularly her due date,
gravidity, and parity. Gravidity and parity affect the duration of labor and the
potential for labor complications. Later, the nurse should ask about chronic
illness, allergies, and support persons.

Option A: After asking for the expected due date, obtain the client’s
problems during this or previous pregnancies.

Option B: Asking about any known allergies may be done after inquiring
about prior ultrasonographic examinations and results, and bleeding during
pregnancy or labor.

Option D: This may be asked if the client’s health history and present health
history, which are some of the most important details, are already obtained.
32. Question

A patient is in the second stage of labor. During this stage, how frequently
should the nurse in charge assess her uterine contractions?

A. Every 5 minutes.

29
C.BY: HOSSAM HAMDY MATERNITY RNPEDIA – ‫موسوعه التمريض‬

B. Every 15 minutes.

C. Every 30 minutes.

D. Every 60 minutes.

Answer: B. Every 15 minutes

During the second stage of labor, the nurse should assess the strength,
frequency, and duration of contraction every 15 minutes. If maternal or fetal
problems are detected, more frequent monitoring is necessary.

Option A: Second stage of labor starts when cervical dilatation reaches 10


cm and ends when the baby is delivered. At this stage, the patient feels an
uncontrollable urge to push. Monitoring every 5 minutes would be too
frequent and inconvenient for the laboring mother.

Option C: After cervical dilation is complete, the fetus descends into the
vaginal canal with or without maternal pushing efforts. The fetus passes
through the birth canal via 7 movements known as the cardinal movements.
These include engagement, descent, flexion, internal rotation, extension,
external rotation, and expulsion. These movements may occur in a few
minutes, so 30 minutes might be too long to assess for contractions.

Option D: Monitoring hourly would be too long and the nurse might miss
the important details of assessment.
33. Question

A patient is in her last trimester of pregnancy. Nurse Vickie should instruct


her to notify her primary health care provider immediately if she notices:

A. Blurred vision

B. Hemorrhoids

C. Increased vaginal mucus

D. Shortness of breath on exertion

30
C.BY: HOSSAM HAMDY MATERNITY RNPEDIA – ‫موسوعه التمريض‬

Answer: A. Blurred vision

Blurred vision or other visual disturbance, excessive weight gain, edema,


and increased blood pressure may signal severe preeclampsia. This condition
may lead to eclampsia, which has potentially serious consequences for both
the patient and fetus.

Option B: Although hemorrhoids may be a problem during pregnancy, they


do not require immediate attention. Hemorrhoids occur when the external
hemorrhoidal veins become varicose (enlarged and swollen), which causes
itching, burning, painful swellings at the anus, dyschezia (painful bowel
movements), and bleeding.

Option C: Almost all women have more vaginal discharge in pregnancy.


This is normal, and helps prevent any infections travelling up from the
vagina to the womb. Towards the end of pregnancy, the amount of discharge
increases further. In the last week or so of pregnancy, it may contain streaks
of sticky, jelly-like pink mucus.

Option D: Dyspnea can begin before any upward displacement of the


diaphragm, suggesting that factors other than mechanical pressure may be
involved. It probably results from the subjective awareness of
hyperventilation that is universally present in pregnancy. Hyperventilation in
pregnancy is predominantly due to an increase in the depth of the tidal
volume, with little change in the respiratory rate
34. Question

The nurse in-charge is reviewing a patient’s prenatal history. Which finding


indicates a genetic risk factor?

A. The patient is 25 years old.

B. The patient has a child with cystic fibrosis.

C. The patient was exposed to rubella at 36 weeks’ gestation.

D. The patient has a history of preterm labor at 32 weeks’ gestation.

Answer: B. The patient has a child with cystic fibrosis

31
C.BY: HOSSAM HAMDY MATERNITY RNPEDIA – ‫موسوعه التمريض‬

Cystic fibrosis is a recessive trait; each offspring has a one in four chance of
having the trait or the disorder.

Option A: Maternal age is not a risk factor until age 35, when the incidence
of chromosomal defects increases.

Option C: Maternal exposure to rubella during the first trimester may cause
congenital defects.

Option D: Although a history or preterm labor may place the patient at risk
for preterm labor, it does not correlate with genetic defects.
35. Question

An adult female patient is using the rhythm (calendar-basal body


temperature) method of family planning. In this method, the unsafe period
for sexual intercourse is indicated by:

A. Return preovulatory basal body temperature.

B. Basal body temperature increase of 0.1 degrees to 0.2 degrees on the 2nd
or 3rd day of cycle.

C. 3 full days of elevated basal body temperature and clear, thin cervical
mucus.

D. Breast tenderness and mittelschmerz.

Answer: C. 3 full days of elevated basal body temperature and clear,


thin cervical mucus.

Ovulation (the period when pregnancy can occur) is accompanied by a basal


body temperature increase of 0.7 degrees F to 0.8 degrees F and clear, thin
cervical mucus.

Option A: A return to the preovulatory body temperature indicates a safe


period for sexual intercourse.

Option B: A slight rise in basal temperature early in the cycle is not


significant. Ovulation may cause a slight increase in basal body temperature.

32
C.BY: HOSSAM HAMDY MATERNITY RNPEDIA – ‫موسوعه التمريض‬

Option D: Breast tenderness and mittelschmerz are not reliable indicators of


ovulation. Mittelschmerz is one-sided, lower abdominal pain associated with
ovulation. It occurs midway through a menstrual cycle.
36. Question

During a nonstress test (NST), the electronic tracing displays a relatively flat
line for fetal movement, making it difficult to evaluate the fetal heart rate
(FHR). To mark the strip, the nurse in charge should instruct the client to
push the control button at which time?

A. At the beginning of each fetal movement.

B. At the beginning of each contraction.

C. After every three fetal movements

D. At the end of fetal movement.

Answer: A. At the beginning of each fetal movement

An NST assesses the FHR during fetal movement. In a healthy fetus, the
FHR accelerates with each movement. By pushing the control button when a
fetal movement starts, the client marks the strip to allow easy correlation of
fetal movement with the FHR.

Option B: The FHR is assessed during uterine contractions in the oxytocin


contraction test, not the NST. The Non-Stress Test (NST) is an assessment
tool used from 32 weeks of gestation to term to evaluate fetal health through
the use of electronic fetal monitors that continuously record the fetal heart
rate (FHR).

Option C: Pushing the control button after every three fetal movements
wouldn’t allow accurate comparison of fetal movement and FHR change.
Fetal activity may be recorded by the patient using an event marker or noted
by the staff performing the test.

Option D: The presence of fetal heart rate acceleration with fetal movement
is the principle behind the non-stress test. It is vital to start monitoring the
moment the client recognizes a fetal movement. The NST recognizes the
coupling of fetal neurological status to cardiovascular reflex responses. It is
33
C.BY: HOSSAM HAMDY MATERNITY RNPEDIA – ‫موسوعه التمريض‬

one of the factors that tends to disappear earliest during progressive fetal
compromise.
37. Question

When evaluating a client’s knowledge of symptoms to report during her


pregnancy, which statement would indicate to the nurse in charge that the
client understands the information given to her?

A. “I’ll report increased frequency of urination.”

B. “If I have blurred or double vision, I should call the clinic immediately.”

C. “If I feel tired after resting, I should report it immediately.”

D. “Nausea should be reported immediately.”

Answer: B. “If I have blurred or double vision, I should call the clinic
immediately.”

Blurred or double vision may indicate hypertension or preeclampsia and


should be reported immediately. It can affect the visual pathways, from the
anterior segment to the visual cortex.

Option A: Urinary frequency is a common problem during pregnancy


caused by increased weight pressure on the bladder from the uterus. The
anatomical and physiological changes affecting the lower urinary tract in
pregnancy, as well as the hormonal milieu of pregnancy, have been
postulated to underlie the pathogenesis of lower urinary symptoms in
pregnancy

Option C: Clients generally experience fatigue during pregnancy.


Pregnancy is accompanied by several psychological, emotional, and physical
changes that may predispose the woman to fatigue, which can range from
mild tiredness to severe exhaustion.

Option D: The pathophysiology of nausea and vomiting during early


pregnancy is unknown, although metabolic, endocrine, GI, and psychologic
factors probably all play a role. Estrogen may contribute because estrogen
levels are elevated in patients with hyperemesis gravidarum.
38. Question
34
C.BY: HOSSAM HAMDY MATERNITY RNPEDIA – ‫موسوعه التمريض‬

When assessing a client during her first prenatal visit, the nurse discovers
that the client had a reduction mammoplasty. The mother indicates she wants
to breast-feed. What information should the nurse give to this mother
regarding breastfeeding success?

A. “It’s contraindicated for you to breastfeed following this type of surgery.”

B. “I support your commitment; however, you may have to supplement each


feeding with formula.”

C. “You should check with your surgeon to determine whether breast-


feeding would be possible.”

D. “You should be able to breastfeed without difficulty.”

Answer: B. “I support your commitment; however, you may have to


supplement each feeding with formula.”

Recent breast reduction surgeries are done in a way to protect the milk sacs
and ducts, so breastfeeding after surgery is possible. Still, it’s good to check
with the surgeon to determine what breast reduction procedure was done.
There is the possibility that reduction surgery may have decreased the
mother’s ability to meet all of her baby’s nutritional needs, and some
supplemental feeding may be required. Preparing the mother for this
possibility is extremely important because the client’s psychological
adaptation to mothering may be dependent on how successfully she breast-
feeds.

Option A: While there is evidence that both breastfeeding and breast


reduction surgery are beneficial, it is unknown whether breast reduction
surgery impacts breastfeeding and whether any breast reduction technique
differentially preserves the ability to breastfeed.

Option C: Women considering breast reduction surgery should be told not


only the name of the proposed breast reduction technique but its
characteristics, including the extent the column of subareolar parenchyma
will be preserved and pedicle width, to allow them to gain a better
understanding of its impact on breastfeeding.

Option D: Breast reduction techniques have been in a continuous state of


development since the early 1900s, with new techniques developed, refined,
35
C.BY: HOSSAM HAMDY MATERNITY RNPEDIA – ‫موسوعه التمريض‬

and modified by subsequent plastic surgeons. This has led to many diverse
breast reduction techniques. Its effect on breastfeeding remains entirely
unclear, so telling the client that she could breastfeed without difficulty
would give her a false sense of reassurance.
39. Question

Following a precipitous delivery, examination of the client’s vagina reveals a


fourth-degree laceration. Which of the following would be contraindicated
when caring for this client?

A. Applying cold to limit edema during the first 12 to 24 hours.

B. Instructing the client to use two or more peri pads to cushion the area.

C. Instructing the client on the use of sitz baths if ordered.

D. Instructing the client about the importance of perineal (Kegel) exercises.

Answer: B. Instructing the client to use two or more peri pads to


cushion the area

Using two or more peripads would do little to reduce the pain or promote
perineal healing. A fourth-degree perineal laceration is the injury to the
perineum involving the anal sphincter complex and anorectal mucosa.

Option A: Ice packs can help reduce pain and swelling in the perineum. Use
ice cubes in a clean, disposable glove. Wrapped in a damp cloth or place the
ice pack inside a pad. Never apply directly on skin. Apply for 10-20 minutes.
Repeat every 2-3 hours until pain and swelling decrease.

Option C: Hot sitz bath may help speed up the healing process. Use sitz
baths a few times a day, 24 hours after giving birth. Sit in water that covers
the vulvar area.

Option D: The muscles lie deep in the pelvis and support the pelvic organs
and control the bladder and bowel function. The pelvic floor muscles attach
to the pubic bone at the front, tail bone at the back, and from one sitting bone
to the other sitting bone. It is important to retrain the muscles after a tear, to
prevent problems such as incontinence.
40. Question
36
C.BY: HOSSAM HAMDY MATERNITY RNPEDIA – ‫موسوعه التمريض‬

A client makes a routine visit to the prenatal clinic. Although she is 14


weeks pregnant, the size of her uterus approximates that in an 18- to 20-
week pregnancy. Dr. Charles diagnoses gestational trophoblastic disease and
orders ultrasonography. The nurse expects ultrasonography to reveal:

A. An empty gestational sac.

B. Grapelike clusters.

C. A severely malformed fetus.

D. An extrauterine pregnancy.

Answer: B. Grapelike clusters.

In a client with gestational trophoblastic disease, an ultrasound performed


after the 3rd month shows grapelike clusters of transparent vesicles rather
than a fetus. The vesicles contain a clear fluid and may involve all or part of
the decidual lining of the uterus. Usually, no embryo (and therefore no fetus)
is present because it has been absorbed.

Option A: An anembryonic pregnancy is characterized by a gestational sac


that forms and grows while an embryo fails to develop. Etiologies include
morphological abnormalities of an embryo that prevents implantation or
prevents long term survival of the embryo after implantation; chromosomal
abnormalities that collectively include autosomal trisomy, polyploidy, sex
chromosomal polysomy, and monosomy X likely represent the most
common etiologies for early pregnancy loss; and other genetic and
chromosomal abnormalities include translocations, inversions, single-gene
perturbations, and placental mosaicism.

Option C: Congenital anomalies are also known as birth defects, congenital


disorders or congenital malformations. Congenital anomalies can be defined
as structural or functional anomalies (for example, a severely malformed
fetus) that occur during intrauterine life and can be identified prenatally, at
birth, or sometimes may only be detected later in infancy, such as hearing
defects.

Option D: Because there is no fetus, there can be no extrauterine pregnancy.


An extrauterine pregnancy is seen with an ectopic pregnancy.

37
C.BY: HOSSAM HAMDY MATERNITY RNPEDIA – ‫موسوعه التمريض‬

41. Question

After completing a second vaginal examination of a client in labor, the


nurse-midwife determines that the fetus is in the right occiput anterior
position and at (–1) station. Based on these findings, the nurse-midwife
knows that the fetal presenting part is:

A. 1 cm below the ischial spines.

B. Directly in line with the ischial spines.

C. 1 cm above the ischial spines.

D. In no relationship to the ischial spines.

Answer: C. 1 cm above the ischial spines.

Fetal station — the relationship of the fetal presenting part to the maternal
ischial spines — is described in the number of centimeters above or below
the spines. A presenting part above the ischial spines is designated as –1, –2,
or –3.

Option A: A presenting part below the ischial spines, as +1, +2, or +3.

Option B: 0 station is when the baby’s head is even with the ischial spines.
The baby is said to be “engaged” when the largest part of the head has
entered the pelvis.

Option D: If the head is high and not yet engaged in the birth canal, it may
float away from the physician’s fingers during the vaginal exam.
42. Question

Which of the following would be inappropriate to assess in a mother who’s


breastfeeding?

A. The attachment of the baby to the breast.

B. The mother’s comfort level with positioning the baby.

38
C.BY: HOSSAM HAMDY MATERNITY RNPEDIA – ‫موسوعه التمريض‬

C. Audible swallowing.

D. The baby’s lips smacking.

Answer: D. The baby’s lips smacking

Assessing the attachment process for breast-feeding should include all of the
answers except the smacking of lips. A baby who’s smacking his lips isn’t
well attached and can injure the mother’s nipples.

Option A: A good attachment shows much of the areola and the tissues
underneath it, including the larger ducts, are in the baby’s mouth; the breast
is stretched out to form a long “teat”, but the nipple only forms about one-
third of the “teat”; the baby’s tongue is forward over the lower gums,
beneath the milk ducts; and the baby is suckling from the breast, not from
the nipple.

Option B: To be well attached at the breast, a baby and his or her mother
need to be appropriately positioned. The mother can be sitting or lying down,
or standing, if she wishes. However, she needs to be relaxed and
comfortable, and without strain, particularly of her back. The baby can
breastfeed in several different positions in relation to the mother: across her
chest and abdomen, under her arm, or alongside her body.

Option C: When the milk ejection reflex is triggered, the baby may swallow
after every suck in order to handle the rapid flow of milk. You should hear
suck, swallow, pause, suck, swallow, pause. Audible swallowing after every
couple of sucks should continue for about ten minutes.
43. Question

During a prenatal visit at 4 months gestation, a pregnant client asks whether


tests can be done to identify fetal abnormalities. Between 18 and 40 weeks
gestation, which procedure is used to detect fetal anomalies?

A. Amniocentesis

B. Chorionic villi sampling

C. Fetoscopy

D. Ultrasound
39
C.BY: HOSSAM HAMDY MATERNITY RNPEDIA – ‫موسوعه التمريض‬

Answer: D. Ultrasound

Ultrasound is used between 18 and 40 weeks’ gestation to identify normal


fetal growth and detect fetal anomalies and other problems.

Option A: Amniocentesis is done during the third trimester to determine


fetal lung maturity.

Option B: Chorionic villi sampling is performed at 8 to 12 weeks’ gestation


to detect genetic disease.

Option C: Fetoscopy is done at approximately 18 weeks’ gestation to


observe the fetus directly and obtain a skin or blood sample.
44. Question

A client, 30 weeks pregnant, is scheduled for a biophysical profile (BPP) to


evaluate the health of her fetus. Her BPP score is 8. What does this score
indicate?

A. The fetus should be delivered within 24 hours.

B. The client should repeat the test in 24 hours.

C. The fetus isn’t in distress at this time.

D. The client should repeat the test in 1 week.

Answer: C. The fetus isn’t in distress at this time.

The BPP evaluates fetal health by assessing five variables: fetal breathing
movements, gross body movements, fetal tone, reactive fetal heart rate, and
qualitative amniotic fluid volume. A normal response for each variable
receives 2 points; an abnormal response receives 0 points. A score between 8
and 10 is considered normal, indicating that the fetus has a low risk of
oxygen deprivation and isn’t in distress. A fetus with a score of 6 or lower is
at risk for asphyxia and premature birth; this score warrants detailed
investigation. The BPP may or may not be repeated if the score isn’t within
normal limits.

40
C.BY: HOSSAM HAMDY MATERNITY RNPEDIA – ‫موسوعه التمريض‬

Option A: The biophysical profile is a test used to evaluate the well-being of


the fetus. It is commonly done at the last trimester of pregnancy, but it does
not indicate that the fetus should be delivered within 24 hours.

Option B: If the score is 6, the health care provider will likely repeat the test
within 24 hours.

Option D: The test is most commonly done when there’s an increased risk
of problems that could lead to complications or pregnancy loss. The health
care provider will determine the necessity and timing of a biophysical profile
based on whether the baby could survive if delivered early, the severity of
the mother’s condition, and the risk of pregnancy loss.
45. Question

A client who is 36 weeks pregnant comes to the clinic for a prenatal


checkup. To assess the client’s preparation for parenting, the nurse might ask
which question?

A. “Are you planning to have epidural anesthesia?”

B. “Have you begun prenatal classes?”

C. “What changes have you made at home to get ready for the baby?”

D. “Can you tell me about the meals you typically eat each day?”

Answer: C. “What changes have you made at home to get ready for the
baby?”

During the third trimester, the pregnant client typically perceives the fetus as
a separate being. To verify that this has occurred, the nurse should ask
whether she has made appropriate changes at home such as obtaining infant
supplies and equipment.

Option A: The type of anesthesia planned doesn’t reflect the client’s


preparation for parenting.

Option B: The client should have begun prenatal classes earlier in the
pregnancy.

41
C.BY: HOSSAM HAMDY MATERNITY RNPEDIA – ‫موسوعه التمريض‬

Option D: The nurse should have obtained dietary information during the
first trimester to give the client time to make any necessary changes.
46. Question

A client who’s admitted to labor and delivery has the following assessment
findings: gravida 2 para 1, estimated 40 weeks gestation, contractions 2
minutes apart, lasting 45 seconds, vertex +4 station. Which of the following
would be the priority at this time?

A. Placing the client in bed to begin fetal monitoring.

B. Preparing for immediate delivery.

C. Checking for ruptured membranes.

D. Providing comfort measures.

Answer: B. Preparing for immediate delivery.

This question requires an understanding of station as part of the intrapartum


assessment process. Based on the client’s assessment findings, this client is
ready for delivery, which is the nurse’s top priority.

Option A: Fetal heart rate monitoring may help detect changes in the normal
heart rate pattern during labor. If certain changes are detected, steps can be
taken to help treat the underlying problem. Fetal heart rate monitoring also
can help prevent treatments that are not needed.

Option C: The membranes can break by themselves. This is called a


spontaneous rupture of the membranes. It most often happens after active
labor has started.

Option D: Comfort measures may be given to the woman after ensuring all
necessary measures to help her deliver successfully.
47. Question

42
C.BY: HOSSAM HAMDY MATERNITY RNPEDIA – ‫موسوعه التمريض‬

The nurse is caring for a client in labor. The external fetal monitor shows a
pattern of variable decelerations in fetal heart rate. What should the nurse do
first?

A. Change the client’s position.

B. Prepare for an emergency cesarean section.

C. Check for placenta previa.

D. Administer oxygen.

Answer: A. Change the client’s position.

Variable decelerations in fetal heart rate are an ominous sign, indicating


compression of the umbilical cord. Changing the client’s position from
supine to side-lying may immediately the problem.

Option B: An emergency cesarean section is necessary only if other


measures, such as changing position and amnioinfusion with sterile saline,
prove unsuccessful.

Option C: Common causes of variable decelerations include vagal reflex


triggered by head compression during pushing and cord compression such as
that caused by short cord, nuchal cord, body entanglement, prolapsed cord,
decreased amniotic fluid, and fetal descent. It does not include placenta
previa.

Option D: Administering oxygen may be helpful, but the priority is to


change the woman’s position and relieve cord compression. The fetus
already has a surplus of oxygen due to high basal blood flow to its organs
and vascular shunts.
48. Question

The nurse in charge is caring for a postpartum client who had a vaginal
delivery with a midline episiotomy. Which nursing diagnosis takes priority
for this client?

A. Risk for deficient fluid volume related to hemorrhage.

B. Risk for infection related to the type of delivery.


43
C.BY: HOSSAM HAMDY MATERNITY RNPEDIA – ‫موسوعه التمريض‬

C. Pain related to the type of incision.

D. Urinary retention related to periurethral edema.

Answer: A. Risk for deficient fluid volume related to hemorrhage

Hemorrhage jeopardizes the client’s oxygen supply — the first priority


among human physiological needs. Therefore, the nursing diagnosis of Risk
for deficient fluid volume related to hemorrhage takes priority over-
diagnoses of Risk for Infection, Pain, and Urinary retention.

Option B: Episiotomy infections are classically reported as being rare at a


rate of 0.1% and increasing up to 2% if a third or fourth-degree tear occurs.

Option C: Episiotomy pain may be relieved by an ice pack, warm or cold


shallow baths or sitz baths, or medicated creams or local numbing sprays.

Option D: Postpartum urinary retention (PPUR) is an upsetting condition


that has no standard literature definition. It has been variably defined as the
abrupt onset of aching or inability to completely micturate, requiring urinary
catheterization, over 12 hours after giving birth or not to void spontaneously
within 6 hours of vaginal delivery.
49. Question

Which change would the nurse identify as a progressive physiological


change in the postpartum period?

A. Lactation

B. Lochia

C. Uterine involution

D. Diuresis

Answer: A. Lactation

Lactation is an example of a progressive physiological change that occurs


during the postpartum period. Lactation is the process of milk production.
44
C.BY: HOSSAM HAMDY MATERNITY RNPEDIA – ‫موسوعه التمريض‬

Option B: Lochia is the vaginal discharge a woman has after a vaginal


delivery. It has a stale, musty odor like menstrual discharge. Lochia for the
first 3 days after delivery is dark red in color. For the fourth through tenth
day after delivery, the lochia will be more watery and pinkish to brownish
color. From about the seventh to tenth day through the fourteenth day after
delivery, the lochia is creamy or yellowish in color.

Option C: Involution is the process by which the uterus is transformed from


pregnant to non-pregnant state. This period is characterized by the
restoration of ovarian function in order to prepare the body for a new
pregnancy.

Option D: Diuresis is increased urination and the physiologic process that


produces such an increase. It involves extra urine production in the kidneys
as part of the body’s homeostatic maintenance of fluid balance.
50. Question

A 39-year-old at 37 weeks gestation is admitted to the hospital with


complaints of vaginal bleeding following the use of cocaine 1 hour earlier.
Which complication is most likely causing the client’s complaint of vaginal
bleeding?

A. Placenta previa

B. Abruptio placentae

C. Ectopic pregnancy

D. Spontaneous abortion

Answer: B. Abruptio placentae

The major maternal adverse reactions from cocaine use in pregnancy include
spontaneous abortion first, not third, trimester abortion and abruptio
placentae. The hypertension and increased levels of catecholamines caused
by cocaine abuse are thought to be responsible for a vasospasm in the uterine
blood vessels that causes placental separation and abruption.

Option A: A pregnant woman who uses cocaine experiences a constriction


of the blood vessels throughout her body. A fetus needs this blood flow for
45
C.BY: HOSSAM HAMDY MATERNITY RNPEDIA – ‫موسوعه التمريض‬

its oxygen supply. After cocaine abuse, the heart rate of the fetus goes up
along with the blood pressure, but it may suffer a lack of oxygen (hypoxia).
This restricted blood supply can also permanently damage sections of the
placenta which can result in loss of the baby.

Option C: Ectopic pregnancy (EP) is defined as the implantation and


development of a fertilized ovum anywhere outside of the uterine cavity.
Such a pregnancy may lead to tubal rupture and intra abdominal hemorrhage
and represents the major cause of maternal death in the first trimester.

Option D: Cocaine use early in pregnancy decreases uterine and placental


blood flow by inhibiting the reuptake of norepinephrine, which causes
arterial vasoconstriction. In most, but not all, previous studies of cocaine use
during pregnancy and spontaneous abortion, the women’s current use of
cocaine, as assessed by self-reports or urine analysis, was related to their
history of spontaneous abortion.

51. Question

A client with type 1 diabetes mellitus who is a multigravida visits the clinic
at 27 weeks gestation. The nurse should instruct the client that for most
pregnant women with type 1 diabetes mellitus:

A. Weekly fetal movement counts are made by the mother.

B. Contraction stress testing is performed weekly.

C. Induction of labor begins at 34 weeks’ gestation.

D. Nonstress testing is performed weekly until 32 weeks’ gestation.

Answer: D. Nonstress testing is performed weekly until 32 weeks’


gestation

For most clients with type 1 diabetes mellitus, non-stress testing is done
weekly until 32 weeks’ gestation and twice a week to assess fetal well-being.

46
C.BY: HOSSAM HAMDY MATERNITY RNPEDIA – ‫موسوعه التمريض‬

Option A: Increased fetal activity may minimize the impact of


hyperglycemia on subsequent birth weight. The inactive fetus appears to be
at a higher risk for glucose-mediated macrosomia.

Option B: Contraction stress test may be done weekly with reassuring


results of no heart rate deceleration in response to 3 contractions in 10
minutes.

Option C: Nonstress test may be done twice a week with reassuring results
of 2 heart rate acceleration in 20 minutes.
52. Question

When administering magnesium sulfate to a client with preeclampsia, the


nurse understands that this drug is given to:

A. Prevent seizures.

B. Reduce blood pressure.

C. Slow the process of labor.

D. Increase diuresis.

Answer: A. Prevent seizures

The chemical makeup of magnesium is similar to that of calcium and,


therefore, magnesium will act like calcium in the body. As a result,
magnesium will block seizure activity in a hyper-stimulated neurologic
system by interfering with signal transmission at the neuromuscular junction.

Option B: Magnesium sulfate may attenuate blood pressure by decreasing


the vascular response to pressor substances.

Option C: Since the primary therapeutic goal of tocolysis is to delay preterm


delivery within 48 hours from the initiation of steroid prophylaxis, little
evidence suggests that extended MgSO4 therapy is beneficial.

Option D: There are rare cases of pregnant women who develop polyuria
after receiving intravenous therapy of magnesium sulfate. It can be
considered as another cause of solute diuresis.

47
C.BY: HOSSAM HAMDY MATERNITY RNPEDIA – ‫موسوعه التمريض‬

53. Question

What is the approximate time that the blastocyst spends traveling to the
uterus for implantation?

A. 2 days

B. 7 days

C. 10 days

D. 14 weeks

Answer: B. 7 days

The blastocyst takes approximately 1 week to travel to the uterus for


implantation. Implantation is a process in which a developing embryo,
moving as a blastocyst through a uterus, makes contact with the uterine wall
and remains attached to it until birth.

Option A: The zygote moves through the fallopian tube and undergoes cell
division, a process called cleavage. These cell divisions produce the inner
cell mass (ICM), which will become the embryo, and the trophoblast, which
surrounds the ICM and interacts with maternal tissues. Together, the ICM
and the trophoblast are called the blastocyst.

Option C: A blastocyst successfully implants in the uterus when, as the zona


pellucida exits the fallopian tube, the blastocyst leaves the zona pellucida
and binds to the endometrium.

Option D: 14 weeks is too long a time to wait for implantation. If the


blastocyst does not implant within 7 days, the pregnancy may not occur at
all.
54. Question

After teaching a pregnant woman who is in labor about the purpose of the
episiotomy, which of the following purposes stated by the client would
indicate to the nurse that the teaching was effective?

48
C.BY: HOSSAM HAMDY MATERNITY RNPEDIA – ‫موسوعه التمريض‬

A. Shortens the second stage of labor.

B. Enlarges the pelvic inlet.

C. Prevents perineal edema.

D. Ensures quick placenta delivery.

Answer: A. Shortens the second stage of labor

An episiotomy serves several purposes. It shortens the second stage of labor,


substitutes a clean surgical incision for a tear, and decreases undue stretching
of perineal muscles. An episiotomy helps prevent tearing of the rectum but it
does not necessarily relieve pressure on the rectum. Tearing may still occur.

Option B: The pelvic inlet or superior aperture of the pelvis is a planar


surface that defines the boundary between the pelvic cavity and the
abdominal cavity. It is not involved during an episiotomy.

Option C: To prevent perineal edema, ice packs may be applied in the first
24 hours after birth to decrease swelling and pain.

Option D: Placenta delivery may be sped up by either pulling the cord


gently with one hand while pressing and kneading the uterus with the other,
or exerting downward pressure on the top of the uterus, asking the woman to
push at the appropriate time.
55. Question

A primigravida client at about 35 weeks gestation in active labor has had no


prenatal care and admitted to cocaine use during the pregnancy. Which of
the following persons must the nurse notify?

A. Nursing unit manager so appropriate agencies can be notified.

B. Head of the hospital’s security department.

C. Chaplain in case the fetus dies in utero.

D. Physician who will attend the delivery of the infant.

49
C.BY: HOSSAM HAMDY MATERNITY RNPEDIA – ‫موسوعه التمريض‬

Answer: D. Physician who will attend the delivery of the infant.

The fetus of a cocaine-addicted mother is at risk for hypoxia, meconium


aspiration, and intrauterine growth retardation (IUGR). Therefore, the nurse
must notify the physician of the client’s cocaine use because this knowledge
will influence the care of the client and neonate. The information is used
only in relation to the client’s care.

Option A: Informing the nursing unit manager would be inappropriate since


the physician would be the one who will have the knowledge on how to
manage the fetus.

Option B: The knowledge should only be used in relation to the client’s


care. Notifying the head of the security department is unnecessary and would
be against the data privacy act.

Option C: Informing the physician first of the cocaine use would most likely
save the fetus’ life in utero.
56. Question

When preparing a teaching plan for a client who is to receive a rubella


vaccine during the postpartum period, the nurse in charge should include
which of the following?

A. The vaccine prevents a future fetus from developing congenital


anomalies.

B. Pregnancy should be avoided for 3 months after the immunization.

C. The client should avoid contact with children diagnosed with rubella.

D. The injection will provide immunity against the 7-day measles.

Answer: B. Pregnancy should be avoided for 3 months after the


immunization

After administration of rubella vaccine, the client should be instructed to


avoid pregnancy for at least 3 months to prevent the possibility of the
vaccine’s toxic effects to the fetus.

50
C.BY: HOSSAM HAMDY MATERNITY RNPEDIA – ‫موسوعه التمريض‬

Option A: The role of the vaccine postpartum is to protect the mother


against rubella in the future pregnancies as well as measles and mumps,
since it is given together.

Option C: Protection against measles, mumps, and rubella starts to develop


around 2 weeks after having the MMR vaccine.

Option D: One dose of MMR vaccine is 93% effective against measles,


78% effective against mumps, and 97% effective against rubella.
57. Question

A client with eclampsia begins to experience a seizure. Which of the


following would the nurse in charge do first?

A. Pad the side rails.

B. Place a pillow under the left buttock.

C. Insert a padded tongue blade into the mouth.

D. Maintain a patent airway.

Answer: D. Maintain a patent airway

The priority for the pregnant client having a seizure is to maintain a patent
airway to ensure adequate oxygenation to the mother and the fetus.
Additionally, oxygen may be administered by face mask to prevent fetal
hypoxia.

Option A: Padding the side rails should be done as a precaution before a


seizure, not during the seizure.

Option B: The client should be placed on a flat, firm surface to avoid any
injuries.

Option C: There should be nothing inserted inside the client’s mouth to


maintain airway patency and prevent obstruction and aspiration.
58. Question

51
C.BY: HOSSAM HAMDY MATERNITY RNPEDIA – ‫موسوعه التمريض‬

While caring for a multigravida client in early labor in a birthing center,


which of the following foods would be best if the client requests a snack?

A. Yogurt

B. Cereal with milk

C. Vegetable soup

D. Peanut butter cookies

Answer: A. Yogurt

In some birth settings, intravenous therapy is not used with low-risk clients.
Thus, clients in early labor are encouraged to eat healthy snacks and drink
fluid to avoid dehydration. Yogurt, which is an excellent source of calcium
and riboflavin, is soft and easily digested. During pregnancy, gastric
emptying time is delayed. In most hospital settings, clients are allowed only
ice chips or clear liquids.

Option B: Most institutions would only allow clear liquids for clients in
early labor. This prevents gastrointestinal problems during labor and
delivery.

Option C: Vegetables may cause gastric discomfort for the woman during
labor.

Option D: Fluids are mostly recommended during this stage of labor,


instead of solids, to avoid dehydration.
59. Question

The multigravida mother with a history of rapid labor who is in active labor
calls out to the nurse, “The baby is coming!” Which of the following would
be the nurse’s first action?

A. Inspect the perineum.

B. Time the contractions.

C. Auscultate the fetal heart rate.

52
C.BY: HOSSAM HAMDY MATERNITY RNPEDIA – ‫موسوعه التمريض‬

D. Contact the birth attendant.

Answer: A. Inspect the perineum

When the client says the baby is coming, the nurse should first inspect the
perineum and observe for crowning to validate the client’s statement. If the
client is not delivering precipitously, the nurse can calm her and use
appropriate breathing techniques.

Option B: The woman has a history of rapid labor and is already


experiencing true labor contractions. There is no need to time the
contractions experienced.

Option C: Fetal heart rate monitoring is being consistently monitored during


labor. The client’s concerns about the delivery of the baby must be
prioritized.

Option D: Before contacting a birth attendant or the physician, validate the


client’s claims first. If she is not yet delivering, instruct about breathing
techniques that may ease her discomfort.
60. Question

While assessing a primipara during the immediate postpartum period, the


nurse in charge plans to use both hands to assess the client’s fundus to:

A. Prevent uterine inversion.

B. Promote uterine involution.

C. Hasten the puerperium period.

D. Determine the size of the fundus.

Answer: A. Prevent uterine inversion

Using both hands to assess the fundus is useful for preventing uterine
inversion. The recent uterine inversion with placenta already separated from
it may often be replaced by manually pushing up on the fundus with the
palm and fingers in the direction of the long axis of the vagina.
53
C.BY: HOSSAM HAMDY MATERNITY RNPEDIA – ‫موسوعه التمريض‬

Option B: Involution is the process by which the uterus is transformed from


pregnant to non-pregnant state. This period is characterized by the
restoration of ovarian function in order to prepare the body for a new
pregnancy.

Option C: The puerperium, or postpartum period, generally lasts 6 weeks


and is the period of adjustment after delivery when the anatomic and
physiologic changes of pregnancy are reversed, and the body returns to the
normal, nonpregnant state.

Option D: A fundal height measurement is typically done to determine if a


baby is small for its gestational age. The measurement is generally defined
as the distance in centimeters from the pubic bone to the top of the uterus.
The expectation is that after week 24 of pregnancy the fundal height for a
normally growing baby will match the number of weeks of pregnancy —
plus or minus 2 centimeters.
61. Question

Which behaviors would be exhibited during the letting-go phase of maternal


role adaptation. Select all that apply.

A. Emergence of the family unit.

B. Dependent behaviors.

C. Sexual intimacy continues.

D. Defining one's individual roles.

E. Being talkative and excited about becoming a mother.

Answer: Answer: A, C, & D

The emergence of family unit, sexual intimacy relationship continuing and


defining one’s individual roles represent interdependent behaviors associated
with the letting-go phase. During the letting go phase, the woman finally
accepts her new role and gives up her old roles like being a childless woman
or just a mother of one child.

54
C.BY: HOSSAM HAMDY MATERNITY RNPEDIA – ‫موسوعه التمريض‬

Option B: Dependent behaviors are exhibited in the taking-in phase. The


taking-in phase is the time of reflection for the woman because within the 2
to 3 day period, the woman is passive.

Option E: Being talkative and excited about becoming a mother represents


the taking-hold phase and is an example of dependent-independent
behaviors. The woman starts to initiate actions on her own and makes
decisions without relying on others.
62. Question

While making a visit to the home of a postpartum woman 1 week after birth,
the nurse should recognize that the woman would characteristically:

A. Express a strong need to review the events and her behavior during the
process of labor and birth.

B. Exhibit a reduced attention span, limiting readiness to learn.

C. Vacillate between the desire to have her own nurturing needs met and the
need to take charge of her own care and that of her newborn.

D. Have reestablished her role as a spouse or partner.

Answer: C. Vacillate between the desire to have her own nurturing


needs met and the need to take charge of her own care and that of her
newborn.

One week after birth the woman should exhibit behaviors characteristic of
the dependent-independent or taking-hold stage. She still has needs for
nurturing and acceptance by others.

Option A: Wanting to discuss the events of her labor and delivery are
characteristics of the taking-in stage; this stage lasts from the first 24 hours
until 2 days after delivery.

Option B: A reduced attention span and limiting readiness to learn is also


characteristic of the taking-in stage. This dependence is mainly due to her
physical discomfort from hemorrhoids or the after pains, from the
uncertainty of how she could care for the newborn, and also from the
extreme tiredness she feels that follows childbirth.
55
C.BY: HOSSAM HAMDY MATERNITY RNPEDIA – ‫موسوعه التمريض‬

Option D: Having reestablished her role as a spouse reflects the letting-go


stage, which indicates that psychosocial recovery is complete.
63. Question

Which of the following is the most common kind of placental adherence


seen in pregnant women?

A. Accreta

B. Placenta previa

C. Percreta

D. Increta

Answer: A. Accreta

Placenta accreta is the most common kind of placental adherence seen in


pregnant women and is characterized by slight penetration of myometrium.

Option B: In placenta previa, the placenta does not embed ly and results in
what is known as a low-lying placenta. It can be marginal, partial, or
complete in how it covers the cervical os, and it increases the patient’s risk
for painless vaginal bleeding during the pregnancy and/or delivery process.

Option C: Placenta percreta leads to perforation of the uterus and is the


most serious and invasive of all types of accrete.

Option D: Placenta increta leads to deep penetration of the myometrium.


64. Question

A 40-year-old woman with a high body mass index (BMI) is 10 weeks


pregnant. Which diagnostic tool is appropriate to suggest to her at this time?

A. Biophysical profile

B. Amniocentesis

56
C.BY: HOSSAM HAMDY MATERNITY RNPEDIA – ‫موسوعه التمريض‬

C. Maternal serum alpha-fetoprotein (MSAFP)

D. Transvaginal ultrasound

Answer: D. Transvaginal ultrasound

An ultrasound is the method of biophysical assessment of the infant that is


performed at this gestational age. Transvaginal ultrasound is especially
useful for obese women, whose thick abdominal layers cannot be penetrated
adequately with the abdominal approach.

Option A: A biophysical profile is a method of biophysical assessment of


fetal well-being in the third trimester.

Option B: An amniocentesis is performed after the fourteenth week of


pregnancy.

Option C: A MSAFP test is performed from week 15 to week 22 of the


gestation (weeks 16 to 18 are ideal).
65. Question

A nurse providing care for the antepartum woman should understand that the
contraction stress test (CST):

A. Sometimes uses vibroacoustic stimulation.

B. Is an invasive test; however, contractions are stimulated.

C. Is considered to have a negative result if no late decelerations are


observed with the contractions.

D. Is more effective than nonstress test (NST) if the membranes have already
been ruptured.

Answer: C. Is considered to have a negative result if no late


decelerations are observed with the contractions.

No late decelerations indicate a positive CST result.

57
C.BY: HOSSAM HAMDY MATERNITY RNPEDIA – ‫موسوعه التمريض‬

Option A: Vibroacoustic stimulation is sometimes used with NST.


Vibroacoustic stimulation (VAS) of the fetus has been used as both a
primary and adjunctive method of FHR testing. This device produces a
broadband acoustic signal and a complex vibratory component.

Option B: CST is invasive if stimulation is performed by IV oxytocin but


not if by nipple stimulation.

Option D: CST is contraindicated if the membranes have ruptured.


66. Question

In the past, factors to determine whether a woman was likely to have a high-
risk pregnancy were evaluated primarily from a medical point of view. A
broader, more comprehensive approach to high-risk pregnancy has been
adopted. There are now four categories based on threats to the health of the
woman and the outcome of pregnancy. Which of the options listed here is
not included as a category?

A. Biophysical

B. Psychosocial

C. Geographic

D. Environmental

Answer: C. Geographic

The fourth category is ly referred to as the sociodemographic risk category.


Several risk factors for high-risk pregnancy were present before pregnancy,
including multiple pregnancies, maternal age under 16 or over 35 years, and
interval between pregnancies less than one year.

Option A: A fetal biophysical profile is a prenatal test used to check on a


baby’s well-being. The test combines fetal heart rate monitoring (nonstress
test) and fetal ultrasound to evaluate a baby’s heart rate, breathing,
movements, muscle tone and amniotic fluid level.

Option B: A pregnancy may be determined to be at high risk because of


obstetric factors in previous pregnancies or the present one; conditions that
are, themselves, psychosocial: anxiety disorders (GAD, OCD, panic
58
C.BY: HOSSAM HAMDY MATERNITY RNPEDIA – ‫موسوعه التمريض‬

disorder, PTSD), mood disorders, and schizophrenia, all of which are a


background for a disturbed pregnancy and might complicate a pregnancy
denominated high risk for some other reason.

Option D: Environmental factors that have been implicated in adverse


pregnancy outcomes include smoking, video display terminals, anesthetic
gases, antineoplastic drugs and exposure to lead, selenium and inorganic
mercury.
67. Question

A woman who is at 36 weeks of gestation is having a nonstress test. Which


statement indicates her understanding of the test?

A. "I will need to have a full bladder for the test to be done accurately."

B. "I should have my husband drive me home after the test because I may be
nauseated."

C. "This test will help to determine whether the baby has Down syndrome or
a neural tube defect."

D. "This test observes for fetal activity and an acceleration of the fetal heart
rate to determine the well-being of the baby."

Answer: D. “This test observes for fetal activity and an acceleration of


the fetal heart rate to determine the well-being of the baby.”

The nonstress test is one of the most widely used techniques to determine
fetal well-being and is accomplished by monitoring fetal heart rate in
conjunction with fetal activity and movements.

Option A: An ultrasound requires a full bladder. A full bladder creates a


reservoir fluid that enhances the movement of sound waves through the
abdominal cavity. This creates a clearer view of the structures that need to be
observed.

Option B: An amniocentesis is a test after which a pregnant woman should


be driven home.

59
C.BY: HOSSAM HAMDY MATERNITY RNPEDIA – ‫موسوعه التمريض‬

Option C: A maternal serum alpha-fetoprotein test is used in conjunction


with unconjugated estriol levels and human chorionic gonadotropin helps to
detect Down syndrome.
68. Question

What is an appropriate indicator for performing a contraction stress test?

A. Increased fetal movement and small for gestational age.

B. Maternal diabetes mellitus and postmaturity.

C. Adolescent pregnancy and poor prenatal care.

D. History of preterm labor and intrauterine growth restriction.

Answer: B. Maternal diabetes mellitus and postmaturity.

The contraction stress test helps predict how the baby will do during labor.
The test triggers contractions and registers how the baby’s heart reacts. A
normal heartbeat is a good sign that the baby will be healthy during labor.

Option A: Decreased fetal movement is an indicator for performing a


contraction stress test; the size (small for gestational age) is not an indicator.

Option C: Although adolescent pregnancy and poor prenatal care are risk
factors for poor fetal outcomes, they are not indicators for performing a
contraction stress test.

Option D: Intrauterine growth restriction is an indicator; history of a


previous stillbirth, not preterm labor, is another indicator.
69. Question

The nurse sees a woman for the first time when she is 30 weeks pregnant.
The woman has smoked throughout the pregnancy, and fundal height
measurements now are suggestive of growth restriction in the fetus. In
addition to ultrasound to measure fetal size, what would be another tool
useful in confirming the diagnosis?

60
C.BY: HOSSAM HAMDY MATERNITY RNPEDIA – ‫موسوعه التمريض‬

A. Doppler blood flow analysis

B. Contraction stress test (CST)

C. Amniocentesis

D. Daily fetal movement counts

Answer: A. Doppler blood flow analysis

Doppler blood flow analysis allows the examiner to study the blood flow
noninvasively in the fetus and the placenta. It is a helpful tool in the
management of high-risk pregnancy due to intrauterine growth restriction
(IUGR), diabetes mellitus, multiple fetuses, or preterm labor.

Option B: Because of the potential risk of inducing labor and causing fetal
distress, a CST is not performed on a woman whose fetus is preterm.

Option C: Indications for an amniocentesis include diagnosis of genetic


disorders or congenital anomalies, assessment of the pulmonary maturity,
and the diagnosis of fetal hemolytic disease, not IUGR.

Option D: Fetal kick count monitoring is performed to monitor the fetus in


pregnancies complicated by conditions that may affect fetal oxygenation.
Although it may be a useful tool at some point later in this woman’s
pregnancy, it is not used to diagnose IUGR.
70. Question

A nurse is providing instruction for an obstetrical patient to perform a daily


fetal movement count (DFMC). Which instructions could be included in the
plan of care? Select all that apply.

A. The fetal alarm signal is reached when there are no fetal movements
noted for 5 hours.

B. The patient can monitor fetal activity once daily for a 60-minute period
and note activity.

C. Monitor fetal activity two times a day either after meals or before bed for
a period of 2 hours or until 10 fetal movements are noted.

61
C.BY: HOSSAM HAMDY MATERNITY RNPEDIA – ‫موسوعه التمريض‬

D. Count all fetal movements in a 12-hour period daily until 10 fetal


movements are noted.

Answer: B, C, & D

The fetal alarm signal is reached when no fetal movements are noted for a
period of 12 hours. Fetal movement is one show of a baby’s health in the
womb. Each woman should learn the normal pattern and number of
movements for her own baby. A change in the normal pattern or number of
fetal movements may mean the baby is under stress. And it’s not normal for
a baby to stop moving with the start of labor.

Option A: In general, the woman should feel 10 movements in 2 hours. Sit


or lie on the side in a comfortable spot during a time of day when the baby is
usually active. This may be after eating or moving around. If the woman lies
down, lie on the left side, since the baby will have better circulation.
71. Question

A patient has undergone an amniocentesis for evaluation of fetal well-being.


Which intervention would be included in the nurse’s plan of care after the
procedure? Select all that apply.

A. Perform ultrasound to determine fetal positioning.

B. Observe the patient for possible uterine contractions.

C. Administer RhoGAM to the patient if she is Rh-negative.

D. Perform a mini catheterization to obtain a urine specimen to assess for


bleeding.

Answer: B & C

Ultrasound is used prior to the procedure as a visualization aid to assist with


insertion of the transabdominal needle. RhoGAM is a prescription medicine
that is used to prevent Rh immunization, a condition in which an individual
with Rh-negative blood develops antibodies after exposure to Rh-positive
blood. RhoGAM is administered by intramuscular (IM) injection. RhoGAM
is purified from human plasma containing anti-Rh (anti-D).
62
C.BY: HOSSAM HAMDY MATERNITY RNPEDIA – ‫موسوعه التمريض‬

Option A: The position of the baby in the uterus is called the presentation of
the fetus. Ideally for labor, the baby is positioned head-down, facing the
mother’s back with the chin tucked to its chest and the back of the head
ready to enter the pelvis. This position is called cephalic presentation.

Option D: There is no need to assess the urine for bleeding as this is not
considered to be a typical presentation or complication.
72. Question

With regard to small-for-gestational-age (SGA) infants and intrauterine


growth restriction (IUGR), nurses should be aware that:

A. In the first trimester, diseases or abnormalities result in asymmetric


IUGR.

B. Infants with asymmetric IUGR have the potential for normal growth and
development.

C. In asymmetric IUGR, weight is slightly more than SGA, whereas length


and head circumference are somewhat less than SGA.

D. Symmetric IUGR occurs in the later stages of pregnancy.

Answer: B. Infants with asymmetric IUGR have the potential for


normal growth and development.

The infant with asymmetric IUGR has the potential for normal growth and
development. SGA infants have reduced brain capacity. The asymmetric
form occurs in the later stages of pregnancy.

Option A: IUGR is either symmetric or asymmetric. The symmetric form


occurs in the first trimester, as a result of disease or abnormalities.

Option C: Weight is less than the 10th percentile, but the head
circumference is greater than the 10th percentile (within normal limits).

Option D: IUGR is either symmetric or asymmetric. The symmetric form


occurs in the first trimester, as a result of disease or abnormalities.
73. Question

63
C.BY: HOSSAM HAMDY MATERNITY RNPEDIA – ‫موسوعه التمريض‬

A client who delivered by cesarean section 24 hours ago is using a patient-


controlled analgesia (PCA) pump for pain control. Her oral intake has been
ice chips only since surgery. She is now complaining of nausea and bloating,
and states that because she had nothing to eat, she is too weak to breastfeed
her infant. Which nursing diagnosis has the highest priority?

A. Altered nutrition, less than body requirements for lactation.

B. Alteration in comfort related to nausea and abdominal distention.

C. Impaired bowel motility related to pain medication and immobility.

D. Fatigue related to cesarean delivery and physical care demands of infant.

Answer: C. Impaired bowel motility related to pain medication and


immobility

Impaired bowel motility caused by surgical anesthesia, pain medication, and


immobility is the priority nursing diagnosis and addresses the potential
problem of a paralytic ileus.

Option A: Altered nutrition is also an appropriate diagnosis since the


woman was not able to eat adequately since the surgery, hindering her ability
to breastfeed. However, it can be managed and is not the priority at the time.

Option B: The woman’s comfort is also altered due to nausea and bloating,
but it is not considered a priority.

Option D: After cesarean delivery, fatigue may overcome the client’s desire
to eat and breastfeed her infant. This is a diagnosis but it does not take
priority over impaired bowel motility.
74. Question

The nurse is teaching care of the newborn to a childbirth preparation class


and describes the need for administering antibiotic ointment into the eyes of
the newborn. An expectant father asks, “What type of disease causes
infections in babies that can be prevented by using this ointment?” Which
response by the nurse is accurate?

64
C.BY: HOSSAM HAMDY MATERNITY RNPEDIA – ‫موسوعه التمريض‬

A. Herpes

B. Trichomonas

C. Gonorrhea

D. Syphilis

Answer: C. Gonorrhea

Erythromycin ointment is instilled into the lower conjunctiva of each eye


within 2 hours after birth to prevent ophthalmia neonatorum, an infection
caused by gonorrhea (C), and inclusion conjunctivitis, an infection caused by
Chlamydia. The infant may be exposed to these bacteria when passing
through the birth canal.

Option A: Antibiotics do not work for viruses like herpes. Taking antibiotics
when they are not really needed increases the risk of getting infection later
that resists antibiotic treatment.

Option B: Trichomoniasis can be cured with a single dose of prescription


antibiotic medication (either metronidazole or tinidazole), pills that can be
taken by mouth.

Option D: A single intramuscular injection of long-acting Benzathine


penicillin G (2.4 million units administered intramuscularly) will cure a
person who has primary, secondary, or early latent syphilis. Three doses of
long-acting Benzathine penicillin G (2.4 million units administered
intramuscularly) at weekly intervals is recommended for individuals with
late latent syphilis or latent syphilis of unknown duration. Treatment will kill
the syphilis bacterium and prevent further damage, but it will not repair
damage already done.
75. Question

A new mother is having trouble breastfeeding her newborn. The child is


making frantic rooting motions and will not grasp the nipple. Which
intervention should the nurse implement?

A. Encourage frequent use of a pacifier so that the infant becomes


accustomed to sucking.
65
C.BY: HOSSAM HAMDY MATERNITY RNPEDIA – ‫موسوعه التمريض‬

B. Hold the infant's head firmly against the breast until he latches onto the
nipple.

C. Encourage the mother to stop feeding for a few minutes and comfort the
infant.

D. Provide a formula for the infant until he becomes calm, and then offer the
breast again.

Answer: C. Encourage the mother to stop feeding for a few minutes and
comfort the infant.

The infant is becoming frustrated and so is the mother; both need a time out.
The mother should be encouraged to comfort the infant and to relax herself.
After such a time out, breastfeeding is often more successful.

Option A: Concern about pacifiers for breastfeeding infants focuses on


“nipple confusion”—that is, that pacifiers (and supplemental bottles) do not
facilitate effective breast sucking and may contribute to latch. Findings
from earlier observational studies suggest that early exposure to pacifiers
leads to cessation of exclusive breastfeeding by 3 to 6 months and an end to
all breastfeeding by 12 months.

Option B: Option B would only cause the infant to be more resistant,


resulting in the mother and infant to become more frustrated.

Option D: When using an average baby bottle, babies don’t have to work as
hard because gravity and the nipple cause the milk flow to be more
continuous for the baby. Nipple confusion occurs when the baby switches
back to breast, and doesn’t understand why the milk flows differently than it
did with the bottle.

66
C.BY: HOSSAM HAMDY MATERNITY RNPEDIA – ‫موسوعه التمريض‬

Maternity Nursing NU.2 (Quiz #1: 75 Questions)

1. Question

1 point(s)

The nurse is counseling a couple who has sought information about


conceiving. The couple asks the nurse to explain when ovulation usually
occurs. Which statement by the nurse is ?

A. Two weeks before menstruation.

B. Immediately after menstruation.

C. Immediately before menstruation.

D. Three weeks before menstruation.

Answer: A. Two weeks before menstruation

Ovulation occurs 14 days before the first day of the menstrual period (A).
Although ovulation can occur in the middle of the cycle or 2 weeks after
menstruation, this is only true for a woman who has a perfect 28-day cycle.
For many women, the length of the menstrual cycle varies.

Option B: After the follicle releases its egg, it changes into the corpus
luteum. This structure releases hormones, mainly progesterone and some
estrogen. The rise in hormones keeps the uterine lining thick and ready for a
fertilized egg to implant. If the woman does get pregnant, her body will
produce human chorionic gonadotropin (hCG). This is the hormone
pregnancy tests detect. It helps maintain the corpus luteum and keeps the
uterine lining thick. If the woman doesn’t get pregnant, the corpus luteum
will shrink away and be resorbed. This leads to decreased levels of estrogen
and progesterone, which causes the onset of the period. The uterine lining
will shed during this period.

Option C: The menstrual phase is the first stage of the menstrual cycle. It’s
also when the woman gets her period. This phase starts when an egg from
the previous cycle isn’t fertilized. Because pregnancy hasn’t taken place,
levels of the hormones estrogen and progesterone drop. The thickened lining
of the uterus, which would support a pregnancy, is no longer needed, so it
67
C.BY: HOSSAM HAMDY MATERNITY RNPEDIA – ‫موسوعه التمريض‬

sheds through the vagina. During this period, there is a release of a


combination of blood, mucus, and tissue from the uterus.

Option D: During each menstrual cycle, an egg develops and is released


from the ovaries. The lining of the uterus builds up. If a pregnancy doesn’t
happen, the uterine lining sheds during a menstrual period. Then the cycle
starts again.
2. Question

1 point(s)

The nurse instructs a laboring client to use accelerated blow breathing. The
client begins to complain of tingling fingers and dizziness. Which action
should the nurse take?

A. Administer oxygen by face mask.

B. Notify the health care provider of the client's symptoms.

C. Have the client breathe into her cupped hands.

D. Check the client's blood pressure and fetal heart rate.

Answer: C. Have the client breathe into her cupped hands.

Tingling fingers and dizziness are signs of hyperventilation (blowing off too
much carbon dioxide). Hyperventilation is treated by retaining carbon
dioxide. This can be facilitated by breathing into a paper bag or cupped
hands.

Option A: Giving oxygen is inappropriate because the carbon dioxide level


is low, not the oxygen level.

Option B: Before notifying the healthcare provider, the nurse may first
intervene by having the client breathe into her cupped hands. If the client’s
situation does not improve, the nurse may notify the physician.

Option D: The client’s blood pressure and fetal heart rate are not related to
the low levels of carbon dioxide in her body. It is due to blowing off too
much carbon dioxide while using blow breathing exercises.
3. Question

68
C.BY: HOSSAM HAMDY MATERNITY RNPEDIA – ‫موسوعه التمريض‬

1 point(s)

When assessing a client at 12 weeks of gestation, the nurse recommends that


she and her husband consider attending childbirth preparation classes. When
is the best time for the couple to attend these classes?

A. At 16 weeks of gestation.

B. At 20 weeks of gestation.

C. At 24 weeks of gestation.

D. At 30 weeks of gestation.

Answer: D. At 30 weeks of gestation.

Learning is facilitated by an interested pupil. The couple is most interested in


childbirth toward the end of the pregnancy when they are beginning to
anticipate the onset of labor and the birth of their child. At 30 weeks, is
closest to the time when parents would be ready for such classes.

Option A: This would not be the best time during pregnancy for the couple
to attend childbirth education classes. At these times they will have other
teaching needs. Early pregnancy classes often include topics such as
nutrition, physiologic changes, coping with normal discomforts of
pregnancy, fetal development, maternal and fetal risk factors, and evolving
roles of the mother and her significant others.

Option B: At 20 weeks gestation, the couple may have an ultrasound to


check the fetus’ gender. This is also the time when the scan may show the
beating of the fetus’ heart, the curve of his spine, or his arms and legs
kicking.

Option C: During the 24th week of gestation, the couple may think about
where they should have the baby; learn about the signs of preterm labor; and
how they could introduce the new baby to his siblings.
4. Question

1 point(s)

One hour following a normal vaginal delivery, a newborn infant boy’s


axillary temperature is 96° F, his lower lip is shaking and, when the nurse

69
C.BY: HOSSAM HAMDY MATERNITY RNPEDIA – ‫موسوعه التمريض‬

assesses for a Moro reflex, the boy’s hands shake. Which intervention should
the nurse implement first?

A. Stimulate the infant to cry.

B. Wrap the infant in warm blankets.

C. Feed the infant formula.

D. Obtain a serum glucose level.

Answer: D. Obtain a serum glucose level.

This infant is demonstrating signs of hypoglycemia, possibly secondary to a


low body temperature. The nurse should first determine the serum glucose
level.

Option A: Stimulating the infant to cry is an intervention for a lethargic


infant.

Option B: This intervention should be done based on the temperature, but


first the glucose level should be obtained.

Option C: Feeding the infant with formula helps raise the blood sugar, but
first, the nurse should determine the glucose level.
5. Question

1 point(s)

Which statement made by the client indicates that the mother understands
the limitations of breastfeeding her newborn?

A. "Breastfeeding my infant consistently every 3 to 4 hours stops ovulation


and my period."

B. "Breastfeeding my baby immediately after drinking alcohol is safer than


waiting for the alcohol to clear my breast milk."

C. "I can start smoking cigarettes while breastfeeding because it will not
affect my breast milk."

D. "When I take a warm shower after I breastfeed, it relieves the pain from
being engorged between breastfeedings."
70
C.BY: HOSSAM HAMDY MATERNITY RNPEDIA – ‫موسوعه التمريض‬

Answer: A. “Breastfeeding my infant consistently every 3 to 4 hours


stops ovulation and my period.”

Continuous breastfeeding on a 3- to 4-hour schedule during the day will


cause a release of prolactin, which will suppress ovulation and menses, but is
not completely effective as a birth control method.

Option B: Drinking alcohol immediately before breastfeeding is because


alcohol can immediately enter breast milk.

Option C: Nicotine is transferred to the infant in breast milk

Option D: Taking a warm shower will stimulate the production of milk,


which will be more painful after breastfeedings
6. Question

1 point(s)

When assessing the adequacy of sperm for conception to occur, which of the
following is the most useful criterion?

A. Sperm count

B. Sperm motility

C. Sperm maturity

D. Semen volume

Answer: B. Sperm motility

Although all of the factors listed are important, sperm motility is the most
significant criterion when assessing male infertility. To reach and fertilize an
egg, sperm must move — wriggling and swimming through a woman’s
cervix, uterus, and fallopian tubes. This is known as motility. Males are most
likely to be fertile if at least 40% of their sperm are moving.

Option A: A normal sperm count ranges from 15 million sperm to more


than 200 million sperm per milliliter (mL) of semen. Anything less than 15
million sperm per milliliter, or 39 million sperm per ejaculate, is considered
low.
71
C.BY: HOSSAM HAMDY MATERNITY RNPEDIA – ‫موسوعه التمريض‬

Option C: Sperm cells are continually being produced by the testes, but not
all areas of the seminiferous tubules produce sperm cells at the same time.
One immature germ cell takes as long as 74 days to reach final maturation,
and during this growth process, there are intermittent resting phases.

Option D: According to the International Society for Sexual Medicine, the


average semen volume per ejaculate ranges from 1.25 to 5 milliliters (ml).
This amount is the equivalent of one-quarter to 1 teaspoon of semen. It is
important to note that semen volumes can vary from one time to another.
7. Question

1 point(s)

A couple who wants to conceive but has been unsuccessful during the last 2
years has undergone many diagnostic procedures. When discussing the
situation with the nurse, one partner states, “We know several friends in our
age group, and all of them have their own child already, Why can’t we have
one?”. Which of the following would be the most appropriate nursing
diagnosis for this couple?

A. Fear related to the unknown.

B. Pain related to numerous procedures.

C. Ineffective family coping related to infertility.

D. Self-esteem disturbance related to infertility.

Answer: D. Self-esteem disturbance related to infertility.

Based on the partner’s statement, the couple is verbalizing feelings of


inadequacy and negative feelings about themselves and their capabilities.
Thus, the nursing diagnosis of self-esteem disturbance is most appropriate.

Option A: The couple may have felt fear that they may never conceive a
child because of unsuccessful attempts. This is an appropriate nursing
diagnosis but one that should not be prioritized.

Option B: The couple has undergone a lot of diagnostic procedures for the
last 2 years and most likely has felt pain and uncomfortability, however, this
comes only as a secondary diagnosis.

72
C.BY: HOSSAM HAMDY MATERNITY RNPEDIA – ‫موسوعه التمريض‬

Option C: They may experience ineffective coping due to the unsuccessful


attempts of conceiving, but this is not a priority diagnosis
8. Question

1 point(s)

Which of the following urinary symptoms does the pregnant woman most
frequently experience during the first trimester?

A. Dysuria

B. Frequency

C. Incontinence

D. Burning

Answer: B. Frequency

Pressure and irritation of the bladder by the growing uterus during the first
trimester is responsible for causing urinary frequency.

Option A: The term dysuria is used to describe painful urination, which


often signifies an infection of the lower urinary tract. The discomfort is
usually described by the patient as burning, stinging, or itching.

Option C: Urinary incontinence — the loss of bladder control — is a


common and often embarrassing problem. It can be caused by everyday
habits, underlying medical conditions, or physical problems.

Option D: A burning sensation with urination can be caused by infectious


(including sexually transmitted infections, or STDs such as chlamydia and
gonorrhea) and noninfectious conditions, but it is most commonly due to
bacterial infection of the urinary tract affecting the bladder.
9. Question

1 point(s)

Heartburn and flatulence, common in the second trimester, are most likely
the result of which of the following?

A. Increased plasma HCG levels


73
C.BY: HOSSAM HAMDY MATERNITY RNPEDIA – ‫موسوعه التمريض‬

B. Decreased intestinal motility

C. Decreased gastric acidity

D. Elevated estrogen levels

Answer: C. Decreased gastric acidity

During the second trimester, the reduction in gastric acidity in conjunction


with pressure from the growing uterus and smooth muscle relaxation can
cause heartburn and flatulence.

Option A: HCG levels increase in the first, not the second, trimester.
Typically, the hCG levels will double every 72 hours. The level will reach its
peak in the first 8-11 weeks of pregnancy and then will decline and level off
for the remainder of the pregnancy.

Option B: Decrease intestinal motility would most likely be the cause of


constipation and bloating. The gallbladder enlarges and empties sluggishly in
response to meals during pregnancy. Small bowel transit is slowed, and the
resting pressure of the lower esophageal sphincter is reduced. All these
effects are reversed by delivery; motility reverts toward normal in the
postpartum period. The rapid return of normal motility suggests that the
effects of pregnancy are hormonally related.

Option D: Estrogen levels decrease in the second trimester. Estrogen is a


major hormone that involves a lot of minor hormones like estradiol which
was mentioned above. The estrogen hormones in women are critical as they
are produced by the placenta and ovaries and play a pivotal role in seeing the
pregnancy to term. They do so by maintaining the uterine lining which
provides a safe space for the growth of the baby. They also help regulate
other hormones like progesterone which are essential for fetal growth.
10. Question

1 point(s)

In which of the following areas would the nurse expect to observe chloasma?

A. Breast, areola, and nipples

B. Chest, neck, arms, and legs

C. Abdomen, breast, and thighs


74
C.BY: HOSSAM HAMDY MATERNITY RNPEDIA – ‫موسوعه التمريض‬

D. Cheeks, forehead, and nose

Answer: D. Cheeks, forehead, and nose

Chloasma, also called the mask of pregnancy, is an irregular hyperpigmented


area found on the face. It is not seen on the breasts, areola, nipples, chest,
neck, arms, legs, abdomen, or thighs.

Option A: The area around the nipples and the skin on the breast might
darken, possibly due to hormonal changes.

Option B: The skin on the inner thigh and neck may also darken as a result
of an increase in the hormones.

Option C: The woman might notice a dark line from the navel to the pubic
bone. This is called linea nigra.
11. Question

1 point(s)

A pregnant client states that she “waddles” when she walks. The nurse’s
explanation is based on which of the following is the cause?

A. The large size of the newborn.

B. Pressure on the pelvic muscles.

C. Relaxation of the pelvic joints.

D. Excessive weight gain.

Answer: C. Relaxation of the pelvic joints

During pregnancy, hormonal changes cause relaxation of the pelvic joints,


resulting in the typical “waddling” gait. The loosening of the joints and the
downward pressure from the growing belly actually cause the pelvis to get
wider. A wider pelvis means a wider stance. That explains part of the
waddle. The other part is caused by a shift in the center of balance.

Option A: Changes in posture are related to the growing fetus. The


abdominal muscles become stretched as the baby grows. These muscles are
75
C.BY: HOSSAM HAMDY MATERNITY RNPEDIA – ‫موسوعه التمريض‬

less able to contract and keep the lower back in proper alignment. Hormone
levels increase during pregnancy and cause joints and ligaments to loosen.

Option B: Pressure on the surrounding muscles causing discomfort is due to


the growing uterus. The extra weight of pregnancy often becomes more
noticeable in the second trimester. As pregnancy progresses, the uterus puts
more and more pressure on the lower body. As the pelvic floor weakens, this
pressure can cause a feeling of fullness in the vagina or generalized pain and
pressure in the hips and pelvis.

Option D: Weight gain has no effect on gait. Pregnancy leads to several


changes in body composition and morphology of women. It is not clear
whether the biomechanical changes occurring in this period are due
exclusively to body composition and size or to other physiological factors.
12. Question

1 point(s)

Which of the following represents the average amount of weight gained


during pregnancy?

A. 12 to 22 lb

B 15 to 25 lb

C. 24 to 30 lb

D. 25 to 40 lb

Answer: C. 24 to 30 lb

The average amount of weight gained during pregnancy is 24 to 30 lb. This


weight gain consists of the following: fetus – 7.5 lb; placenta and membrane
– 1.5 lb; amniotic fluid – 2 lb; uterus – 2.5 lb; breasts – 3 lb; and increased
blood volume – 2 to 4 lb; extravascular fluid and fat – 4 to 9 lb.

Option A: A gain of 12 to 22 lb is insufficient.

Option B: Whereas a weight gain of 15 to 25 lb is marginal.

Option D: A weight gain of 25 to 40 lb is considered excessive.


13. Question
76
C.BY: HOSSAM HAMDY MATERNITY RNPEDIA – ‫موسوعه التمريض‬

1 point(s)

When talking with a pregnant client who is experiencing aching swollen leg
veins, the nurse would explain that this is most probably the result of which
of the following?

A. Thrombophlebitis

B. Pregnancy-induced hypertension

C. Pressure on blood vessels from the enlarging uterus

D. The force of gravity pulling down on the uterus

Answer: C. Pressure on blood vessels from the enlarging uterus

The pressure of the growing uterus on blood vessels results in an increased


risk for venous stasis in the lower extremities. Subsequently, edema and
varicose vein formation may occur.

Option A: Thrombophlebitis is an inflammation of the veins due to


thrombus formation. The hypercoagulable condition of the immediate
antepartum period is responsible, in large part, for the development of
superficial thrombophlebitis and DVT in 0.15% and 0.04% of this patient
population, respectively.

Option B: Pregnancy-induced hypertension is not associated with these


symptoms. Pregnancy-induced hypertension is associated with significant
elevations in total peripheral resistance, enhanced responsiveness to
angiotensin II, and marked reductions in renal blood flow and glomerular
filtration rate, and proteinuria.

Option D: Gravity plays only a minor role with these symptoms. The center
of gravity of pregnant women is displaced anteriorly and superiorly,
compared to non-pregnant women. Furthermore, changes are seen in body
shape. Because the volume of the lower trunk increases structurally, it
becomes unstable. Nagai et al. reported that the postural sway of anterior-
posterior movements increased during pregnancy because of the increase in
the abdominal circumference
14. Question

1 point(s)
77
C.BY: HOSSAM HAMDY MATERNITY RNPEDIA – ‫موسوعه التمريض‬

Cervical softening and uterine souffle are classified as which of the


following?

A. Diagnostic signs

B. Presumptive signs

C. Probable signs

D. Positive signs

Answer: C. Probable signs

Cervical softening (Goodell sign) and uterine soufflé are two probable signs
of pregnancy. Probable signs are objective findings that strongly suggest
pregnancy. Other probable signs include Hegar sign, which is softening of
the lower uterine segment; Piskacek sign, which is an enlargement and
softening of the uterus; serum laboratory tests; changes in skin pigmentation;
and ultrasonic evidence of a gestational sac.

Option A: There are no diagnostic signs classified in pregnancy changes.

Option B: Presumptive signs are subjective signs and include amenorrhea;


nausea and vomiting; urinary frequency; breast tenderness and changes;
excessive fatigue; uterine enlargement; and quickening.

Option D: Positive signs of pregnancy are those signs that are definitely
confirmed as a pregnancy. They include fetal heart sounds, ultrasound
scanning of the fetus, palpation of the entire fetus, palpation of fetal
movements, x-ray, and actual delivery of an infant.
15. Question

1 point(s)

Which of the following would the nurse identify as a presumptive sign of


pregnancy?

A. Hegar sign

B. Nausea and vomiting

C. Skin pigmentation changes

78
C.BY: HOSSAM HAMDY MATERNITY RNPEDIA – ‫موسوعه التمريض‬

D. Positive serum pregnancy test

Answer: B. Nausea and vomiting

Presumptive signs of pregnancy are subjective signs. Of the signs listed, only
nausea and vomiting are presumptive signs.

Option A: Hegar’s sign is a non-sensitive indication of pregnancy in women


— its absence does not exclude pregnancy. It pertains to the features of the
cervix and the uterine isthmus. It is demonstrated as a softening in the
consistency of the uterus, and the uterus and cervix seem to be two separate
regions. It is a probable sign of pregnancy.

Option C: The area around the nipples and the skin on the inner thighs,
genitals, and neck might darken, possibly due to hormonal changes. The
woman might notice a dark line from the navel to the pubic bone (linea
nigra). Dark patches might develop on the face (chloasma). Avoid sun
exposure, which can worsen chloasma. After childbirth, skin typically
returns to its normal pigment over a period of several months.

Option D: A positive serum pregnancy test is considered a probable sign,


which is strongly suggestive of pregnancy.
16. Question

1 point(s)

Which of the following common emotional reactions to pregnancy would the


nurse expect to occur during the first trimester?

A. Introversion, egocentrism, narcissism.

B. Awkwardness, clumsiness, and unattractiveness.

C. Anxiety, passivity, extroversion.

D. Ambivalence, fear, fantasies.

Answer: D. Ambivalence, fear, fantasies

During the first trimester, common emotional reactions include ambivalence,


fear, fantasies, or anxiety.
79
C.BY: HOSSAM HAMDY MATERNITY RNPEDIA – ‫موسوعه التمريض‬

Option A: The second trimester is a period of well-being accompanied by


the increased need to learn about fetal growth and development. Common
emotional reactions during this trimester include narcissism, passivity, or
introversion. At times the woman may seem egocentric and self-centered.

Option B: During the third trimester, the woman typically feels awkward,
clumsy, and unattractive, often becoming more introverted or reflective of
her own childhood.

Option C: The effects of anxiety during pregnancy on offspring’s health are


serious and thought-provoking to which the need for identifying and
screening of anxiety disorders in prenatal care is necessary.
17. Question

1 point(s)

During which of the following would the focus of classes be mainly on


physiologic changes, fetal development, sexuality, during pregnancy, and
nutrition?

A. Prepregnant period

B. First trimester

C. Second trimester

D. Third trimester

Answer: B. First trimester

First-trimester classes commonly focus on such issues as early physiologic


changes, fetal development, sexuality during pregnancy, and nutrition. Some
early classes may include pregnant couples.

Option A: Preconception health and health care focus on things the woman
can do before and between pregnancies to increase the chances of having a
healthy baby. For some women, getting their body ready for pregnancy takes
a few months. For other women, it might take longer. Before getting
pregnant, the woman should talk to her physician about preconception health
care. The physician will want to discuss health history and any medical
conditions the woman currently has that could affect a pregnancy. He or she
also will discuss any previous pregnancy problems, medicines that the
80
C.BY: HOSSAM HAMDY MATERNITY RNPEDIA – ‫موسوعه التمريض‬

woman is currently taking, vaccinations that she might need, and steps she
can take before pregnancy to prevent certain birth defects.

Option C: The second trimester is a good time to start a regular, pregnancy-


friendly workout. It is a wonderful way to bond with and get support from
other moms-to-be. This is also the time when screening tests or
amniocentesis will be offered, as well as other chromosomal abnormalities,
genetic disorders, and neural tube defects.

Option D: Third-trimester classes may focus on preparation for birth,


parenting, and newborn care.
18. Question

1 point(s)

Which of the following would be a disadvantage of breastfeeding?

A. Involution occurs more rapidly.

B. The incidence of allergies increases due to maternal antibodies.

C. The father may resent the infant’s demands on the mother’s body.

D. There is a greater chance for error during preparation.

Answer: C. The father may resent the infant’s demands on the mother’s
body

With breastfeeding, the father’s body is not capable of providing the milk for
the newborn, which may interfere with feeding the newborn, providing fewer
chances for bonding, or he may be jealous of the infant’s demands on his
wife’s time and body.

Option A: Breastfeeding is advantageous because uterine involution occurs


more rapidly, thus minimizing blood loss.

Option B: The presence of maternal antibodies in breast milk helps decrease


the incidence of allergies in the newborn.

Option D: A greater chance for error is associated with bottle feeding. No


preparation is required for breastfeeding.
19. Question
81
C.BY: HOSSAM HAMDY MATERNITY RNPEDIA – ‫موسوعه التمريض‬

1 point(s)

Which of the following would cause a false-positive result on a pregnancy


test?

A. The test was performed less than 10 days after an abortion.

B. The test was performed too early or too late in the pregnancy.

C. The urine sample was stored too long at room temperature.

D. A spontaneous abortion or a missed abortion is impending.

Answer: A. The test was performed less than 10 days after an abortion

A false-positive reaction can occur if the pregnancy test is performed less


than 10 days after an abortion.

Option B: Shortly after a fertilized egg attaches to the uterine lining


(implantation), the placenta forms and produces the hormone human
chorionic gonadotropin (HCG). This hormone enters the bloodstream and
urine. During early pregnancy, the HCG concentration increases rapidly —
doubling every two to three days. The earlier the woman takes the home
pregnancy test, the harder it might be for the test to detect HCG.

Option C: Leaving the dipstick in the urine stream for the exact amount of
time allotted is also important. Consider setting a timer on a stopwatch or the
phone. That can help the woman track how long the dipstick has been in her
urine stream.

Option D: During pregnancy, hCG levels continue to rise as the placenta


grows, doubling every few days and peaking at around 10 weeks. When a
pregnancy ends, hCG levels begin to recede, but it’s a slow process. The
hormone can remain in the blood and urine for up to six weeks following the
end of the pregnancy. It’s possible to have a false-positive test until the hCG
levels return to their pre-pregnancy state.
20. Question

1 point(s)

FHR can be auscultated with a fetoscope as early as which of the following?

A. 5 weeks gestation
82
C.BY: HOSSAM HAMDY MATERNITY RNPEDIA – ‫موسوعه التمريض‬

B. 10 weeks gestation

C. 15 weeks gestation

D. 20 weeks gestation

Answer: D. 20 weeks gestation

The FHR can be auscultated with a fetoscope at about 20 week’s gestation.


FHR usually is auscultated at the midline suprapubic region with a Doppler
ultrasound transducer at 10 to 12 week’s gestation. FHR, cannot be heard
any earlier than 10 weeks’ gestation.

Option A: A fetal heartbeat may first be detected by a vaginal ultrasound as


early as 5 1/2 to 6 weeks after gestation. That’s when a fetal pole, the first
visible sign of a developing embryo, can sometimes be seen.

Option B: With all the rapid growth, the woman will probably be able to
hear her baby’s heartbeat for the first time around week 9 or week 10 of
pregnancy, though it can vary a bit. It will be about 170 beats per minute by
this time, a rate that will slow from here on out. Her doctor or midwife will
place a handheld ultrasound device called a Doppler on your belly to amplify
the sound.

Option C: The baby’s heartbeat may be heard as early as the twelfth week
of pregnancy using a highly sensitive Doppler that allows hearing the baby’s
heartbeat. The normal range for the baby’s heart rate is 115 to 160 beats per
minute.
21. Question

1 point(s)

A client LMP began July 5, 2020. Her estimated date of delivery (EDD)
should be which of the following?

A. January 2, 2021

B. March 28, 2021

C. April 12, 2021

D. October 12, 2020

83
C.BY: HOSSAM HAMDY MATERNITY RNPEDIA – ‫موسوعه التمريض‬

Answer: C. April 12, 2021

LMP of the woman is July 5, 2020. Using Naegele’s rule: Add seven days to
the first day of the LMP (5+7=12), then subtract three months (July-3
months= April), lastly add 1 year. The EDD will be on April 12, 2021.

Option A: On January 2, 2021, the woman is at 25 weeks gestation.

Option B: On March 28, 2021, the woman is at 38 weeks gestation.

Option D: On October 12, 2020, the woman is at 14 weeks gestation.


22. Question

1 point(s)

Which of the following fundal heights indicates less than 12 weeks’


gestation when the date of the LMP is unknown?

A. Uterus in the pelvis

B. Uterus at the xiphoid

C. Uterus in the abdomen

D. Uterus at the umbilicus

Answer: A. Uterus in the pelvis

When the LMP is unknown, the gestational age of the fetus is estimated by
uterine size or position (fundal height). The presence of the uterus in the
pelvis indicates less than 12 weeks’ gestation. At approximately 12 to 14
weeks, the fundus is out of the pelvis above the symphysis pubis.

Option B: When the fundal height is at the level of the xiphoid, the woman
is already at 40 weeks gestation.

Option C: When the physician measures how high the top of the uterus has
reached in the mother’s abdomen, he or she is measuring the fundal height.
This is a much more accurate way of estimating fetal growth than weighing
the mother.

84
C.BY: HOSSAM HAMDY MATERNITY RNPEDIA – ‫موسوعه التمريض‬

Option D: When the fundal height is at the level of the umbilicus, the
woman is approximately 20 weeks gestation.
23. Question

1 point(s)

Which of the following danger signs should be reported promptly during the
antepartum period?

A. Constipation

B. Breast tenderness

C. Nasal stuffiness

D. Leaking amniotic fluid

Answer: D. Leaking amniotic fluid

Danger signs that require prompt reporting leaking of amniotic fluid, vaginal
bleeding, blurred vision, rapid weight gain, and elevated blood pressure.

Option A: Postpartum constipation, with symptoms such as pain or


discomfort, straining, and hard stool, is a common condition affecting
mothers. Hemorrhoids, pain at the episiotomy site, effects of pregnancy
hormones, and haematinics used in pregnancy can increase the risk of
postpartum constipation. Eating a high-fiber diet and increasing fluid intake
is usually encouraged, although laxatives are commonly used in relieving
constipation.

Option B: The breasts may be extra tender as early as one or two weeks
after conception. This is because the body is making so much estrogen and
progesterone in early pregnancy that the glands in the breasts start growing.
This hormone surge causes breasts to retain more fluids and feel heavy, sore,
or more sensitive than normal PMS tenderness.

Option C: Pregnancy rhinitis is an inflammation of the mucous membranes


lining the nose. This causes nasal congestion. Increased blood flow to the
nasal passages and enlargement of the nasal veins also play a role.
24. Question

1 point(s)
85
C.BY: HOSSAM HAMDY MATERNITY RNPEDIA – ‫موسوعه التمريض‬

Which of the following prenatal laboratory test values would the nurse
consider as significant?

A. Hematocrit 33.5%

B. Rubella titer less than 1:8

C. White blood cells 8,000/mm3

D. One hour glucose challenge test 110 g/dL

Answer: B. Rubella titer less than 1:8

A rubella titer should be 1:8 or greater. Thus, a finding of a titer less than 1:8
is significant, indicating that the client may not possess immunity to rubella.

Option A: Hemoglobin levels in the first and second half of pregnancy can
predict preeclampsia and premature preterm rupture of membranes.
Increased hematocrit levels in the second half of pregnancy or lack of
reduction of hematocrit levels in the second half compared to the first half
can estimate preeclampsia. Normal values of hematocrit have been
determined from 36 to 48 percent for women in childbearing age. The cause
of its decrease in adults and during pregnancy is anemia, and the reasons for
its increase are myeloproliferative disorders, chronic obstructive pulmonary
disease, and other hypoxic lung conditions.

Option C: The average white cell count during pregnancy is about 9-15k. It
increases up to term and can go as high as 25k during labor.

Option D: A glucose screening test is a routine test during pregnancy that


checks a pregnant woman’s blood glucose (sugar) level. This test checks for
gestational diabetes. Most pregnant women have a glucose screening test
between 24 and 28 weeks of pregnancy. The test may be done earlier if the
woman has a high glucose level in her urine during her routine prenatal
visits, or if she has a high risk for diabetes. Most of the time, a normal result
for the glucose screening test is a blood sugar that is equal to or less than 140
mg/dL (7.8 mmol/L) 1 hour after drinking the glucose solution.
25. Question

1 point(s)

86
C.BY: HOSSAM HAMDY MATERNITY RNPEDIA – ‫موسوعه التمريض‬

Which of the following characteristics of contractions would the nurse


expect to find in a client experiencing true labor?

A. Occurring at irregular intervals.

B. Starting mainly in the abdomen.

C. Gradually increasing intervals.

D. Increasing intensity with walking.

Answer: D. Increasing intensity with walking

With true labor, contractions increase in intensity with walking. In addition,


true labor contractions occur at regular intervals, usually starting in the back
and sweeping around to the abdomen. The interval of true labor contractions
gradually shortens.

Option A: Unlike false labor contractions or Braxton Hicks contractions,


true labor contractions don’t stop when you change your position or relax. It
occurs at regular intervals.

Option B: Labor contractions usually cause discomfort or a dull ache at the


back and lower abdomen, along with pressure in the pelvis.

Option C: True labor contractions come at regular intervals and get closer
together as time goes on. (Contractions last about 30 to 70 seconds.).
26. Question

1 point(s)

During which of the following stages of labor would the nurse assess
“crowning”?

A. First stage

B. Second stage

C. Third stage

D. Fourth stage

87
C.BY: HOSSAM HAMDY MATERNITY RNPEDIA – ‫موسوعه التمريض‬

Answer: B. Second stage

Crowing, which occurs when the newborn’s head or presenting part appears
at the vaginal opening, occurs during the second stage of labor.

Option A: During the first stage of labor, cervical dilation and effacement
occur. Effacement means that the cervix stretches and gets thinner.
Dilatation means that the cervix opens. As labor nears, the cervix may start
to thin or stretch (efface) and open (dilate). This prepares the cervix for the
baby to pass through the birth canal (vagina).

Option C: During the third stage of labor, the newborn and placenta are
delivered. This stage is often called delivery of the “afterbirth” and is the
shortest stage of labor. It may last from a few minutes to 20 minutes.

Option D: The fourth stage of labor lasts from 1 to 4 hours after birth,
during which time the mother and newborn recover from the physical
process of birth and the mother’s organs undergo the initial readjustment to
the nonpregnant state.
27. Question

1 point(s)

Barbiturates are usually not given for pain relief during active labor for
which of the following reasons?

A. The neonatal effects include hypotonia, hypothermia, generalized


drowsiness, and reluctance to feed for the first few days.

B. These drugs readily cross the placental barrier, causing depressive effects
in the newborn 2 to 3 hours after intramuscular injection.

C. They rapidly transfer across the placenta, and the lack of an antagonist
makes them generally inappropriate during labor.

D. Adverse reactions may include maternal hypotension, allergic or toxic


reaction, or partial or total respiratory failure.

Answer: C. They rapidly transfer across the placenta, and the lack of an
antagonist makes them generally inappropriate during labor.

Barbiturates are rapidly transferred across the placental barrier, and the lack
of an antagonist makes them generally inappropriate during active labor.
88
C.BY: HOSSAM HAMDY MATERNITY RNPEDIA – ‫موسوعه التمريض‬

Option A: Neonatal side effects of barbiturates include central nervous


system depression, prolonged drowsiness, delayed establishment of feeding
(e.g. due to poor sucking reflex or poor sucking pressure). Tranquilizers are
associated with neonatal effects such as hypotonia, hypothermia, generalized
drowsiness, and reluctance to feed for the first few days.

Option B: Narcotic analgesic readily crosses the placental barrier, causing


depressive effects in the newborn 2 to 3 hours after intramuscular injection.

Option D: Regional anesthesia is associated with adverse reactions such as


maternal hypotension, allergic or toxic reaction, or partial or total respiratory
failure.
28. Question

1 point(s)

Which of the following nursing interventions would the nurse perform


during the third stage of labor?

A. Obtain a urine specimen and other laboratory tests.

B. Assess uterine contractions every 30 minutes.

C. Coach for effective client pushing.

D. Promote parent-newborn interaction.

Answer: D. Promote parent-newborn interaction.

During the third stage of labor, which begins with the delivery of the
newborn, the nurse would promote parent-newborn interaction by placing
the newborn on the mother’s abdomen and encouraging the parents to touch
the newborn.

Option A: Collecting a urine specimen and other laboratory tests is done on


admission during the first stage of labor.

Option B: Assessing uterine contractions every 30 minutes is performed


during the latent phase of the first stage of labor.

Option D: Coaching the client to push effectively is appropriate during the


second stage of labor.

89
C.BY: HOSSAM HAMDY MATERNITY RNPEDIA – ‫موسوعه التمريض‬

29. Question

1 point(s)

Which of the following actions demonstrates the nurse’s understanding of


the newborn’s thermoregulatory ability?

A. Placing the newborn under a radiant warmer.

B. Suctioning with a bulb syringe.

C. Obtaining an Apgar score.

D. Inspecting the newborn’s umbilical cord.

Answer: A. Placing the newborn under a radiant warmer.

The newborn’s ability to regulate body temperature is poor. Therefore,


placing the newborn under a radiant warmer aids in maintaining his or her
body temperature.

Option B: Suctioning with a bulb syringe helps maintain a patent airway.

Option C: Obtaining an Apgar score measures the newborn’s immediate


adjustment to extrauterine life.

Option D: Inspecting the umbilical cord aids in detecting cord anomalies.


30. Question

1 point(s)

Immediately before expulsion, which of the following cardinal movements


occur?

A. Descent

B. Flexion

C. Extension

D. External rotation

90
C.BY: HOSSAM HAMDY MATERNITY RNPEDIA – ‫موسوعه التمريض‬

Answer: D. External rotation

Immediately before expulsion or birth of the rest of the body, the cardinal
movement of external rotation occurs. During this pause, the baby must
rotate so that his/her face moves from face-down to facing either of the
laboring woman’s inner thighs. This movement, also called restitution, is
necessary as the shoulders must fit around and under the pubic arch.

Option A: The baby’s head moves deep into the pelvic cavity and is
commonly called lightening. The baby’s head becomes markedly molded
when these distances are closely the same. When the occiput is at the level of
the ischial spines, it can be assumed that the biparietal diameter is engaged
and then descends into the pelvic inlet.

Option B: Flexion occurs during descent and is brought about by the


resistance felt by the baby’s head against the soft tissues of the pelvis. The
resistance brings about a flexion in the baby’s head so that the chin meets the
chest. The smallest diameter of the baby’s head (or suboccipitobregmatic
plane) presents into the pelvis.

Option C: After internal rotation is complete and the head passes through
the pelvis at the nape of the neck, a rest occurs as the neck is under the pubic
arch. Extension occurs as the head, face, and chin are born.
31. Question

1 point(s)

Before birth, which of the following structures connects the right and left
auricles of the heart?

A. Umbilical vein

B. Foramen ovale

C. Ductus arteriosus

D. Ductus venosus

Answer: B. Foramen ovale

The foramen ovale is an opening between the right and left auricles (atria)
that should close shortly after birth so the newborn will not have a murmur
or mixed-blood traveling through the vascular system.
91
C.BY: HOSSAM HAMDY MATERNITY RNPEDIA – ‫موسوعه التمريض‬

Option A: The umbilical vein carries oxygenated, nutrient-rich blood from


the placenta to the fetus, and the umbilical arteries carry deoxygenated,
nutrient-depleted blood from the fetus to the placenta. Any impairment in
blood flow within the cord can be a catastrophic event for the fetus.

Option C: At birth, the lungs fill with air with the first breaths, pulmonary
vascular resistance drops, and blood flows from the right ventricle to the
lungs for oxygenation. The increased arterial oxygen tension and the
decreased flow through the ductus arteriosus allow the ductus to constrict.

Option D: In utero, the ductus venosus connects the left portal vein to the
inferior vena cava, allowing a portion of the venous blood to bypass the liver
and return to the heart. After birth, the ductus venosus generally closes
between days of life 2 to 18 in term infants
32. Question

1 point(s)

Which of the following when present in the urine may cause a reddish stain
on the diaper of a newborn?

A. Mucus

B. Uric acid crystals

C. Bilirubin

D. Excess iron

Answer: B. Uric acid crystals

Uric acid crystals in the urine may produce the reddish “brick dust” stain on
the diaper. Urate crystals are made up of uric acid, an end product of normal
metabolism. Babies are born with a high blood uric acid level because of the
amount they get across the placenta, and this is quickly excreted in the urine
and stool. If a baby is not making much urine at this time, these urate
crystals will be especially concentrated and easy to see. This does not mean,
however, that your baby is dehydrated. Instead, you can make sure your
baby is properly drinking and urinating enough with a few simple questions.

Option A: Mucus in urine may be positive in other clinical conditions such


as urinary tract infection and stones. Secreted from the vesicle glands of the
92
C.BY: HOSSAM HAMDY MATERNITY RNPEDIA – ‫موسوعه التمريض‬

genitourinary tract, mucus is not generally seen in the urine samples of


children under the age of 12.

Option C: Bilirubin is a brownish-yellow substance that is produced after


red blood cells break down. The body gets rid of bilirubin through the stool
and urine.

Option D: Neonates and young children with certain liver disorders


characteristically have a very high serum ferritin level. These conditions are
gestational alloimmune liver disease (GALD) and hemophagocytic
lymphohistiocytosis (HLH). It is not clear what the iron content of the
ferritin is in these neonates. Knowing this will be a step toward
understanding whether the pathogenesis of these conditions involves iron
overload. Additionally, if urine ferritin and iron levels correlate with serum
ferritin and iron levels, urine may be used as a non-invasive way to monitor
iron status.
33. Question

1 point(s)

When assessing the newborn’s heart rate, which of the following ranges
would be considered normal if the newborn were sleeping?

A. 80 beats per minute

B. 100 beats per minute

C. 120 beats per minute

D. 140 beats per minute

Answer: B. 100 beats per minute

The normal heart rate for a newborn that is sleeping is approximately 100
beats per minute. If the newborn was awake, the normal heart rate would
range from 120 to 160 beats per minute.

Option A: 80 beats per minute is below the normal range of a newborn’s


heart rate. Neonatal bradycardia is defined as a decrease in heart by 30 bpm
from baseline. Regarding neonatal resuscitation, bradycardia is concerning
when the heart rate is less than 100 bpm. The primary cause of neonatal
bradycardia is hypoxia. Other causes of bradycardia in this age group include
93
C.BY: HOSSAM HAMDY MATERNITY RNPEDIA – ‫موسوعه التمريض‬

hypothermia, hypovolemia, and pneumothorax, head injury, and


medications.

Option C: Newborns 0 to 1 month old has a normal range of 70 to 190 beats


per minute

Option D: 140 beats per minute is still within the normal range of a
newborn’s heart rate.
34. Question

1 point(s)

Which of the following is true regarding the fontanels of the newborn?

A. The anterior is triangular shaped; the posterior is diamond-shaped.

B. The posterior closes at 18 months; the anterior closes at 8 to 12 weeks.

C. The anterior is large in size when compared to the posterior fontanel.

D. The anterior is bulging; the posterior appears sunken.

Answer: C. The anterior is large in size when compared to the posterior


fontanel.

The anterior fontanel is larger in size than the posterior fontanel.


Additionally, the anterior fontanel, which is diamond-shaped, closes at 18
months, whereas the posterior fontanel, which is triangular shaped, closes at
8 to 12 weeks. Neither fontanel should appear bulging, which may indicate
increased intracranial pressure, or sunken, which may indicate dehydration.

Option A: The anterior fontanelle is the largest of the six fontanelles, and it
resembles a diamond-shape ranging in size from 0.6 cm to 3.6 cm with a
mean of 2.1 cm. Unlike the anterior fontanelle, the posterior fontanelle is
triangular and completely closes within about six to eight weeks after birth.

Option B: The average closure time of the anterior fontanelle ranges from
13 to 24 months. The posterior fontanel completely closes within about six
to eight weeks after birth.

Option D: In addition to being the largest, the anterior fontanelle is also the
most important clinically. This structure offers insight into the newborn’s
state of health, especially hydration and intracranial pressure status. A
94
C.BY: HOSSAM HAMDY MATERNITY RNPEDIA – ‫موسوعه التمريض‬

sunken fontanelle is primarily due to dehydration. Other clinical indicators


that support the diagnosis of dehydration are dry mucous membranes,
sunken eyes, poor tear production, decreased peripheral perfusion, and lack
of wet diapers. Furthermore, a bulging fontanelle may indicate a rise in
intracranial pressure, suggesting multiple pathologies: hydrocephalus,
hypoxemia, meningitis, trauma, or hemorrhage.
35. Question

1 point(s)

Which of the following groups of newborn reflexes below are present at


birth and remain unchanged through adulthood?

A. Blink, cough, rooting, and gag

B. Blink, cough, sneeze, gag

C. Rooting, sneeze, swallowing, and cough

D. Stepping, blink, cough, and sneeze

Answer: B. Blink, cough, sneeze, gag

Blink, cough, sneeze, swallowing and gag reflexes are all present at birth and
remain unchanged through adulthood. Reflexes such as rooting and stepping
subside within the first year.

Option A: The rooting reflex is one of the involuntary primitive motor


reflexes, which are also known as the frontal release reflexes, that are
mediated by the brainstem. It initiates when the corner of an infant’s mouth
is stimulated. When the mouth is touched or stroked, the newborn will turn
his or her head towards the stimulus and open the mouth with tongue
thrusting. The rooting reflex is present at birth (approximately 28 weeks) and
lasts about 4 to 6 months until the frontal lobe of the cerebral cortex
develops and suppresses the primitive motor reflexes.

Option C: The rooting reflex is essential for survival and growth for it helps
the newborn find the source of food (breast or bottle) and initiate feeding. As
the frontal lobe matures, the primitive reflexes are replaced with voluntary
motor functions. The age when each primitive reflex disappears varies.

95
C.BY: HOSSAM HAMDY MATERNITY RNPEDIA – ‫موسوعه التمريض‬

Option D: The stepping reflex is also called the walking or dance reflex
because a baby appears to take steps or dance when held upright with his or
her feet touching a solid surface. This reflex lasts about 2 months.
36. Question

1 point(s)

Which of the following describes the Babinski reflex?

A. The newborn’s toes will hyperextend and fan apart from the dorsiflexion
of the big toe when one side of the foot is stroked upward from the ball of
the heel and across the ball of the foot.

B. The newborn abducts and flexes all extremities and may begin to cry
when exposed to sudden movement or loud noise.

C. The newborn turns the head in the direction of the stimulus, opens the
mouth, and begins to suck when the cheek, lip, or corner of the mouth is
touched.

D. The newborn will attempt to crawl forward with both arms and legs when
he is placed on his abdomen on a flat surface.

Answer: A. The newborn’s toes will hyperextend and fan apart from the
dorsiflexion of the big toe when one side of the foot is stroked upward
from the ball of the heel and across the ball of the foot.

With the Babinski reflex, the newborn’s toes hyperextend and fan apart from
dorsiflexion of the big toe when one side of the foot is stroked upward from
the heel and across the ball of the foot.

Option B: With the startle reflex, the newborn abducts and flexes all
extremities and may begin to cry when exposed to sudden movement of loud
noise.

Option C: With the rooting and sucking reflex, the newborn turns his head
in the direction of the stimulus, opens the mouth, and begins to suck when
the cheeks, lip, or corner of the mouth is touched.

Option D: With the crawl reflex, the newborn will attempt to crawl forward
with both arms and legs when he is placed on his abdomen on a flat surface.
37. Question
96
C.BY: HOSSAM HAMDY MATERNITY RNPEDIA – ‫موسوعه التمريض‬

1 point(s)

Which of the following statements best describes hyperemesis gravidarum?

A. Severe anemia leading to an electrolyte, metabolic, and nutritional


imbalances in the absence of other medical problems.

B. Severe nausea and vomiting leading to an electrolyte, metabolic, and


nutritional imbalances in the absence of other medical problems

C. Loss of appetite and continuous vomiting that commonly results in


dehydration and ultimately decreasing maternal nutrients.

D. Severe nausea and diarrhea that can cause gastrointestinal irritation and
possibly internal bleeding.

Answer: B. Severe nausea and vomiting leading to the electrolyte,


metabolic, and nutritional imbalances in the absence of other medical
problems.

The description of hyperemesis gravidarum includes severe nausea and


vomiting, leading to the electrolyte, metabolic, and nutritional imbalances in
the absence of other medical problems.

Option A: Hyperemesis is not a form of anemia. The exact cause of nausea


and vomiting during pregnancy is not known. However, it is believed to be
caused by a rapidly rising blood level of a hormone called human chorionic
gonadotropin (HCG). HCG is released by the placenta. Mild morning
sickness is common. Hyperemesis gravidarum is less common and more
severe.

Option C: Loss of appetite may occur secondary to nausea and vomiting of


hyperemesis, which, if it continues, can deplete the nutrients transported to
the fetus. Women with hyperemesis gravidarum have extreme nausea and
vomiting during pregnancy. It can cause a weight loss of more than 5% of
body weight. The condition can happen in any pregnancy, but is a little more
likely if the woman is pregnant with twins (or more babies), or if she has a
hydatidiform mole. Women are at higher risk for hyperemesis if they have
had the problem in previous pregnancies or are prone to motion sickness.

Option D: Diarrhea does not occur with hyperemesis. Constipation is one of


the symptoms. Increase fluids during times of the day when there is a feeling
of nausea. Seltzer, ginger ale, or other sparkling drinks may help. The
97
C.BY: HOSSAM HAMDY MATERNITY RNPEDIA – ‫موسوعه التمريض‬

woman can also try using low-dose ginger supplements or acupressure wrist
bands to ease symptoms.
38. Question

1 point(s)

Which of the following would the nurse identify as a classic sign of PIH?

A. Edema of the feet and ankles

B. Edema of the hands and face

C. Weight gain of 1 lb/week

D. Early morning headache

Answer: B. Edema of the hands and face

Edema of the hands and face is a classic sign of PIH. Aggressive volume
resuscitation may lead to pulmonary edema, which is a common cause of
maternal morbidity and mortality. Pulmonary edema occurs most frequently
48-72 hours postpartum, probably due to mobilization of extravascular fluid.
Because volume expansion has no demonstrated benefit, patients should be
fluid restricted when possible, at least until the period of postpartum diuresis.

Option A: Many healthy pregnant women experience foot and ankle edema.
During pregnancy, the extra fluid in the body and the pressure from the
growing uterus can cause swelling (or “edema”) in the ankles and feet. The
swelling tends to get worse as a woman’s due date nears, particularly near
the end of the day and during hotter weather.

Option C: A weight gain of 2 lb or more per week indicates a problem. High


pregnancy weight gain was more strongly associated with term preeclampsia
than early preterm preeclampsia (eg, 64% versus 43% increased odds per 1 z
score difference in weight gain in normal-weight women, and 30% versus
0% in obese women, respectively).

Option D: Early morning headache is not a classic sign of PIH. Dull or


severe, throbbing headaches, often described as migraine-like that just won’t
go away are cause for concern.
39. Question

98
C.BY: HOSSAM HAMDY MATERNITY RNPEDIA – ‫موسوعه التمريض‬

1 point(s)

In which of the following types of spontaneous abortions would the nurse


assess dark brown vaginal discharge and a negative pregnancy test?

A. Threatened

B. Imminent

C. Missed

D. Incomplete

Answer: C. Missed

In a missed abortion, there is early fetal intrauterine death, and products of


conception are not expelled. The cervix remains closed; there may be a dark
brown vaginal discharge, negative pregnancy test, and cessation of uterine
growth and breast tenderness.

Option A: A threatened abortion is evidenced with cramping and vaginal


bleeding in early pregnancy, with no cervical dilation.

Option B: An imminent-abortion indicated by bleeding and pain along with


an effaced cervix.

Option D: An incomplete abortion involves only expulsion of part of the


products of conception and bleeding occurs with cervical dilation.
40. Question

1 point(s)

Which of the following factors would the nurse suspect as predisposing a


client to placenta previa?

A. Multiple gestation

B. Uterine anomalies

C. Abdominal trauma

D. Renal or vascular disease

99
C.BY: HOSSAM HAMDY MATERNITY RNPEDIA – ‫موسوعه التمريض‬

Answer: A. Multiple gestation

Multiple gestation is one of the predisposing factors that may cause placenta
previa. Placenta previa is more common in older and multiparous women.
The reason is not clear but it may be associated with the aging of the
vasculature of the uterus. This causes placental hypertrophy and enlargement
which increases the likelihood of the placenta encroaching on lower segment

Option B: Patients with a unicornuate uterus had high rates of placenta or


vasa previa, and three of five pregnancies with placenta previa also had
placenta accreta. While this represents a small series, placenta accreta in
those with placenta previa has been reported to occur with this frequency in
women with multiple prior cesarean deliveries.

Option C: The exact etiology of placental abruption is unknown. However,


a number of factors are associated with its occurrence. Risk factors can be
thought of in 3 groups: health history, including behaviors, and past
obstetrical events, current pregnancy, and unexpected trauma.

Option D: Complications of conservative management of placenta percreta


described in the literature include bleeding, infection (endometritis, wound
infection, peritonitis, pyelonephritis, uterine necrosis), sepsis and septic
shock, fistula formation, thrombosis, pulmonary embolism, pulmonary
edema, and the side-effects of methotrexate therapy. Acute renal failure has
only been described in one case with methotrexate injection into the
umbilical cord and was considered an acute side-effect of methotrexate
therapy.
41. Question

1 point(s)

Which of the following would the nurse assess in a client experiencing


abruptio placenta?

A. Bright red, painless vaginal bleeding

B. Concealed or external dark red bleeding

C. Palpable fetal outline

D. Soft and nontender abdomen

100
C.BY: HOSSAM HAMDY MATERNITY RNPEDIA – ‫موسوعه التمريض‬

Answer: B. Concealed or external dark red bleeding

A client with abruptio placentae may exhibit concealed or dark red bleeding,
possibly reporting sudden intense localized uterine pain. The uterus is
typically firm to board-like, and the fetal presenting part may be engaged.

Option A: Painless vaginal bleeding during the second or third trimester of


pregnancy is the usual presentation in placenta previa. The bleeding may be
provoked from intercourse, vaginal examinations, labor, and at times there
may be no identifiable cause. On speculum examination, there may be
minimal bleeding to active bleeding.

Option C: On physical examination, the uterus tends to be soft and fetal


parts are readily palpable. With placenta previa, the presenting part is
unengaged and malpresentation is common, seen in up to 50% of cases.

Option D: Abdominal examination usually finds the uterus non-tender, soft


and relaxed. Leopold’s Maneuvers may find the fetus in an oblique or breech
position or lying transverse as a result of the abnormal position of the
placenta. Malpresentation is found in about 35% cases.
42. Question

1 point(s)

Which of the following is described as premature separation of a normally


implanted placenta during the second half of pregnancy, usually with severe
hemorrhage?

A. Placenta previa

B. Ectopic pregnancy

C. Incompetent cervix

D. Abruptio placenta

Answer: D. Abruptio placentae

Abruptio placenta is described as premature separation of a normally


implanted placenta during the second half of pregnancy, usually with severe
hemorrhage.
101
C.BY: HOSSAM HAMDY MATERNITY RNPEDIA – ‫موسوعه التمريض‬

Option A: Placenta previa refers to implantation of the placenta in the lower


uterine segment, causing painless bleeding in the third trimester of
pregnancy.

Option B: Ectopic pregnancy refers to the implantation of the products of


conception in a site other than the endometrium.

Option C: Incompetent cervix is a conduction characterized by painful


dilation of the cervical os without uterine contractions.
43. Question

1 point(s)

Which of the following may happen if the uterus becomes overstimulated by


oxytocin during the induction of labor?

A. Weak contraction prolonged to more than 70 seconds.

B. Tetanic contractions prolonged to more than 90 seconds.

C. Increased pain with bright red vaginal bleeding.

D. Increased restlessness and anxiety.

Answer: B. Tetanic contractions prolonged to more than 90 seconds

Hyperstimulation of the uterus such as with oxytocin during the induction of


labor may result in tetanic contractions prolonged to more than 90seconds,
which could lead to such complications as fetal distress, abruptio placentae,
amniotic fluid embolism, laceration of the cervix, and uterine rupture.

Option A: With some methods, the uterus can be overstimulated, causing it


to contract too frequently. Too many contractions may lead to changes in the
fetal heart rate, umbilical cord problems, and other problems.

Option C: Painless vaginal bleeding during the second or third trimester of


pregnancy is the usual presentation in placenta previa. The bleeding may be
provoked from intercourse, vaginal examinations, labor, and at times there
may be no identifiable cause. On speculum examination, there may be
minimal bleeding to active bleeding.

Option D: Synthetic oxytocin, also known as Pitocin, is frequently


administered during delivery for the purpose of inducing labor and
102
C.BY: HOSSAM HAMDY MATERNITY RNPEDIA – ‫موسوعه التمريض‬

preventing excessive post-delivery bleeding. One might hypothesize, based


on the role that natural oxytocin plays, that women receiving oxytocin might
receive some degree of benefit from the peri-partum use of Pitocin; however,
a recent study calls this hypothesis into question. This study used population-
based data available through the Massachusetts Integrated Clinical Academic
Research Database (MiCARD) in order to retrospectively examine the
relationship between peripartum synthetic oxytocin administration and the
development of depressive and anxiety disorders within the first year
postpartum. While the authors expected to observe that women exposed to
synthetic oxytocin in this cohort would have a reduced risk of postpartum
depressive and/or anxiety disorders than those without any exposure, they
actually found the opposite.
44. Question

1 point(s)

When preparing a client for cesarean delivery, which of the following key
concepts should be considered when implementing nursing care?

A. Instruct the mother’s support person to remain in the family lounge until
after the delivery.

B. Arrange for a staff member of the anesthesia department to explain what


to expect postoperatively.

C. Modify preoperative teaching to meet the needs of either a planned or


emergency cesarean birth.

D. Explain the surgery, expected outcome, and kind of anesthetics.

Answer: C. Modify preoperative teaching to meet the needs of either a


planned or emergency cesarean birth

A key point to consider when preparing the client for a cesarean delivery is
to modify the preoperative teaching to meet the needs of either a planned or
emergency cesarean birth, the depth and breadth of instruction will depend
on circumstances and time available.

Option A: Allowing the mother’s support person to remain with her as much
as possible is an important concept, although doing so depends on many
variables.

103
C.BY: HOSSAM HAMDY MATERNITY RNPEDIA – ‫موسوعه التمريض‬

Option B: Arranging for necessary explanations by various staff members to


be involved with the client’s care is a nursing responsibility.

Option D: The nurse is responsible for reinforcing the explanations about


the surgery’s expected outcome, and type of anesthetic to be used. The
obstetrician is responsible for explaining the surgery and outcome, and the
anesthesiology staff is responsible for explanations about the type of
anesthesia to be used.
45. Question

1 point(s)

Which of the following best describes preterm labor?

A. Labor that begins after 20 weeks gestation and before 37 weeks gestation.

B. Labor that begins after 15 weeks gestation and before 37 weeks gestation

C. Labor that begins after 24 weeks gestation and before 28 weeks gestation.

D. Labor that begins after 28 weeks gestation and before 40 weeks gestation.

Answer: A. Labor that begins after 20 weeks gestation and before 37


weeks gestation

Preterm labor is best described as labor that begins after 20 weeks’ gestation
and before 37 weeks’ gestation. The other time periods are inaccurate.

Option B: At 15 weeks gestation, the fetus weighs around 4 ounces (oz) and
its facial features will be starting to take shape. The bones in its ears will be
developing for the first time, and the fetus will be able to hear the sounds of
the mother’s heart, digestive system, and voice. Even though the eyes of the
fetus will remain closed, it will be able to sense and respond to light.

Option C: At 24 weeks gestation, the branches of the baby’s lungs are


forming, as well as the cells that make surfactant, a natural substance that
lines the tiny air sacs (called alveoli) in the lungs to make breathing
possible.4 While a small amount of surfactant is now present, the lungs are
still immature. Babies born this early have a hard time breathing.

Option D: Babies begin having eye movements as early as 14 weeks, but


these movements increase around 28 weeks. The higher frequency of eye
movements is associated with REM sleep and healthy brain development.
104
C.BY: HOSSAM HAMDY MATERNITY RNPEDIA – ‫موسوعه التمريض‬

46. Question

1 point(s)

When PROM occurs, which of the following provides evidence of the


nurse’s understanding of the client’s immediate needs?

A. The chorion and amnion rupture 4 hours before the onset of labor.

B. PROM removes the fetus's most effective defense against infection.

C. Nursing care is based on fetal viability and gestational age.

D. PROM is associated with malpresentation and possibly incompetent


cervix.

Answer: B. PROM removes the fetus most effective defense against


infection

PROM can precipitate many potential and actual problems; one of the most
serious is the fetus loss of an effective defense against infection. This is the
client’s most immediate need at this time.

Option A: Typically, PROM occurs about 1 hour, not 4 hours, before labor
begins.

Option C: Fetal viability and gestational age are less immediate


considerations that affect the plan of care.

Option D: Malpresentation and an incompetent cervix may be causes of


PROM.
47. Question

1 point(s)

Which of the following factors is the underlying cause of dystocia?

A. Nutritional

B. Mechanical

C. Environmental

105
C.BY: HOSSAM HAMDY MATERNITY RNPEDIA – ‫موسوعه التمريض‬

D. Medical

Answer: B. Mechanical

Dystocia is difficult, painful, prolonged labor due to mechanical factors


involving the fetus (passenger), uterus (powers), pelvis (passage), or psyche.

Option A: Recognizing the causes of obstructed labor is important if the


complications are to be prevented. Adequate prevention, however, can be
achieved only through a multidisciplinary approach aimed in the short term
at identifying high-risk cases and in the long term at improving nutrition.
Early motherhood should be discouraged, and efforts are needed to improve
nutrition during infancy, childhood, early adulthood, and pregnancy.

Option C: Dystocia is a complex disorder of poor uterine action that is


influenced by a significant genetic component as well as environmental
factors. The amount of genetic influence makes it interesting to study the
gene expression in these patients. Detection of the genes related to dystocia
might lead to better pathophysiological understanding of this condition and
detection of these mothers before parturition.

Option D: A number of researchers have regarded factors such as mother’s


age, height, weight before pregnancy, body mass index (BMI), weight gain
during pregnancy, fundal height, birth weight, and foot length of the mother
as risk factors.
48. Question

1 point(s)

When uterine rupture occurs, which of the following would be the priority?

A. Limiting hypovolemic shock.

B. Obtaining blood specimens.

C. Instituting complete bed rest.

D. Inserting a urinary catheter.

Answer: A. Limiting hypovolemic shock

106
C.BY: HOSSAM HAMDY MATERNITY RNPEDIA – ‫موسوعه التمريض‬

With uterine rupture, the client is at risk for hypovolemic shock. Therefore,
the priority is to prevent and limit hypovolemic shock. Immediate steps
should include giving oxygen, replacing lost fluids, providing drug therapy
as needed, evaluating fetal responses, and preparing for surgery.

Option B: Obtaining blood specimens can be done once the client is already
in a stable condition.

Option C: Complete bed rest is applicable for the patient who has uterine
rupture. A pregnant uterus after laparoscopic adenomyomectomy might
rupture easily by rather weak and short uterine contractions. Furthermore,
uterine contractions followed by uterine bleeding might be useful for the
diagnosis of uterine rupture. When uterine contractions are followed by
uterine bleeding in pregnant women that have had a prior
adenomyomectomy, this must be considered a potential sign of uterine
rupture.

Option D: Inserting a urinary catheter is necessary for preparation for


surgery to remedy the rupture.
49. Question

1 point(s)

Which of the following is the nurse’s initial action when umbilical cord
prolapse occurs?

A. Begin monitoring maternal vital signs and FHR.

B. Place the client in a knee-chest position in bed.

C. Notify the physician and prepare the client for delivery.

D. Apply a sterile warm saline dressing to the exposed cord.

Answer: B. Place the client in a knee-chest position in bed

The immediate priority is to minimize pressure on the cord. Thus the nurse’s
initial action involves placing the client on bed rest and then placing the
client in a knee-chest position or lowering the head of the bed and elevating
the maternal hips on a pillow to minimize the pressure on the cord.

Option A: Monitoring maternal vital signs and FHR is important, but it does
not have an effect on minimizing the pressure on the cord.
107
C.BY: HOSSAM HAMDY MATERNITY RNPEDIA – ‫موسوعه التمريض‬

Option C: Before notifying the physician, the nurse must do her


independent nursing intervention to minimize the pressure on the cord.

Option D: Applying sterile, warm saline dressing on the exposed cord


prevents its drying out and the growth of infectious organisms, however, it
does not minimize cord pressure.
50. Question

1 point(s)

Which of the following amounts of blood loss following birth marks the
criterion for describing postpartum hemorrhage?

A. More than 200 ml

B. More than 300 ml

C. More than 400 ml

D. More than 500 ml

Answer: D. More than 500 ml

Postpartum hemorrhage is defined as blood loss of more than 500 ml


following birth. Any amount less than this is not considered postpartum
hemorrhage.

Option A: The amount of 200 ml is less than the amount considered as


postpartum hemorrhage. Postpartum hemorrhage (PPH) is the leading cause
of maternal mortality. All women who carry a pregnancy beyond 20 weeks’
gestation are at risk for PPH and its sequelae. Although maternal mortality
rates have declined greatly in the developed world, PPH remains a leading
cause of maternal mortality elsewhere.

Option B: 300 ml is less than the amount considered as postpartum


hemorrhage. PPH is defined as blood loss of more than 500 mL following
vaginal delivery or more than 1000 mL following cesarean delivery. A loss
of these amounts within 24 hours of delivery is termed early or primary PPH,
whereas such losses are termed late or secondary PPH if they occur 24 hours
after delivery.

Option C: At term, the estimated blood flow to the uterus is 500-800


mL/min, which constitutes 10-15% of cardiac output. Most of this flow
108
C.BY: HOSSAM HAMDY MATERNITY RNPEDIA – ‫موسوعه التمريض‬

traverses the low-resistance placental bed. The uterine blood vessels that
supply the placental site traverse a weave of myometrial fibers. As these
fibers contract following delivery, myometrial retraction occurs. Retraction
is the unique characteristic of the uterine muscle to maintain its shortened
length following each successive contraction. The blood vessels are
compressed and kinked by this crisscross latticework, and, normally, blood
flow is quickly occluded. This arrangement of muscle bundles has been
referred to as the “living ligatures” or “physiologic sutures” of the uterus.
51. Question

1 point(s)

Which of the following is the primary predisposing factor related to mastitis?

A. Epidemic infection from nosocomial sources localizing in the lactiferous


glands and ducts.

B. Endemic infection occurring randomly and localizing in the peri glandular


connective tissue.

C. Temporary urinary retention due to decreased perception of the urge to


avoid.

D. Breast injury caused by overdistention, stasis, and cracking of the nipples.

Answer: D. Breast injury caused by overdistention, stasis, and cracking


of the nipples

With mastitis, injury to the breast, such as overdistention, stasis, and


cracking of the nipples, is the primary predisposing factor.

Option A: If a breast doesn’t completely empty at feedings, one of the milk


ducts can become clogged. The blockage causes milk to back up, leading to
breast infection.

Option B: Bacteria from your skin’s surface and baby’s mouth can enter the
milk ducts through a crack in the skin of your nipple or through a milk duct
opening. Stagnant milk in a breast that isn’t emptied provides a breeding
ground for the bacteria.

109
C.BY: HOSSAM HAMDY MATERNITY RNPEDIA – ‫موسوعه التمريض‬

Option C: Temporary urinary retention due to decreased perception of the


urge to void is a contributory factor to the development of urinary tract
infection, not mastitis.
52. Question

1 point(s)

Which of the following best describes thrombophlebitis?

A. Inflammation and clot formation that result when blood components


combine to form an aggregate body.

B. Inflammation and blood clots that eventually become lodged within the
pulmonary blood vessels.

C. Inflammation and blood clots that eventually become lodged within the
femoral vein.

D. Inflammation of the vascular endothelium with clot formation on the


vessel wall.

Answer: D. Inflammation of the vascular endothelium with clot


formation on the vessel wall

Thrombophlebitis refers to an inflammation of the vascular endothelium


with clot formation on the wall of the vessel.

Option A: Blood components combining to form an aggregate body


describe a thrombus or thrombosis

Option B: Clots lodging in the pulmonary vasculature refers to pulmonary


embolism. Pulmonary embolism usually arises from a thrombus that
originates in the deep venous system of the lower extremities; however, it
rarely also originates in the pelvic, renal, upper extremity veins, or the right
heart chambers

Option C: The femoral vein runs along the inside of the legs from the groin
area downward. Femoral vein thrombosis refers to a blood clot present in
those veins. These veins are superficial, or close to the surface of the skin,
and are often more prone to blood clots than deeper veins.
53. Question

110
C.BY: HOSSAM HAMDY MATERNITY RNPEDIA – ‫موسوعه التمريض‬

1 point(s)

Which of the following assessment findings would the nurse expect if the
client develops DVT?

A. Mid Calf pain, tenderness, and redness along the vein.

B. Chills, fever, malaise, occurring 2 weeks after delivery.

C. Muscle pain, the presence of Homans sign, and swelling in the affected
limb.

D. Chills, fever, stiffness, and pain occurring 10 to 14 days after delivery.

Answer: C. Muscle pain the presence of Homans sign, and swelling in


the affected limb

Classic symptoms of DVT include muscle pain, the presence of Homans


sign, and swelling of the affected limb.

Option A: Midcalf pain, tenderness, and redness, along the vein reflect
superficial thrombophlebitis. In the absence of a triggering event, neither
venous stasis nor abnormal coagulability alone causes clinically important
thrombosis, but vascular endothelial injury does reliably result in thrombus
formation. The initiating injury triggers an inflammatory response that
results in immediate platelet adhesion at the injury site. Further platelet
aggregation is mediated by thromboxane A2 (TxA2) and by thrombin.

Option B: Chills, fever, and malaise occurring 2 weeks after delivery reflect
pelvic thrombophlebitis. The body naturally produces more clotting proteins
during pregnancy. This ensures that the blood forms clots quickly after
delivery to avoid excess bleeding. These natural changes are meant to protect
you from complications during your pregnancy. But they also increase your
risk of having a blood clot. Any medical procedure, including delivery of a
baby, also carries a risk of infection. Septic pelvic vein thrombophlebitis is
caused when a blood clot forms in the pelvic veins and becomes infected by
bacteria present in the uterus.

Option D: Chills, fever, stiffness, and pain occurring 10 to 14 days after


delivery suggest femoral thrombophlebitis. The femoral vein runs along the
inside of the legs from the groin area downward. Femoral vein thrombosis
refers to a blood clot present in those veins. These veins are superficial, or

111
C.BY: HOSSAM HAMDY MATERNITY RNPEDIA – ‫موسوعه التمريض‬

close to the surface of the skin, and are often more prone to blood clots than
deeper veins.
54. Question

1 point(s)

Which of the following are the most commonly assessed findings in cystitis?

A. Frequency, urgency, dehydration, nausea, chills, and flank pain.

B. Nocturia, frequency, urgency, dysuria, hematuria, fever, and suprapubic


pain.

C. Dehydration, Hypertension, dysuria, suprapubic pain, chills, and fever.

D. High fever, chills, flank pain nausea, vomiting, dysuria, and frequency.

Answer: B. Nocturia, frequency, urgency dysuria, hematuria, fever and


suprapubic pain

Manifestations of cystitis include frequency, urgency, dysuria, hematuria


nocturia, fever, and suprapubic pain. Based on a systematic review
examining history and examination findings of women with uncomplicated
UTI, the combination of dysuria and urinary frequency in the absence of
vaginal discharge or irritation is highly predictive of uncomplicated cystitis.
Symptoms may be subtle or atypical in the very young and the very old.
Elderly patients with UTI may present with confusion or altered mental
status.

Option A: Cystitis refers to infection of the lower urinary tract, or more


specifically the urinary bladder. It may be broadly categorized as either
uncomplicated or complicated. Uncomplicated cystitis refers to lower
urinary tract infection (UTI) in either men or non-pregnant women who are
otherwise healthy. Complicated cystitis, on the other hand, is associated with
risk factors that increase the risk of infection or the risk of failing antibiotic
therapy.

Option C: Cystitis usually develops due to the colonization of the


periurethral mucosa by bacteria from the fecal or vaginal flora and ascension
of such pathogens to the urinary bladder. Uropathogens may have microbial
virulence factors that allow them to escape host defenses and invade host
tissues in the urinary tract. UTI in males is much less common due to the
112
C.BY: HOSSAM HAMDY MATERNITY RNPEDIA – ‫موسوعه التمريض‬

longer anatomic urethra and antibacterial defenses provided by the prostatic


fluid.

Option D: High fever chills, flank pain, nausea, vomiting, dysuria, and
frequency are associated with pyelonephritis. Acute pyelonephritis is a
bacterial infection causing inflammation of the kidneys and is one of the
most common diseases of the kidney. Pyelonephritis occurs as a
complication of an ascending urinary tract infection (UTI) which spreads
from the bladder to the kidneys and their collecting systems.
55. Question

1 point(s)

Which of the following best reflects the frequency of reported postpartum


“blues”?

A. Between 10% and 40% of all new mothers report some form of
postpartum blues.

B. Between 30% and 50% of all new mothers report some form of
postpartum blues.

C. Between 50% and 80% of all new mothers report some form of
postpartum blues.

D. Between 25% and 70% of all new mothers report some form of
postpartum blues.

Answer: C. Between 50% and 80% of all new mothers report some form
of postpartum blues

According to statistical reports, between 50% and 80% of all new mothers
report some form of postpartum blues. The ranges of 10% to 40%, 30% to
50%, and 25% to 70% are in .

Option A: Postpartum blues, also known as “baby blues,” affect


approximately 50% to 80% of new mothers. Symptoms may include mood
swings with times of feeling anxious, irritable, or tearful interspersed with
times of feeling well. Sleeping difficulties may also occur.

Option B: The symptoms usually begin 3-4 days after delivery, worsen by
days 5-7, and tend to resolve by day 12. For symptoms that last longer than 2
113
C.BY: HOSSAM HAMDY MATERNITY RNPEDIA – ‫موسوعه التمريض‬

weeks, it is important for the individual to seek medical attention, since


approximately 1 in 5 women with postpartum blues develops postpartum
major depression.

Option D: Symptoms peak on the fourth or fifth day after delivery and last
for several days, but they are generally time-limited and spontaneously remit
within the first 2 postpartum weeks. Symptoms do not interfere with a
mother’s ability to function and to care for her child.
56. Question

1 point(s)

For the client who is using oral contraceptives, the nurse informs the client
about the need to take the pill at the same time each day to accomplish which
of the following?

A. Decrease the incidence of nausea.

B. Maintain hormonal levels.

C. Reduce side effects.

D. Prevent drug interactions.

Answer: B. Maintain hormonal levels

Regular timely ingestion of oral contraceptives is necessary to maintain


hormonal levels of the drugs to suppress the action of the hypothalamus and
anterior pituitary leading to inappropriate secretion of FSH and LH.
Therefore, follicles do not mature, ovulation is inhibited, and pregnancy is
prevented.

Option A: The estrogen content of the oral site contraceptive may cause
nausea, regardless of when the pill is taken. Nausea can be avoided by taking
the medication at night before sleep.

Option C: Most side effects of OCP’s are mild and disappear with continued
use or switching to another pill formulation. The most common adverse
effect of combined oral contraceptive pills is breakthrough bleeding. Women
will also complain of nausea, headaches, abdominal cramping, breast
tenderness, and an increase in vaginal discharge or decreased libido.

114
C.BY: HOSSAM HAMDY MATERNITY RNPEDIA – ‫موسوعه التمريض‬

Option D: If the patient has medical conditions that put them at increased
risk for taking combined OC’s then there exist many alternatives to provide
pregnancy prevention. If a patient takes too many oral contraceptive pills at
one time the most likely complications will be severe headaches and nausea
or vomiting. There is no antidote to treat this condition, just treatment of the
symptoms with antiemetics and analgesics.
57. Question

1 point(s)

When teaching a client about contraception. Which of the following would


the nurse include as the most effective method for preventing sexually
transmitted infections?

A. Spermicides

B. Diaphragm

C. Condoms

D. Vasectomy

Answer: C. Condoms

Condoms, when used ly and consistently, are the most effective


contraceptive method or barrier against bacterial and viral sexually
transmitted infections.

Option A: Although spermicides kill sperm, they do not provide reliable


protection against the spread of sexually transmitted infections, especially
intracellular organisms such as HIV.

Option B: Insertion and removal of the diaphragm along with the use of the
spermicides may cause vaginal irritations, which could place the client at
risk for infection transmission.

Option D: Male sterilization eliminates spermatozoa from the ejaculate, but


it does not eliminate bacterial and/or viral microorganisms that can cause
sexually transmitted infections.
58. Question

1 point(s)
115
C.BY: HOSSAM HAMDY MATERNITY RNPEDIA – ‫موسوعه التمريض‬

When preparing a woman who is 2 days postpartum for discharge,


recommendations for which of the following contraceptive methods would
be avoided?

A. Diaphragm

B. Female condom

C. Oral contraceptives

D. Rhythm method

Answer: A. Diaphragm

The diaphragm must be fitted individually to ensure effectiveness. Because


of the changes to the reproductive structures during pregnancy and following
delivery, the diaphragm must be refitted, usually at the 6 weeks’ examination
following childbirth or after a weight loss of 15 lbs or more. In addition, for
maximum effectiveness, the spermicidal jelly should be placed in the dome
and around the rim. However, the spermicidal jelly should not be inserted
into the vagina until involution is completed at approximately 6 weeks.

Option B: Use of a female condom protects the reproductive system from


the introduction of semen or spermicides into the vagina and may be used
after childbirth.

Option C: Oral contraceptives may be started within the first postpartum


week to ensure suppression of ovulation.

Option D: For the couple who has determined the female’s fertile period,
using the rhythm method, avoidance of intercourse during this period, is safe
and effective.
59. Question

1 point(s)

For which of the following clients would the nurse expect that an intrauterine
device would not be recommended?

A. Woman over age 35

B. Nulliparous woman

116
C.BY: HOSSAM HAMDY MATERNITY RNPEDIA – ‫موسوعه التمريض‬

C. Promiscuous young adult

D. Postpartum client

Answer: C. Promiscuous young adult

An IUD may increase the risk of pelvic inflammatory disease, especially in


women with more than one sexual partner, because of the increased risk of
sexually transmitted infections. An IUD should not be used if the woman has
an active or chronic pelvic infection, postpartum infection, endometrial
hyperplasia or carcinoma, or uterine abnormalities.

Option A: Age is not a factor in determining the risks associated with IUD
use. Most IUD users are over the age of 30.

Option B: Although there is a slightly higher risk for infertility in women


who have never been pregnant, the IUD is an acceptable option as long as
the risk-benefit ratio is discussed.

Option D: IUDs may be inserted immediately after delivery, but this is not
recommended because of the increased risk and rate of expulsion at this
time.
60. Question

1 point(s)

A client in her third trimester tells the nurse, “I’m constipated all the time!”
Which of the following should the nurse recommend?

A. Daily enemas

B. Laxatives

C. Increased fiber intake

D. Decreased fluid intake

Answer: C. Increased fiber intake

During the third trimester, the enlarging uterus places pressure on the
intestines. This coupled with the effect of hormones on smooth muscle
117
C.BY: HOSSAM HAMDY MATERNITY RNPEDIA – ‫موسوعه التمريض‬

relaxation causes decreased intestinal motility (peristalsis). Increasing fiber


in the diet will help fecal matter pass more quickly through the intestinal
tract, thus decreasing the amount of water that is absorbed. As a result, the
stool is softer and easier to pass.

Option A: Enemas could precipitate preterm labor and electrolyte loss and
should be avoided.

Option B: Laxatives may cause preterm labor by stimulating peristalsis and


may interfere with the absorption of nutrients. Use for more than 1 week can
also lead to laxative dependency.

Option D: Liquid in the diet helps provide a semisolid, soft consistency to


the stool. Eight to ten glasses of fluid per day are essential to maintain
hydration and promote stool evacuation
61. Question

1 point(s)

Which of the following would the nurse use as the basis for the teaching plan
when caring for a pregnant teenager concerned about gaining too much
weight during pregnancy?

A. 10 pounds per trimester.

B. 1 pound per week for 40 weeks.

C. ½ pound per week for 40 weeks.

D. A total gain of 25 to 30 pounds.

Answer: D. A total gain of 25 to 30 pounds

To ensure adequate fetal growth and development during the 40 weeks of a


pregnancy, a total weight gain of 25 to 30 pounds is recommended:

Option A: 1.5 pounds in the first 10 weeks; 9 pounds by 30 weeks; and 27.5
pounds by 40 weeks. The pregnant woman should gain less weight in the
first and second trimester than in the third.

Option B: During the first trimester, the client should only gain 1.5 pounds
in the first 10 weeks, not 1 pound per week.

118
C.BY: HOSSAM HAMDY MATERNITY RNPEDIA – ‫موسوعه التمريض‬

Option C: A weight gain of ½ pound per week would be 20 pounds for the
total pregnancy, less than the recommended amount.
62. Question

1 point(s)

The client tells the nurse that her last menstrual period started on January 14
and ended on January 20. Using Nagele’s rule, the nurse determines her
EDD to be which of the following?

A. September 27

B. October 21

C. November 7

D. December 27

Answer: B. October 21

To calculate the EDD by Nagele’s rule, add 7 days to the first day of the last
menstrual period and count back 3 months, changing the year appropriately.

Option A: To obtain a date of September 27, 7 days have been added to the
last day of the LMP (rather than the first day of the LMP), plus 4 months
(instead of 3 months) were counted back.

Option C: To obtain the date of November 7, 7 days have been subtracted


(instead of added) from the first day of LMP plus November indicates
counting back 2 months (instead of 3 months) from January.

Option D: To obtain the date of December 27, 7 days were added to the last
day of the LMP (rather than the first day of the LMP) and December
indicates counting back only 1 month (instead of 3 months) from January.
63. Question

1 point(s)

When taking an obstetrical history on a pregnant client who states, “I had a


son born at 38 weeks gestation, a daughter born at 30 weeks gestation and I
lost a baby at about 8 weeks,” the nurse should record her obstetrical history
as which of the following?
119
C.BY: HOSSAM HAMDY MATERNITY RNPEDIA – ‫موسوعه التمريض‬

A. G2 T2 P0 A0 L2

B. G3 T1 P1 A0 L2

C. G3 T2 P0 A0 L2

D. G4 T1 P1 A1 L2

Answer: D. G4 T1 P1 A1 L2

The client has been pregnant four times, including current pregnancy (G).
Birth at 38 weeks’ gestation is considered full term (T), while birth form 20
weeks to 38 weeks is considered preterm (P). A spontaneous abortion
occurred at 8 weeks (A). She has two living children (L).

Option A: Gravidity (G) is the number of times a woman has been pregnant,
regardless of the outcome. G should be 4 times. T should be 1 since she has
carried one full-term birth. P should be 1 since she has one preterm birth. A
should be 1 since she has one abortion.

Option B: Parity (P) is the total number of times a woman has given birth to
a child with a gestational age of 24 weeks or more, regardless of whether the
child was born alive or not (stillbirth). G should be 4 since she has been
pregnant 4 times. A should be 1 since she had one abortion.

Option C: G should be 4 times. T should be 1 since she has carried one full-
term birth. P should be 1 since she has one preterm birth. A should be 1 since
she has one abortion.
64. Question

1 point(s)

When preparing to listen to the fetal heart rate at 12 weeks’ gestation, the
nurse would use which of the following?

A. Stethoscope placed midline at the umbilicus.

B. Doppler placed midline at the suprapubic region.

C. Fetoscope placed midway between the umbilicus and the xiphoid process.

D. External electronic fetal monitor placed at the umbilicus.

120
C.BY: HOSSAM HAMDY MATERNITY RNPEDIA – ‫موسوعه التمريض‬

Answer: B. Doppler placed midline at the suprapubic region

At 12 weeks gestation, the uterus rises out of the pelvis and is palpable
above the symphysis pubis. The Doppler intensifies the sound of the fetal
pulse rate so it is audible. The uterus has merely risen out of the pelvis into
the abdominal cavity and is not at the level of the umbilicus.

Option A: The fetal heart rate at this age is not audible with a stethoscope.
Exciting circulatory developments continue at 12 weeks when baby-to-be’s
bone marrow begins busily producing blood cells. By 17 weeks, the fetal
brain begins to regulate the heartbeat in preparation for supporting a baby in
the outside world. (Up until this point, the heart has been beating
spontaneously.) In three more weeks, by around week 20, the mother may
hear her baby’s heartbeat with a stethoscope.

Option C: The uterus at 12 weeks is just above the symphysis pubis in the
abdominal cavity, not midway between the umbilicus and the xiphoid
process. At 12 weeks the FHR would be difficult to auscultate with a
fetoscope. A fetoscope, or a fetal stethoscope, works much like a regular
stethoscope except that it has a bell-shaped end that magnifies sound waves
from the fetal heartbeat in order to make them audible. One can usually hear
a fetal heartbeat with the stethoscope starting around 20 weeks of pregnancy.

Option D: Although the external electronic fetal monitor would project the
FHR, the uterus has not risen to the umbilicus at 12 weeks.
65. Question

1 point(s)

When developing a plan of care for a client newly diagnosed with gestational
diabetes, which of the following instructions would be the priority?

A. Dietary intake

B. Medication

C. Exercise

D. Glucose monitoring

Answer: A. Dietary intake


121
C.BY: HOSSAM HAMDY MATERNITY RNPEDIA – ‫موسوعه التمريض‬

Although all of the choices are important in the management of diabetes, diet
therapy is the mainstay of the treatment plan and should always be the
priority. The goal of dietary therapy is to avoid single large meals and foods
with a large percentage of simple carbohydrates.

Option B: Women diagnosed with gestational diabetes generally need only


diet therapy without medication to control their blood sugar levels. A total of
6 feedings per day is preferred, with 3 major meals and 3 snacks to limit the
amount of energy intake presented to the bloodstream at any interval.

Option C: Exercise, is important for all pregnant women and especially for
diabetic women, because it burns up glucose, thus decreasing blood sugar.
However, dietary intake, not exercise, is the priority. The diet should include
foods with complex carbohydrates and cellulose, such as whole-grain bread
and legumes.

Option D: All pregnant women with diabetes should have periodic


monitoring of serum glucose. However, those with gestational diabetes
generally do not need daily glucose monitoring. The standard of care
recommends a fasting and 2-hour postprandial blood sugar level every 2
weeks.
66. Question

1 point(s)

A client at 24 weeks gestation has gained 6 pounds in 4 weeks. Which of the


following would be the priority when assessing the client?

A. Glucosuria

B. Depression

C. Hand/face edema

D. Dietary intake

Answer: C. Hand/face edema

After 20 weeks’ gestation, when there is a rapid weight gain, preeclampsia


should be suspected, which may be caused by fluid retention manifested by
edema, especially of the hands and face. The three classic signs of
preeclampsia are hypertension, edema, and proteinuria.
122
C.BY: HOSSAM HAMDY MATERNITY RNPEDIA – ‫موسوعه التمريض‬

Option A: Although urine is checked for glucose at each clinic visit, this is
not the priority. Routine dipstick screening for protein and glucose at each
prenatal visit should be abandoned. Women who are known or perceived to
be at high risk for gestational diabetes or preeclampsia should continue to be
monitored closely at the discretion of their clinician.

Option B: Depression may cause either anorexia or excessive food intake,


leading to excessive weight gain or loss. This is not, however, the priority
consideration at this time. About 6% of women, including up to 10% (one in
10) of women who are pregnant, will experience depression at some time
during their lives.

Option D: Weight gain thought to be caused by excessive food intake would


require a 24-hour diet recall. However, excessive intake would not be the
primary consideration for this client at this time. High gestational weight
gain (GWG) is related to several complications in mothers, such as
hypertension, diabetes, pre-eclampsia, macrosomia, and maternal weight
retention postpartum. In addition, GWG may also influence body
composition in childhood and later life. Studies have also suggested that
excessive GWG is associated with higher fat mass in childhood and greater
body mass index (BMI) and fat mass in later adulthood.
67. Question

1 point(s)

A client 12 weeks’ pregnant came to the emergency department with


abdominal cramping and moderate vaginal bleeding. Speculum examination
reveals 2 to 3 cm cervical dilation.The nurse would document these findings
as which of the following?

A. Threatened abortion

B. Imminent abortion

C. Complete abortion

D. Missed abortion

Answer: B. Imminent abortion

123
C.BY: HOSSAM HAMDY MATERNITY RNPEDIA – ‫موسوعه التمريض‬

Cramping and vaginal bleeding coupled with cervical dilation signify that
termination of the pregnancy is inevitable and cannot be prevented. Thus, the
nurse would document an imminent abortion.

Option A: In a threatened abortion, cramping and vaginal bleeding are


present, but there is no cervical dilation. The symptoms may subside or
progress to abortion.

Option C: In a complete abortion all the products of conception are


expelled.

Option D: A missed abortion is early fetal intrauterine death without


expulsion of the products of conception.
68. Question

1 point(s)

Which of the following would be the priority nursing diagnosis for a client
with an ectopic pregnancy?

A. Risk for infection

B. Pain

C. Knowledge Deficit

D. Anticipatory Grieving

Answer: B. Pain

For the client with an ectopic pregnancy, lower abdominal pain, usually
unilateral, is the primary symptom. Thus, pain is the priority.

Option A: Although the potential for infection is always present, the risk is
low in ectopic pregnancy because pathogenic microorganisms have not been
introduced from external sources.

Option C: The client may have limited knowledge of the pathology and
treatment of the condition. The mechanisms responsible for ectopic
implantation are unknown. The four main possibilities are anatomic
obstruction to the passage of the zygote, an abnormal conceptus,
abnormalities in the mechanisms responsible for tubal motility, and
transperitoneal migration of the zygote.
124
C.BY: HOSSAM HAMDY MATERNITY RNPEDIA – ‫موسوعه التمريض‬

Option D: By far the most common emotional reaction after having an


ectopic pregnancy is finding oneself suddenly overcome with intense
emotions of reliving some aspects of the diagnosis and treatment of the
ectopic pregnancy when the woman did not want to. She may also get
palpitations, or feel anxious or agitated when reminded of the ectopic
pregnancy. These are called flashbacks. She may experience nightmares or
bad dreams and have a sense of being “on edge”, irritable, or more anxious.
Some women also experience a sense of being detached and numb and that
the ectopic pregnancy has changed them in some negative way.
69. Question

1 point(s)

Before assessing the postpartum client’s uterus for firmness and position in
relation to the umbilicus and midline, which of the following should the
nurse do first?

A. Assess the vital signs.

B. Administer analgesia.

C. Ambulate her in the hall.

D. Assist her to urinate.

Answer: D. Assist her to urinate

Before the uterine assessment is performed, it is essential that the woman


empties her bladder. A full bladder will interfere with the accuracy of the
assessment by elevating the uterus and displacing to the side of the midline.

Option A: Vital sign assessment is not necessary unless an abnormality in


uterine assessment is identified. Immediately postpartum, check the mother’s
vital signs every hour for 4 hours. Once she is stable, monitor every 4 hours.

Option B: Uterine assessment should not cause acute pain that requires
administration of analgesia. By approximately one hour post-delivery, the
fundus is firm and at the level of the umbilicus. The fundus continues to
descend into the pelvis at the rate of approximately 1 cm or finger-breadth
per day and should be nonpalpable by 14 days postpartum.

125
C.BY: HOSSAM HAMDY MATERNITY RNPEDIA – ‫موسوعه التمريض‬

Option C: Ambulating the client is an essential component of postpartum


care but is not necessary prior to assessment of the uterus. Since it is not
justified anymore to keep healthy women in bed after giving birth to a child,
most studies on early ambulation are performed with pregnant women who
have to remain in bed for diseases or complications. A small increased risk
of venous thrombosis of extended bed rest has been found. However, similar
to studies performed in the beginning of the twentieth century, women who
are obligatorily bedridden most often have a poorer health status, which
results in a higher risk of venous thrombosis, compared to those who are
allowed to leave the bed at an early stage.
70. Question

1 point(s)

Which of the following should the nurse do when a primipara who is


lactating tells the nurse that she has sore nipples?

A. Tell her to breastfeed more frequently.

B. Administer a narcotic before breastfeeding.

C. Encourage her to wear a nursing brassiere.

D. Use soap and water to clean the nipples.

Answer: A. Tell her to breastfeed more frequently

Feeding more frequently, about every 2 hours, will decrease the infant’s
frantic, vigorous sucking from hunger and will decrease breast engorgement,
soften the breast, and promote ease of latching on for feeding.

Option B: Narcotics administered prior to breastfeeding are passed through


the breast milk to the infant, causing excessive sleepiness. Nipple soreness is
not severe enough to warrant narcotic analgesia.

Option C: All postpartum clients, especially lactating mothers, should wear


a supportive brassiere with wide cotton straps. They can improve posture and
prevent a lot of spine problems and back pain during breastfeeding. This
does not, however, prevent or reduce nipple soreness.

126
C.BY: HOSSAM HAMDY MATERNITY RNPEDIA – ‫موسوعه التمريض‬

Option D: Soaps are drying to the skin of the nipples and should not be used
on the breasts of lactating mothers. Dry nipple skin predisposes to cracks and
fissures, which can become sore and painful.
71. Question

1 point(s)

The nurse assesses the vital signs of a client, 4 hours’ postpartum that are as
follows: BP 90/60; temperature 100.4ºF; pulse 100 weak, thready; R 20 per
minute. Which of the following should the nurse do first?

A. Report the temperature to the physician.

B. Recheck the blood pressure with another cuff.

C. Assess the uterus for firmness and position.

D. Determine the amount of lochia.

Answer: D. Determine the amount of lochia

A weak, thready pulse elevated to 100 BPM may indicate impending


hemorrhagic shock. An increased pulse is a compensatory mechanism of the
body in response to decreased fluid volume. Thus, the nurse should check
the amount of lochia present.

Option A: Temperatures up to 100.48F in the first 24 hours after birth are


related to the dehydrating effects of labor and are considered normal.

Option B: Although rechecking the blood pressure may be a choice of


action; it is not the first action that should be implemented in light of the
other data. The data indicate a potential impending hemorrhage.

Option C: Assessing the uterus for firmness and position in relation to the
umbilicus and midline is important, but the nurse should check the extent of
vaginal bleeding first. Then it would be appropriate to check the uterus,
which may be a possible cause of the hemorrhage.
72. Question

1 point(s)

127
C.BY: HOSSAM HAMDY MATERNITY RNPEDIA – ‫موسوعه التمريض‬

The nurse assesses the postpartum vaginal discharge (lochia) on four clients.
Which of the following assessments would warrant notification of the
physician?

A. A dark red discharge on a 2-day postpartum client.

B. A pink to brownish discharge on a client who is 5 days postpartum.

C. Almost colorless to creamy discharge on a client 2 weeks after delivery.

D. A bright red discharge 5 days after delivery.

Answer: D. A bright red discharge 5 days after delivery

Any bright red vaginal discharge would be considered abnormal, but


especially 5 days after delivery, when the lochia is typically pink to
brownish. Lochia rubra, a dark red discharge, is present for 2 to 3 days after
delivery. Bright red vaginal bleeding at this time suggests late postpartum
hemorrhage, which occurs after the first 24 hours following delivery and is
generally caused by retained placental fragments or bleeding disorders.

Option A: Lochia rubra is the normal dark red discharge occurring in the
first 2 to 3 days after delivery, containing epithelial cells, erythrocyTes,
leukocytes, and decidua.

Option B: Lochia serosa is a pink to brownish serosanguineous discharge


occurring from 3 to 10 days after delivery that contains decidua,
erythrocytes, leukocytes, cervical mucus, and microorganisms.

Option C: Lochia alba is an almost colorless to yellowish discharge


occurring from 10 days to 3 weeks after delivery and containing leukocytes,
decidua, epithelial cells, fat, cervical mucus, cholesterol crystals, and
bacteria.
73. Question

1 point(s)

A postpartum client has a temperature of 101.4ºF, with a uterus that is tender


when palpated, remains unusually large, and not descending as normally
expected. Which of the following should the nurse assess next?

A. Lochia

128
C.BY: HOSSAM HAMDY MATERNITY RNPEDIA – ‫موسوعه التمريض‬

B. Breasts

C. Incision

D. Urine

Answer: A. Lochia

The data suggests an infection of the endometrial lining of the uterus. The
lochia may be decreased or copious, dark brown in appearance, and foul-
smelling, providing further evidence of a possible infection.

Option B: All the client’s data indicate a uterine problem, not a breast
problem. Typically, transient fever, usually 101ºF, may be present with
breast engorgement. Symptoms of mastitis include influenza-like
manifestations.

Option C: Localized infection of an episiotomy or C-section incision rarely


causes systemic symptoms, and uterine involution would not be affected.

Option D: The client data do not include dysuria, frequency, or urgency,


symptoms of urinary tract infections, which would necessitate assessing the
client’s urine.
74. Question

1 point(s)

Which of the following is the priority focus of nursing practice with the
current early postpartum discharge?

A. Promoting comfort and restoration of health.

B. Exploring the emotional status of the family.

C. Facilitating safe and effective self and newborn care.

D. Teaching about the importance of family planning.

Answer: C. Facilitating safe and effective self and newborn care

129
C.BY: HOSSAM HAMDY MATERNITY RNPEDIA – ‫موسوعه التمريض‬

Because of early postpartum discharge and limited time for teaching, the
nurse’s priority is to facilitate the safe and effective care of the client and
newborn.

Option A: After a vaginal birth, recovery can take anywhere from three
weeks if the woman didn’t tear to six weeks or more if she had a perineal
tear or an episiotomy. If the woman is delivered by C-section, expect to
spend the first three to four days postpartum in the hospital recovering; it
will take four to six weeks before the woman will feel back to normal.

Option B: Having a baby is a life-changing experience. Almost every mom


faces a bout of the baby blues due to a roller coaster of hormones, lack of
sleep, and the struggle to adjust to that tiny new human at home. That said, if
the woman has symptoms of postpartum depression — including feeling
persistently hopeless, sad, isolated, irritable, worthless, or anxious — for
more than two weeks postpartum, she should talk to a doctor.

Option D: Teaching about family planning is important in


postpartum/newborn nursing care, but they are not the priority focus in the
limited time presented by early postpartum discharge.
75. Question

1 point(s)

Which of the following actions would be least effective in maintaining a


neutral thermal environment for the newborn?

A. Place the infant under a radiant warmer after bathing.

B. Covering the scale with a warmed blanket prior to weighing.

C. Placing the crib close to the nursery window for family viewing.

D. Covering the infant’s head with a knit stockinette.

Answer: C. Placing the crib close to the nursery window for family
viewing.

Heat loss by radiation occurs when the infant’s crib is placed too near cold
walls or windows. Thus placing the newborn’s crib close to the viewing
window would be least effective. Body heat is lost through evaporation
during bathing.
130
C.BY: HOSSAM HAMDY MATERNITY RNPEDIA – ‫موسوعه التمريض‬

Option A: Placing the infant under the radiant warmer after bathing will
assist the infant to be rewarmed.

Option B: Covering the scale with a warmed blanket prior to weighing


prevents heat loss through conduction.

Option D: A knit cap prevents heat loss from the head, a large head, a large
body surface area of the newborn’s body.

131
C.BY: HOSSAM HAMDY MATERNITY RNPEDIA – ‫موسوعه التمريض‬

Maternity Nursing NU.3 (Quiz #1: 75 Questions)

1. Question

1 point(s)

A newborn who has an asymmetrical Moro reflex response should be further


assessed for which of the following?

A. Talipes equinovarus

B. Fractured clavicle

C. Congenital hypothyroidism

D. Increased intracranial pressure

Answer: B. Fractured clavicle

A fractured clavicle would prevent the normal Moro response of


symmetrical sequential extension and abduction of the arms followed by
flexion and adduction.

Option A: In talipes equinovarus (clubfoot) the foot is turned medially, and


in plantar flexion, with the heel elevated. The feet are not involved with the
Moro reflex.

Option C: Hypothyroidism has no effect on the primitive reflexes.


Hypothyroidism means that the thyroid gland can’t make enough thyroid
hormones to keep the body running normally.

Option D: The absence of the Moro reflex is the most significant single
indicator of central nervous system status, but it is not a sign of increased
intracranial pressure.
2. Question

1 point(s)

During the first 4 hours after a male circumcision, assessing for which of the
following is the priority?

132
C.BY: HOSSAM HAMDY MATERNITY RNPEDIA – ‫موسوعه التمريض‬

A. Infection

B. Hemorrhage

C. Discomfort

D. Dehydration

Answer: B. Hemorrhage

Hemorrhage is a potential risk following any surgical procedure. Although


the infant has been given vitamin K to facilitate clotting, the prophylactic
dose is often not sufficient to prevent bleeding.

Option A: Although infection is a possibility, signs will not appear within 4


hours after the surgical procedure.

Option C: The primary discomfort of circumcision occurs during the


surgical procedure, not afterward.

Option D: Although feedings are withheld prior to the circumcision, the


chances of dehydration are minimal.
3. Question

1 point(s)

The mother asks the nurse. “What’s wrong with my son’s breasts? Why are
they so enlarged?” Which of the following would be the best response by the
nurse?

A. “The breast tissue is inflamed from the trauma experienced with birth.”

B. “A decrease in material hormones present before birth causes


enlargement,”

C. “You should discuss this with your doctor. It could be a malignancy.”

D. “The tissue has hypertrophied while the baby was in the uterus.”

Answer: B. “A decrease in material hormones present before birth


causes enlargement,”
133
C.BY: HOSSAM HAMDY MATERNITY RNPEDIA – ‫موسوعه التمريض‬

The presence of excessive estrogen and progesterone in the maternal-fetal


blood followed by prompt withdrawal at birth precipitates breast
engorgement, which will spontaneously resolve in 4 to 5 days after birth.

Option A: The trauma of the birth process does not cause inflammation of
the newborn’s breast tissue.

Option C: Newborns do not have breast malignancy. This reply by the nurse
would cause the mother to have undue anxiety.

Option D: Breast tissue does not hypertrophy in the fetus or newborns.


4. Question

1 point(s)

Immediately after birth, the nurse notes the following on a male newborn:
respirations 78; apical heart rate 160 BPM, nostril-flaring; mild intercostal
retractions; and grunting at the end of expiration. Which of the following
should the nurse do?

A. Call the assessment data to the physician’s attention

B. Start oxygen per nasal cannula at 2 L/min.

C. Suction the infant’s mouth and nares.

D. Recognize this as a normal first period of reactivity.

Answer: D. Recognize this as normal first period of reactivity

The first 15 minutes to 1 hour after birth is the first period of reactivity
involving respiratory and circulatory adaptation to extrauterine life. The data
given reflect the normal changes during this time period.

Option A: The infant’s assessment data reflect normal adaptation. Thus, the
physician does not need to be notified.

Option B: Babies with heart or lung problems may need to breathe increased
amounts of oxygen to get normal levels of oxygen in their blood. Oxygen
therapy provides babies with extra oxygen.

134
C.BY: HOSSAM HAMDY MATERNITY RNPEDIA – ‫موسوعه التمريض‬

Option C: The data do not indicate the presence of choking, gagging, or


coughing, which are signs of excessive secretions. Suctioning is not
necessary.
5. Question

1 point(s)

The nurse hears a mother telling a friend on the telephone about umbilical
cord care. Which of the following statements by the mother indicates
effective teaching?

A. “Daily soap and water cleansing is best.”

B. “Alcohol helps it dry and kills germs.”

C. “An antibiotic ointment applied daily prevents infection.”

D. “He can have a tub bath each day.”

Answer: B. “Alcohol helps it dry and kills germs”

Application of 70% isopropyl alcohol to the cord minimizes microorganisms


(germicidal) and promotes drying. The cord should be kept dry until it falls
off and the stump has healed.

Option A: Research now shows that the stump might actually heal faster if
left alone. If the stump becomes dirty or sticky, clean it with plain water,
after which you dry it by holding a clean, absorbent cloth around it or
fanning it with a piece of paper.

Option C: Antibiotic ointment should only be used to treat an infection, not


as prophylaxis.

Option D: Infants should not be submerged in a tub of water until the cord
falls off and the stump has completely healed.
6. Question

1 point(s)

A newborn weighing 3000 grams and feeding every 4 hours needs 120
calories/kg of body weight every 24 hours for proper growth and

135
C.BY: HOSSAM HAMDY MATERNITY RNPEDIA – ‫موسوعه التمريض‬

development. How many ounces of 20 cals/oz formula should this newborn


receive at each feeding to meet nutritional needs?

A. 2 ounces

B. 3 ounces

C. 4 ounces

D. 6 ounces

Answer: B. 3 ounces

To determine the amount of formula needed, do the following mathematical


calculation. 3 kg x 120 cal/kg per day = 360 calories/day feeding q 4 hours =
6 feedings per day = 60 calories per feeding: 60 calories per feeding; 60
calories per feeding with formula 20 cal/oz = 3 ounces per feeding. Based on
the calculation. 2, 4 or 6 ounces are .

Option A: 2 ounces is less than the recommended intake. Newborns will


nurse about 8 to 12 times per day during the first weeks of life. In the
beginning, mothers may want to try nursing 10–15 minutes on each breast,
then adjust the time as necessary.

Option C: With 4 ounces, the newborn would be overfed and not meet the
adequate nutritional requirement. A baby who is fussing, crying, seems
hungry, and does not appear satisfied after feeding may not be getting
enough to eat. A baby is getting enough to eat if he or she seems satisfied,
produces about six to eight wet diapers a day, has regular bowel movements,
sleeps well, is alert when awake, and is gaining weight.

Option D: 6 ounces is too much intake for a newborn every feeding. Many
infants “spit up” a small amount after eating or during burping, but a baby
should not vomit after feeding. Vomiting after every feeding might be a sign
of an allergy, digestive problem, or other problem that needs medical
attention.
7. Question

1 point(s)

The post-term neonate with meconium-stained amniotic fluid needs care


designed to especially monitor for which of the following?
136
C.BY: HOSSAM HAMDY MATERNITY RNPEDIA – ‫موسوعه التمريض‬

A. Respiratory problems

B. Gastrointestinal problems

C. Integumentary problems

D. Elimination problems

Answer: A. Respiratory problems

Intrauterine anoxia may cause relaxation of the anal sphincter and emptying
of meconium into the amniotic fluid. At birth some of the meconium fluid
may be aspirated, causing mechanical obstruction or chemical pneumonitis.

Option B: The infant is not at increased risk for gastrointestinal problems.


Postmature infants are alert and appear mature. They have a decreased
amount of soft-tissue mass, particularly subcutaneous fat. The skin may hang
loosely on the extremities and is often dry and peeling. The fingernails and
toenails are long. The nails and umbilical cord may be stained with
meconium passed in utero.

Option C: Even though the skin is stained with meconium, it is


noninfectious (sterile) and non-irritating.

Option D: The post-term meconium-stained infant is not at additional risk


for bowel or urinary problems. Failure to pass meconium beyond 48 hours in
term neonates may indicate disease or obstruction of the infant’s bowel. The
diagnostic differential for the delayed passage of meconium includes
Hirschsprung disease, meconium plug syndrome, meconium ileus, anorectal
malformations, small left colon syndrome, and intestinal atresias.
8. Question

1 point(s)

When measuring a client’s fundal height, which of the following techniques


denotes the method of measurement used by the nurse?

A. From the xiphoid process to the umbilicus.

B. From the symphysis pubis to the xiphoid process.

C. From the symphysis pubis to the fundus.

137
C.BY: HOSSAM HAMDY MATERNITY RNPEDIA – ‫موسوعه التمريض‬

D. From the fundus to the umbilicus.

Answer: C. From the symphysis pubis to the fundus.

The nurse should use a non-elastic, flexible, paper measuring tape, placing
the zero point on the superior border of the symphysis pubis and stretching
the tape across the abdomen at the midline to the top of the fundus.

Option A: The xiphoid process is an landmark to use in measuring for


fundal height. If measured ly, the fundus at 36 weeks should be at the
xiphoid process.

Option B: Measuring from the symphysis pubis to the xiphoid process is .


If the fundal height is measured at the level of the symphysis pubis, then the
woman is at 12 weeks gestation.

Option D: The umbilicus is not an appropriate landmark to use when


measuring the height of the fundus (McDonald’s measurement). When the
fundal height is at the level of the umbilicus, the fundus is at 20 weeks
gestation.
9. Question

1 point(s)

A client with severe preeclampsia is admitted with BP 160/110, proteinuria,


and severe pitting edema. Which of the following would be most important
to include in the client’s plan of care?

A. Daily weights

B. Seizure precautions

C. Right lateral positioning

D. Stress reduction

Answer: B. Seizure precautions

Women hospitalized with severe preeclampsia need decreased CNS


stimulation to prevent a seizure. Seizure precautions provide environmental
safety should a seizure occur.
138
C.BY: HOSSAM HAMDY MATERNITY RNPEDIA – ‫موسوعه التمريض‬

Option A: Because of edema, daily weight is important but not the priority.
High pregnancy weight gain was more strongly associated with term
preeclampsia than early preterm preeclampsia (eg, 64% versus 43%
increased odds per 1 z score difference in weight gain in normal-weight
women, and 30% versus 0% in obese women, respectively).

Option C: Preeclampsia causes vasospasm and therefore can reduce


uteroplacental perfusion. The client should be placed on her left side to
maximize blood flow, reduce blood pressure, and promote diuresis.

Option D: Interventions to reduce stress and anxiety are very important to


facilitate coping and a sense of control, but seizure precautions are the
priority.
10. Question

1 point(s)

A postpartum primipara asks the nurse, “When can we have sexual


intercourse again?” Which of the following would be the nurse’s best
response?

A. “Anytime you both want to.”

B. “As soon as you choose a contraceptive method.”

C. “When the discharge has stopped, and the incision is healed.”

D. “After your 6 weeks examination.”

Answer: C. “When the discharge has stopped and the incision is


healed.”

Cessation of the lochial discharge signifies healing of the endometrium. The


risk of hemorrhage and infection are minimal 3 weeks after a normal vaginal
delivery.

Option A: Telling the client anytime is inappropriate because this response


does not provide the client with the specific information she is requesting.

Option B: Choice of a contraceptive method is important, but not the


specific criteria for safe resumption of sexual activity.

139
C.BY: HOSSAM HAMDY MATERNITY RNPEDIA – ‫موسوعه التمريض‬

Option D: Culturally, the 6-weeks’ examination has been used as the time
frame for resuming sexual activity, but it may be resumed earlier.
11. Question

1 point(s)

When preparing to administer the vitamin K injection to a neonate, the nurse


would select which of the following sites as appropriate for the injection?

A. Deltoid muscle

B. Anterior femoris muscle

C. Vastus lateralis muscle

D. Gluteus maximus muscle

Answer: C. Vastus lateralis muscle

The middle third of the vastus lateralis is the preferred injection site for
vitamin K administration because it is free of blood vessels and nerves and is
large enough to absorb the medication.

Option A: The deltoid muscle of a newborn is not large enough for a


newborn IM injection. Injections into this muscle in a small child might
cause damage to the radial nerve.

Option B: The anterior femoris muscle is the next safest muscle to use in a
newborn but is not the safest. The rectus femoris (the middle third of the
rectus femoris) is no longer a recommended site because it may cause
discomfort and pain. A previous study reported that one disadvantage of this
site is that nerves and numerous blood vessels run very close to it

Option D: Because of the proximity of the sciatic nerve, the gluteus


maximus muscle should not be until the child has been walking 2 years.
12. Question

1 point(s)

When performing a pelvic examination, the nurse observes a red swollen


area on the right side of the vaginal orifice. The nurse would document this
as enlargement of which of the following?
140
C.BY: HOSSAM HAMDY MATERNITY RNPEDIA – ‫موسوعه التمريض‬

A. Clitoris

B. Parotid gland

C. Skene’s gland

D. Bartholin’s gland

Answer: D. Bartholin’s gland

The Bartholin’s glands are located symmetrically at the posterior region of


the vaginal opening and play an important role in the female reproductive
system. These two pea-sized glands are involved in mucus secretion and
vaginal lubrication.

Option A: The clitoris is female erectile tissue found in the perineal area
above the urethra. During sexual stimulation, the clitoris functions much like
a man’s penis in that it becomes erect thanks to signals from the brain. The
shaft under the skin has erectile tissue, which becomes engorged with
increased blood flow. The clitoris is a very sensitive area when stimulated.

Option B: The parotid glands are open into the mouth. The parotid is the
largest of the three glands and is bordered superiorly by the zygomatic arch,
anteriorly and medially by the masseter, and posteriorly by the
sternocleidomastoid. The saliva secreted from these glands moistens the
mouth which helps with chewing, swallowing, phonation, and digestion.

Option C: Skene’s glands open into the posterior wall of the female urinary
meatus. The Skene’s glands, which are also known as the lesser vestibular
glands (homologous to the prostate glands in males), are two glands located
on either side of the urethra. These glands are believed to secrete a substance
to lubricate the urethra opening. This substance is also believed to act as an
antimicrobial.
13. Question

1 point(s)

To differentiate as a female, the hormonal stimulation of the embryo that


must occur involves which of the following?

A. Increase in maternal estrogen secretion

B. Decrease in maternal androgen secretion.


141
C.BY: HOSSAM HAMDY MATERNITY RNPEDIA – ‫موسوعه التمريض‬

C. Secretion of androgen by the fetal gonad.

D. Secretion of estrogen by the fetal gonad.

Answer: D. Secretion of estrogen by the fetal gonad.

The fetal gonad must secrete estrogen for the embryo to differentiate as a
female.

Option A: An increase in maternal estrogen secretion does not affect


differentiation of the embryo, and maternal estrogen secretion occurs in
every pregnancy.

Option B: Maternal androgen secretion remains the same as before


pregnancy and does not affect differentiation.

Option C: Secretion of androgen by the fetal gonad would produce a male


fetus.
14. Question

1 point(s)

A client at 8 weeks’ gestation calls complaining of slight nausea in the


morning hours. Which of the following client interventions should the nurse
question?

A. Taking 1 teaspoon of bicarbonate of soda in an 8-ounce glass of water.

B. Eating a few low-sodium crackers before getting out of bed.

C. Avoiding the intake of liquids in the morning hours.

D. Eating six small meals a day instead of three large meals.

Answer: A. Taking 1 teaspoon of bicarbonate of soda in an 8-ounce glass


of water.

Using bicarbonate would increase the amount of sodium ingested, which can
cause complications.

142
C.BY: HOSSAM HAMDY MATERNITY RNPEDIA – ‫موسوعه التمريض‬

Option B: Eating low-sodium crackers would be appropriate. Foods high in


starch — such as saltines, bread, and toast — help absorb gastric acid and
settle a queasy stomach. The bland nature of a cracker helps to satisfy hunger
(excessive hunger can exacerbate nausea) without the strong smells or tastes
that may increase nausea.

Option C: Since liquids can increase nausea avoiding them in the morning
hours when nausea is usually the strongest is appropriate.

Option D: Eating six small meals a day would keep the stomach full, which
often decreases nausea.
15. Question

1 point(s)

The nurse documents positive ballottement in the client’s prenatal record.


The nurse understands that this indicates which of the following?

A. Palpable contractions on the abdomen.

B. Passive movement of the unengaged fetus.

C. Fetal kicking felt by the client.

D. Enlargement and softening of the uterus.

Answer: B. Passive movement of the unengaged fetus.

Ballottement indicates passive movement of the unengaged fetus.


Ballottement is when the lower uterine segment or the cervix is tapped by
the examiner’s finger and left there, the fetus floats upward, then sinks back
and a gentle tap is felt on the finger.

Option A: Ballottement is not a contraction. Contractions can be felt by


placing a hand on the abdomen and feeling when the uterus becomes hard,
and when it relaxes. It is therefore possible to assess the length of a
contraction by taking the time at the beginning and end of the contraction.

Option C: Fetal kicking felt by the client represents quickening. Usually,


quickening occurs naturally at about the middle of a pregnancy. A woman
pregnant for the first time (i.e., a primigravida woman) typically feels fetal
movements at about 18–20 weeks, whereas a woman who has given birth at

143
C.BY: HOSSAM HAMDY MATERNITY RNPEDIA – ‫موسوعه التمريض‬

least once (i.e., a para woman) will typically feel movements around 15–17
weeks.

Option D: Enlargement and softening of the uterus is known as Piskacek’s


sign. The Piskacek’s sign is an indication of pregnancy. Specifically,
Piskacek’s sign consists of noting a palpable lateral bulge or soft prominence
at one of the locations where the uterine tube meets the uterus. Piskacek’s
sign can be noted in the seventh to eighth week of gestation.
16. Question

1 point(s)

During a pelvic exam, the nurse notes a purple-blue tinge of the cervix. The
nurse documents this as which of the following?

A. Braxton-Hicks sign

B. Chadwick’s sign

C. Goodell’s sign

D. McDonald’s sign

Answer: B. Chadwick’s sign

Chadwick’s sign is a bluish discoloration of the cervix, vagina, and labia


resulting from increased blood flow. It can be observed as early as 6 to 8
weeks after conception, and its presence is an early sign of pregnancy.

Option A: Braxton Hicks contractions are painless contractions beginning


around the 4th month. Braxton Hicks contractions are sporadic contractions
and relaxation of the uterine muscle. Sometimes, they are referred to as
prodromal or “false labor” pains. It is believed they start around 6 weeks
gestation but usually are not felt until the second or third trimester of the
pregnancy. Braxton Hicks contractions are the body’s way of preparing for
true labor, but they do not indicate that labor has begun or is going to start.

Option C: Goodell sign is an indication of pregnancy. It is a significant


softening of the vaginal portion of the cervix from increased vascularization.
This vascularization is a result of hypertrophy and engorgement of the
vessels below the growing uterus. This sign occurs at approximately four
weeks’ gestation.
144
C.BY: HOSSAM HAMDY MATERNITY RNPEDIA – ‫موسوعه التمريض‬

Option D: Flexibility of the uterus against the cervix is known as


McDonald’s sign.
17. Question

1 point(s)

During a prenatal class, the nurse explains the rationale for breathing
techniques during preparation for labor based on the understanding that
breathing techniques are most important in achieving which of the
following?

A. Eliminate pain and give expectant parents something to do.

B. Reduce the risk of fetal distress by increasing uteroplacental perfusion.

C. Facilitate relaxation, possibly reducing the perception of pain.

D. Eliminate pain so that less analgesia and anesthesia are needed.

Answer: C. Facilitate relaxation, possibly reducing the perception of


pain.

Breathing techniques can raise the pain threshold and reduce the perception
of pain. They also promote relaxation.

Option A: Breathing techniques do not eliminate pain, but they can reduce
it. Focused breathing might work by interrupting the transmission of pain
signals to the brain by giving the woman something positive to focus on. It
may also work by stimulating the release of endorphins, which are natural
pain-relieving hormones, and by helping her reframe her thinking about
labor so that she can view it as positive, productive, and manageable.

Option B: Positioning, not breathing, increases uteroplacental perfusion.


Side-lying position aids in better blood circulation.

Option D: Breathing techniques do not eliminate pain. Many different


childbirth classes include components of breathing in their class curriculum.
The techniques may vary but many classes today teach that breathing should
be slow, deep, controlled, and conscious or voluntary, not automatic.
18. Question

1 point(s)
145
C.BY: HOSSAM HAMDY MATERNITY RNPEDIA – ‫موسوعه التمريض‬

After 4 hours of active labor, the nurse notes that the contractions of a
primigravida client are not strong enough to dilate the cervix. Which of the
following would the nurse anticipate doing?

A. Obtaining an order to begin IV oxytocin infusion.

B. Administering a light sedative to allow the patient to rest for several hours

C. Preparing for a cesarean section for failure to progress

Answer: A. Obtaining an order to begin IV oxytocin infusion.

The client’s labor is hypotonic. The nurse should call the physical and obtain
an order for an infusion of oxytocin, which will assist the uterus to contract
more forcefully in an attempt to dilate the cervix.

Option B: Administering a light sedative would be done for hypertonic


uterine contractions.

Option C: Preparing for a cesarean section is unnecessary at this time.

Option D: Oxytocin would increase the uterine contractions and hopefully


progress labor before a cesarean would be necessary. It is too early to
anticipate client pushing with contractions.
19. Question

1 point(s)

A multigravida at 38 weeks’ gestation is admitted with painless, bright red


bleeding and mild contractions every 7 to 10 minutes. Which of the
following assessments should be avoided?

A. Maternal vital sign

B. Fetal heart rate

C. Contraction monitoring

D. Cervical dilation

Answer: D. Cervical dilation


146
C.BY: HOSSAM HAMDY MATERNITY RNPEDIA – ‫موسوعه التمريض‬

The signs indicate placenta previa and vaginal exam to determine cervical
dilation would not be done because it could cause hemorrhage.

Option A: Assessing maternal vital signs can help determine maternal


physiologic status.

Option B: Fetal heart rate is important to assess fetal well-being and should
be done.

Option C: Monitoring the contractions will help evaluate the progress of


labor.
20. Question

1 point(s)

Which of the following would be the nurse’s most appropriate response to a


client who asks why she must have a cesarean delivery if she has a complete
placenta previa?

A. “You will have to ask your physician when he returns.”

B. “You need a cesarean to prevent hemorrhage.”

C. “The placenta is covering most of your cervix.”

D. “The placenta is covering the opening of the uterus and blocking your
baby.”

Answer: D. “The placenta is covering the opening of the uterus and


blocking your baby.”

A complete placenta previa occurs when the placenta covers the opening of
the uterus, thus blocking the passageway for the baby. This response
explains what a complete previa is and the reason the baby cannot come out
except by cesarean delivery.

Option A: Telling the client to ask the physician is a poor response and
would increase the patient’s anxiety.

Option B: Although a cesarean would help to prevent hemorrhage, the


statement does not explain why the hemorrhage could occur.

147
C.BY: HOSSAM HAMDY MATERNITY RNPEDIA – ‫موسوعه التمريض‬

Option C: With a complete previa, the placenta is covering all the cervix,
not just most of it.
21. Question

1 point(s)

The nurse understands that the fetal head is in which of the following
positions with a face presentation?

A. Completely flexed

B. Completely extended

C. Partially extended

D. Partially flexed

Answer: B. Completely extended

In a face presentation, the fetal head and neck are hyperextended, causing the
occiput to come in contact with the upper back of the fetus while lying on a
longitudinal axis.

Option A: With a vertex presentation, the head is completely or partially


flexed.

Option C: With a brow (forehead) presentation, the head would be partially


extended.

Option D: Partially flexed fetal head is categorized as a sinciput


presentation.
22. Question

1 point(s)

With a fetus in the left anterior breech presentation, the nurse would expect
the fetal heart rate would be most audible in which of the following areas?

A. Above the maternal umbilicus and to the right of midline.

B. In the lower-left maternal abdominal quadrant.

148
C.BY: HOSSAM HAMDY MATERNITY RNPEDIA – ‫موسوعه التمريض‬

C. In the lower-right maternal abdominal quadrant.

D. Above the maternal umbilicus and to the left of the midline.

Answer: D. Above the maternal umbilicus and to the left of midline

With this presentation, the fetal upper torso and back face the left upper
maternal abdominal wall. The fetal heart rate would be most audible above
the maternal umbilicus and to the left of the middle. The other positions
would be .

Option A: The fetal heart rate would be most audible above the maternal
umbilicus but to the left of the midline.

Option B: It should not be at the lower-left of the maternal abdominal


quadrant. Fetal heart rate heard in this area may be inaccurate or maybe the
maternal heart rate.

Option C: This would be an inaccurate area to check for the fetal heart rate.
Since the baby is in breech position, the fetal back may be located at the
upper maternal abdominal wall.
23. Question

1 point(s)

The amniotic fluid of a client has a greenish tint. The nurse interprets this to
be the result of which of the following?

A. Lanugo

B. Hydramnios

C. Meconium

D. Vernix

Answer: C. Meconium

The greenish tint is due to the presence of meconium. Meconium is a thick,


green, tar-like substance that lines the baby’s intestines during pregnancy.
Typically this substance is not released in the baby’s bowel movements until
149
C.BY: HOSSAM HAMDY MATERNITY RNPEDIA – ‫موسوعه التمريض‬

after birth. However, sometimes a baby will have a bowel movement prior to
birth, excreting the meconium into the amniotic fluid.

Option A: Lanugo is the soft, downy hair on the shoulders and back of the
fetus. This downy, unpigmented hair is the first type of hair that grows from
hair follicles. It can be found everywhere on a baby’s body, except on the
palms, lips, and soles of the feet. Most fetuses develop lanugo around the
fourth or fifth month of pregnancy.

Option B: Hydramnios represents excessive amniotic fluid.

Option D: Vernix is the white, cheesy substance covering the fetus. It is


produced by dedicated cells and is thought to have some protective roles
during fetal development and for a few hours after birth.
24. Question

1 point(s)

A patient is in labor and has just been told she has a breech presentation. The
nurse should be particularly alert for which of the following?

A. Quickening

B. Ophthalmia neonatorum

C. Pica

D. Prolapsed umbilical cord

Answer: D. Prolapsed umbilical cord

In a breech position, because of the space between the presenting part and
the cervix, prolapse of the umbilical cord is common.

Option A: Quickening is the woman’s first perception of fetal movement.

Option B: Ophthalmia neonatorum usually results from maternal gonorrhea


and is conjunctivitis.

Option C: Pica refers to the oral intake of nonfood substances.


25. Question

150
C.BY: HOSSAM HAMDY MATERNITY RNPEDIA – ‫موسوعه التمريض‬

1 point(s)

When describing dizygotic twins to a couple, on which of the following


would the nurse base the explanation?

A. Two ova fertilized by separate sperm.

B. Sharing of a common placenta.

C. Each ova with the same genotype.

D. Sharing of a common chorion.

Answer: A. Two ova fertilized by separate sperm

Dizygotic (fraternal) twins involve two ova fertilized by separate sperm.


Monozygotic (identical) twins involve a common placenta, same genotype,
and common chorion.

Option B: Monozygotic multiples form from a single egg and sperm


combination that splits after conception. If the split is delayed for a few days,
the embryos will develop with a single, shared placenta.

Option C: Identical twins share all of their genes and are always of the same
sex.

Option D: In the majority of cases, these multiples will be enclosed within a


shared chorion (the outer layer of the sac that contains a fetus).
26. Question

1 point(s)

Which of the following refers to the single cell that reproduces itself after
conception?

A. Chromosome

B. Blastocyst

C. Zygote

D. Trophoblast

151
C.BY: HOSSAM HAMDY MATERNITY RNPEDIA – ‫موسوعه التمريض‬

Answer: C. Zygote

The zygote is the single cell that reproduces itself after conception. It is the
union of the sperm cell and the egg cell. Also known as a fertilized ovum,
the zygote begins as a single cell but divides rapidly in the days following
fertilization. After this two-week period of cell division, the zygote
eventually becomes an embryo. If this goes well, the embryo becomes a
fetus.

Option A: The chromosome is the material that makes up the cell and is
gained from each parent. Chromosomes are thread-like structures located
inside the nucleus of animal and plant cells. Each chromosome is made of
protein and a single molecule of deoxyribonucleic acid (DNA). Passed from
parents to offspring, DNA contains the specific instructions that make each
type of living creature unique.

Option B: The blastocyst consists of cells forming an outer trophectoderm


(TE, trophoblast) layer, an inner cell mass (ICM, embryo blast), and a
blastocoel (fluid-filled cavity). The inner cell mass will form the entire
embryo and is the source of true embryonic stem cells capable of forming all
cell types within the embryo.

Option D: In early development, the blastocyst outer trophectoderm (TE)


layer will generate all the extraembryonic trophoblast cell types:
cytotrophoblast, syncytiotrophoblast, trophoblastic column, and extravillous
trophoblast cells. These cells have an important contribution to extra-
embryonic tissues (fetal placenta and membranes) and processes of early
development (adplantation, implantation, and endocrine support of
pregnancy).
27. Question

1 point(s)

In the late 1950s, consumers and health care professionals began challenging
the routine use of analgesics and anesthetics during childbirth. Which of the
following was an outgrowth of this concept?

A. Labor, delivery, recovery, postpartum (LDRP)

B. Nurse-midwifery

C. Clinical nurse specialist


152
C.BY: HOSSAM HAMDY MATERNITY RNPEDIA – ‫موسوعه التمريض‬

D. Prepared childbirth

Answer: D. Prepared childbirth

Prepared childbirth was the direct result of the 1950s challenging the routine
use of analgesic and anesthetics during childbirth.

Option A: The LDRP was a much later concept and was not a direct result
of the challenging of routine use of analgesics and anesthetics during
childbirth.

Option B: A nurse-midwife is a licensed healthcare professional who


specializes in women’s reproductive health and childbirth. In addition to
attending births, they perform annual exams, give counseling, and write
prescriptions. According to the ACNM, the vast majority of midwives in the
U.S. are CNMs.

Option C: Clinical nurse specialists (CNS) are advanced practice registered


nurses (APRNs) that serve as experts in evidence-based nursing practice
within one of a number of different specialty areas. They integrate their
advanced knowledge of disease processes in assessing, diagnosing, and
treating patient illnesses, but their role extends beyond providing patient
care.
28. Question

1 point(s)

A client has a mid pelvic contracture from a previous pelvic injury due to a
motor vehicle accident as a teenager. The nurse is aware that this could
prevent a fetus from passing through or around which structure during
childbirth?

A. Symphysis pubis

B. Sacral promontory

C. Ischial spines

D. Pubic arch

Answer: C. Ischial spines


153
C.BY: HOSSAM HAMDY MATERNITY RNPEDIA – ‫موسوعه التمريض‬

The ischial spines are located in the mid-pelvic region and could be
narrowed due to the previous pelvic injury.

Option A: The pubic symphysis is a secondary cartilaginous joint (a joint


made of hyaline cartilage and fibrocartilage) located between the left and
right pubic bones near the midline of the body. More specifically, it is
located above any external genitalia and in front of the bladder.

Option B: Superiorly, there is an anterior projection of bone, known as the


sacral promontory. It forms the posterior margin of the pelvic inlet and as a
result, it is serially continuous with the margin of the ala of the sacrum,
arcuate line of the ilium, and the pectin pubis and pubic crest of the pubic
bone.

Option D: The pubic arch, also referred to as the ischiopubic arch, is part of
the pelvis. It is formed by the convergence of the inferior rami of the ischium
and pubis on either side, below the pubic symphysis. The angle at which
they converge is known as the subpubic angle.
29. Question

1 point(s)

When teaching a group of adolescents about variations in the length of the


menstrual cycle, the nurse understands that the underlying mechanism is due
to variations in which of the following phases?

A. Menstrual phase

B. Proliferative phase

C. Secretory phase

D. Ischemic phase

Answer: B. Proliferative phase

Variations in the length of the menstrual cycle are due to variations in the
proliferative phase. In addition to ovarian follicle maturation, changes also
occur in the endometrium during the first 14 days of the cycle, hence the
term ‘proliferative phase.’ The increasing concentrations of estradiol
strongly influence the endometrial changes that happen before ovulation.

154
C.BY: HOSSAM HAMDY MATERNITY RNPEDIA – ‫موسوعه التمريض‬

Option A: Menstruation is the elimination of the thickened lining of the


uterus (endometrium) from the body through the vagina. Menstrual fluid
contains blood, cells from the lining of the uterus (endometrial cells), and
mucus. The average length of a period is between three days and one week.

Option C: The secretory phase always occurs from day 14 to day 28 of the
cycle. Progesterone stimulated by LH is the dominant hormone during this
phase to prepare the corpus luteum and the endometrium for possible
fertilized ovum implantation

Option D: The final part of the secretory phase is the ischemic phase. The
endometrium becomes pale and arteries constrict due to lower hormone
release by the disintegrating corpus luteum.
30. Question

1 point(s)

When teaching a group of adolescents about male hormone production,


which of the following would the nurse include as being produced by the
Leydig cells?

A. Follicle-stimulating hormone

B. Testosterone

C. Luteinizing hormone

D. Gonadotropin-releasing hormone

Answer: B. Testosterone

Testosterone is produced by the Leydig cells in the seminiferous tubules.


The Leydig cells make and secrete testosterone, in response to luteinizing
hormone from the pituitary. This process does not start until puberty when
LH stimulates the Leydig cells to produce testosterone. FSH stimulates the
Sertoli cells to secrete androgen-binding protein into the lumen of the
seminiferous tubules.

Option A: FSH is made by the pituitary gland, a small gland located


underneath the brain. FSH plays an important role in sexual development
and functioning. In women, FSH helps control the menstrual cycle and
stimulates the growth of eggs in the ovaries.
155
C.BY: HOSSAM HAMDY MATERNITY RNPEDIA – ‫موسوعه التمريض‬

Option C: Luteinizing hormone (LH) is produced and released in the


anterior pituitary gland. This hormone is considered a gonadotropic hormone
because of its role in controlling the function of ovaries in females and testes
in males, which are known as the gonads.

Option D: The hypothalamus is responsible for releasing gonadotropin-


releasing hormone.
31. Question

1 point(s)

While performing a physical assessment of a 12 month-old, the nurse notes


that the infant’s anterior fontanel is still slightly open. Which of the
following is the nurse’s most appropriate action?

A. Notify the physician immediately because there is a problem.

B. Perform an intensive neurological examination.

C. Perform an intensive developmental examination.

D. Do nothing because this is a normal finding for the age.

Answer: D. Do nothing because this is a normal finding for the age.

The anterior fontanelle typically closes anywhere between 12 to 18 months


of age. Thus, assessing the anterior fontanelle as still being slightly open is a
normal finding requiring no further action. Because it is a normal finding for
this age, notifying the physician or performing additional examinations is
inappropriate.

Option A: The average size of the anterior fontanel is 2.1 cm, and the
median time of closure is 13.8 months. The most common causes of a large
anterior fontanel or delayed fontanel closure are achondroplasia,
hypothyroidism, Down syndrome, increased intracranial pressure, and
rickets.

Option B: During a neurological exam, the child’s healthcare provider will


test the functioning of the nervous system. The nervous system is very
complex and controls many parts of the body. The nervous system consists
of the brain, spinal cord, 12 nerves that come from the brain, and the nerves
that come from the spinal cord. The nervous system regulates the muscles.
156
C.BY: HOSSAM HAMDY MATERNITY RNPEDIA – ‫موسوعه التمريض‬

The circulation to the brain, arising from the arteries in the neck, is also
frequently examined. In infants and younger children, a neurological exam
includes the measurement of the head circumference.

Option C: The newborn’s skull is molded during birth. The frontal bone
flattens, the occipital bone is pulled outward, and the parietal bones override.
These changes aid delivery through the birth canal and usually resolve after
three to five days. The newborn’s skull should be evaluated for shape,
circumference, suture ridges, and size of anterior and posterior fontanels.
Size is calculated by the average of the anteroposterior and transverse
dimensions.
32. Question

1 point(s)

When teaching a mother about introducing solid foods to her child, which of
the following indicates the earliest age at which this should be done?

A. 1 month

B. 2 months

C. 3 months

D. 4 months

Answer: D. 4 months

Solid foods are not recommended before age 4 to 6 months because of the
sucking reflex and the immaturity of the gastrointestinal tract and immune
system. Therefore, the earliest age at which to introduce foods is 4 months.
Any time earlier would be inappropriate.

Option A: 1-month old infants should stick to breast milk. Most doctors
recommend waiting until at least 6 months before giving water. A 1-month-
old may feed every 2-3 hours. The infant will know when to stop feeding by
stopping, moving away from the breast, or falling asleep.

Option B: At about 2 months of age, babies usually take 4 to 5 ounces per


feeding every 3 to 4 hours.

Option C: How much formula for a 3-month-old baby? Typically five


ounces about six to eight times a day. How often should a 3-month-old
157
C.BY: HOSSAM HAMDY MATERNITY RNPEDIA – ‫موسوعه التمريض‬

nurse? Feedings are typically about every three or four hours at this age but
each breastfed baby may be slightly different. To double-check that the
baby’s getting enough breast milk, check the diapers. How many wet diapers
for a 3-month-old baby? About four or five very wet ones per day.
33. Question

1 point(s)

The infant of a substance-abusing mother is at risk for developing a sense of


which of the following?

A. Mistrust

B. Shame

C. Guilt

D. Inferiority

Answer: A. Mistrust

According to Erikson, infants need to have their needs met consistently and
effectively to develop a sense of trust. An infant whose needs are
consistently unmet or who experiences significant delays in having them
met, such as in the case of the infant of a substance-abusing mother, will
develop a sense of uncertainty, leading to mistrust of caregivers and the
environment.

Option B: Toddlers develop a sense of shame when their autonomy needs


are not met consistently. Autonomy versus shame and doubt is the second
stage of Erik Erikson’s stages of psychosocial development. This stage
occurs between the ages of 18 months to approximately 3 years. According
to Erikson, children at this stage are focused on developing a sense of
personal control over physical skills and a sense of independence.

Option C: Preschoolers develop a sense of guilt when their sense of


initiative is thwarted. Initiative versus guilt is the third stage of Erik
Erikson’s theory of psychosocial development. During the initiative versus
guilt stage, children assert themselves more frequently through directing play
and other social interactions.

158
C.BY: HOSSAM HAMDY MATERNITY RNPEDIA – ‫موسوعه التمريض‬

Option D: Schoolagers develop a sense of inferiority when they do not


develop a sense of industry. Erikson’s fourth psychosocial crisis, involving
industry (competence) vs. Inferiority occurs during childhood between the
ages of five and twelve.
34. Question

1 point(s)

Which of the following toys should the nurse recommend for a 5-month-old?

A. A big red balloon

B. A teddy bear with button eyes

C. A push-pull wooden truck

D. A colorful busy box

Answer: D. A colorful busy box

A busy box facilitates fine motor development that occurs between 4 and 6
months.

Option A: Balloons are contraindicated because small children may aspirate


balloons.

Option B: Because the button eyes of a teddy bear may detach and be
aspirated, this toy is unsafe for children younger than 3 years.

Option C: A 5-month-old is too young to use a push-pull toy. Toddlers are


full of energy, eager to explore everything around them. They love all kinds
of physical activities such as pulling, pushing, lugging, knocking down,
emptying, and filling.
35. Question

1 point(s)

The mother of a 2-month-old is concerned that she may be spoiling her baby
by picking her up when she cries. Which of the following would be the
nurse’s best response?

A. “ Let her cry for a while before picking her up, so you don’t spoil her.”
159
C.BY: HOSSAM HAMDY MATERNITY RNPEDIA – ‫موسوعه التمريض‬

B. “Babies need to be held and cuddled; you won’t spoil her this way.”

C. “Crying at this age means the baby is hungry; give her a bottle.”

D. “If you leave her alone she will learn how to cry herself to sleep.”

Answer: B. “Babies need to be held and cuddled; you won’t spoil her
this way.”

Infants need to have their security needs met by being held and cuddled. At 2
months of age, they are unable to make the connection between crying and
attention. This association does not occur until late infancy or early
toddlerhood.

Option A: Letting the infant cry for a time before picking up the infant or
leaving the infant alone to cry herself to sleep interferes with meeting the
infant’s need for security at this very young age.

Option C: Infants cry for many reasons. Assuming that the child Is hungry
may cause overfeeding problems such as obesity.

Option D: If the care the infant receives is consistent, predictable, and


reliable, they will develop a sense of trust which will carry with them to
other relationships, and they will be able to feel secure even when
threatened.
36. Question

1 point(s)

When assessing an 18-month-old, the nurse notes a characteristic protruding


abdomen. Which of the following would explain the rationale for this
finding?

A. Increased food intake owing to age.

B. Underdeveloped abdominal muscles.

C. Bow Legged posture.

D. Linear growth curve.

160
C.BY: HOSSAM HAMDY MATERNITY RNPEDIA – ‫موسوعه التمريض‬

Answer: B. Underdeveloped abdominal muscles

Underdeveloped abdominal musculature gives the toddler a characteristically


protruding abdomen. It is generally normal for toddlers to have potbellies.
By the time children reach school age, the potbelly will most often disappear
and their bodies seem more proportionate. The belly should feel soft and
NOT tender.

Option A: During toddlerhood, food intake decreases, not increases.


Feeding toddlers (ages 1 to 3) can often be challenging. That’s because
several developmental changes are happening at this time. Toddlers are
striving for independence and control. Their growth rate slows down and
with this comes a decrease in appetite.

Option C: Toddlers are characteristically bowlegged because the leg


muscles must bear the weight of the relatively large trunk. When babies are
born with bow legs it’s because some of the bones had to rotate (twist)
slightly when they were growing in the womb to fit into the small space.
This is called physiological bow legs. It’s considered a normal part of a
child’s growth and development.

Option D: Toddler growth patterns occur in a steplike, not linear pattern.


Between ages 1 and 2, a toddler will gain only about 5 pounds (2.2
kilograms). Weight gain will remain at about 5 pounds (2.2 kilograms) per
year between ages 2 to 5. Between ages 2 to 10 years, a child will grow at a
steady pace. A final growth spurt begins at the start of puberty, sometime
between ages 9 to 15.
37. Question

1 point(s)

If parents keep a toddler dependent in areas where he is capable of using


skills, the toddler will develop a sense of which of the following?

A. Mistrust

B. Shame

C. Guilt

D. Inferiority

161
C.BY: HOSSAM HAMDY MATERNITY RNPEDIA – ‫موسوعه التمريض‬

Answer: B. Shame

According to Erikson, toddlers experience a sense of shame when they are


not allowed to develop appropriate independence and autonomy. If children
are criticized, overly controlled, or not given the opportunity to assert
themselves, they begin to feel inadequate in their ability to survive, and may
then become overly dependent upon others, lack self-esteem, and feel a
sense of shame or doubt in their abilities.

Option A: Infants develop mistrust when their needs are not consistently
gratified. Failing to acquire the virtue of hope will lead to the development
of fear. This infant will carry the basic sense of mistrust with them to other
relationships. It may result in anxiety, heightened insecurities, and an over
feeling of mistrust in the world around them.

Option C: Preschoolers develop guilt when their initiative needs are not
met. If this tendency is squelched, either through criticism or control,
children develop a sense of guilt. The child will often overstep the mark in
his forcefulness, and the danger is that the parents will tend to punish the
child and restrict his initiatives too much.

Option D: While school-agers develop a sense of inferiority when their


industry needs are not met. If this initiative is not encouraged, if it is
restricted by parents or teachers, then the child begins to feel inferior,
doubting his own abilities and therefore may not reach his or her potential.
38. Question

1 point(s)

Which of the following is an appropriate toy for an 18-month-old?

A. Multiple-piece puzzle

B. Miniature cars

C. Finger paints

D. Comic book

Answer: C. Finger paints

Young trent textures. Thus, finger paints would be an appropriate toy choice.

162
C.BY: HOSSAM HAMDY MATERNITY RNPEDIA – ‫موسوعه التمريض‬

Option A: Multiple-piece toddlers are still sensorimotor learners and they


enjoy the experience of feeling dizzy, such as puzzle, are too difficult to
manipulate and may be hazardous if the pieces are small enough to be
aspirated.

Option B: Miniature cars also have a high potential for aspiration.

Option D: Comic books are on too high a level for toddlers. Although they
may enjoy looking at some of the pictures, toddlers are more likely to rip a
comic book apart.
39. Question

1 point(s)

When teaching parents about the child’s readiness for toilet training, which
of the following signs should the nurse instruct them to watch for in the
toddler?

A. Demonstrates dryness for 4 hours.

B. Demonstrates ability to sit and walk.

C. Has a new sibling for stimulation.

D. Verbalizes desire to go to the bathroom.

Answer: D. Verbalizes desire to go to the bathroom

The child must be able to state the need to go to the bathroom to initiate
toilet training. Toilet training is teaching the child to recognize their body
signals for urinating and having a bowel movement. It also means teaching
the child to use a potty chair or toilet ly and at the appropriate times.

Option A: Usually, a child needs to be dry for only 2 hours, not 4 hours.
Children develop at different rates. A child younger than 12 months has no
control over bladder or bowel movements. There is very little control
between 12 to 18 months. Most children don’t have bowel and bladder
control until 24 to 30 months. The average age of toilet training is 27
months.

Option B: The child also must be able to sit, walk, and squat. Toilet training
should start when your child shows signs that he or she is ready. There is no
right age to begin. If you try to toilet train before your child is ready, it can
163
C.BY: HOSSAM HAMDY MATERNITY RNPEDIA – ‫موسوعه التمريض‬

be a battle for both you and your child. The ability to control bowel and
bladder muscles comes with proper growth and development.

Option C: A new sibling would most likely hinder toilet training. If there
are older siblings, ask them to let the younger child see you praising them for
using the toilet.
40. Question

1 point(s)

When teaching parents about typical toddler eating patterns, which of the
following should be included?

A . Food “jags.”

B. Preference to eat alone

C. Consistent table manners

D. Increase in appetite

Answer: A. Food “jags.”

Toddlers become picky eaters, experiencing food jags, and eating large
amounts one day and very little the next. A toddler’s food gags express a
preference for the ritualism of eating one type of food for several days at a
time.

Option B: Toddlers typically enjoy socialization and limiting others at


mealtime.

Option C: Toddlers prefer to feed themselves and thus are too young to
have table manners.

Option D: A toddler’s appetite and the need for calories, protein, and fluid
decrease due to the dramatic slowing of growth rate.
41. Question

1 point(s)

Which of the following suggestions should the nurse offer the parents of a 4-
year-old boy who resists going to bed at night?
164
C.BY: HOSSAM HAMDY MATERNITY RNPEDIA – ‫موسوعه التمريض‬

A. “Allow him to fall asleep in your room, then move him to his own bed.”

B. “Tell him that you will lock him in his room if he gets out of bed one
more time.”

C. “Encourage active play at bedtime to tire him out so he will fall asleep
faster.”

D. “Read him a story and allow him to play quietly in his bed until he falls
asleep.”

Answer: D. “Read him a story and allow him to play quietly in his bed
until he falls asleep.”

Preschoolers commonly have fears of the dark, being left alone especially at
bedtime, and ghosts, which may affect the child’s going to bed at night.
Quiet play and time with parents is a positive bedtime routine that provides
security and also readies the child for sleep.

Option A: The child should sleep in his own bed.

Option B: Telling the child about locking him in his room will be viewed by
the child as a threat. Additionally, a locked door is frightening and
potentially hazardous.

Option C: Vigorous activity at bedtime stirs up the child and makes it more
difficult to fall asleep.
42. Question

1 point(s)

When providing therapeutic play, which of the following toys would best
promote imaginative play in a 4-year-old?

A. Large blocks

B. Dress-up clothes

C. Wooden puzzle

D. Big wheels

165
C.BY: HOSSAM HAMDY MATERNITY RNPEDIA – ‫موسوعه التمريض‬

Answer: B. Dress-up clothes

Dress-up clothes enhance imaginative play and imagination, allowing


preschoolers to engage in rich fantasy play. Play therapy is a form of therapy
primarily geared toward children. In this form of therapy, a therapist
encourages a child to explore life events that may have an effect on current
circumstances, in a manner and pace of the child’s choosing, primarily
through play but also through language.

Option A: Though play is often regarded simply as a way for individuals,


particularly children, to relax, scientific research has proven that play is a
crucial factor in healthy child development. Large blocks are good for fine
motor development.

Option C: Wooden puzzles are appropriate for encouraging fine motor


development. Play is considered to be especially important for the healthy
development of children who have experienced stressful events or past
trauma. While the effects of trauma tend to reside in the nonverbal areas of
the brain—the hippocampus, amygdala, thalamus, and brain stem—a
person’s capacity to communicate and process adverse issues resides in the
brain’s frontal lobes.

Option D: Big wheels and tricycles encourage gross motor development.


Play therapy is thought to be one of the most beneficial means of helping
children who are experiencing emotional or behavioral challenges. Though
the approach may benefit people of all ages, it is specially designed to treat
children under 12.
43. Question

1 point(s)

Which of the following activities, when voiced by the parents following a


teaching session about the characteristics of school-age cognitive
development would indicate the need for additional teaching?

A. Collecting baseball cards and marbles.

B. Ordering dolls according to size.

C. Considering simple problem-solving options.

D. Developing plans for the future.

166
C.BY: HOSSAM HAMDY MATERNITY RNPEDIA – ‫موسوعه التمريض‬

Answer: D. Developing plans for the future

The school-aged child is in the stage of concrete operations, marked by


inductive reasoning, logical operations, and reversible concrete thought. The
ability to consider the future requires formal thought operations, which are
not developed until adolescence.

Option A: Collecting baseball cards and marbles is an example of concrete


operational thinking. Piaget considered the concrete stage a major turning
point in the child’s cognitive development because it marks the beginning of
logical or operational thought.

Option B: Children can conserve number (age 6), mass (age 7), and weight
(age 9). Conservation is the understanding that something stays the same in
quantity even though its appearance changes.

Option C: Simple problem-solving options is an example of the concrete


operational thinking of the school age.
44. Question

1 point(s)

A hospitalized school age child states: “I’m not afraid of this place, I’m not
afraid of anything.” This statement is most likely an example of which of the
following?

A. Regression

B. Repression

C. Reaction formation

D. Rationalization

Answer: C. Reaction formation

Reaction formation is the school ager’s typical defensive response when


hospitalized. In reaction formation, expression of unacceptable thoughts or
behaviors is prevented (or overridden) by the exaggerated expression of
opposite thoughts or types of behaviors.

Option A: Regression is seen in toddlers and preschoolers when they retreat


or return to an earlier level of development.
167
C.BY: HOSSAM HAMDY MATERNITY RNPEDIA – ‫موسوعه التمريض‬

Option B: Repression refers to the involuntary blocking of unpleasant


feelings and experiences from one’s awareness.

Option D: Rationalization is the attempt to make excuses to justify


unacceptable feelings or behaviors.
45. Question

1 point(s)

After teaching a group of parents about accident prevention for school agers,
which of the following statements by the group would indicate the need for
more teaching?

A. “School Agers are more active and adventurous than younger children.”

B. “School Agers are more susceptible to home hazards than are younger
children.”

C. “Schoolagers are unable to understand potential dangers around them.”

D. “Schoolargers are less subject to parental control than are younger


children.”

Answer: C. “Schoolagers are unable to understand potential dangers


around them.”

The school age’s cognitive level is sufficiently developed to enable a good


understanding of and adherence to rules. Thus, school-agers should be able
to understand the potential dangers around them.

Option A: With growth comes greater freedom and children become more
adventurous and daring.

Option B: The school-aged child is also still prone to accidents and home
hazards, especially because of increased motor abilities and independence.
Plus the home hazards differ from other age groups. These hazards, which
are potentially lethal but tempting, may include firearms, alcohol, and
medications.

Option D: School-age children begin to internalize their own controls and


needless outside direction. Plus the child is away from home more often.
Some parental or caregiver assistance is still needed to answer questions and
provide guidance for decisions and responsibilities.
168
C.BY: HOSSAM HAMDY MATERNITY RNPEDIA – ‫موسوعه التمريض‬

46. Question

1 point(s)

Which of the following skills is the most significant one learned during the
school-age period?

A. Collecting

B. Ordering

C. Reading

D. Sorting

Answer: C. Reading

The most significant skill learned during the school-age period is reading.
During this time the child develops formal adult articulation patterns and
learns that words can be arranged in structure.

Option A: School-age children most often have smooth and strong motor
skills. However, their coordination (especially eye-hand), endurance,
balance, and physical abilities vary.

Option B: An ability to pay attention is important for success both at school


and at home. A 6-year-old should be able to focus on a task for at least 15
minutes. By age 9, a child should be able to focus attention for about an
hour.

Option D: Fine motor skills may also vary widely. These skills can affect a
child’s ability to write neatly, dress appropriately, and perform certain
chores, such as making beds or doing dishes.
47. Question

1 point(s)

A child age 7 was unable to receive the measles, mumps, and rubella (MMR)
vaccine at the recommended scheduled time. When would the nurse expect
to administer the MMR vaccine?

A. In a month from now

169
C.BY: HOSSAM HAMDY MATERNITY RNPEDIA – ‫موسوعه التمريض‬

B. In a year from now

C. At age 10

D. At age 13

Answer: C. At age 10

Based on the recommendations of the American Academy of Family


Physicians and the American Academy of Pediatrics, the MMR vaccine
should be given at the age of 10 if the child did not receive it between the
ages of 4 to 6 years as recommended. Immunization for diphtheria and
tetanus is required at age 13.

Option A: Children should get two doses of MMR vaccine, starting with the
first dose at 12 to 15 months of age, and the second dose at 4 through 6 years
of age.

Option B: Children can receive the second dose earlier as long as it is at


least 28 days after the first dose.

Option D: MMR vaccine is given later than some other childhood vaccines
because antibodies transferred from the mother to the baby can provide some
protection from disease and make the MMR vaccine less effective until
about 1 year of age.
48. Question

1 point(s)

The adolescent’s inability to develop a sense of who he is and what he can


become results in the sense of which of the following?

A. Shame

B. Guilt

C. Inferiority

D. Role confusion

Answer: D. Role confusion


170
C.BY: HOSSAM HAMDY MATERNITY RNPEDIA – ‫موسوعه التمريض‬

According to Erikson, role cunfusion develops when the adolescent does not
develop a sense of identity and a sense of where he fits in.

Option A: Toddlers develop a sense of shame when they do not achieve


autonomy. If children are criticized, overly controlled, or not given the
opportunity to assert themselves, they begin to feel inadequate in their ability
to survive, and may then become overly dependent upon others, lack self-
esteem, and feel a sense of shame or doubt in their abilities.

Option B: Preschoolers develop a sense of guilt when they do not develop a


sense of initiative. If this tendency is squelched, either through criticism or
control, children develop a sense of guilt. The child will often overstep the
mark in his forcefulness, and the danger is that the parents will tend to
punish the child and restrict his initiatives too much.

Option C: School-age children develop a sense of inferiority when they do


not develop a sense of industry. If this initiative is not encouraged, if it is
restricted by parents or teachers, then the child begins to feel inferior,
doubting his own abilities and therefore may not reach his or her potential.
49. Question

1 point(s)

Which of the following would be most appropriate for a nurse to use when
describing menarche to a 13-year-old?

A. A female’s first menstruation or menstrual “periods.”

B. The first year of menstruation or “period."

C. The entire menstrual cycle or from one “period” to another.

D. The onset of uterine maturation or peak growth.

Answer: A. A female’s first menstruation or menstrual “periods”.

Menarche refers to the onset of the first menstruation or menstrual period


and refers only to the first cycle. Uterine growth and broadening of the
pelvic girdle occur before menarche. The average age of onset of menarche
is 12.4 years.

Option B: Pulsatile hypothalamic production of gonadotropin-releasing


hormone (GnRH) at puberty stimulates the pituitary production of follicle-
171
C.BY: HOSSAM HAMDY MATERNITY RNPEDIA – ‫موسوعه التمريض‬

stimulating hormone (FSH) and luteinizing hormone (LH). This pulsatile


secretion pattern appears to be necessary as continuous secretion of GnRH,
or its synthetic analogs, inhibits pituitary production of FSH and LH and
delays menarche.

Option C: Most menstrual periods last between 3 and 7 days, and menses
that last more than 10 days is considered abnormal.

Option D: Menarche signals maturation of the adolescent female body. It


commonly is associated with the ability to ovulate and reproduce. However,
the appearance of menarche does not guarantee either ovulation or fertility.
50. Question

1 point(s)

A 14-year-old boy has acne and according to his parents, dominates the
bathroom by using the mirror all the time. Which of the following remarks
by the nurse would be least helpful in talking to the boy and his parents?

A. “This is probably the only concern he has about his body. So don’t worry
about it or the time he spends on it.”

B. “Teenagers are anxious about how their peers perceive them. So they
spend a lot of time grooming.”

C. “A teen may develop a poor self-image when experiencing acne. Do you


feel this way sometimes?”

D. “You appear to be keeping your face well washed. Would you feel
comfortable discussing your cleansing method?”

Answer: A. “This is probably the only concern he has about his body. So
don’t worry about it or the time he spends on it.”

Stating that this is probably the only concern the adolescent has and telling
the parents not to worry about it or the time he spends on it shuts off further
investigation and is likely to make the adolescent and his parents feel
defensive.

Option B: The statement about peer acceptance and time spent in front of
the mirror for the development of self-image provides information about the

172
C.BY: HOSSAM HAMDY MATERNITY RNPEDIA – ‫موسوعه التمريض‬

adolescent’s needs to the parents and may help to gain trust with the
adolescent.

Option C: Asking the adolescent how he feels about the acne will encourage
the adolescent to share his feelings.

Option D: Discussing the cleansing method shows interest and concern for
the adolescent and also can help to identify any patient-teaching needs for
the adolescent regarding cleansing.
51. Question

1 point(s)

Which of the following should the nurse suspect when noting that a 3-year-
old is engaging in explicit sexual behavior during doll play?

A. The child is exhibiting normal preschool curiosity.

B. The child is acting out personal experiences.

C. The child does not know how to play with dolls.

D. The child is probably developmentally delayed.

Answer: B. The child is acting out personal experiences

Preschoolers should be developmentally incapable of demonstrating explicit


sexual behavior. If a child does so, the child has been exposed to such
behavior, and sexual abuse should be suspected.

Option A: As preschoolers seek independence, it’s normal for them to argue


and exercise their right to say “no.” They commonly vacillate between
demanding they are a big kid who can do everything on their own, to using
baby talk to declare they need help with a simple task.

Option C: Explicit sexual behavior during doll play is not a characteristic of


preschool development nor symptomatic of developmental delay. Whether
or not the child knows how to play with dolls is irrelevant.

Option D: Preschoolers may exhibit the occasional tantrum, but they should
be gaining more control over their emotions and impulses compared to when
they were toddlers. Any temper tantrums at this stage should be shorter and
less intense than the toddler years.
173
C.BY: HOSSAM HAMDY MATERNITY RNPEDIA – ‫موسوعه التمريض‬

52. Question

1 point(s)

Which of the following statements by the parents of a child with school


phobia would indicate the need for further teaching?

A. “We’ll keep him at home until the phobia subsides.”

B. “We’ll work with his teachers and counselors at school.”

C. “We’ll try to encourage him to talk about his problem.”

D. “We’ll discuss possible solutions with him and his counselor.”

Answer: A. “We’ll keep him at home until the phobia subsides.”

The parents need more teaching if they state that they will keep the child
home until the phobia subsides. Doing so reinforces the child’s feelings of
worthlessness and dependency.

Option B: The child should attend school even during resolution of the
problem.

Option C: Allowing the child to verbalize helps the child to ventilate


feelings and may help to uncover causes and solutions.

Option D: Collaboration with the teachers and counselors at school may


lead to uncovering the cause of the phobia and to the development of
solutions. The child should participate and play an active role in developing
possible solutions.
53. Question

1 point(s)

When developing a teaching plan for a group of high school students about
teenage pregnancy, the nurse would keep in mind which of the following?

A. The incidence of teenage pregnancies is increasing.

B. Most teenage pregnancies are planned.

C. Denial of the pregnancy is common early on.


174
C.BY: HOSSAM HAMDY MATERNITY RNPEDIA – ‫موسوعه التمريض‬

D. The risk for complications during pregnancy is rare.

Answer: C. Denial of the pregnancy is common early on.

The adolescent who becomes pregnant typically denies the pregnancy early
on. Early recognition by a parent or health care provider may be crucial to
timely initiation of prenatal care.

Option A: The incidence of adolescent pregnancy has declined since 1991,


yet morbidity remains high.

Option B: Most teenage pregnancies are unplanned and occur out of


wedlock.

Option D: The pregnant adolescent is at high risk for physical complications


including premature labor and low-birth-weight infants, high neonatal
mortality, iron deficiency anemia, prolonged labor, and fetopelvic
disproportion as well as numerous psychological crises.
54. Question

1 point(s)

When assessing a child with a cleft palate, the nurse is aware that the child is
at risk for more frequent episodes of otitis media due to which of the
following?

A. Lowered resistance from malnutrition.

B. Ineffective functioning of the Eustachian tubes.

C. Plugging of the Eustachian tubes with food particles.

D. Associated congenital defects of the middle ear.

Answer: B. Ineffective functioning of the Eustachian tubes

Because of the structural defect, children with cleft palate may have
ineffective functioning of their Eustachian tubes creating frequent bouts of
otitis media.

175
C.BY: HOSSAM HAMDY MATERNITY RNPEDIA – ‫موسوعه التمريض‬

Option A: Most children with cleft palate remain well-nourished and


maintain adequate nutrition through the use of proper feeding techniques.

Option C: Food particles do not pass through the cleft and into the
Eustachian tubes. Otitis media with effusion is ubiquitous in children who
have a cleft palate. The cause is simply the lack of proper insertion of the
tensor veli palatini muscle in the soft palate. The muscle is, therefore, unable
to open the eustachian tube on swallowing or wide mouth opening.

Option D: There is no association between cleft palate and congenital ear


deformities. The prevalence of associated anomalies with orofacial clefts can
differ among different populations. At birth, the black population has a lower
prevalence of oral clefts compared to whites; Sullivan found that oral clefts
in the black population are more commonly associated with clubfoot and
polydactyly compared to other ethnic populations.
55. Question

1 point(s)

While performing a neurodevelopmental assessment on a 3-month-old


infant, which of the following characteristics would be expected?

A. A strong Moro reflex.

B. A strong parachute reflex.

C. Rolling from front to back.

D. Lifting of head and chest when prone.

Answer: D. Lifting of head and chest when prone

A 3-month-old infant should be able to lift the head and chest when prone.

Option A: The Moro reflex typically diminishes or subsides by 3 months.


The Moro reflex is a normal primitive, infantile reflex. The Moro reflex is an
involuntary protective motor response against abrupt disruption of body
balance or extremely sudden stimulation.

Option B: The parachute reflex appears at 9 months. This reflex occurs in


slightly older infants when the child is held upright and the baby’s body is
rotated quickly to face forward (as in falling). The baby will extend his arms

176
C.BY: HOSSAM HAMDY MATERNITY RNPEDIA – ‫موسوعه التمريض‬

forward as if to break a fall, even though this reflex appears long before the
baby walks.

Option C: Rolling from front to back usually is accomplished at about 5


months.

56. Question

1 point(s)

By the end of which of the following would the nurse most commonly
expect a child’s birth weight to triple?

A. 4 months

B. 7 months

C. 9 months

D. 12 months

Answer: D. 12 months

A child’s birth weight usually triples by 12 months and doubles by 4 months.


No specific birth weight parameters are established for 7 or 9 months.

Option A: Most infants will gain about a pound over their birth weight by
month one. At this age, infants are not as sleepy, they begin developing a
regular feeding pattern, and they have a stronger suck during feedings.

Option B: On average, babies gain about one pound each month for the first
six months. The average weight at six months is about 16 pounds 2 ounces
(7.3 kg) for girls and 17 pounds 8 ounces (7.9 kg) for boys.

Option C: Between six months and one year, weight gain slows down a
little. Most babies double their birth weight by five to six months of age and
triple it by the time they are a year old. By one year, the average weight of a
baby girl is approximately 19 pounds 10 ounces (8.9 kg), with boys
weighing about 21 pounds 3 ounces (9.6 kg).

177
C.BY: HOSSAM HAMDY MATERNITY RNPEDIA – ‫موسوعه التمريض‬

57. Question

1 point(s)

Which of the following best describes parallel play between two toddlers?

A. Sharing crayons to color separate pictures.

B. Playing a board game with a nurse.

C. Sitting near each other while playing with separate dolls.

D. Sharing their dolls with two different nurses.

Answer: C. Sitting near each other while playing with separate dolls

Toddlers engaging in parallel play will play near each other, but not with
each other. Thus, when two toddlers sit near each other but play with
separate dolls, they are exhibiting parallel play.

Option A: Cooperative play involves children playing and working with


others towards a common goal or purpose.

Option B: Being able to participate in cooperative play is extremely


important. It means that the child has the skills they’ll need later to
collaborate and cooperate at school and in other typical social settings, like
sports.

Option D: Sharing dolls with two different nurses is an example of


cooperative play.
58. Question

1 point(s)

Which of the following would the nurse identify as the initial priority for a
child with acute lymphocytic leukemia?

A. Instituting infection control precautions.

B. Encouraging adequate intake of iron-rich foods.

C. Assisting with coping with chronic illness.

178
C.BY: HOSSAM HAMDY MATERNITY RNPEDIA – ‫موسوعه التمريض‬

D. Administering medications via IM injections.

Answer: A. Instituting infection control precautions

Acute lymphocytic leukemia (ALL) causes leukopenia, resulting in


immunosuppression and increasing the risk of infection, a leading cause of
death in children with ALL. Therefore, the initial priority nursing
intervention would be to institute infection control precautions to decrease
the risk of infection.

Option B: Iron-rich foods help with anemia, but dietary iron is not an initial
intervention. For the treatment of iron deficiency anemia in adults, 100 to
200 mg of elemental iron per day has been recommended. The best way to
take the supplement so that it can be absorbed in the greatest amount of iron
is to take it in two or more doses during the day.

Option C: The prognosis of ALL usually is good. However, later on, the
nurse may need to assist the child and family with coping since death and
dying may still be an issue in need of discussion.

Option D: Injections should be discouraged, owing to increased risk of


bleeding due to thrombocytopenia.
59. Question

1 point(s)

Which of the following information, when voiced by the mother, would


indicate to the nurse that she understands home care instructions following
the administration of diphtheria, tetanus, and pertussis injection?

A. Measures to reduce fever.

B. Need for dietary restrictions.

C. Reasons for subsequent rash.

D. Measures to control subsequent diarrhea.

Answer: A. Measures to reduce fever

179
C.BY: HOSSAM HAMDY MATERNITY RNPEDIA – ‫موسوعه التمريض‬

The pertussis component may result in fever and the tetanus component may
result in injection soreness. Therefore, the mother’s verbalization of
information about measures to reduce fever indicates understanding.

Option B: No dietary restrictions are necessary after this injection is given.

Option C: Subsequent rash is more likely to be seen 5 to 10 days after


receiving the MMR vaccine, not diphtheria, pertussis, and tetanus vaccine.

Option D: Diarrhea is not associated with this vaccine. Common side effects
include soreness or swelling where the shot was given, fever, irritability,
feeling tired, loss of appetite, and vomiting.
60. Question

1 point(s)

Which of the following actions by a community health nurse is most


appropriate when noting multiple bruises and burns on the posterior trunk of
an 18-month-old child during a home visit?

A. Report the child’s condition to Protective Services immediately.

B. Schedule a follow-up visit to check for more bruises.

C. Notify the child’s physician immediately.

D. Don nothing because this is a normal finding in a toddler.

Answer: A. Report the child’s condition to Protective Services


immediately.

Multiple bruises and burns on a toddler are signs of child abuse. Therefore,
the nurse is responsible for reporting the case to Protective Services
immediately to protect the child from further harm.

Option B: Scheduling a follow-up visit is inappropriate because additional


harm may come to the child if the nurse waits for further assessment data.

Option C: Although the nurse should notify the physician, the goal is to
initiate measures to protect the child’s safety. Notifying the physician
immediately does not initiate the removal of the child from harm nor does it
absolve the nurse from responsibility.

180
C.BY: HOSSAM HAMDY MATERNITY RNPEDIA – ‫موسوعه التمريض‬

Option D: Multiple bruises and burns are not normal toddler injuries.
Bruises form when the soft tissues of the body get bumped. When they do,
small veins and capillaries (the tiniest blood vessels) under the skin
sometimes break. Red blood cells leak out of these blood vessels. These red
blood cells that collect under your skin cause that bluish, purplish, reddish,
or blackish marks.
61. Question

1 point(s)

Which of the following is being used when the mother of a hospitalized child
calls the student nurse and states, “You idiot, you have no idea how to care
for my sick child”?

A. Displacement

B. Projection

C. Repression

D. Psychosis

Answer: B. Projection

The mother is using projection, the defense mechanism used when a person
attributes his or her own undesirable traits to another.

Option A: Displacement is the transfer of emotion onto an unrelated object,


such as when the mother would kick a chair or bang the door shut.

Option C: Repression is the submerging of painful ideas into the


unconscious. Repression is an unconscious mechanism employed by the ego
to keep disturbing or threatening thoughts from becoming conscious.

Option D: Psychosis is a state of being out of touch with reality. During a


period of psychosis, a person’s thoughts and perceptions are disturbed and
the individual may have difficulty understanding what is real and what is
not. Symptoms of psychosis include delusions (false beliefs) and
hallucinations (seeing or hearing things that others do not see or hear).
62. Question

1 point(s)
181
C.BY: HOSSAM HAMDY MATERNITY RNPEDIA – ‫موسوعه التمريض‬

Which of the following should the nurse expect to note as a frequent


complication for a child with congenital heart disease?

A. Susceptibility to respiratory infection

B. Bleeding tendencies

C. Frequent vomiting and diarrhea

D. Seizure disorder

Answer: A. Susceptibility to respiratory infection

Children with congenital heart disease are more prone to respiratory


infections. Children with congenital heart disease (CHD) are at risk for
increased morbidity from viral lower respiratory tract infections because of
anatomical cardiac lesions that can worsen an already compromised
respiratory status.

Option B: It has been recognized that patients with Cyanotic Congenital


Heart Disease (CCHD) show significant bleeding tendency which can be
secondary to coagulopathies in these patients. Some coagulation
abnormalities are thrombocytopenia, factor deficiencies, fibrinolysis, and
Disseminated Intravascular Coagulation (DIC).

Option C: Vomiting and diarrhea are most likely experienced with a heart
attack. Women are somewhat more likely than men to experience some of
the other common symptoms, particularly shortness of breath,
nausea/vomiting, and back or jaw pain.

Option D: Although neurological morbidity has been consistently described


in the congenital heart disease (CHD) population,1 no studies to date have
examined the long-term risk of epilepsy in subjects with CHD compared
with the general population.
63. Question

1 point(s)

Which of the following would the nurse do first for a 3-year-old boy who
arrives in the emergency room with a temperature of 105 degrees, inspiratory
stridor, and restlessness, who is leaning forward and drooling?

A. Auscultate his lungs and place him in a mist tent.


182
C.BY: HOSSAM HAMDY MATERNITY RNPEDIA – ‫موسوعه التمريض‬

B. Have him lie down and rest after encouraging fluids.

C. Examine his throat and perform a throat culture.

D. Notify the physician immediately and prepare for intubation.

Answer: D. Notify the physician immediately and prepare for


intubation.

The child is exhibiting classic signs of epiglottitis, always a pediatric


emergency. The physician must be notified immediately and the nurse must
be prepared for an emergency intubation or tracheostomy.

Option A: Further assessment with auscultating lungs and placing the child
in a mist tent wastes valuable time. The situation is a possible life-
threatening emergency. In children, stridor may become louder in the supine
position. Causes of stridor are pertussis, croup, epiglottis, aspirations. The
recommended auscultation position for the stethoscope is the chest wall
position.

Option B: Having the child lie down would cause additional distress and
may result in respiratory arrest. Inspiratory stridor is often a medical
emergency. Assessment of vital signs and degree of respiratory distress is the
first step. In some cases, securing the airway may be necessary before or in
parallel with the physical examination.

Option C: Throat examination may result in laryngospasm that could be


fatal. Physical examination of a patient with suspected acute epiglottitis is
contraindicated. The patient may prefer certain positions that alleviate the
stridor.
64. Question

1 point(s)

Which of the following would the nurse need to keep in mind as a


predisposing factor when formulating a teaching plan for a child with a
urinary tract infection?

A. A shorter urethra in females.

B. Frequent emptying of the bladder.

C. Increased fluid intake.


183
C.BY: HOSSAM HAMDY MATERNITY RNPEDIA – ‫موسوعه التمريض‬

D. Ingestion of acidic juices.

Answer: A. A shorter urethra in females

In females, the urethra is shorter than in males. This decreases the distance
for organisms to travel, thereby increasing the chance of the child developing
a urinary tract infection.

Option B: Frequent emptying of the bladder would help to decrease urinary


tract infections by avoiding sphincter stress.

Option C: Increased fluid intake enables the bladder to be cleared more


frequently, thus helping to prevent urinary tract infections.

Option D: The intake of acidic juices helps to keep the urine pH acidic and
thus decrease the chance of flora development.
65. Question

1 point(s)

Which of the following should the nurse do first for a 15-year-old boy with a
full leg cast who is screaming in unrelenting pain and exhibiting right foot
pallor signifying compartment syndrome?

A. Medicate him with acetaminophen.

B. Notify the physician immediately.

C. Release the traction.

D. Monitor him every 5 minutes.

Answer: B. Notify the physician immediately

Compartment syndrome is an emergent situation and the physician needs to


be notified immediately so that interventions can be initiated to relieve the
increasing pressure and restore circulation. Compartment syndrome is a
painful condition that occurs when pressure within the muscles builds to
dangerous levels. This pressure can decrease blood flow, which prevents
nourishment and oxygen from reaching nerve and muscle cells.

184
C.BY: HOSSAM HAMDY MATERNITY RNPEDIA – ‫موسوعه التمريض‬

Option A: Acetaminophen (Tylenol) will be ineffective since the pain is


related to the increasing pressure and tissue ischemia. Acute compartment
syndrome is a surgical emergency. There is no effective non-surgical
treatment.

Option C: The cast, not traction, is being used in this situation for
immobilization, so releasing the traction would be inappropriate. Casts and
tight bandages may lead to compartment syndrome. If symptoms of
compartment syndrome develop, remove or loosen any constricting
bandages. If there is a cast, contact the doctor immediately.

Option D: In this situation, specific action not continued monitoring is


indicated. In acute compartment syndrome, unless the pressure is relieved
quickly, permanent disability and tissue death may result.
66. Question

1 point(s)

At which of the following ages would the nurse expect to administer the
varicella zoster vaccine to a child?

A. At birth

B. 2 months

C. 6 months

D. 12 months

Answer: D. 12 months

The varicella zoster vaccine (VZV) is a live vaccine given after age 12
months. The first dose of hepatitis B vaccine is given at birth to 2 months,
then at 1 to 4 months, and then again at 6 to 18 months. DTaP is routinely
given at 2, 4, 6, and 15 to 18 months and a booster at 4 to 6 years.

Option A: CDC recommends two doses of chickenpox vaccine for children,


adolescents, and adults. Children should receive two doses of the vaccine—
the first dose at 12 through 15 months old and a second dose at 4 through 6
years old.

185
C.BY: HOSSAM HAMDY MATERNITY RNPEDIA – ‫موسوعه التمريض‬

Option B: If the second dose is administered after the 7th birthday, the
minimum interval between doses is ?3 months for children age <13 years
and 4 weeks for persons age ?13 years

Option C: If it has been more than 8 weeks since the first dose, the second
dose may be given without restarting the schedule.
67. Question

1 point(s)

When discussing normal infant growth and development with parents, which
of the following toys would the nurse suggest as most appropriate for an 8-
month-old?

A. Push-pull toys

B. Rattle

C. Large blocks

D. Mobile

Answer: C. Large blocks

Because the 8-month-old is refining his gross motor skills, being able to sit
unsupported, and also improving his fine motor skills, probably capable of
making hand-to-hand transfers, large blocks would be the most appropriate
toy selection.

Option A: Push-pull toys would be more appropriate for the 10 to 12-


month-old as he or she begins to cruise the environment. Push toys provide
support for babies who aren’t quite ready to stand or walk on their own.
Teetering behind a push toy helps build strength, balance, and confidence —
three essential ingredients to becoming a champion walker. Like push toys,
pull toys and ride-ons also boost balance and coordination.

Option B: Rattles are more appropriate for infants in the 1 to 3 month age
range. The sounds rattles make can also alert babies to noise. If they hear the
sound of a rattle, babies will eventually turn their heads towards the sound.
Many rattles also have moving parts that can be twisted, turned, and spun,
which can help further develop a baby’s attention span and fine motor skills.

186
C.BY: HOSSAM HAMDY MATERNITY RNPEDIA – ‫موسوعه التمريض‬

Option D: Mobiles pose a danger to older infants because of possible


strangulation.
68. Question

1 point(s)

Which of the following aspects of psychosocial development is necessary for


the nurse to keep in mind when providing care for the preschool child?

A. The child can use complex reasoning to think out of situations.

B. Fear of body mutilation is a common preschool fear.

C. The child engages in competitive types of play.

D. Immediate gratification is necessary to develop initiative.

Answer: B. Fear of body mutilation is a common preschool fear

During the preschool period, the child has mastered a sense of autonomy and
goes on to master a sense of initiative. During this period, the child
commonly experiences more fears than at any other time. One common fear
is the fear of body mutilation, especially associated with painful experiences.

Option A: In addition to the social aspects of play with peers already


described, the type of play a child prefers reflects cognitive, fine and gross
motor, and visual perceptual motor skills. Children will not play for long at
activities that frustrate them because of a lack of ability. Fine motor and
visual perceptual motor skills are being refined during these years, but there
is a broad range of time for normal acquisition.

Option C: By age 4, children usually can play with three others fairly well.
Fantasy or pretend play gains prominence at about age 3. Children can play
out longer stories as they mature, with each child taking a specific role. By
age 5, the child has many social skills expected of adults, such as responding
to the good fortune of others spontaneously with positive verbal messages,
apologizing for unintentional mistakes, and relating to a group of friends.

Option D: Almost all preschool children are noncompliant, at least some of


the time—on the average, they comply with adult requests about 50% of the
time. This struggle for autonomy can be viewed as a positive milestone of

187
C.BY: HOSSAM HAMDY MATERNITY RNPEDIA – ‫موسوعه التمريض‬

development, with passivity representing a potential symptom of depression


or intimidation.
69. Question

1 point(s)

Which of the following is characteristic of a preschooler with mild-mental


retardation?

A. Slow to feed self.

B. Lack of speech.

C. Marked motor delays.

D. Gait disability.

Answer: A. Slow to feed self.

Mild mental retardation refers to developmental disability involving an IQ of


50 to 70. Typically, the child is not noted as being retarded, but exhibits
slowness in performing tasks, such as self-feeding, walking, and taking.

Option B: A speech delay might be due to an oral impairment, like problems


with the tongue or palate (the roof of the mouth) or a short frenulum (the
fold beneath the tongue), which can limit tongue movement.

Option C: Gross motor delays are common and vary in severity and
outcome. Some children with gross motor delays attain typical milestones at
a later age. Other children have a permanent motor disability, such as
cerebral palsy, which has a prevalence of 3.3 per 1000.

Option D: Gait disabilities would be seen in more severe forms of mental


retardation. During a child’s first few years walking, they may have obvious
gait abnormalities — disturbances in what is considered the normal walking
cycle for that age group.
70. Question

1 point(s)

Which of the following assessment findings would lead the nurse to suspect
Down syndrome in an infant?
188
C.BY: HOSSAM HAMDY MATERNITY RNPEDIA – ‫موسوعه التمريض‬

A. Small tongue

B. Transverse palmar crease

C. Large nose

D. Restricted joint movement

Answer: B. Transverse palmar crease

Down syndrome is characterized by the following a transverse palmar crease


(simian crease), separated sagittal suture, oblique palpebral fissures, small
nose, depressed nasal bridge, high arched palate, excess and lax skin, wide
spacing and plantar crease between the second and big toes, hyperextensible
and lax joints, large protruding tongue, and muscle weakness.

Option A: Tongue-tie, also known as ankyloglossia, is a condition some


babies are born with that limits their tongue movements. But for babies with
tongue-tie, there’s a problem with the lingual frenulum. That’s the small
stretch of tissue that connects the underside of the tongue to the bottom of
the mouth. It might be too short and tight or attached way up near the tip of
the tongue.

Option C: A newborn’s nose may be pushed in or flat because of the tight


squeeze during labor and delivery. It may take a week or longer before his or
her nose looks more normal.

Option D: Muscle tone represents one of the important concepts for


characterizing changes in the state of the developing nervous system. It can
be manifested in the level of activity of flexors and extensors and in muscle
reactions to its passive stretching (StR) or shortening (ShR).
71. Question

1 point(s)

While assessing a newborn with cleft lip, the nurse would be alert that which
of the following will most likely be compromised?

A. Sucking ability

B. Respiratory status

C. Locomotion
189
C.BY: HOSSAM HAMDY MATERNITY RNPEDIA – ‫موسوعه التمريض‬

D. GI function

Answer: A. Sucking ability

Because of the defect, the child will be unable to form the mouth adequately
around the nipple, thereby requiring special devices to allow for feeding and
sucking gratification.

Option B: Respiratory status may be compromised if the child is fed


improperly or during the postoperative period

Option C: Locomotion would be a problem for the older infant because of


the use of restraints.

Option D: GI functioning is not compromised in the child with a cleft lip.


One of the most immediate concerns after birth is feeding. While most
babies with cleft lips can breast-feed, a cleft palate may make sucking
difficult.
72. Question

1 point(s)

When providing postoperative care for the child with a cleft palate, the nurse
should position the child in which of the following positions?

A. Supine

B. Prone

C. In an infant seat

D. On the side

Answer: B. Prone

Postoperatively children with cleft palate should be placed on their


abdomens to facilitate drainage. A child who has had a cleft lip repair should
be positioned on their back to keep them from rubbing their face in the bed.
A child with only a cleft palate repair may sleep on their stomach.

190
C.BY: HOSSAM HAMDY MATERNITY RNPEDIA – ‫موسوعه التمريض‬

Option A: If the child is placed in the supine position, he or she may


aspirate. The goal after surgery is to protect the new repair and stitches. For
this reason, there will be some changes in the child’s feeding, positioning,
and activity for a short time.

Option C: Using an infant seat does not facilitate drainage.

Option D: Side-lying does not facilitate drainage as well as the prone


position.
73. Question

1 point(s)

While assessing a child with pyloric stenosis, the nurse is likely to note
which of the following?

A. Regurgitation

B. Steatorrhea

C. Projectile vomiting

D. “Currant jelly” stools

Answer: C. Projectile vomiting

Projectile vomiting is a key symptom of pyloric stenosis. Vomiting intensity


also increases until pathognomonic projectile vomiting ensues. Although
vomiting may initially be infrequent, over several days it becomes more
predictable, occurring at nearly every feeding.

Option A: Regurgitation is seen more commonly with GERD. Regurgitation


occurs with varying degrees of severity in approximately 80% of GERD
patients. This symptom is usually described as a sour taste in the mouth or a
sense of fluid moving up and down in the chest.

Option B: Steatorrhea occurs in malabsorption disorders such as celiac


disease. During celiac disease, steatorrhea was caused by the decreased
enzymatic function of the pancreas, asynchronism of the food, and bile
supply to the intestinal lumen, disorders of absorption of lipolysis products.

Option D: “Currant jelly” stools are characteristic of intussusception. The


trapped section of the bowel may have its blood supply cut off, which causes
191
C.BY: HOSSAM HAMDY MATERNITY RNPEDIA – ‫موسوعه التمريض‬

ischemia. The mucosa is sensitive to ischemia and responds by causing


sloughing off into the gut. This creates a “red currant jelly” stool, which is
sloughed mucosa, blood, and mucus.
74. Question

1 point(s)

Which of the following nursing diagnoses would be inappropriate for the


infant with gastroesophageal reflux (GER)?

A. Fluid volume deficit

B. Risk for aspiration

C. Altered nutrition: less than body requirements

D. Altered oral mucous membranes

Answer: D. Altered oral mucous membranes

GERD is the backflow of gastric contents into the esophagus resulting from
relaxation or incompetence of the lower esophageal (cardiac) sphincter. No
alteration in the oral mucous membranes occurs with this disorder.

Option A: Another common symptom of GERD is bringing swallowed food


up again to the mouth (regurgitation). Some people can have trouble
swallowing.

Option B: GERD can cause stomach contents to flow back into the
esophagus and dysphagia can cause food and/or liquid to remain in the
esophagus after swallowing. If these substances are inhaled and move into
the lungs, it can lead to serious respiratory problems, such as aspiration
pneumonia.

Option C: It is equally important to receive prompt treatment for each of


these disorders, as undiagnosed GERD can lead to Barrett’s esophagus and
untreated gastroparesis can result in malnutrition and weight loss.
75. Question

1 point(s)

192
C.BY: HOSSAM HAMDY MATERNITY RNPEDIA – ‫موسوعه التمريض‬

Which of the following parameters would the nurse monitor to evaluate the
effectiveness of thickened feedings for an infant with gastroesophageal
reflux disease (GERD)?

A. Vomiting

B. Stools

C. Uterine

D. Weight

Answer: A. Vomiting

Thickened feedings are used with GER to stop the vomiting. Therefore, the
nurse would monitor the child’s vomiting to evaluate the effectiveness of
using the thickened feedings.

Option B: Feed thickeners are commonly used for managing infants with
GOR despite the lack of strong supporting evidence. It is postulated that feed
thickener reduces GOR by increasing the viscosity or ‘stickiness’ of the
liquid content, enabling the feed to be retained in the stomach.

Option C: However, feed thickeners can increase the energy density and
osmolality of the feed which may increase the frequency of relaxation of the
lower esophageal sphincter and delay gastric emptying, worsening GOR.

Option D: If feedings are ineffective, this should be noted before there is


any change in the child’s weight

193
C.BY: HOSSAM HAMDY MATERNITY RNPEDIA – ‫موسوعه التمريض‬

Maternity Nursing NU.4 (Quiz #1: 75 Questions)

1. Question

1 point(s)

Discharge teaching for a child with celiac disease would include instructions
about avoiding which of the following?

A. Rice

B. Milk

C. Wheat

D. Chicken

Answer: C. Wheat

Children with celiac disease cannot tolerate or digest gluten. Therefore,


because of its gluten content, wheat and wheat-containing products must be
avoided.

Option A: Rice is one of the most popular gluten-free grains for people with
celiac disease. Many gluten-free packaged goods are made with rice instead
of wheat.

Option B: It’s also important to note that while milk is gluten-free, for those
newly diagnosed with celiac disease, secondary lactose intolerance is
common due to the loss of lactase, an enzyme that digests milk sugar along
the lining of the small intestine.

Option D: There are many naturally gluten-free foods to enjoy on the celiac
disease diet, including: Animal proteins: Beef, chicken, dairy products, eggs,
game meat, lamb, pork, seafood, and turkey.
2. Question

1 point(s)

194
C.BY: HOSSAM HAMDY MATERNITY RNPEDIA – ‫موسوعه التمريض‬

Which of the following would the nurse expect to assess in a child with
celiac disease having a celiac crisis secondary to an upper respiratory
infection?

A. Respiratory distress

B. Lethargy

C. Watery diarrhea

D. Weight gain

Answer: C. Watery diarrhea

Episodes of celiac crises are precipitated by infections, ingestion of gluten,


prolonged fasting, or exposure to anticholinergic drugs. Celiac crisis is
typically characterized by severe watery diarrhea. Celiac crisis is a rare
initial presentation of CD characterized by severe diarrhea, dehydration,
weight loss, hypoproteinemia, and metabolic and electrolyte disturbances.
Although rare, it should be considered in patients with apparently
unexplained chronic diarrhea.

Option A: Respiratory distress is unlikely in a routine upper respiratory


infection. Mainly present in children, celiac crisis causes profuse intractable
diarrhea with severe metabolic disturbances (such as acidosis and
hypokalemia), hypotension, neuromuscular weakness, cardiac arrhythmias,
and respiratory failure.

Option B: Irritability, rather than lethargy, is more likely. Due to the wide
variety of symptoms that may present themselves, it can sometimes be
difficult to diagnose celiac disease. One person might have diarrhea and
abdominal pain, while another person has irritability or depression.

Option D: Because of the fluid loss associated with severe watery diarrhea,
the child’s weight is more likely to be decreased.
3. Question

1 point(s)

Which of the following should the nurse do first after noting that a child
with Hirschsprung disease has a fever and watery explosive diarrhea?

A. Notify the physician immediately.


195
C.BY: HOSSAM HAMDY MATERNITY RNPEDIA – ‫موسوعه التمريض‬

B. Administer antidiarrheal medications.

C. Monitor child every 30 minutes.

D. Nothing, this is characteristic of Hirschsprung disease.

Answer: A. Notify the physician immediately.

For the child with Hirschsprung disease, fever and explosive diarrhea
indicate enterocolitis, a life-threatening situation. Therefore, the physician
should be notified immediately.

Option B: Generally, because of the intestinal obstruction and inadequate


propulsive intestinal movement, antidiarrheals are not used to treat
Hirschsprung disease.

Option C: The child is acutely ill and requires intervention, with monitoring
more frequently than every 30 minutes.

Option D: Hirschsprung disease typically presents with chronic


constipation. Hirschsprung’s disease (congenital megacolon) is caused by
the failed migration of colonic ganglion cells during gestation. Varying
lengths of the distal colon are unable to relax, causing functional colonic
obstruction.
4. Question

1 point(s)

A newborn’s failure to pass meconium within the first 24 hours after birth
may indicate which of the following?

A. Hirschsprung disease

B. Celiac disease

C. Intussusception

D. Abdominal wall defect

Answer: A. Hirschsprung disease

196
C.BY: HOSSAM HAMDY MATERNITY RNPEDIA – ‫موسوعه التمريض‬

Failure to pass meconium within the first 24 hours after birth may be an
indication of Hirschsprung disease, a congenital anomaly resulting in
mechanical obstruction due to inadequate motility in an intestinal segment.

Option B: Celiac disease is a serious autoimmune disease that occurs in


genetically predisposed people where the ingestion of gluten leads to damage
in the small intestine. It is estimated to affect 1 in 100 people worldwide.
Two and one-half million Americans are undiagnosed and are at risk for
long-term health complications.

Option C: Intussusception is a process in which a segment of intestine


invaginates into the adjoining intestinal lumen, causing bowel obstruction.
With early diagnosis, appropriate fluid resuscitation, and therapy, the
mortality rate from intussusception in children is less than 1%. If left
untreated, however, this condition is uniformly fatal in 2-5 days.

Option D: Types of abdominal wall defects may include gastroschisis.


Contents of the abdomen protrude out of the body through an opening in the
abdominal muscles near the umbilical cord. Without a protective covering,
the organs are exposed to amniotic fluid and may swell or become damaged.
5. Question

1 point(s)

When assessing a child for possible intussusception, which of the following


would be least likely to provide valuable information?

A. Stool inspection

B. Pain pattern

C. Family history

D. Abdominal palpation

Answer: C. Family history

Because intussusception is not believed to have a familial tendency,


obtaining a family history would provide the least amount of information.

Option A: Later signs include rectal bleeding, often with “red currant jelly”
stool, and lethargy. Physical examination may reveal a “sausage-shaped”
mass.
197
C.BY: HOSSAM HAMDY MATERNITY RNPEDIA – ‫موسوعه التمريض‬

Option B: Early symptoms include periodic abdominal pain, nausea,


vomiting (green from bile), pulling legs to the chest, and cramping
abdominal pain. Pain is intermittent because the bowel segment transiently
stops contracting.

Option D: A sausage-shaped mass may be palpated in the right upper


quadrant. Almost all intussusceptions occur with the intussusceptum having
been located proximally to the intussuscipiens. This is because a peristaltic
action of the intestine pulls the proximal segment into the distal segment.
6. Question

1 point(s)

Nurse Barry is performing Leopold’s maneuver and found the following:


breech presentation, fetal back at the right side of the mother. Based on these
findings, the nurse can hear the fetal heartbeat (PMI) best in which location?

A. Left lower quadrant

B. Right lower quadrant

C. Left upper quadrant

D. Right upper quadrant

Answer: B. Right lower quadrant

Right lower quadrant. The landmark to look for when looking for PMI is the
location of the fetal back in relation to the right or left side of the mother and
the presentation, whether cephalic or breech. The best site is the fetal back
nearest the head.

Option A: The fetal limbs may be palpated at the left lower quadrant.

Option C: The fetal head may be facing the direction of the left upper
quadrant.

Option D: The right upper quadrant has the fetal back, but it is nearer to the
lower extremities of the fetus.
7. Question

1 point(s)
198
C.BY: HOSSAM HAMDY MATERNITY RNPEDIA – ‫موسوعه التمريض‬

In Leopold’s maneuver step #1, the nurse palpated a soft, broad mass that
moves with the rest of the mass. The interpretation of this finding is:

A. The mass palpated at the fundal part is the head part.

B. The presentation is breech.

C. The mass palpated is the back.

D. The mass palpated is the buttocks.

Answer: D. The mass palpated is the buttocks.

The palpated mass is the fetal buttocks since it is broad and soft and moves
with the rest of the mass. The first maneuver also called the fundal grip,
assesses the uterine fundus to determine its height and which fetal pole—that
is, cephalic or podalic—occupies the fundus.

Option A: The head feels hard and round with a smooth surface of uniform
consistency, is very mobile and ballotable.

Option B: The breech gives the sensation of a large, nodular mass, and its
surface is uneven, non-ballotable, and not very mobile. The first maneuver
aims to determine the gestational age and the fetal lie.

Option C: The uterine fundus is pressed with force using one hand, which
accentuates the curvature of the fetal back, allowing for easier palpation with
the other hand. The fetal heart can be auscultated at this time, which can also
provide information on fetal orientation. The heart is well perceived when
the stethoscope or the doppler transducer is placed on the back of the fetus.
8. Question

1 point(s)

In Leopold’s maneuver step #3, the nurse palpated a hard round movable
mass at the suprapubic area. The interpretation is that the mass palpated is:

A. The buttocks because the presentation is breech.

B. The mass palpated is the head.

C. The mass is the fetal back.

199
C.BY: HOSSAM HAMDY MATERNITY RNPEDIA – ‫موسوعه التمريض‬

D. The mass palpated is the small fetal part.

Answer: B. The mass palpated is the head.

When the mass palpated is hard round and movable, it is the fetal head. The
head feels hard and round with a smooth surface of uniform consistency, is
very mobile and ballotable. The third maneuver aids in confirmation of fetal
presentation. The first Pawlík grip, sometimes called the first pelvic grip,
helps to define which presenting part of the fetus is situated in the
hypogastrium.

Option A: The palpated mass is the fetal buttocks since it is broad and soft
and moves with the rest of the mass.

Option C: The uterine fundus is pressed with force using one hand, which
accentuates the curvature of the fetal back, allowing for easier palpation with
the other hand. The fetal heart can be auscultated at this time, which can also
provide information on fetal orientation. The heart is well perceived when
the stethoscope or the doppler transducer is placed on the back of the fetus.

Option D: The second maneuver, sometimes called the umbilical grip,


involves palpation of the lateral uterine surfaces. Still facing the maternal
xiphoid cartilage, both hands slide down from the uterine fundus towards the
lateral uterine walls. The clinician’s hands are placed flat and parallel to each
other along the abdominal wall at the level of the umbilicus. It allows
establishing if the fetus is in a longitudinal, transverse, or oblique situation,
and to determine the position of the back and small parts.
9. Question

1 point(s)

The hormone responsible for a positive pregnancy test is:

A. Estrogen

B. Progesterone

C. Human Chorionic Gonadotropin

D. Follicle Stimulating Hormone

200
C.BY: HOSSAM HAMDY MATERNITY RNPEDIA – ‫موسوعه التمريض‬

Answer: C. Human Chorionic Gonadotropin

Human chorionic gonadotropin (HCG) is the hormone secreted by the


chorionic villi which is the precursor of the placenta. In the early stage of
pregnancy, while the placenta is not yet fully developed, the major hormone
that sustains the pregnancy is HCG.

Option A: Estrogen helps control the menstrual cycle and is important for
childbearing. Estrogen also has other functions: it keeps cholesterol in
control and protects bone health for both women and men.

Option B: Progesterone prepares the endometrium for the potential of


pregnancy after ovulation. It triggers the lining to thicken to accept a
fertilized egg. It also prohibits the muscle contractions in the uterus that
would cause the body to reject an egg.

Option D: In women, FSH helps control the menstrual cycle and stimulates
the growth of eggs in the ovaries. FSH levels in women change throughout
the menstrual cycle, with the highest levels happening just before an egg is
released by the ovary. This is known as ovulation. In men, FSH helps control
the production of sperm.
10. Question

1 point(s)

The hormone responsible for the maturation of the Graafian follicle is:

A. Follicle-stimulating hormone

B. Progesterone

C. Estrogen

D. Luteinizing hormone

Answer: A. Follicle-stimulating hormone

The hormone that stimulates the maturation of the Graafian follicle is the
Follicle Stimulating Hormone which is released by the anterior pituitary
gland.

Option B: Progesterone prepares the endometrium for the potential of


pregnancy after ovulation. It triggers the lining to thicken to accept a
201
C.BY: HOSSAM HAMDY MATERNITY RNPEDIA – ‫موسوعه التمريض‬

fertilized egg. It also prohibits the muscle contractions in the uterus that
would cause the body to reject an egg.

Option C: Estrogen helps control the menstrual cycle and is important for
childbearing. Estrogen also has other functions: it keeps cholesterol in
control and protects bone health for both women and men.

Option D: Luteinizing hormone is crucial in regulating the function of the


testes in men and ovaries in women. In men, luteinizing hormone stimulates
Leydig cells in the testes to produce testosterone, which acts locally to
support sperm production.
11. Question

1 point(s)

The most common normal position of the fetus in utero is:

A. Transverse position

B. Vertical position

C. Oblique position

D. None of the above

Answer: B. Vertical position

Vertical position means the fetal spine is parallel to the maternal spine thus
making it easy for the fetus to go out of the birth canal. Most babies are lying
vertically by the seventh month, with the baby’s head towards the cervix of
the uterus. This is the safest position for normal delivery.

Option A: The transverse lie position is where the fetus’s head is on one
side of the mother’s body and the feet on the other, rather than having the
head close to the cervix or close to the heart. The fetus can also be slightly at
an angle, but still more sideways, than up or down.

Option C: If a fetus is lying diagonally across the uterus, the position is


called oblique. It’s very unusual for a fetus to stay in this position right up
until labor. Only one percent of babies will be transverse or oblique.
12. Question

202
C.BY: HOSSAM HAMDY MATERNITY RNPEDIA – ‫موسوعه التمريض‬

1 point(s)

In the later part of the 3rd trimester, the mother may experience shortness of
breath. This complaint may be explained as:

A. A normal occurrence in pregnancy because the fetus is using more


oxygen.

B. The fundus of the uterus is high pushing the diaphragm upwards.

C. The woman is having an allergic reaction to the pregnancy and its


hormones.

D. The woman may be experiencing complications of pregnancy.

Answer: B. The fundus of the uterus is high pushing the diaphragm


upwards

From the 32nd week of the pregnancy, the fundus of the enlarged uterus is
pushing the respiratory diaphragm upwards. Thus, the lungs have reduced
space for expansion consequently reducing the oxygen supply.

Option A: At the same time that the lung capacity decreases due to the
physical constraint of a growing uterus, the respiratory center in the brain is
stimulated by the hormone progesterone to get the pregnant woman to take
slower breaths. Progesterone is released during pregnancy. Although each
breath may bring in less air, the air stays in the lungs longer so that the
woman can extract the oxygen she and her baby needs.

Option C: Swelling or narrowing of the throat or the airways to the lungs


can cause wheezing. It can also result in shortness of breath because the
lungs can’t hold as much air when they are affected by swelling or mucus
buildup.

Option D: If asthma cannot be controlled, the woman may be at risk for a


serious health problem called preeclampsia. Preeclampsia is a condition that
can happen after the 20th week of pregnancy or right after pregnancy. It’s
when a pregnant woman has high blood pressure and signs that some of her
organs, like her kidneys and liver, may not be working properly. Some of
these signs include having protein in the urine, changes in vision, and severe
headache.
13. Question

203
C.BY: HOSSAM HAMDY MATERNITY RNPEDIA – ‫موسوعه التمريض‬

1 point(s)

Which of the following findings in a woman would be consistent with a


pregnancy of two months duration?

A. Weight gain of 6-10 lbs. And the presence of striae gravidarum.

B. Fullness of the breast and urinary frequency.

C. Braxton Hicks contractions and quickening.

D. Increased respiratory rate and ballottement.

Answer: B. Fullness of the breast and urinary frequency.

The fullness of the breast is due to the increased amount of progesterone in


pregnancy. The urinary frequency is caused by the compression of the
urinary bladder by the gravid uterus which is still within the pelvic cavity
during the first trimester.

Option A: In the first trimester, most women don’t need to gain much
weight — which is good news if she is struggling with morning sickness. If
the woman starts out at a healthy or normal weight, she needs to gain only
about 1 to 4 pounds (0.5 to 1.8 kilograms) in the first few months of
pregnancy.

Option C: Braxton Hicks contractions are sporadic contractions and


relaxation of the uterine muscle. Sometimes, they are referred to as
prodromal or “false labor” pains. It is believed they start around 6 weeks
gestation but usually are not felt until the second or third trimester of the
pregnancy.

Option D: Minute ventilation (V?E) starts to increase significantly (by up to


48%) during the first trimester of gestation, due to higher tidal volume (VT)
with unchanged respiratory rate. This ventilatory pattern is then maintained
throughout the course of pregnancy. Ballottement is a sharp upward push
against the uterine wall with a finger inserted into the vagina for diagnosing
pregnancy by feeling the return impact of the displaced fetus also.
14. Question

1 point(s)

Which of the following is a positive sign of pregnancy?


204
C.BY: HOSSAM HAMDY MATERNITY RNPEDIA – ‫موسوعه التمريض‬

A. Fetal movement felt by mother

B. Enlargement of the uterus

C. (+) pregnancy test

D. (+) ultrasound

Answer: D. (+) ultrasound

A positive ultrasound will confirm that a woman is pregnant since the fetus
in utero is directly visualized.

Option A: The first fetal movements which are felt by the mother are called
quickening. One function of these movements is to alert the pregnant woman
that she has a fetus growing in her uterus. Quickening often occurs between
the 16th to the 22nd week of pregnancy. This is called a presumptive sign of
pregnancy as the other movements of the woman’s body can mimic early
fetal movements such as flatus, peristalsis, and abdominal muscle
contractions.

Option B: From conception to delivery, a woman’s uterus can grow from


the size of a pear to the size of a watermelon. But pregnancy isn’t the only
potential reason for an enlarged uterus. An enlarged uterus is common and
can be a symptom of a variety of medical conditions, some of which require
treatment.

Option C: An elevated ?-hCG in the absence of viable pregnancy can occur


for multiple reasons and has a broad differential diagnosis including
miscarriage, ectopic pregnancy, pituitary hCG production, trophoblastic
disease, and phantom hCG.
15. Question

1 point(s)

What event occurring in the second trimester helps the expectant mother to
accept the pregnancy?

A. Lightening

B. Ballotment

C. Pseudocyesis
205
C.BY: HOSSAM HAMDY MATERNITY RNPEDIA – ‫موسوعه التمريض‬

D. Quickening

Answer: D. Quickening

Quickening is the first fetal movement felt by the mother that makes the
woman realize that she is truly pregnant. In early pregnancy, the fetus is
moving but too weak to be felt by the mother. In the 18th-20th week of
gestation, the fetal movements become stronger thus the mother already feels
the movements.

Option A: Lightening is one of the major signs that labor is approaching. It


happens when the baby’s head literally “drops” lower into the pelvis,
becoming engaged within the pubic bones. This starts the baby’s descent
down and out into the world. Lightening can start as early as a few weeks
before labor actually begins.

Option B: Ballottement is a sharp upward push against the uterine wall with
a finger inserted into the vagina for diagnosing pregnancy by feeling the
return impact of the displaced fetus also.

Option C: Pseudocyesis is defined by the DSM-5 as a false belief of being


pregnant that is associated with objective signs and reported symptoms of
pregnancy, which may include abdominal enlargement, reduced menstrual
flow, amenorrhea, subjective sensation of fetal movement, nausea, breast
engorgement and secretions, and labor pains at the expected date of delivery.
16. Question

1 point(s)

Shoes with low, broad heels, plus a good posture will prevent which prenatal
discomfort?

A. Backache

B. Vertigo

C. Leg cramps

D. Nausea

Answer: A. Backache
206
C.BY: HOSSAM HAMDY MATERNITY RNPEDIA – ‫موسوعه التمريض‬

Backache usually occurs in the lumbar area and becomes more problematic
as the uterus enlarges. The pregnant woman in her third trimester usually
assumes a lordotic posture to maintain balance causing an exaggeration of
the lumbar curvature. Low broad heels provide the pregnant woman with
good support.

Option B: Neurotologists and gynecologists should be aware of pregnant


women with vertigo. To improve the quality of life during gestation,
clinicians should consider this pathology in their differential diagnosis,
which will help avoid vertigo-related undesirable conditions that may
compromise both maternal and fetal status.

Option C: It may have to do with changes in blood circulation and stress on


the leg muscles from carrying extra weight. A growing baby also puts
pressure on the nerves and blood vessels that go to the legs. And some
doctors say low calcium, or a change in the way the body processes calcium,
may cause cramps.

Option D: The pathophysiology of nausea and vomiting during early


pregnancy is unknown, although metabolic, endocrine, GI, and psychologic
factors probably all play a role. Estrogen may contribute because estrogen
levels are elevated in patients with hyperemesis gravidarum.
17. Question

1 point(s)

When a pregnant woman experiences leg cramps, the nursing intervention


to relieve the muscle cramps is:

A. Allow the woman to exercise.

B. Let the woman walk for a while.

C. Let the woman lie down and dorsiflex the foot towards the knees.

D. Ask the woman to raise her legs.

Answer: C. Let the woman lie down and dorsiflex the foot towards the
knees

207
C.BY: HOSSAM HAMDY MATERNITY RNPEDIA – ‫موسوعه التمريض‬

Leg cramps are caused by the contraction of the gastrocnemius (leg muscle).
Thus, the intervention is to stretch the muscle by dorsiflexing the foot of the
affected leg towards the knee.

Option A: During pregnancy, exercise can reduce backaches, constipation,


bloating, and swelling; boost the mood and energy levels; help the woman
sleep better; prevent excess weight gain; and promote muscle tone, strength,
and endurance.

Option B: For most pregnant women, at least 30 minutes of moderate-


intensity exercise is recommended on most, if not all, days of the week.
Walking is a great exercise for beginners. It provides moderate aerobic
conditioning with minimal stress on the joints.

Option D: Elevating the legs is a great way to relieve leg pain during
pregnancy. Lie back on the couch or bed and place both legs on a pillow.
Raising both legs just 6-12 inches above the heart allows gravity to help pull
the blood back toward the heart.
18. Question

1 point(s)

From the 33rd week of gestation till full term, a healthy mother should have
a prenatal check-up every:

A. Week

B. 2 weeks

C. 3 weeks

D. 4 weeks

Answer: A. Week

In the 9th month of pregnancy, the mother needs to have a weekly visit to the
prenatal clinic to monitor fetal condition and to ensure that she is adequately
prepared for the impending labor and delivery.

Option B: Starting from week 28 to week 36 of pregnancy, the woman


should have a prenatal visit every 2 weeks. The healthcare provider will
probably measure the height of the woman’s uterus, which can very much
help to determine how big the baby is growing. The healthcare provider will
208
C.BY: HOSSAM HAMDY MATERNITY RNPEDIA – ‫موسوعه التمريض‬

also listen to her heart rate with a fetal doppler. In addition, if the pregnant
woman did not undergo any screening at her last appointment, the healthcare
provider may recommend that she takes a test like the quad screen, which
can identify possible cases of chromosomal, or other developmental
abnormalities.

Option C: In the third trimester, the pregnant woman will have a prenatal
visit every 2 weeks until week 36. After that, she will see her provider every
week.

Option D: The next prenatal visits — often scheduled about every four
weeks during the first trimester — might be shorter than the first. Near the
end of the first trimester — by about 12 to 14 weeks of pregnancy — the
woman might be able to hear her baby’s heartbeat with a small device that
bounces sound waves off the baby’s heart (Doppler).
19. Question

1 point(s)

The expected weight gain in a normal pregnancy during the 3rd trimester is:

A. 1 pound a week

B. 2 pounds a week

C. 10 lbs a month

D. 10 lbs total weight gain in the 3rd trimester

Answer: A. 1 pound a week

During the 3rd trimester, the fetus is gaining more subcutaneous fat and is
growing fast in preparation for extrauterine life. Thus, one pound a week is
expected.

Option B: In the first trimester, most women don’t need to gain much
weight — which is good news if she is struggling with morning sickness. If a
pregnant woman starts out at a healthy or normal weight, she needs to gain
only about 1 to 4 pounds (0.5 to 1.8 kilograms) in the first few months of
pregnancy.

Option C: Gaining too much weight during pregnancy can increase the
baby’s risk of health problems, such as being born significantly larger than
209
C.BY: HOSSAM HAMDY MATERNITY RNPEDIA – ‫موسوعه التمريض‬

average (fetal macrosomia). The woman might also be at increased risk of


pregnancy-related hypertension, gestational diabetes, prolonged labor, and
the need for a C-section or delivery before her due date. Excessive weight
gain during pregnancy can also increase the risk of postpartum weight
retention and increases the risk of blood clots in the postpartum period.

Option D: Steady weight gain is more important in the second and third
trimesters — especially if a pregnant woman starts out at a healthy weight or
she is underweight. According to the guidelines, the pregnant woman will
gain about 1 pound (0.5 kilogram) a week until delivery. An extra 300
calories a day — half a sandwich and a glass of skim milk — might be
enough to help her meet this goal.
20. Question

1 point(s)

In Bartholomew’s Rule of 4, when the level of the fundus is midway


between the umbilicus and xiphoid process the estimated age of gestation
(AOG) is:

A. 5th month

B. 6th month

C. 7th month

D. 8th month

Answer: C. 7th month

In Bartholomew’s Rule of 4, the landmarks used are the symphysis pubis,


umbilicus, and xiphoid process. At the level of the umbilicus, the AOG is
approximately 5 months and at the level of the xiphoid process 9 months.
Thus, midway between these two landmarks would be considered as 7
months AOG.

Option A: The uterus has been described as a soft and globular pelvic organ.
In pregnancy, the uterus increases in size to accommodate the developing
fetus. At approximately 12 weeks gestation the uterus becomes large enough
to be palpable just above the pubic symphysis.

210
C.BY: HOSSAM HAMDY MATERNITY RNPEDIA – ‫موسوعه التمريض‬

Option B: At 16 weeks gestation, the fundus of the uterus can be palpated at


the midpoint between the umbilicus and the pubic symphysis.

Option D: At 20 weeks gestation, the fundus can be palpable at the level of


the umbilicus. After 20 weeks of gestation, the pubic symphysis to fundal
height in centimeters should correlate with the week of gestation.
21. Question

1 point(s)

The following are ways of determining the expected date of delivery (EDD)
when the LMP is unknown EXCEPT:

A. Naegele’s rule

B. Quickening

C. McDonald’s rule

D. Batholomew’s rule of 4

Answer: A. Naegele’s rule

Naegele’s Rule is determined based on the last menstrual period of the


woman. Establish the date of the last menstrual period by obtaining a history
from the patient. From this date, add 1 year and 7 days, then subtract 3
months. This will approximate the estimated delivery date. The date of the
last known menstrual period will give the approximate start date for the age
of the fetus.

Option B: The first fetal movements which are felt by the mother are called
quickening. One function of these movements is to alert the pregnant woman
that she has a fetus growing in her uterus. Quickening often occurs between
the 16th to the 22nd week of pregnancy. This is called a presumptive sign of
pregnancy as the other movements of the woman’s body can mimic early
fetal movements such as flatus, peristalsis, and abdominal muscle
contractions.

Option C: Fundal height, or McDonald’s rule, is a measure of the size of the


uterus used to assess fetal growth and development during pregnancy. It is
measured from the top of the mother’s uterus to the top of the mother’s pubic
symphysis.
211
C.BY: HOSSAM HAMDY MATERNITY RNPEDIA – ‫موسوعه التمريض‬

Option D: In Bartholomew’s Rule of 4, the landmarks used are the


symphysis pubis, umbilicus, and xiphoid process.
22. Question

1 point(s)

If the LMP is Jan. 30, the expected date of delivery (EDD) is:

A. Oct. 7

B. Oct. 24

C. Nov. 7

D. Nov. 8

Answer: C. Nov. 7

Based on the last menstrual period, the expected date of delivery is Nov. 7.
The formula for Naegele’s Rule is to subtract 3 from the month and add 7 to
the day.

Option A: This is an EDD. An average pregnancy lasts 280 days from the
first day of the last menstrual period (LMP) or 266 days after conception.
Historically, an accurate LMP is the best estimator to determine the due date.

Option B: Oct 24 is . It is prudent for the obstetrician to get a detailed


menstrual history, including duration, flow, previous menstrual periods, and
hormonal contraceptives. These factors are used to determine the length of
her cycles and ovulation period.

Option D: Nov 8 is not the , exact EDD. Having accurate birth dating
might decrease maternal/fetal morbidity and or mortality through timely
consulting with experts in the field of maternal-fetal medicine,
obstetrics/gynecology, oncology, or genetics. For example, fetal genetic
abnormalities can be detected in a timely fashion providing the mother with
sufficient time to make a lifetime decision.
23. Question

1 point(s)

Kegel’s exercise is done in pregnancy in order to:


212
C.BY: HOSSAM HAMDY MATERNITY RNPEDIA – ‫موسوعه التمريض‬

A. Strengthen perineal muscles.

B. Relieve backache.

C. Strengthen abdominal muscles.

D. Prevent leg varicosities and edema.

Answer: A. Strengthen perineal muscles

Kegel’s exercise is done by contracting and relaxing the muscles


surrounding the vagina and anus in order to strengthen the perineal muscles.

Option B: The expanding uterus shifts the center of gravity and stretches out
and weakens the abdominal muscles. This changes the pregnant woman’s
posture and puts a strain on her back. Wear supportive clothing and shoes.
Pay attention to position when sitting, sleeping, and lifting things. If the
woman needs to stand for a long time, rest one foot on a stool or a box to
take the strain off the back. She also can use heat or cold to soothe sore
muscles.

Option C: The abdominal muscles support the spine and play an important
role in the health of the back. During pregnancy, these muscles stretch and
weaken. These changes also can increase the risk of hurting the back during
exercise. Look for an abdominal support garment. It looks like a girdle and
helps take the weight of the belly of the back muscles. Also, some maternity
pants come with a wide elastic band that fits under the curve of the belly to
help support its weight.

Option D: Varicose veins occur when veins of the legs swell. Many changes
in pregnancy can increase the risk of varicose veins, such as increased blood
volume, which enlarges the vein; the heavyweight of the growing baby,
which presses on the large blood vessels in the pelvis; and altering blood
flow. Most varicose veins that develop during pregnancy get better within
the first year after birth. But for now, the pregnant woman should limit
standing or sitting for a long time without a break, and try not to cross the
legs. She should also try to raise her legs and feet whenever she is sitting or
lying down.
24. Question

1 point(s)

213
C.BY: HOSSAM HAMDY MATERNITY RNPEDIA – ‫موسوعه التمريض‬

Pelvic rocking is an appropriate exercise in pregnancy to relieve which


discomfort?

A. Leg cramps

B. Urinary frequency

C. Orthostatic hypotension

D. Backache

Answer: D. Backache

Backache is caused by the stretching of the muscles of the lower back


because of the pregnancy. Pelvic rocking is good to relieve backache.

Option A: Straightening the leg and flexing the foot helps when a pregnant
woman gets a cramp. A gentle massage of the calf may help relax the
muscle. Make sure there is enough fluid intake during the day. If cleared by
the doctor, get regular exercise, which can help reduce cramps.

Option B: HCG increases the blood flow to the pelvic area, which in turn
increases the pregnant woman’s need to pee. The growing uterus also puts
pressure on the bladder, giving it less room to store urine. To prevent
frequent urination, the pregnant woman should already empty her bladder
every time she urinates. She should also skip diuretics like caffeine, and try
to limit fluids right before bedtime.

Option C: It’s not uncommon to have a drop in blood pressure during


pregnancy. Many women don’t realize that pregnancy can have an effect on
blood pressure. It occurs because the circulation expands during pregnancy
and hormonal changes cause the blood vessels to dilate, leading to a
lowering of blood pressure. The woman should take time to wake up slowly
in the morning to prevent dizziness or fainting. If she does feel faint, she
should sit or lie down gently to avoid falling and take steady breaths. Lying
on the left side may also help increase blood flow to the heart, which may
help stabilize the body.
25. Question

1 point(s)

The main reason for an expected increased need for iron in pregnancy is:
214
C.BY: HOSSAM HAMDY MATERNITY RNPEDIA – ‫موسوعه التمريض‬

A. The mother may have physiologic anemia due to the increased need for
red blood cell mass as well as the fetal requires about 350-400 mg of iron to
grow.

B. The mother may suffer anemia because of poor appetite.

C. The fetus has an increased need for RBC which the mother must supply.

D. The mother may have a problem with digestion because of pica.

Answer: A. The mother may have physiologic anemia due to the


increased need for red blood cell mass, as well as the fetal, requires
about 350-400 mg of iron to grow.

About 400 mg of iron is needed by the mother in order to produce more


RBC mass to be able to provide the needed increase in blood supply for the
fetus. Also, about 350-400 mg of iron is needed for the normal growth of the
fetus. Thus, about 750-800 mg iron supplementation is needed by the mother
to meet this additional requirement.

Option B: If the woman loses her appetite, she may experience a general
disinterest in all foods or a lack of desire to eat. Keep in mind that appetite
loss differs from an aversion to a few specific foods, which is also fairly
common during pregnancy. Pregnant women with chronically poor appetites
run a risk of anemia, fetal growth abnormalities, and preterm birth.

Option C: Anemia during pregnancy is especially a concern because it is


associated with low birth weight, premature birth, and maternal mortality.
Women who are pregnant are at a higher risk for developing anemia due to
the excess amount of blood the body produces to help provide nutrients for
the baby.

Option D: Pica is the practice of craving substances with little or no


nutritional value. Most pregnancy and pica-related cravings involve non-
food substances such as dirt or chalk. Eating non-food substances is
potentially harmful to both the mother and the baby. Eating non-food
substances may interfere with the nutrient absorption of healthy food
substances and actually cause a deficiency.
26. Question

1 point(s)

215
C.BY: HOSSAM HAMDY MATERNITY RNPEDIA – ‫موسوعه التمريض‬

The diet that is appropriate in normal pregnancy should be high in:

A. Protein, minerals, and vitamins

B. Carbohydrates and vitamins

C. Proteins, carbohydrates, and fats

D. Fats and minerals

Answer: A. Protein, minerals, and vitamins

In a normal pregnancy, there is a higher demand for protein (bodybuilding


foods), vitamins (esp. vitamin A, B, C, folic acid), and minerals (esp. iron,
calcium, phosphorous, zinc, iodine, magnesium) because of the need of the
growing fetus.

Option B: Maternal glucose is substantially influenced by the type of


carbohydrates in the diet through its direct effect on glycemia. The rate at
which each carbohydrate raises blood glucose levels after ingestion can be
measured via the dietary glycemic index (GI). Carbohydrate type and the GI
of the diet enhance or inhibit abnormal hyperglycemia during pregnancy
caused by either pathological conditions or the inability of the mother to
cope with the physiological IR of pregnancy.

Option C: During pregnancy, the body needs more fat. Roughly 25 percent
to 35 percent of the daily calories should come from fat, depending on the
woman’s carbohydrate goals. Eating monounsaturated fat is preferred over
saturated varieties.

Option D: Unsaturated fats provide vital nutrients to help build and develop
cells in both the woman’s body and her baby’s. Polyunsaturated fats are rich
in omega-3s — EPA (or eicosapentaenoic acid, found in plant sources) and
DHA (or docosahexaenoic acid, found in fish) — to help develop and sustain
the health of the baby’s heart, immune system, brain, eyes, and more. Some
monounsaturated fats may also be a good source of folate, otherwise known
as folic acid, which helps protect the baby against birth defects
27. Question

1 point(s)

216
C.BY: HOSSAM HAMDY MATERNITY RNPEDIA – ‫موسوعه التمريض‬

Which of the following signs will require a mother to seek immediate


medical attention?

A. When the first fetal movement is felt.

B. No fetal movement is felt on the 6th month.

C. Mild uterine contraction.

D. Slight dyspnea on the last month of gestation.

Answer: B. No fetal movement is felt on the 6th month.

Fetal movement is usually felt by the mother during 4.5 – 5 months. If the
pregnancy is already in its 6th month and no fetal movement is felt, the
pregnancy is not normal either the fetus is already dead intra-uterine or it is
an H-mole.

Option A: The first fetal movements which are felt by the mother are called
quickening. One function of these movements is to alert the pregnant woman
that she has a fetus growing in her uterus. Quickening often occurs between
the 16th to the 22nd week of pregnancy. This is called a presumptive sign of
pregnancy as the other movements of the woman’s body can mimic early
fetal movements such as flatus, peristalsis, and abdominal muscle
contractions.

Option C: Sometime during the second or third trimester, the woman might
start to feel mild, sporadic contractions, especially when she is tired or
dehydrated, or after sex. These normal contractions, called Braxton Hicks
contractions or false labor, are the body’s way of rehearsing for birth.

Option D: In the first few weeks of pregnancy, a normal increase in the


hormone progesterone causes the woman to breathe more often. This can
look and feel like shortness of breath. This hormone expands the lung
capacity, allowing blood to carry large quantities of oxygen to the baby.
28. Question

1 point(s)

You want to perform a pelvic examination on one of your pregnant clients.


You prepare your client for the procedure by:

A. Asking her to void.


217
C.BY: HOSSAM HAMDY MATERNITY RNPEDIA – ‫موسوعه التمريض‬

B. Taking her vital signs and recording the readings.

C. Giving the client perineal care.

D. Doing a vaginal prep.

Answer: A. Asking her to void.

A pelvic examination includes abdominal palpation. If the pregnant woman


has a full bladder, the manipulation may cause discomfort and accidental
urination because of the pressure applied during the abdominal palpation.
Also, a full bladder can impede the accuracy of the examination because the
bladder (which is located in front of the uterus) can block the uterus.

Option B: A pelvic exam is a procedure followed by the physician to


examine the vaginal area and the adjoining regions of the pelvis. General
anesthesia is usually given prior to the pelvic exam. Also, the vaginal fluids
may be collected during a pelvic exam to check for any infections. The
vulva, uterus, cervix, ovaries, bladder, or even rectum is checked when a
pregnant woman undergoes a pelvic exam.

Option C: Because the pelvic organs, including the uterus and ovaries, can’t
be seen from outside the body, the doctor needs to feel (palpate) the
abdomen and pelvis for this part of the exam. The doctor will insert two
lubricated, gloved fingers into the vagina with one hand, while the other
hand presses gently on the outside of the lower abdomen.

Option D: During this part of the exam, the doctor will check the size and
shape of the uterus and ovaries, noting any tender areas or unusual growths.
After the vaginal exam, the doctor will insert a gloved finger into the rectum
to check for tenderness, growth, or other irregularities.
29. Question

1 point(s)

When preparing the mother who is in her 4th month of pregnancy for an
abdominal ultrasound, the nurse should instruct her to:

A. Observe NPO from midnight to avoid vomiting.

B. Do perineal flushing properly before the procedure.

218
C.BY: HOSSAM HAMDY MATERNITY RNPEDIA – ‫موسوعه التمريض‬

C. Drink at least 2 liters of fluid 2 hours before the procedure and not void
until the procedure is done.

D. Void immediately before the procedure for better visualization.

Answer: C. Drink at least 2 liters of fluid 2 hours before the procedure


and not void until the procedure is done.

Drinking at least 2 liters of water 2 hours before the procedure will result in
a distended bladder. A full bladder is needed when doing an abdominal
ultrasound to serve as a “window” for the ultrasonic sound waves to pass
through and allow visualization of the uterus (located behind the urinary
bladder).

Option A: Eat a low-fat dinner on the evening before the examination- (no
fried, fatty or greasy foods and no dairy products) and nothing to eat or drink
for 12 hours prior to the appointment.

Option B: There is no need for perineal flushing. The sonographer gently


presses the transducer against the stomach area, moving it back and forth.
The device sends signals to a computer, which creates images that show how
blood flows through the structures in the abdomen.

Option D: Food and liquids in the stomach (and urine in the bladder) can
make it difficult for the technician to get a clear picture of the structures in
the abdomen.
30. Question

1 point(s)

The nursing intervention to relieve “morning sickness” in a pregnant woman


is by giving:

A. Dry carbohydrate food like crackers

B. Low sodium diet

C. Intravenous infusion

D. Antacid

219
C.BY: HOSSAM HAMDY MATERNITY RNPEDIA – ‫موسوعه التمريض‬

Answer: A. Dry carbohydrate food like crackers

Morning sickness may be caused by hypoglycemia early in the morning thus


giving carbohydrate food will help. Foods high in starch — such as saltines,
bread, and toast — help absorb gastric acid and settle a queasy stomach. The
bland nature of a cracker helps to satisfy hunger (excessive hunger can
exacerbate nausea) without the strong smells or tastes that may increase
nausea, according to Erin Palinski-Wade, RD, CDE.

Option B: A beverage containing sodium, such as a broth, may also help to


promote hydration — which is important when the woman may be
dehydrated from vomiting.

Option C: In treating ailments like cramping, electrolyte loss, dehydration,


and nausea, mobile IV drip therapy can effectively and quickly relieve even
the most persistent cases of stomach upset. Nausea, especially when
persistent, can significantly affect daily life. Intravenous fluid administration
(20-30 mL/kg of isotonic sodium chloride 0.9% solution over 1-2 h) may
also be used until oral rehydration is tolerated.

Option D: Antacids containing aluminum, calcium, and magnesium were


not found to be teratogenic in animal studies and are recommended as first-
line treatment of heartburn and acid reflux during pregnancy.
31. Question

1 point(s)

The common normal site of nidation/implantation in the uterus is:

A. Upper uterine portion

B. Mid-uterine area

C. Lower uterine segment

D. Lower cervical segment

Answer: A. Upper uterine portion

The embryo’s normal nidation site is the upper portion of the uterus. If the
implantation is in the lower segment, this is an abnormal condition called
placenta previa.

220
C.BY: HOSSAM HAMDY MATERNITY RNPEDIA – ‫موسوعه التمريض‬

Option B: Implantation begins with apposition of the blastocyst at the


uterine epithelium, generally about 2-4 days after the morula enters the
uterine cavity. The implantation site in the human uterus is usually in the
upper and posterior wall in the midsagittal plane.

Option C: When the implantation takes place in the lower part of the uterus,
the placenta will later develop in the cervix uteri. This type of implantation is
called placenta previa. A birth through the birth canal would detach the
placenta before the fetus is born. This can lead to serious hemorrhages.

Option D: Aberrant implantation in the lower segment of the human uterus


can occur in the cervix, cervico-isthmus (close to internal os of the uterine
cervix), and previous cesarean scars. Cervical pregnancy (CP) is a rare form
of ectopic pregnancy and its incidence is about 1 in 1000 to 1 in 18,000 live
births
32. Question

1 point(s)

Mrs. Santos is on her 5th pregnancy and has a history of abortion in the 4th
pregnancy, and the first pregnancy was a twin. She is considered to be:

A. G 4 P 3

B. G 5 P 3

C. G 5 P 4

D. G 4 P 4

Answer: B. G 5 P 3

Gravida refers to the total number of pregnancies including the current one.
Para refers to the number of pregnancies that have reached viability. Thus, if
the woman has had one abortion, she would be considered Para 3. Twin
pregnancy is counted only as 1.

Option A: Gravida should be 5 since the woman is on her 5th pregnancy.

Option C: Para should be 3 because twin pregnancies are counted as one


and the woman has one abortion.

Option D: Gravida should be 5 since the woman is on her 5th pregnancy.


221
C.BY: HOSSAM HAMDY MATERNITY RNPEDIA – ‫موسوعه التمريض‬

33. Question

1 point(s)

The following are skin changes in pregnancy EXCEPT:

A. Chloasma

B. Striae gravidarum

C. Linea negra

D. Chadwick’s sign

Answer: D. Chadwick’s sign

Chadwick’s sign is bluish discoloration of the vaginal mucosa as a result of


the increased vascularization in the area.

Option A: It is also sometimes known as ‘melasma’ or the ‘mask of


pregnancy’. Chloasma is thought to be due to stimulation of pigment-
producing cells by female sex hormones so that they produce more melanin
pigments (dark colored pigments) when the skin is exposed to the sun.

Option B: Striae gravidarum (SG) are atrophic linear scars that represent
one of the most common connective tissue changes during pregnancy.
Histologically, the appearance of SG is similar to striae distensae (SD) and
contingent on lesion age. Early on, active lesions are comprised
predominantly of fine elastic fibers but aging lesions demonstrate a thinning
of the dermis and decrease of collagen content in the upper dermis

Option C: Linea Nigra is the dark line that develops across the belly during
pregnancy. The specific cause of the linea nigra is unknown, but it is
believed to be related to the changing hormones or the imbalance of
hormones as a result of the developing baby. One theory is that the
melanocyte-stimulating hormone created by the placenta is the main
contributing factor.
34. Question

1 point(s)

Which of the following statements is TRUE of conception?

222
C.BY: HOSSAM HAMDY MATERNITY RNPEDIA – ‫موسوعه التمريض‬

A. Within 2-4 hours after intercourse, conception is possible in a fertile


woman.

B. Generally, fertilization is possible 4 days after ovulation.

C. Conception is possible during menstruation in a long menstrual cycle.

D. To avoid conception, intercourse must be avoided 5 days before and 3


days after menstruation.

Answer: A. Within 2-4 hours after intercourse conception is possible in


a fertile woman.

The sperms when deposited near the cervical os will be able to reach the
fallopian tubes within 4 hours. If the woman has just ovulated (within
24hours after the rupture of the Graafian follicle), fertilization is possible.

Option B: Following ovulation, the egg is capable of fertilization for only


12 to 24 hours. Contact between the egg and sperm is random. Once the egg
arrives at a specific portion of the tube, called the ampullary-isthmic
junction, it rests for another 30 hours.

Option C: Conception during menstrual period is unlikely because the


ovulation time is several days away, decreasing any chances of getting
pregnant during this time. However, there are exceptions. This applies to
women who have a typical 28 to 30 day or longer cycle. If the woman has a
shorter cycle, that means that she may ovulate earlier in the cycle.

Option D: The likelihood of getting pregnant right before menstruation is


extremely low. For women with a typical 28- to 30-day cycle or longer and
their cycles are regular, it is fairly safe to say that ovulation occurred
between Day 11 and Day 21. The egg is only available for 12 to 24 hours for
conception.
35. Question

1 point(s)

Which of the following are the functions of amniotic fluid? Select all that
apply.

A. Cushions the fetus from abdominal trauma

B. Serves as the fluid for the fetus


223
C.BY: HOSSAM HAMDY MATERNITY RNPEDIA – ‫موسوعه التمريض‬

C. Maintains the internal temperature

D. Facilitates fetal movement

Answer: A, B, C, & D

All the four functions enumerated are true of amniotic fluid. Amniotic fluid
surrounds the embryo and fetus during development and has a myriad of
functions.

Option A: Physically, it protects the fetus in the event the maternal abdomen
is the object of trauma. It protects the umbilical cord by providing a cushion
between the fetus and the umbilical cord thus reducing risk of compression
between the fetus and the uterine wall.

Option B: It serves as a reservoir of fluid and nutrients for the fetus


containing: proteins, electrolytes, immunoglobulins, and vitamins from the
mother.
Option C: The fluid insulates the fetus, keeping it warm and maintaining a
regular temperature.

Option D: It provides the necessary fluid, space, and growth factors to allow
normal development and growth of fetal organs such as the musculoskeletal
system, gastrointestinal system, and pulmonary system.
36. Question

1 point(s)

You are performing an abdominal exam on a 9th-month pregnant woman.


While lying supine, she felt breathless, had pallor, tachycardia, and cold
clammy skin. The assessment of the woman’s condition is that she is:

A. Experiencing the beginning of labor.

B. Having supine hypotension.

C. Having sudden elevation of BP.

D. Going into shock.

Answer: B. Having supine hypotension.


224
C.BY: HOSSAM HAMDY MATERNITY RNPEDIA – ‫موسوعه التمريض‬

Supine hypotension is characterized by breathlessness, pallor, tachycardia,


and cold, clammy skin. This is due to the compression of the abdominal
aorta by the gravid uterus when the woman is in a supine position.

Option A: The woman may be at the beginning of labor if the fetus drops or
moves lower into the pelvis, increase in vaginal discharge that is clear, pink,
or slightly bloody, and cervical effacement and dilatation occurs.

Option C: Preeclampsia occurs when hypertension develops after 20 weeks


of pregnancy and is associated with signs of damage to other organ systems,
including the kidneys, liver, blood, or brain. Untreated preeclampsia can lead
to serious — even fatal — complications for mother and baby, including
development of seizures (eclampsia).

Option D: Shock is a state of compromised tissue perfusion that causes


cellular hypoxia and is defined as a syndrome initiated by acute
hypoperfusion, leading to tissue hypoxia and vital organ dysfunction. The
treatment of shock in a pregnant woman differs in two important respects
from the treatment of shock in other adults. [1] First, normal physiologic
changes occur in most organ systems during pregnancy. Second, the mother
and the fetus are both vulnerable during pregnancy. Therefore, obstetric
critical care involves simultaneous assessment and management of the
mother and fetus, who have differing physiological profiles.
37. Question

1 point(s)

Smoking is contraindicated in pregnancy because:

A. Nicotine causes vasodilation of the mother’s blood vessels.

B. Carbon monoxide binds with the hemoglobin of the mother reducing


available hemoglobin for the fetus.

C. The smoke will make the fetus, and the mother feels dizzy.

D. Nicotine will cause vasoconstriction of the fetal blood vessels.

Answer: B. Carbon monoxide binds with the hemoglobin of the mother


reducing available hemoglobin for the fetus.

225
C.BY: HOSSAM HAMDY MATERNITY RNPEDIA – ‫موسوعه التمريض‬

Carbon monoxide is one of the substances found in cigarette smoke. This


substance diminishes the ability of the hemoglobin to bind with oxygen thus
reducing the amount of oxygenated blood reaching the fetus.

Option A: There is blood flow restriction to the placenta due to the


vasoconstrictive effects of catecholamines released from the adrenals and
nerve cells after nicotine activation.

Option C: Nicotine is rapidly absorbed when the tobacco smoke reaches the
small airways and alveoli of the lung. This causes a quick rise in blood
nicotine concentrations, but due to the eventual burnout of the cigarette,
these levels also peak early and thereafter drop to lower levels.

Option D: Direct effects on nicotinic acetylcholine receptors (nAChRs),


which are present and functional very early in the fetal brain [5] are also
likely to contribute.
38. Question

1 point(s)

Which of the following is the most likely effect on the fetus if the woman is
severely anemic during pregnancy?

A. Large for gestational age (LGA) fetus

B. Hemorrhage

C. Small for gestational age (SGA) baby

D. Erythroblastosis fetalis

Answer: C. Small for gestational age (SGA) baby

Anemia is a condition where there is a reduced amount of hemoglobin.


Hemoglobin is needed to supply the fetus with adequate oxygen. Oxygen is
needed for normal growth and development of the fetus.

Option A: Women who gain a lot of weight during pregnancy often give
birth to babies who are large for gestational age. Diabetes in the mother is
the most common cause of babies who are large for gestational age. When a
pregnant woman has high blood sugar, she can pass that along to her baby.

226
C.BY: HOSSAM HAMDY MATERNITY RNPEDIA – ‫موسوعه التمريض‬

Option B: Subchorionic bleeding affects the chorionic membranes. These


lift apart and form another sac between the placenta and the uterus. The
movement and resulting clots are what cause this type of bleeding. These
hematomas can range in size, with the smallest being most common.

Option D: Erythroblastosis fetalis is hemolytic anemia in the fetus (or


neonate, as erythroblastosis neonatorum) caused by transplacental
transmission of maternal antibodies to fetal red blood cells. The disorder
usually results from an incompatibility between maternal and fetal blood
groups, often Rho(D) antigens.
39. Question

1 point(s)

Which of the following signs and symptoms will most likely make the nurse
suspect that the patient has hydatidiform mole?

A. Slight bleeding

B. Passage of clear vesicular mass per vagina

C. Absence of fetal heartbeat

D. Enlargement of the uterus

Answer: B. Passage of clear vesicular mass per vagina

Hydatidiform mole (H-mole) is characterized by the degeneration of the


chorionic villi wherein the villi becomes vesicle-like. These vesicle-like
substances when expelled per vagina and is a definite sign that the woman
has H-mole.

Option A: Implantation bleeding is a common cause of spotting early on in


pregnancy. Implantation bleeding happens when the fertilized egg attaches to
the uterine lining. This can trigger a few days of light bleeding or spotting.

Option C: If the crown-rump length (CRL) is > 7 mm and there is no


embryonic cardiac activity, this is defined as a missed miscarriage, or. If the
mean gestational sac diameter is > 25 mm and there is no yolk sac or
embryonic pole, this is defined as an empty sac miscarriage.

Option D: Two of the most common causes of an enlarged uterus are uterine
fibroids and adenomyosis. Uterine fibroids are commonly noncancerous
227
C.BY: HOSSAM HAMDY MATERNITY RNPEDIA – ‫موسوعه التمريض‬

tumors of the muscular wall of the uterus, affecting as many as eight in 10


women by the age of 50. Fibroids more commonly affect women over age
30.
40. Question

1 point(s)

Upon assessment, the nurse found the following: fundus at 2 fingerbreadths


above the umbilicus, last menstrual period (LMP) 5 months ago, fetal
heartbeat (FHB) not appreciated. Which of the following is the most
possible diagnosis of this condition?

A. Hydatidiform mole

B. Missed abortion

C. Pelvic inflammatory disease

D. Ectopic pregnancy

Answer: A. Hydatidiform mole

Hydatidiform mole begins as a pregnancy but early in the development of


the embryo degeneration occurs. The proliferation of the vesicle-like
substances is rapid causing the uterus to enlarge bigger than the expected
size based on ages of gestation (AOG). In the situation given, the pregnancy
is only 5 months but the size of the uterus is already above the umbilicus
which is compatible with 7 months AOG. Also, no fetal heartbeat is
appreciated because the pregnancy degenerated thus there is no appreciable
fetal heartbeat.

Option B: A missed abortion is a nonviable intrauterine pregnancy that has


been retained within the uterus without spontaneous abortion. Typically, no
symptoms exist besides amenorrhea, and the patient finds out that the
pregnancy stopped developing earlier when a fetal heartbeat is not observed
or heard at the appropriate time. An ultrasound usually confirms the
diagnosis.

Option C: Pelvic inflammatory disease (PID) is an infection of one or more


of the upper reproductive organs, including the uterus, fallopian tubes, and
ovaries. Untreated PID can cause scar tissue and pockets of infected fluid

228
C.BY: HOSSAM HAMDY MATERNITY RNPEDIA – ‫موسوعه التمريض‬

(abscesses) to develop in the reproductive tract, which can cause permanent


damage.

Option D: An ectopic pregnancy is when a fertilized egg implants itself


outside of the womb, usually in one of the fallopian tubes. The fallopian
tubes are the tubes connecting the ovaries to the womb. If an egg gets stuck
in them, it won’t develop into a baby and the mother’s health may be at risk
if the pregnancy continues.
41. Question

1 point(s)

When a pregnant woman goes into a convulsive seizure, the MOST


immediate action of the nurse to ensure the safety of the patient is:

A. Apply restraint so that the patient will not fall out of bed.

B. Put a mouth gag so that the patient will not bite her tongue and the tongue
will not fall back.

C. Position the mother on her side to allow the secretions to drain from her
mouth and prevent aspiration.

D. Check if the woman is also having precipitate labor.

Answer: C. Position the mother on her side to allow the secretions to


drain from her mouth and prevent aspiration.

Positioning the mother on her side will allow the secretions that may
accumulate in her mouth to drain by gravity thus preventing aspiration
pneumonia.

Option A: Placing a patient who is in seizure in restraints would further


injure him or her. Place the patient on a flat, firm surface during seizure.

Option B: Putting a mouth gag is not safe since during the convulsive
seizure the jaw will immediately lock.

Option D: The mother may go into labor also during the seizure, but the
immediate concern of the nurse is the safety of the baby. After the seizure,
check the perineum for signs of precipitate labor.
42. Question
229
C.BY: HOSSAM HAMDY MATERNITY RNPEDIA – ‫موسوعه التمريض‬

1 point(s)

A gravidocardiac mother is advised to observe bed rest primarily to:

A. Allow the fetus to achieve normal intrauterine growth.

B. Minimize oxygen consumption which can aggravate the condition of the


compromised heart of the mother.

C. Prevent perinatal infection.

D. Reduce incidence of premature labor.

Answer: B. Minimize oxygen consumption which can aggravate the


condition of the compromised heart of the mother.

The activity of the mother will require more oxygen consumption. Since the
heart of a gravido-cardiac is compromised, there is a need to put a mother on
bedrest to reduce the need for oxygen.

Option A: In cases of maternal decompensation, fetal monitoring should


also be done to ensure fetal well-being. Women with moderate-risk or high-
risk lesions, especially cyanotic lesions, have an increased risk of fetal
growth restriction and should be followed with monthly ultrasound
examinations for fetal growth.

Option C: The 2011 update to the American Heart Association guideline for
the prevention of cardiovascular disease (CVD) in women recommends that
risk assessment at any stage of life include a detailed history of pregnancy
complications. Gestational diabetes, preeclampsia, preterm birth, and birth of
an infant small for gestational age are ranked as major risk factors for CVD.

Option D: During the third trimester, cardiac output is further influenced by


body position, where the supine position causes caval compression by the
gravid uterus. This leads to a decrease in venous return, which can cause
supine hypotension of pregnancy. Stroke volume normally increases in the
first and second trimester and decreases in the third trimester. This decrease
is due to partial vena cava obstruction.
43. Question

1 point(s)

230
C.BY: HOSSAM HAMDY MATERNITY RNPEDIA – ‫موسوعه التمريض‬

A pregnant mother is admitted to the hospital with the chief complaint of


profuse vaginal bleeding, AOG 36 wks, not in labor. The nurse must always
consider which of the following precautions:

A. The internal exam is done only at the delivery under strict asepsis with a
double set-up.

B. The preferred manner of delivering the baby is vaginal.

C. An emergency delivery set for vaginal delivery must be made ready


before examining the patient.

D. Internal exams must be done following routine procedures.

Answer: A. The internal exam is done only at the delivery under strict
asepsis with a double set-up.

Painless vaginal bleeding during the third trimester may be a sign of placenta
praevia.

Option B: If the bleeding is due to soft tissue injury in the birth canal,
immediate vaginal delivery may still be possible so the set up for vaginal
delivery will be used.

Option C: A double set-up means there is a set-up for cesarean section and a
set-up for vaginal delivery to accommodate immediately the necessary type
of delivery needed. In both cases, strict asepsis must be observed.

Option D: If an internal examination is done in this kind of condition, this


can lead to even more bleeding and may require immediate delivery of the
baby by cesarean section.
44. Question

1 point(s)

Which of the following signs will distinguish threatened abortion from


imminent abortion?

A. Severity of bleeding.

B. Dilation of the cervix.

C. Nature and location of pain.


231
C.BY: HOSSAM HAMDY MATERNITY RNPEDIA – ‫موسوعه التمريض‬

D. Presence of uterine contraction.

Answer: B. Dilation of the cervix

In imminent abortion, the pregnancy will definitely be terminated because


the cervix is already open unlike in threatened abortion where the cervix is
still closed.

Option A: Nearly 25% of pregnant women have some degree of vaginal


bleeding during the first two trimesters and about 50% of these progress to
loss of the pregnancy. The bleeding during a threatened abortion is typically
mild to moderate.

Option C: A threatened abortion occurs when a pregnant patient at less than


20 weeks gestation presents with vaginal bleeding. The cervical os is closed
on a physical exam. The patient may also experience abdominal cramping,
pelvic pain, pelvic pressure, and/or back pain.

Option D: A pelvic exam is mandatory to determine the type of abortion.


Determining factors include the amount and site of bleeding, whether the
cervix is dilated, and whether fetal tissue has passed. In a threatened
abortion, the vaginal exam may reveal a closed cervical os with no tissue.
There is usually no cervical motion tenderness.
45. Question

1 point(s)

The nursing measure to relieve fetal distress due to maternal supine


hypotension is:

A. Place the mother in semi-Fowler's position.

B. Put the mother on the left side-lying position.

C. Place mother on a knee-chest position.

D. Any of the above.

Answer: B. Put the mother on left side-lying position.

232
C.BY: HOSSAM HAMDY MATERNITY RNPEDIA – ‫موسوعه التمريض‬

When a pregnant woman lies in a supine position, the weight of the gravid
uterus would be compressing on the vena cava against the vertebrae
obstructing blood flow from the lower extremities. This causes a decrease in
blood return to the heart and consequently immediate decreased cardiac
output and hypotension. Hence, putting the mother on side-lying will relieve
the pressure exerted by the gravid uterus on the vena cava.

Option A: Placing the pregnant woman in a Semi-Fowler’s position would


still place the weight of the gravid uterus on the vena cava and obstruct the
blood flow from the lower extremities.

Option C: A pregnant woman would be incapable to perform the knee-chest


position due to her gravid belly. The primary treatment used for non-
reassuring fetal status is intrauterine resuscitation. This will help prevent any
unnecessary procedures.
46. Question

1 point(s)

To prevent preterm labor from progressing, drugs are usually prescribed to


halt the labor. The drugs commonly given are:

A. Magnesium sulfate and terbutaline

B. Prostaglandin and oxytocin

C. Progesterone and estrogen

D. Dexamethasone and prostaglandin

Answer: A. Magnesium sulfate and terbutaline

Magnesium sulfate acts as a CNS depressant as well as a smooth muscle


relaxant. Terbutaline is a drug that inhibits the uterine smooth muscles from
contracting. On the other hand, oxytocin and prostaglandin stimulate the
contraction of smooth muscles.

Option B: Prostaglandins act to mediate cervical ripening and to stimulate


uterine contractions and indirectly to increase fundally dominant myometrial
contractility by up-regulation of gap junctions, oxytocin, and arginine
vasopressin receptors, and synchronizations of contractions. Oxytocin
receptor antagonists (ORA), such as atosiban, have been specially developed
233
C.BY: HOSSAM HAMDY MATERNITY RNPEDIA – ‫موسوعه التمريض‬

for the treatment of preterm labor. ORA has been proposed as effective
tocolytic agent for women in preterm labor to prolong pregnancy with fewer
side effects than other tocolytic agents.

Option C: In the first trimester, progesterone produced by the corpus luteum


is critical to the maintenance of early pregnancy until the placenta takes over
this function at 7 to 9 weeks of gestation, hence its name (pro-gestational
steroidal ketone). Indeed, removal of the source of progesterone (the corpus
luteum) or administration of a progesterone receptor antagonist readily
induces abortion before 7 weeks (49 days) of gestation. Estrogen is one of
the key hormones of labor. As the labor comes closer, the high levels of
estrogens stimulate many different processes necessary for delivery. As the
levels of estrogen rise, an increase in oxytocin receptors in the uterus is
stimulated, as well as prostaglandins in the cervix.

Option D: Dexamethasone accelerates maturation of fetal lungs, decreases


the number of neonates with respiratory distress syndrome, and improves
survival in preterm delivered neonates. Optimal gestational age for use of
dexamethasone therapy is 31 to 34 weeks of gestation.
47. Question

1 point(s)

In placenta previa marginalis, the placenta is found at the:

A. Internal cervical os partly covering the opening.

B. External cervical os slightly covering the opening.

C. Lower segment of the uterus with the edges near the internal cervical os.

D. Lower portion of the uterus completely covering the cervix.

Answer: C. Lower segment of the uterus with the edges near the
internal cervical os

Placenta marginalis is a type of placenta previa wherein the placenta is


implanted at the lower segment of the uterus thus the edges of the placenta
are touching the internal cervical opening/os. The normal site of placental
implantation is the upper portion of the uterus.

234
C.BY: HOSSAM HAMDY MATERNITY RNPEDIA – ‫موسوعه التمريض‬

Option A: Marginal placenta previa is where the placental edge is within


2cm of the internal os. Nearly 90% of placentas identified as “low lying”
will ultimately resolve by the third trimester due to placental migration. The
placenta itself does not move but grows toward the increased blood supply at
the fundus, leaving the distal portion of the placenta at the lower uterine
segment with relatively poor blood supply to regress and atrophy.

Option B: The trophoblast adheres to the decidua basalis of the


endometrium, forming a normal pregnancy. Prior uterine scars provide an
environment that is rich in oxygen and collagen. The trophoblast can adhere
to the uterine scar leading to the placenta covering the cervical os or the
placenta invading the walls of the myometrium.

Option D: Migration can also take place by the growing lower uterine
segment thus increasing the distance from the lower margin of the placenta
to the cervix.
48. Question

1 point(s)

In which of the following conditions can the causative agent pass through the
placenta and affect the fetus in utero?

A. Gonorrhea

B. Rubella

C. Candidiasis

D. Moniliasis

Answer: B. Rubella

Rubella is caused by a virus and viruses have low molecular weight thus can
pass through the placental barrier. Relatively few pathogens are capable of
placental and fetal infections in humans and even for these, maternal
infection does not guarantee placental or fetal infection.

Option A: Other STIs, like gonorrhea, chlamydia, hepatitis B, and genital


herpes, can pass from the mother to the baby as the baby passes through the
birth canal. This infection in an infant can cause eye infections, pneumonia,

235
C.BY: HOSSAM HAMDY MATERNITY RNPEDIA – ‫موسوعه التمريض‬

or infections of the joints or blood. Treating gonorrhea as soon as it is


detected in pregnant women will reduce the risk of transmission.

Option C: Candida infection of the fetus results in prematurity and death,


infection can occur across intact membranes, and systemic candidiasis in the
fetus is likely to be associated with an intrauterine device. The pathology of
the placenta includes microscopic granulomata and presence of filaments or
spores on the cord and histological change of the membrane or chorionic
plate revealing intense chorioamnionitis with occasional focal granuloma.

Option D: Bacterial or viral infection of the mother during the course of


pregnancy can cross the placenta and actively infect the fetus. However,
especially for bacteria, it is more common for mothers to experience an
infection that can be treated without overt fetal infection.
49. Question

1 point(s)

Which of the following can lead to infertility in adult males?

A. German measles

B. Orchitis

C. Chickenpox

D. Rubella

Answer: B. Orchitis

Orchitis is a complication that may accompany mumps in adult males. This


condition is characterized by unilateral inflammation of one of the testes
which can lead to atrophy of the affected testis. About 20-30% of males who
get mumps after puberty may develop this complication.

Option A: Since rubella infection is extremely dangerous for unborn babies,


women of childbearing age are advised to undergo rubella immunity testing
before trying to conceive. If immunity cannot be established, they are asked
to be vaccinated. Before infertility treatment, doctors will advise to vaccinate
for rubella due to the harmful effect it has on the unborn baby. It does not
have an effect on the fertility of men.

236
C.BY: HOSSAM HAMDY MATERNITY RNPEDIA – ‫موسوعه التمريض‬

Option C: Chickenpox can affect male fertility, but typically only for a short
time during and after the viral infection. Sperm production and fertility
should return to normal after the viral infection subsides and testicular
inflammation and/or infection passes.

Option D: German measles, also called rubella, does not affect a man’s
fertility. However, in a woman, pregnancy should be delayed for a period of
28 days after the mother is vaccinated against rubella. Since rubella causes a
range of birth defects in babies, fertility specialists are extremely cautious
about rubella immunity testing and vaccinating before they start the fertility
treatment.
50. Question

1 point(s)

Papanicolaou smear is usually done to determine cancer of:

A. Cervix

B. Ovaries

C. Fallopian tubes

D. Breast

Answer: A. Cervix

Papanicolaou (Paps) smear is done to detect cervical cancer. It can’t detect


cancer in ovaries and fallopian tubes because these organs are outside of the
uterus and the abnormal cells from these organs will not be detected from a
smear done on the cervix.

Option B: The 2 tests used most often (in addition to a complete pelvic
exam) to screen for ovarian cancer are transvaginal ultrasound (TVUS) and
the CA-125 blood test. TVUS (transvaginal ultrasound) is a test that uses
sound waves to look at the uterus, fallopian tubes, and ovaries by putting an
ultrasound wand into the vagina.

Option C: Through biopsy, the doctor removes a sample of cells from the
fallopian tubes. A technician in a lab looks at these cells under a microscope
to see if they are cancer. Fallopian tube cancers release a protein called
CA125 that can show up in the blood.
237
C.BY: HOSSAM HAMDY MATERNITY RNPEDIA – ‫موسوعه التمريض‬

Option D: A biopsy is the only definitive way to make a diagnosis of breast


cancer. During a biopsy, the doctor uses a specialized needle device guided
by X-ray or another imaging test to extract a core of tissue from the
suspicious area.
51. Question

1 point(s)

Which of the following causes of infertility in the female is primarily


psychological in origin?

A. Vaginismus

B. Dyspareunia

C. Endometriosis

D. Impotence

Answer: A. Vaginismus

Vaginismus is primarily psychological in origin. Vaginismus is involuntary


contraction of muscles around the opening of the vagina in women with no
abnormalities in the genital organs. The tight muscle contraction makes
sexual intercourse or any sexual activity that involves penetration painful or
impossible.

Option B: Dyspareunia is usually caused by infection, endometriosis, or


hormonal changes in menopause although may sometimes be psychological
in origin. Dyspareunia is pain when sexual intercourse or other sexual
activity that involves penetration is attempted or pain during these activities.

Option C: Endometriosis is a condition that is caused by organic


abnormalities. Endometriosis is defined as the presence of normal
endometrial mucosa (glands and stroma) abnormally implanted in locations
other than the uterine cavity. Approximately 30-40% of women with
endometriosis will be subfertile.

Option D: Erectile dysfunction (impotence) is the inability to get and keep


an erection firm enough for sex. Having erection trouble from time to time
isn’t necessarily a cause for concern. If erectile dysfunction is an ongoing

238
C.BY: HOSSAM HAMDY MATERNITY RNPEDIA – ‫موسوعه التمريض‬

issue, however, it can cause stress, affect self-confidence and contribute to


relationship problems.
52. Question

1 point(s)

Before giving a repeat dose of magnesium sulfate to a pre-eclamptic patient,


the nurse should assess the patient’s condition. Which of the following
conditions will require the nurse to temporarily suspend a repeat dose of
magnesium sulfate?

A. 100 cc. urine output in 4 hours

B. Knee jerk reflex is (+)2

C. Serum magnesium level is 10mEg/L.

D. Respiratory rate of 16/min

Answer: A. 100 cc. urine output in 4 hours

The minimum urine output expected for a repeat dose of MgSO4 is 30 cc/hr.
If in 4 hours the urine output is only 100 cc this is low and can lead to poor
excretion of Magnesium with a possible cumulative effect, which can be
dangerous to the mother.

Option B: As the plasma levels increase the muscle weakness becomes


more pronounced and there is a marked reduction and then loss of deep
tendon reflexes eventually leading to flaccid paralysis and respiratory arrest.

Option C: Magnesium sulfate is the ideal drug for the prevention and
treatment of eclampsia, and, indeed, its universal use is recommended by the
World Health Organization. Nevertheless, the best regimen remains to be
established and there is still no evidence that serum magnesium levels
between 4 and 7?mEq/L, established in a retrospective study and still
considered therapeutic, represent a guarantee that pregnant women with
hypertensive disorders are protected against eclampsia.

Option D: Magnesium sulfate has CNS and respiratory depressant effects. It


acts peripherally, causing vasodilation; moderate doses cause flushing and
sweating, whereas high doses cause hypotension. It prevents or controls
seizures by blocking neuromuscular transmission.
239
C.BY: HOSSAM HAMDY MATERNITY RNPEDIA – ‫موسوعه التمريض‬

53. Question

1 point(s)

Which of the following is TRUE in Rh incompatibility?

A. The condition can occur if the mother is Rh(+) and the fetus is Rh(-).

B. Every pregnancy of an Rh(-) mother will result in erythroblastosis fetalis.

C. On the first pregnancy of the Rh(-) mother, the fetus will not be affected.

D. RhoGam is given only during the first pregnancy to prevent


incompatibility.

Answer: C. On the first pregnancy of the Rh(-) mother, the fetus will not
be affected

On the first pregnancy, the mother still has no contact with Rh(+) blood thus
it has not antibodies against Rh(+). After the first pregnancy, even if
terminated into an abortion, there is already the possibility of mixing of
maternal and fetal blood so this can trigger the maternal blood to produce
antibodies against Rh(+) blood. The fetus takes its blood type usually from
the father.

Option A: The most common cause of Rh incompatibility is exposure from


an Rh-negative mother by Rh-positive fetal blood during pregnancy or
delivery. As a consequence, blood from the fetal circulation may leak into
the maternal circulation, and, after a significant exposure, sensitization
occurs leading to maternal antibody production against the foreign Rh
antigen.

Option B: In women who are prone to Rh incompatibility, the second


pregnancy with an Rh-positive fetus often produces a mildly anemic infant,
whereas succeeding pregnancies produce more seriously affected infants
who ultimately may die in utero from massive antibody-induced hemolytic
anemia.

Option D: The exact mechanism by which passive administration of Rh IgG


prevents Rh immunization is unknown. The most likely hypothesis is that the
Rh immune globulin coats the surface of fetal RBCs containing Rh antigens.
These exogenous antibody-antigen complexes cross the placenta before they

240
C.BY: HOSSAM HAMDY MATERNITY RNPEDIA – ‫موسوعه التمريض‬

can stimulate the maternal endogenous immune system B cells to produce


IgG antibodies.
54. Question

1 point(s)

Which of the following are the most commonly assessed findings in cystitis?

A. Frequency, urgency, dehydration, nausea, chills, and flank pain

B. Nocturia, frequency, urgency dysuria, hematuria, fever, and suprapubic


pain

C. Dehydration, hypertension, dysuria, suprapubic pain, chills, and fever

D. High fever, chills, flank pain nausea, vomiting, dysuria, and frequency

Answer: B. Manifestations of cystitis include, frequency, urgency,


dysuria, hematuria nocturia, fever, and suprapubic pain.

Dehydration, hypertension, and chills are not typically associated with


cystitis. High fever chills, flank pain, nausea, vomiting, dysuria, and
frequency are associated with pyelonephritis.

Option A: Cystitis usually develops due to the colonization of the


periurethral mucosa by bacteria from the fecal or vaginal flora and ascension
of such pathogens to the urinary bladder. Uropathogens may have microbial
virulence factors that allow them to escape host defenses and invade host
tissues in the urinary tract.

Option C: Acute cystitis often presents with urinary symptoms which


include dysuria, urinary frequency urgency, suprapubic pain or tenderness,
and occasionally hematuria. Based on a systematic review examining history
and examination findings of women with uncomplicated UTI, the
combination of dysuria and urinary frequency in the absence of vaginal
discharge or irritation is highly predictive of uncomplicated cystitis.

Option D: Cystitis may be differentiated from pyelonephritis by the absence


of systemic findings such as fever, chills, or sepsis. Findings such as flank
pain, costovertebral angle tenderness, nausea, and vomiting are also more
indicative of upper UTI or pyelonephritis.
55. Question
241
C.BY: HOSSAM HAMDY MATERNITY RNPEDIA – ‫موسوعه التمريض‬

1 point(s)

Which of the following best reflects the frequency of reported postpartum


“blues”?

A. Between 10% and 40% of all new mothers report some form of
postpartum blues.

B. Between 30% and 50% of all new mothers report some form of
postpartum blues.

C. Between 50% and 80% of all new mothers report some form of
postpartum blues.

D. Between 25% and 70% of all new mothers report some form of
postpartum blues.

Answer: C. According to statistical reports, between 50% and 80% of


all new mothers report some form of postpartum blues.

The ranges of 10% to 40%, 30% to 50%, and 25% to 70% are . Postpartum
blues, also known as “baby blues,” affect approximately 50% to 80% of new
mothers.

Option A: Symptoms may include mood swings with times of feeling


anxious, irritable, or tearful interspersed with times of feeling well. Sleeping
difficulties may also occur. The symptoms usually begin 3-4 days after
delivery, worsen by days 5-7, and tend to resolve by day 12.

Option B: For symptoms that last longer than 2 weeks, it is important for the
individual to seek medical attention since approximately 1 in 5 women with
postpartum blues develops postpartum major depression.

Option D: In developed countries, PPD occurs in about 12% to 13% of


postpartum women. More recently, the rates in the United States have been
reported as 10% to20%. Transculturally, the rates are estimated at 10% to
15%, with a higher rate in adolescent mothers.
56. Question

1 point(s)

Which of the following conditions will lead to a small-for-gestational-age


fetus due to less blood supply to the fetus?
242
C.BY: HOSSAM HAMDY MATERNITY RNPEDIA – ‫موسوعه التمريض‬

A. Diabetes in the mother

B. Maternal cardiac condition

C. Premature labor

D. Abruptio placenta

Answer: B. Maternal cardiac condition

In general, when the heart is compromised such as in maternal cardiac


condition, the condition can lead to less blood supply to the uterus
consequently to the placenta which provides the fetus with the essential
nutrients and oxygen. Thus if the blood supply is less, the baby will suffer
from chronic hypoxia leading to a small-for-gestational-age condition.

Option A: Infants of diabetic mothers are prone to various neonatal adverse


outcomes, including metabolic and hematologic disorders, respiratory
distress, cardiac disorders, and neurologic impairment due to perinatal
asphyxia and birth traumas, among others. Macrosomia is the most constant
consequence of diabetes and its severity is mainly influenced by maternal
blood glucose level.

Option C: Preterm labor and delivery are associated with the impaired
neurodevelopmental outcome which includes impaired cognitive abilities,
motor deficits, cerebral palsy, and vision and hearing losses. These risks
increase with decreasing gestational age. Behavioral issues such as anxiety,
depression, autism spectrum disorders, and ADHD also associated with
preterm labor.

Option D: Neonatal consequences include preterm birth and low birth


weight, perinatal asphyxia, stillbirth, and neonatal death. In many countries,
the rate of placental abruption has been increasing, even with improved
obstetrical care and monitoring techniques.
57. Question

1 point(s)

The lower limit of viability for infants in terms of age of gestation is:

A. 21-24 weeks

B. 25-27 weeks
243
C.BY: HOSSAM HAMDY MATERNITY RNPEDIA – ‫موسوعه التمريض‬

C. 28-30 weeks

D. 38-40 weeks

Answer: A. 21-24 weeks

Viability means the capability of the fetus to live/survive outside of the


uterine environment. With the present technological and medical advances,
21 weeks AOG is considered as the minimum fetal age for viability.

Option B: Fetal viability is a major issue that is dependent on the evolution


and progress of modern neonatology (Beauthier, 2007). It is generally
accepted that a 28-week-old fetus that doesn’t need resuscitation is viable.
However, according to WHO, fetal viability is possible after 20 weeks of
fetal life (22 weeks of amenorrhea).

Option C: A simple way to calculate fetal age (in lunar months) is to divide
the fetal length (in cm) by 4 for fetuses less than 5 months’ gestation. If it is
less than 5 months’ gestation the length (in cm) is divided by 5.

Option D: Anthropometric measurements collected during examination of


the fetus are used to estimate its age more accurately (Beauthier, 2011b).
Three types of data can be gathered from radiologic investigations: direct
fetal age estimation from measurement of the length of long bones; fetal age
estimation from measurement of the long bones and calculation of fetal
stature (crown–heel or crown-rump length); and a more difficult method
involving the degree of deciduous teeth calcification; this method requires
the conservation of dental crowns.
58. Question

1 point(s)

A nurse in the labor room is monitoring a client with dysfunctional labor for
signs of maternal or fetal compromise. Which of the following assessment
findings would alert the nurse to a compromise?

A. Coordinated uterine contractions

B. Meconium in the amniotic fluid

C. Progressive changes in the cervix

D. Maternal fatigue
244
C.BY: HOSSAM HAMDY MATERNITY RNPEDIA – ‫موسوعه التمريض‬

Answer: B. Meconium in the amniotic fluid

Signs of maternal or fetal compromise include passage of meconium,


decreased movement felt by the mother, nonreassuring fetal heart rate, and
fetal metabolic acidosis.

Option A: Technically, effective uterine contractions include three factors:


intensity, synchronization, and frequency of contractions. Most studies are
based on single-lead recordings that can reflect the severity and frequency of
uterine contractions. Therefore, uterine synchronization topography can be
used to display labor progress in the labor room.

Option C: A prolonged latent phase may result from oversedation or from


entering labor early with a thickened or uneffaced cervix. It may be
misdiagnosed in the face of frequent prodromal contractions.

Option D: Maternal fatigue can occur with prolonged labor, but do not
indicate maternal or fetal compromise. Fatigue is one of the most common
complaints in pregnant women that often continues until delivery. Maternal
fatigue prolongs the labor process and increases the rate of cesarean section.
Studies on the pattern of uterine contractions have shown that the length of
the fall time is longer in prolonged labors than in normal deliveries.
59. Question

1 point(s)

While assessing a G2P2 client who had a normal spontaneous vaginal


delivery 30 minutes ago, the nurse notes a large amount of red vaginal
bleeding. What would be the initial priority nursing action?

A. Notify the physician.

B. Encourage breastfeed soon after birth.

C. Monitor vital signs

D. Provide fundal massage.

Answer: D. Provide fundal massage

245
C.BY: HOSSAM HAMDY MATERNITY RNPEDIA – ‫موسوعه التمريض‬

Fundal massage also called uterine massage is done to reduce bleeding and
cramping of the uterus after childbirth. This would be the priority nursing
action since it directly addresses the problem.

Option A: The goal is to stop the bleeding immediately, notifying the


physician may come after the nurse’s intervention.

Option B: Breastfeeding the baby will stimulate the release of oxytocin,


which will cause uterine contraction, but it will be slower to do so than the
fundal massage.

Option C: Monitor the vital after the bleeding has stopped or has been
reduced.
60. Question

1 point(s)

The preferred manner of delivering the baby in a gravido-cardiac is vaginal


delivery assisted by forceps under epidural anesthesia. The main rationale
for this is:

A. To allow atraumatic delivery of the baby.

B. To allow a gradual shifting of the blood into the maternal circulation.

C. To make the delivery effort-free and the mother does not need to push
with contractions.

D. To prevent perineal laceration with the expulsion of the fetal head.

Answer: C. To make the delivery effort-free and the mother does not
need to push with contractions.

Forceps delivery under epidural anesthesia will make the delivery process
less painful and require less effort to push for the mother. Pushing requires
more effort which a compromised heart may not be able to endure.

Option A: The benefit of avoiding hemodynamic fluctuations resulting from


pushing must be weighed against an increased risk of perineal trauma,
hemorrhage, and fetal head injury with forceps or vacuum delivery. For most
women, pushing may be preferred above primary assisted delivery.

246
C.BY: HOSSAM HAMDY MATERNITY RNPEDIA – ‫موسوعه التمريض‬

Option B: Several studies show that cesarean section is performed more


often in women with heart disease than in a healthy population. For most
cardiac patients, however, vaginal delivery is preferred and cesarean section
is reserved for obstetric indications since cesarean section is associated with
more blood loss and higher thromboembolic and infection risk.

Option D: Using a technique called super crowning, avoiding an


episiotomy, and reaching for a vacuum device rather than forceps during
operative vaginal deliveries are among the strategies that can help reduce the
number of third- and fourth-degree lacerations.
61. Question

1 point(s)

When giving narcotic analgesics to a mother in labor, the special


consideration to follow is:

A. The progress of labor is well established reaching the transitional stage.

B. Uterine contraction is progressing well, and delivery of the baby is


imminent.

C. Cervical dilatation has already reached at least 8 cm. and the station is at
least (+)2.

D. Uterine contractions are strong and the baby will not be delivered yet
within the next 3 hours.

Answer: D. Uterine contractions are strong and the baby will not be
delivered yet within the next 3 hours.

Narcotic analgesics must be given when uterine contractions are already well
established so that it will not cause stoppage of the contraction thus
protracting labor. Also, it should be given when delivery of a fetus is
imminent or too close because the fetus may suffer respiratory depression as
an effect of the drug that can pass through the placental barrier.

Option A: Opioid analgesia offers a systemic alternative to regional


analgesia procedures. Since the early 1940s, the most commonly used
systemic analgesic has been meperidine (pethidine). As with all opioids,
meperidine crosses the placenta and presents a dose-dependent risk of
neonatal respiratory depression and reduction of fetal heart frequency. The
247
C.BY: HOSSAM HAMDY MATERNITY RNPEDIA – ‫موسوعه التمريض‬

mother may suffer from nausea, vomiting, respiratory depression, dysphoria,


and delayed gastric emptying.

Option B: The effects of systemic opioids in labor are predominantly


sedative rather than analgesic; other opioids, when used in labor, are usually
administered as patient-controlled analgesia.

Option C: Visceral labor pain occurs during the early first stage and the
second stage of childbirth. With each uterine contraction, pressure is
transmitted to the cervix causing stretching and distension and activating
excitatory nociceptive afferents. These afferents innervate the endocervix
and lower segment from T10 – L1.
62. Question

1 point(s)

The cervical dilatation taken at 8:00 AM in a G1P0 patient was 6


centimeters. A repeat I.E. done at 10 A. M. showed that cervical dilation was
7 cm. The interpretation of this result is:

A. Labor is progressing as expected.

B. The latent phase of Stage 1 is prolonged.

C. The active phase of Stage 1 is protracted.

D. The duration of labor is normal.

Answer: C. The active phase of Stage 1 is protracted

The active phase of Stage I starts from 4cm cervical dilatation and is
expected that the uterus will dilate by 1cm every hour. Since the time
elapsed is already 2 hours, the dilatation is expected to be already 8 cm.
Hence, the active phase is protracted.

Option A: In the active phase, the cervix changes more rapidly and
predictably until it reaches 10 centimeters and cervical dilation and
effacement are complete. Active labor with more rapid cervical dilation
generally starts around 6 centimeters of dilation. During the active phase, the
cervix typically dilated at a rate of 1.2 to 1.5 centimeters per hour.

Option B: During the latent phase, the cervix dilates slowly to


approximately 6 centimeters. The latent phase is generally considerably
248
C.BY: HOSSAM HAMDY MATERNITY RNPEDIA – ‫موسوعه التمريض‬

longer and less predictable with regard to the rate of cervical change than is
observed in the active phase. A normal latent phase can last up to 20 hours
and 14 hours in nulliparous and multiparous women respectively, without
being considered prolonged.

Option D: Sedation can increase the duration of the latent phase of labor.
Multiparas, or women with a history of prior vaginal delivery, tend to
demonstrate more rapid cervical dilation. The absence of cervical change for
greater than 4 hours in the presence of adequate contractions or six hours
with inadequate contractions is considered the arrest of labor and may
warrant clinical intervention.
63. Question

1 point(s)

Which of the following techniques during labor and delivery can lead to
uterine inversion?

A. Fundal pressure applied to assist the mother in bearing down during


delivery of the fetal head.

B. Strongly tugging on the umbilical cord to deliver the placenta and hasten
placental separation.

C. Massaging the fundus to encourage the uterus to contract.

D. Applying light traction when delivering the placenta that has already
detached from the uterine wall.

Answer: B. Strongly tugging on the umbilical cord to deliver the


placenta and hasten placental separation.

When the placenta is still attached to the uterine wall, tugging on the cord
while the uterus is relaxed can lead to inversion of the uterus. Light tugging
on the cord when placenta has detached is alright in order to help deliver the
placenta that is already detached.

Option A: The purpose of fundal pressure is to shorten the second stage of


labor. The clinical indications for this maneuver can be fetal distress, failure
to progress in the second stage of labor, and/or maternal exhaustion, or
medical conditions whereby (prolonged) pushing is contraindicated, for
example, maternal heart disease.
249
C.BY: HOSSAM HAMDY MATERNITY RNPEDIA – ‫موسوعه التمريض‬

Option C: Fundal massage, also called uterine massage, is a technique used


to reduce bleeding and cramping of the uterus after childbirth or after an
abortion. As the uterus returns to its nonpregnant size, its muscles contract
strongly, which can cause pain.

Option D: Controlled cord traction (CCT) is traction applied to the


umbilical cord once the woman’s uterus has contracted after the birth of her
baby, and her placenta is felt to have separated from the uterine wall, whilst
counter-pressure is applied to her uterus beneath her pubic bone until her
placenta delivers.
64. Question

1 point(s)

The fetal heart rate is checked following rupture of the bag of waters in order
to:

A. Check if the fetus is suffering from head compression.

B. Determine if cord compression followed the rupture.

C. Determine if there is uteroplacental insufficiency.

D. Check if the fetal presenting part has adequately descended following the
rupture.

Answer: B. Determine if cord compression followed the rupture.

After the rupture of the bag of waters, the cord may also go with the water
because of the pressure of the rupture and flow. If the cord goes out of the
cervical opening, before the head is delivered (cephalic presentation), the
head can compress on the cord causing fetal distress. Fetal distress can be
detected through the fetal heart tone. Thus, it is essential to check the FHB
right after rupture of the bag to ensure that the cord is not being compressed
by the fetal head.

Option A: Head compression during normal uterine contractions may also


result in early decelerations, but usually does not harm the fetus. Early
decelerations are caused by head compression. Head compression results in
fetal vagal stimulation which slows the fetal heart rate during contractions.

250
C.BY: HOSSAM HAMDY MATERNITY RNPEDIA – ‫موسوعه التمريض‬

Option C: Placental insufficiency is a process whereby there is a


progressive deterioration in placental functioning such that oxygen and
nutrient transfer to the fetus via the placenta is decreased, culminating in a
decompensated hypoxia and acidosis. This process leads to fetal hypoxemia
that then stimulates a downregulation of fetal metabolic demands to preserve
what nutrients are already accessible, thus resulting in intrauterine fetal
growth restriction.

Option D: The downward passage of the presenting part through the pelvis
is called descent. This occurs intermittently with contractions. The rate is
greatest during the second stage of labor.
65. Question

1 point(s)

Upon assessment, the nurse got the following findings: two (2) perineal pads
highly saturated with blood within 2 hours postpartum, PR= 80 bpm, fundus
soft, and boundaries not well defined. The appropriate nursing diagnosis is:

A. Normal blood loss

B. Blood volume deficiency

C. Inadequate tissue perfusion related to hemorrhage

D. Hemorrhage secondary to uterine atony

Answer: D. Hemorrhage secondary to uterine atony

All the signs in the stem of the question are signs of hemorrhage. If the
fundus is soft and boundaries not well defined, the cause of the hemorrhage
could be uterine atony.

Option A: It is normal to lose some blood after giving birth. Women usually
lose about half a quart (500 milliliters) during vaginal birth or about 1 quart
(1,000 milliliters) after a cesarean birth.

Option B: Trauma is among the most frequent causes of hypovolemia, with


its often profuse attendant blood loss. The consequences of hypovolemia
include reduction in circulating blood volume, lower venous return, and in
profound cases, arterial hypotension.

251
C.BY: HOSSAM HAMDY MATERNITY RNPEDIA – ‫موسوعه التمريض‬

Option C: Insufficient arterial blood flow causes decreased nutrition and


oxygenation at the cellular level. Decreased tissue perfusion can be
temporary, with few or minimal consequences to the health of the patient, or
it can be more acute or protracted, with potentially destructive effects on the
patient.
66. Question

1 point(s)

The following are signs and symptoms of fetal distress EXCEPT:

A. Fetal heart rate (FHR) decreases during a contraction and persists even
after the uterine contraction ends.

B. The FHR is less than 120 bpm or over 160 bpm.

C. The pre-contraction FHR is 130 bpm, FHR during contraction is 118


bpm, and FHR after uterine contraction is 126 bpm.

D. FHR is 160 bpm, weak and irregular.

Answer: C. The pre-contraction FHR is 130 bpm, FHR during


contraction is 118 bpm, and FHR after uterine contraction is 126 bpm.

The normal range of FHR is 120-160 bpm, strong and regular. During a
contraction, the FHR usually goes down but must return to its pre-
contraction rate after the contraction ends.

Option A: Usually, doctors identify fetal distress based on an abnormal


heart rate pattern in the fetus. Throughout labor, the fetus’s heart rate is
monitored. It is usually monitored continuously with electronic fetal heart
monitoring. Or a handheld Doppler ultrasound device may be used to check
the heart rate every 15 minutes during early labor and after each contraction
during late labor.

Option B: Contractions that are too strong and/or too close together may
cause fetal distress. If oxytocin was used to stimulate contractions, it is
stopped immediately. The woman may be repositioned and given analgesics.
If no drug was used to stimulate contractions, the woman may be given a
drug that can slow labor (such as terbutaline, given by injection) to stop or
slow the contractions.

252
C.BY: HOSSAM HAMDY MATERNITY RNPEDIA – ‫موسوعه التمريض‬

Option D: Fetal rhythm abnormalities, which include fetal heart rates that
are irregular, too fast or too slow, occur in up to 2% of pregnancies and
account for 10–20% of the referrals to fetal cardiologists.
67. Question

1 point(s)

If the labor period lasts only for 3 hours, the nurse should suspect that the
following conditions may occur. Select all that apply.

A. Laceration of cervix

B. Laceration of perineum

C. Cranial hematoma in the fetus

D. Fetal anoxia

Answer: A, B, C, & D

All the above conditions can occur following precipitate labor and delivery
of the fetus because there was little time for the baby to adapt to the
passageway. If the presentation is cephalic, the fetal head serves as the main
part of the fetus that pushes through the birth canal which can lead to cranial
hematoma, and possible compression of the cord may occur which can lead
to less blood and oxygen to the fetus (hypoxia).

Option A: The maternal passageway (cervix, vaginal canal, and perineum)


did not have enough time to stretch which can lead to a laceration.

Option B: Fetal anoxia may occur from inadequate oxygenation of the


mother, low maternal blood pressure, or abnormalities in the uterus,
placenta, or umbilical cord that result in inadequate blood flow to the fetus.
After birth, anoxia may result from blood loss, shock, or inadequate
respiration.

Option C: Fetal intracranial hemorrhage is generally diagnosed in the late


second trimester as an asymmetric echogenic mass within the ventricles,
mostly associated with some degree of ventriculomegaly. Causes to be
considered include drug use (warfarin, cocaine), alloimmune
thrombocytopenia, coagulation disorders, or trauma.

253
C.BY: HOSSAM HAMDY MATERNITY RNPEDIA – ‫موسوعه التمريض‬

Option D: Anoxia occurs when the infant undergoes a total lack of oxygen.
If the brain is deprived of oxygen for even a brief period of time, the infant’s
brain is not getting enough oxygen to function smoothly and will begin to
malfunction.
68. Question

1 point(s)

The primary power involved in labor and delivery is:

A. Bearing down ability of a mother.

B. Cervical effacement and dilatation.

C. Uterine contraction.

D. Valsalva technique.

Answer: C. Uterine contraction

Uterine contraction is the primary force that will expel the fetus out through
the birth canal Maternal bearing down is considered the secondary
power/force that will help push the fetus out.

Option A: During the second stage of labor a common technique is to


encourage women to take a deep breath at the beginning of a contraction
then hold it and bear down throughout the contraction (this is known as
directed pushing). Maternal pushing during the second stage of labor is an
important and indispensable contributor to the involuntary expulsive force
developed by uterine contraction.

Option B: During the first stage of labor, the cervix opens (dilates) and thins
out (effaces) to allow the baby to move into the birth canal. The cervix must
be 100 percent effaced and 10 centimeters dilated before a vaginal delivery.

Option D: When a person forcefully expires against a closed glottis, changes


occur in intrathoracic pressure that dramatically affects venous return,
cardiac output, arterial pressure, and heart rate. This forced expiratory effort
is called a Valsalva maneuver.
69. Question

1 point(s)
254
C.BY: HOSSAM HAMDY MATERNITY RNPEDIA – ‫موسوعه التمريض‬

The proper technique to monitor the intensity of a uterine contraction is:

A. Place the palm of the hands-on the abdomen and time the contraction.

B. Place the fingertips lightly on the suprapubic area and time the
contraction.

C. Put the tip of the fingers lightly on the fundal area and try to indent the
abdominal wall at the height of the contraction.

D. Put the palm of the hands-on the fundal area and feel the contraction at
the fundal area.

Answer: C. Put the tip of the fingers lightly on the fundal area and try to
indent the abdominal wall at the height of the contraction.

In monitoring the intensity of the contraction the best place is to place the
fingertips at the fundal area. The fundus is the contractile part of the uterus
and the fingertips are more sensitive than the palm of the hand.

Option A: Using the fingertips rather than the palm of the hands yields more
accurate results. Fingertips are more sensitive than the palm of the hand.

Option B: Place the fingertips on the fundal area because this area is the
most contractile part of the uterus.

Option D: Use the fingertips instead of the palms of the hand because it is
more sensitive.
70. Question

1 point(s)

To monitor the frequency of the uterine contraction during labor, the right
technique is to time the contraction is:

A. From the beginning of one contraction to the end of the same contraction.

B. From the beginning of one contraction to the beginning of the next


contraction.

C. From the end of one contraction to the beginning of the next contraction.

255
C.BY: HOSSAM HAMDY MATERNITY RNPEDIA – ‫موسوعه التمريض‬

D. From the deceleration of one contraction to the acme of the next


contraction.

Answer: B. From the beginning of one contraction to the beginning of


the next contraction.

The frequency of the uterine contraction is defined as from the beginning of


one contraction to the beginning of another contraction.

Option A: Duration is timed from when the contraction is first felt until it is
over. This time is usually measured in seconds.

Option C: The time between contractions includes the length or duration of


the contraction and the minutes in between the contractions (called the
interval). Mild contractions generally begin 15 to 20 minutes apart and last
60 to 90 seconds. The contractions become more regular until they are less
than 5 minutes apart.

Option D: Acme is the highest point or peak of a contraction. It is the period


during which the contraction is most intense.
71. Question

1 point(s)

The peak point of a uterine contraction is called the:

A. Acceleration

B. Acme

C. Deceleration

D. Axiom

Answer: B. Acme

Acme is the technical term for the highest point of intensity of a uterine
contraction.

Option A: An acceleration is an abrupt increase in FHR above baseline with


onset to peak of the acceleration less than < 30 seconds and less than 2
256
C.BY: HOSSAM HAMDY MATERNITY RNPEDIA – ‫موسوعه التمريض‬

minutes in duration. The duration of the acceleration is defined as the time


from the initial change in heart rate from the baseline to the time of return to
the FHR to baseline.

Option C: A deceleration is a decrease in the fetal heart rate below the fetal
baseline heart rate. Early deceleration is defined as a waveform with a
gradual decrease and returns to baseline with time from onset of the
deceleration to the lowest point of the deceleration (nadir) >30 seconds.

Option D: Axiom is not a part of labor and delivery.


72. Question

1 point(s)

When determining the duration of a uterine contraction the right technique is


to time it from:

A. The beginning of one contraction to the end of the same contraction.

B. The end of one contraction to the beginning of another contraction.

C. The acme point of one contraction to the acme point of another


contraction.

D. The beginning of one contraction to the end of another contraction.

Answer: A. The beginning of one contraction to the end of the same


contraction.

Duration of a uterine contraction refers to one contraction. Thus it is ly


measured from the beginning of one contraction to the end of the same
contraction and not of another contraction.

Option B: The time between contractions includes the length or duration of


the contraction and the minutes in between the contractions (called the
interval). Mild contractions generally begin 15 to 20 minutes apart and last
60 to 90 seconds. The contractions become more regular until they are less
than 5 minutes apart.

Option C: Acme is the technical term for the highest point of intensity of a
uterine contraction.

257
C.BY: HOSSAM HAMDY MATERNITY RNPEDIA – ‫موسوعه التمريض‬

Option D: The frequency of the uterine contraction is defined as from the


beginning of one contraction to the beginning of another contraction.
73. Question

1 point(s)

When the bag of water ruptures, the nurse should check the characteristic of
the amniotic fluid. The normal color of amniotic fluid is:

A. Clear as water

B. Bluish

C. Greenish

D. Yellowish

Answer: A. Clear as water

The normal color of the amniotic fluid is clear like water. If it is yellowish,
there is probably Rh incompatibility. If the color is greenish, it is probably
meconium stained.

Option B: A color change from yellow to blue (nitrazine indicator) indicates


the possible presence of amniotic fluid.

Option C: Fluid that looks green or brown usually means that the baby has
passed his first bowel movement (meconium) while in the womb.

Option D: Amniotic fluid is usually clear to pale yellow in color.


74. Question

1 point(s)

When the bag of waters ruptures spontaneously, the nurse should inspect the
vaginal introitus for possible cord prolapse. If there is part of the cord that
has prolapsed into the vaginal opening the nursing intervention is:

A. Push back the prolapsed cord into the vaginal canal.

B. Place the mother in a semi fowlers position to improve circulation.

258
C.BY: HOSSAM HAMDY MATERNITY RNPEDIA – ‫موسوعه التمريض‬

C. Cover the prolapsed cord with sterile gauze wet with sterile NSS and
place the woman in Trendelenburg position.

D. Push back the cord into the vagina and place the woman in Sim's position.

Answer: C. Cover the prolapsed cord with sterile gauze wet with sterile
NSS and place the woman in Trendelenburg position.

The action of the nurse is to cover the cord with sterile gauze wet with
sterile NSS. Observe strict asepsis in the care of the cord to prevent
infection. The cord has to be kept moist to prevent it from drying. Don’t
attempt to put back the cord into the vagina but relieve pressure on the cord
by positioning the mother either on Trendelenburg or Sims position

Option A: Avoid handling the cord to reduce vasospasm. Manually elevate


the presenting part by lifting the presenting part of the cord by vaginal digital
examination. Alternatively, if in the community, fill the maternal bladder
with 500ml of normal saline (warmed if possible) via a urinary catheter and
arrange immediate hospital transfer.

Option B: Encourage into left lateral position with head down and a pillow
placed under left hip OR knee-chest position. This will relieve pressure off
the cord from the presenting part.

Option D: Umbilical cord prolapse is an acute obstetric emergency that


requires immediate delivery of the baby. The route of delivery is usually by
cesarean section. The doctor will relieve cord compression by manually
elevating the fetal presentation part until a cesarean section is performed.
75. Question

1 point(s)

The fetal heartbeat should be monitored every 15 minutes during the 2nd
stage of labor. The characteristic of a normal fetal heart rate is:

A. The heart rate will decelerate during a contraction and then go back to its
pre-contraction rate after the contraction.

B. The heart rate will accelerate during a contraction and remain slightly
above the pre-contraction rate at the end of the contraction.

C. The rate should not be affected by the uterine contraction.


259
C.BY: HOSSAM HAMDY MATERNITY RNPEDIA – ‫موسوعه التمريض‬

D. The heart rate will decelerate at the middle of a contraction and remain so
for about a minute after the contraction.

Answer: A. The heart rate will decelerate during a contraction and then
go back to its pre-contraction rate after the contraction.

The normal fetal heart rate will decelerate (go down) slightly during a
contraction because of the compression on the fetal head. However, the heart
rate should go back to the pre-contraction rate as soon as the contraction is
over since the compression on the head has also ended.

Option B: The presence of accelerations is considered a reassuring sign of


fetal well-being. An acceleration pattern preceding or following a variable
deceleration (the “shoulders” of the deceleration) is seen only when the fetus
is not hypoxic.

Option C: Uterine contractions can compress the blood vessels in the uterus,
potentially interfering in the transfer of oxygen to the placenta and the baby.
Contractions can also compress the umbilical cord, which may affect the
flow of oxygenated blood to the baby.

Option D: Early decelerations are caused by fetal head compression during


uterine contraction, resulting in vagal stimulation and slowing of the heart
rate. This type of deceleration has a uniform shape, with a slow onset that
coincides with the start of the contraction and a slow return to the baseline
that coincides with the end of the contraction.

260
C.BY: HOSSAM HAMDY MATERNITY RNPEDIA – ‫موسوعه التمريض‬

Maternity Nursing NU.5 (Quiz #1: 75 Questions)

Question

1 point(s)

Mechanism of labor, also known as the cardinal movements, refers to the


sequencing of events involved in posturing and positioning that allows the
baby to find the easiest to come out of the birth canal. Arrange the
following mechanisms of labor in the order during the course of labor
and fetal delivery.

View Answers:

Flexion

Descent

External rotation

Expulsion

Internal rotation

Engagement

Extension

The order is shown above.

The mechanism of fetal delivery begins with descent into the pelvic inlet
which may occur several days before true labor sets in the primigravida.
Flexion, internal rotation, and extension are mechanisms that the fetus must
perform as it accommodates through the passageway/birth canal. Eternal
rotation is done after the head is delivered so that the shoulders will be easily
delivered through the vaginal introitus.
2. Question

1 point(s)
261
C.BY: HOSSAM HAMDY MATERNITY RNPEDIA – ‫موسوعه التمريض‬

The first thing that a nurse must ensure when the baby’s head comes out is

A. The cord is intact

B. No part of the cord is encircling the baby’s neck

C. The cord is still attached to the placenta

D. The cord is still pulsating

Answer: B. No part of the cord is encircling the baby’s neck

The nurse should check right away for possible cord coil around the neck
because if it is present, the baby can be strangled by it and the fetal head will
have difficulty being delivered.

Option A: In a newborn who was born a few hours ago, the cord may look
plump and pale yellow. One of the umbilical arteries may be visible and
protruding from the cut edge. A normal cord has two arteries and one vein.

Option C: The cord is expelled from the mother within a half-hour after
birth. It is still attached to the placenta, which is commonly called “the
afterbirth.” With its function completed, it is no longer needed and so is
discarded by the mother’s body.

Option D: Some cords may pulsate (the pulsation assists the transfer of your
baby’s blood back into their body) for as long as 30 minutes or more, where
others may stop pulsating at 5 minutes or less after the baby is born.
3. Question

1 point(s)

To ensure that the baby will breathe as soon as the head is delivered, the
nurse’s priority action is to

A. Slap the baby’s buttocks to make the baby cry.

B. Suction the nose and mouth to remove mucous secretions.

C. Clamp the cord about 6 inches from the base.

D. Check the baby’s color to make sure it is not cyanotic.

262
C.BY: HOSSAM HAMDY MATERNITY RNPEDIA – ‫موسوعه التمريض‬

Answer: B. Suction the nose and mouth to remove mucous secretions.

Suctioning the nose and mouth of the fetus as soon as the head is delivered
will remove any obstruction that may be present allowing for better
breathing. Also, if mucus is in the nose and mouth, aspiration of the mucus is
possible which can lead to aspiration pneumonia. (Remember that only the
baby’s head has come out as given in the situation.)

Option A: Earlier, many doctors would hold the baby upside down firmly
around his legs and then slap the butt gently. This not only causes slight pain
to the child, but the motion also helps loosen any residues that might be
obstructing the airways. Constantly doing so can irritate the child enough to
begin crying.

Option C: Late cord clamping (performed approximately 1–3 min after


birth) is recommended for all births while initiating simultaneous essential
neonatal care. Early umbilical cord clamping (less than 1 min after birth) is
not recommended unless the neonate is asphyxiated and needs to be moved
immediately for resuscitation.

Option D: When a baby is first born, the skin is a dark red to purple color.
As the baby starts to breathe air, the color changes to red. This redness
normally starts to fade on the first day. A baby’s hands and feet may stay
bluish in color for several days. This is a normal response to a baby’s
underdeveloped blood circulation. But blue coloring of other parts of the
body isn’t normal.
4. Question

1 point(s)

When doing perineal care in preparation for delivery, the nurse should
observe the following, except?

A. Use up-down technique with one stroke.

B. Clean from the mons veneris to the anus.

C. Use mild soap and warm water.

D. Paint the inner thighs going towards the perineal area.

263
C.BY: HOSSAM HAMDY MATERNITY RNPEDIA – ‫موسوعه التمريض‬

Answer: D. Paint the inner thighs going towards the perineal area

Painting of the perineal area in preparation for delivery of the baby must
always be done but the stroke should be from the perineum going outwards
to the thighs. The perineal area is the one being prepared for the delivery and
must be kept clean

Option A: Wipe the perineum in one stroke to prevent the transfer of


infectious microorganisms from the anal area to the perineum.

Option B: Always wash from front to back to prevent spreading fecal matter
from the anal area to the vagina or urethra.

Option C: Use mild soap and warm water. Mild soap would avoid killing
the normal flora that lives in and around the perineum.
5. Question

1 point(s)

What are the important considerations that the nurse must remember after
the placenta is delivered? Select all that apply.

A. Check if the placenta is complete including the membranes

B. Check if the cord is long enough for the baby

C. Check if the umbilical cord has 3 blood vessels

D. Check if the cord has a meaty portion and a shiny portion

Answer: A & C

The nurse after delivering the placenta must ensure that all the cotyledons
and the membranes of the placenta are complete. Also, the nurse must check
if the umbilical cord is normal which means it contains the 3 blood vessels: 1
vein and 2 arteries.

Option B: At term, the typical umbilical cord is 55 to 60 cm in length, with


a diameter of 2.0 to 2.5 cm. The structure should have abundant Wharton’s
jelly, and no true knots or thromboses should be present. The total cord
length should be estimated in the delivery room, since the delivering
physician has access to both the placental and fetal ends.

264
C.BY: HOSSAM HAMDY MATERNITY RNPEDIA – ‫موسوعه التمريض‬

Option C: The maternal surface of the placenta should be dark maroon in


color and should be divided into lobules or cotyledons. The structure should
appear complete, with no missing cotyledons. The fetal surface of the
placenta should be shiny, gray and translucent enough that the color of the
underlying maroon villous tissue may be seen.

Option D: The normal cord contains two arteries and one vein. During the
placental examination, the delivering physician should count the vessels in
either the middle third of the cord or the fetal third of the cord, because the
arteries are sometimes fused near the placenta and are therefore difficult to
differentiate.
6. Question

1 point(s)

The following are statements about false labor, except?

A. The pain is irregular in intensity and frequency.

B. The duration of contraction progressively lengthens over time.

C. There is no bloody vaginal discharge.

D. The cervix is still closed.

Answer: B. The duration of contraction progressively lengthens over


time

In false labor, the contractions remain to be irregular in intensity and


duration while in true labor, the contractions become stronger, longer and
more frequent. Braxton Hicks contractions can be described as tightening in
the abdomen that comes and goes. These contractions do not get closer
together, do not increase in how long they last or how often they occur, and
do not feel stronger over time. They often come with a change of position
and stop with rest.

Option A: In false labor, the pain concentrates in the lower abdomen and
groin. It is irregular in intensity and frequency. The pain often ceases
regardless of the mother’s activity.

Option C: There is no evidence of a bloody show. A bloody show or a


“mucus plug” could mean a cervical change, which means labor is close.
265
C.BY: HOSSAM HAMDY MATERNITY RNPEDIA – ‫موسوعه التمريض‬

Option D: Some women have painful contractions for days with no cervical
changes while other women might feel only a little pressure and backache.
7. Question

1 point(s)

The passageway in labor and delivery of the fetus include the following,
except?

A. Distensibility of lower uterine segment

B. Cervical dilatation and effacement

C. Distensibility of vaginal canal and introitus

D. Flexibility of the pelvis

Answer: D. Flexibility of the pelvis

The pelvis is a bony structure that is part of the passageway but is not
flexible. The lower uterine segment including the cervix as well as the
vaginal canal and introitus are all part of the passageway in the delivery of
the fetus.

Option A: As uterine contractions cause pressure on the membranes, the


hydrostatic action of the amniotic sac in turn dilates the cervical canal like a
wedge. In the absence of intact membranes, the pressure of the presenting
fetal part against the cervix and lower uterine segment is similarly effective.

Option B: Effacement may be compared to a funneling process in which the


whole length of a narrow cylinder is converted into a very obtuse, flaring
funnel with a small circular opening. Because of increased myometrial
activity during uterine preparedness for labor, appreciable effacement of a
softened cervix sometimes is accomplished before active labor begins.
Effacement causes expulsion of the mucous plug as the cervical canal is
shortened. Because the lower segment and cervix have lesser resistance
during a contraction, a centrifugal pull is exerted on the cervix and creates
cervical dilatation.

Option C: The tailbone (sacrum or coccyx) needs to be sufficiently mobile


to be gently pressed back out of the way when the baby moves through. The
sacroiliac joint allows this nutation or counter-nutation of the sacrum. The
266
C.BY: HOSSAM HAMDY MATERNITY RNPEDIA – ‫موسوعه التمريض‬

symphysis pubis is a cartilaginous joint in the front of the pelvis. It also


needs to be properly mobile to help the pelvis flex to allow the baby to pass
through. The relaxin hormone in your body helps both the tailbone and the
symphysis pubis become more mobile to facilitate birth.
8. Question

1 point(s)

The normal umbilical cord is composed of:

A. 2 arteries and 1 vein

B. 2 veins and 1 artery

C. 2 arteries and 2 veins

D. None of the above

Answer: A. 2 arteries and 1 vein

Three vessels comprise the umbilical cord: two umbilical arteries and one
umbilical vein. The umbilical cord is a soft, tortuous cord with a smooth
outer covering of amnion. It extends from the umbilicus of the fetus to the
center of the placenta. Its length ranges from 50 cm to 60 cm, with a
diameter of about 1 cm.[6] The umbilical cord is composed of a gelatinous
ground substance called Wharton’s jelly or substantia gelatinea funiculi
umbilicalis.

Option B: The umbilical arteries carry deoxygenated blood from fetal


circulation to the placenta. The two umbilical arteries converge together
about at 5 mm from the insertion of the cord, forming a type of vascular
connection called the Hyrtl’s anastomosis. The primary function of Hartl’s
anastomosis is to equalize blood flow and pressure between the umbilical
and placental arteries.

Option C: The two umbilical arteries arise from the internal iliac arteries of
the fetus and enter the umbilical cord before further branching at the level of
the placenta. At the placental level, each umbilical artery bifurcates into
smaller arterioles that continue to branch further to distribute blood to the
chorionic villi. The capillaries of the villi fuse to form venules that converge
to form the umbilical vein. The umbilical vein carries oxygenated blood and
nutrients from the mother to the fetus.
267
C.BY: HOSSAM HAMDY MATERNITY RNPEDIA – ‫موسوعه التمريض‬

Option D: The umbilical cord is considered both the physical and emotional
attachment between mother and fetus. This structure allows for the transfer
of oxygen and nutrients from the maternal circulation into fetal circulation
while simultaneously removing waste products from fetal circulation to be
eliminated maternally.
9. Question

1 point(s)

At what stage of labor and delivery does a primigravida differ mainly from a
multigravida?

A. Stage 1

B. Stage 2

C. Stage 3

D. Stage 4

Answer: A. Stage 1

In stage 1 during normal vaginal delivery of a vertex presentation, the


multigravida may have about 8 hours of labor while the primigravida may
have up to 12 hours labor.

Option B: The second stage of labor commences with complete cervical


dilation to 10 centimeters and ends with the delivery of the neonate. In
women who have delivered vaginally previously, whose bodies have
acclimated to delivering a fetus, the second stage may only require a brief
trial, whereas a longer duration may be required for a nulliparous female.

Option C: The third stage of labor commences when the fetus is delivered
and concludes with the delivery of the placenta. Separation of the placenta
from the uterine interface is hallmarked by three cardinal signs including a
gush of blood at the vagina, lengthening of the umbilical cord, and a globular
shaped uterine fundus on palpation.

Option D: During the fourth stage of labor, the baby is born, the placenta
has delivered, and the woman and her partner will probably feel joy, relief,
and fatigue. Most babies are ready to nurse within a short period after birth.
Others wait a little longer. If the woman is planning to breastfeed, it is
268
C.BY: HOSSAM HAMDY MATERNITY RNPEDIA – ‫موسوعه التمريض‬

strongly encouraged to try to nurse as soon as possible after the baby is born.
Nursing right after birth will help the uterus to contract and will decrease the
amount of bleeding.
10. Question

1 point(s)

The second stage of labor begins with ___ and ends with __?

A. Begins with full dilatation of cervix and ends with delivery of placenta.

B. Begins with true labor pains and ends with delivery of the baby.

C. Begins with complete dilatation and effacement of cervix and ends with
delivery of the baby.

D. Begins with passage of show and ends with full dilatation and effacement
of cervix.

Answer: C. Begins with complete dilatation and effacement of cervix


and ends with delivery of baby

Stage 2 of labor and delivery process begins with full dilatation of the cervix
and ends with the delivery of the baby. Stage 1 begins with true labor pains
and ends with full dilatation and effacement of the cervix.

Option A: The second stage of labor commences with complete cervical


dilation to 10 centimeters and ends with the delivery of the neonate. This
was also defined as the pelvic division phase by Friedman. After cervical
dilation is complete, the fetus descends into the vaginal canal with or without
maternal pushing efforts. The fetus passes through the birth canal via 7
movements known as the cardinal movements.

Option B: The first stage of labor begins when labor starts and ends with
full cervical dilation to 10 centimeters. Labor often begins spontaneously or
may be induced medically for a variety of maternal or fetal indications.

Option D: During the latent phase, the cervix dilates slowly to


approximately 6 centimeters. The latent phase is generally considerably
longer and less predictable with regard to the rate of cervical change than is
observed in the active phase. Active labor with more rapid cervical dilation
generally starts around 6 centimeters of dilation. During the active phase, the
269
C.BY: HOSSAM HAMDY MATERNITY RNPEDIA – ‫موسوعه التمريض‬

cervix typically dilated at a rate of 1.2 to 1.5 centimeters per hour.


Multiparas, or women with a history of prior vaginal delivery, tend to
demonstrate more rapid cervical dilation.
11. Question

1 point(s)

The following are signs that the placenta has detached, except?

A. Lengthening of the cord

B. Uterus becomes more globular

C. Sudden gush of blood

D. Mother feels like bearing down

Answer: D. Mother feels like bearing down

Placental detachment does not require the mother to bear down. A normal
placenta will detach by itself without any effort from the mother.

Option A: The most reliable sign is the lengthening of the umbilical cord as
the placenta separates and is pushed into the lower uterine segment by
progressive uterine retraction. Placing a clamp on the cord near the perineum
makes it easier to appreciate this lengthening. Never place traction on the
cord without countertraction on the uterus above the symphysis; otherwise,
one may mistake cord lengthening due to impending prolapse or inversion
for that of uncomplicated placental separation.

Option B: The uterus takes on a more globular shape and becomes firmer.
This occurs as the placenta descends into the lower segment and the body of
the uterus continues to retract. This change may be clinically difficult to
appreciate.

Option C: As the placenta detaches, the spiral arteries are exposed in the
placental bed; massive hemorrhage would occur if not for the structure of the
uterus. The vessels supplying the placental bed traverse a latticework of
crisscrossing muscle bundles that occlude and kink-off the vessels as they
contract and retract following the expulsion of the placenta.
12. Question

270
C.BY: HOSSAM HAMDY MATERNITY RNPEDIA – ‫موسوعه التمريض‬

1 point(s)

When the shiny portion of the placenta comes out first is called which of the
following mechanisms?

A. Marmets

B. Ritgens

C. Duncan

D. Schultze

Answer: D. Schultze

There are 2 mechanisms possible during the delivery of the placenta. If the
shiny portion comes out first, it is called the Schultze mechanism; while if
the meaty portion comes out first, it is called the Duncan mechanism.

Option A: Developed by a mother who needed to express her milk over a


long period of time for medical reasons, the Marmet technique mimics the
actions of a breastfeeding baby and is the most recommended method of
expressing breastmilk by hand.

Option B: Ritgen’s maneuver means that the fetal chin is reached between
the anus and the coccyx and pulled anteriorly while using the fingers of the
other hand on the fetal occiput to control the speed of delivery and keep
flexion of the fetal neck.

Option C: Duncan’s mechanism is the expulsion of the placenta with the


presentation of the maternal rough side first, rather than the usual fetal side
of the placenta.
13. Question

1 point(s)

When the baby’s head is out, the immediate action of the nurse is

A. Cut the umbilical cord

B. Wipe the baby’s face and suction mouth first

C. Check if there is a cord coiled around the neck


271
C.BY: HOSSAM HAMDY MATERNITY RNPEDIA – ‫موسوعه التمريض‬

D. Deliver the anterior shoulder

Answer: C. Check if there is a cord coiled around the neck.

The nurse should check if there is a cord coil because the baby will not be
delivered safely if the cord is coiled around its neck. Normally the umbilical
cord coils to the left. Regardless of its origin, umbilical coiling appears to
confer turgor to the umbilical unit, producing a cord that is strong but
flexible. The role of umbilical cord coiling is not clear; nonetheless, it is
thought to play a role in protecting the umbilical cord from external forces
such as tension, pressure, stretching or entanglement.

Option A: The World Health Organization currently recommends clamping


the umbilical cord between one and three minutes after birth, “for improved
maternal and infant health and nutrition outcomes,” while the American
College of Obstetricians and Gynecologists recommends clamping within 30
to 60 seconds.

Option B: Wiping off the face should be done seconds after you have
ensured that there is no cord coil but suctioning of the nose should be done
after the mouth because the baby is a “nasal obligate” breather. If the nose is
suctioned first before the mouth, the mucus plugging the mouth can be
aspirated by the baby.

Option D: Anterior shoulder in obstetrics refers to that shoulder of the fetus


that faces the pubic symphysis of the mother during delivery. Depending
upon the original position of the fetus, either the left or the right shoulder can
be the anterior shoulder.
14. Question

1 point(s)

When delivering the baby’s head the nurse supports the mother’s perineum
to prevent a tear. This technique is called

A. Marmet’s technique

B. Ritgen’s technique

C. Duncan maneuver

D. Schultze maneuver
272
C.BY: HOSSAM HAMDY MATERNITY RNPEDIA – ‫موسوعه التمريض‬

Answer: B. Ritgen’s technique

Ritgen’s technique is done to prevent the perineal tear. This is done by the
nurse by supporting the perineum with a sterile towel and pushing the
perineum downward with one hand while the other hand is supporting the
baby’s head as it goes out of the vaginal opening.

Option A: Developed by a mother who needed to express her milk over a


long period of time for medical reasons, the Marmet technique mimics the
actions of a breastfeeding baby and is the most recommended method of
expressing breastmilk by hand.

Option C: Duncan’s mechanism is the expulsion of the placenta with the


presentation of the maternal rough side first, rather than the usual fetal side
of the placenta.

Option D: There are 2 mechanisms possible during the delivery of the


placenta. If the shiny portion comes out first, it is called the Schultze
mechanism; while if the meaty portion comes out first, it is called the
Duncan mechanism.
15. Question

1 point(s)

The basic delivery set for normal vaginal delivery includes the following
instruments/articles, except?

A. 2 clamps

B. Pair of scissors

C. Kidney Basin

D. Retractor

Answer: D. Retractor

For normal vaginal delivery, the nurse needs only the instruments for cutting
the umbilical cord such as 2 clamps (straight or curve) and a pair of scissors
as well as the kidney basin to receive the placenta. The retractor is not part of
the basic set. In the hospital setting, needle holders and tissue forceps are
273
C.BY: HOSSAM HAMDY MATERNITY RNPEDIA – ‫موسوعه التمريض‬

added especially if the woman delivering the baby is a primigravida wherein


episiotomy is generally done.

Option A: The clamps are used for clamping the umbilical cord. After the
cord has stopped pulsating, or after at least 1-3 minutes after birth, the first
clamp is placed near the neonate’s umbilicus, then the other one just farther
down the cord.

Option B: Using sterile scissors, cut between the two clamps. Keep in mind
that the cord is thick and hard to cut. One pair will be used to cut the baby’s
umbilical cord. Another will be used if there is a need for an episiotomy. In
that case, the doctor will have to cut into the perineum (the skin between the
vagina and anus) to help the baby fit through.

Option C: The kidney basin is used to receive the placenta. Inspect the
placenta for completeness before disposing it properly.
16. Question

1 point(s)

As soon as the placenta is delivered, the nurse must do which of the


following actions?

A. Inspect the placenta for completeness including the membranes.

B. Place the placenta in a receptacle for disposal.

C. Label the placenta properly.

D. Leave the placenta in the kidney basin for the nursing aide to dispose
properly.

Answer: A. Inspect the placenta for completeness including the


membranes.

The placenta must be inspected for completeness to include the membranes


because an incomplete placenta could mean that there is retention of
placental fragments which can lead to uterine atony. If the uterus does not
contract adequately, hemorrhage can occur.

Option B: During the examination, the size, shape, consistency and


completeness of the placenta should be determined, and the presence of

274
C.BY: HOSSAM HAMDY MATERNITY RNPEDIA – ‫موسوعه التمريض‬

accessory lobes, placental infarcts, hemorrhage, tumors and nodules should


be noted. Once deemed complete, it may be disposed of properly.

Option C: The placenta is not necessarily labeled. For inspection, keep in


mind that the maternal surface of the placenta should be dark maroon in
color and should be divided into lobules or cotyledons. The structure should
appear complete, with no missing cotyledons. The fetal surface of the
placenta should be shiny, gray, and translucent enough that the color of the
underlying maroon villous tissue may be seen.

Option D: Before the proper disposal of the placenta, it should be assessed


properly. Evaluating placental completeness is of critical, immediate
importance in the delivery room. Retained placental tissue is associated with
postpartum hemorrhage and infection.
17. Question

1 point(s)

In vaginal delivery done in the hospital setting, the doctor routinely orders
oxytocin to be given to the mother parenterally. The oxytocin is usually
given after the placenta has been delivered and not before because:

A. Oxytocin will prevent bleeding.

B. Oxytocin can make the cervix close and thus trap the placenta inside.

C. Oxytocin will facilitate placental delivery.

D. Giving oxytocin will ensure complete delivery of the placenta.

Answer: B. Oxytocin can make the cervix close and thus trap the
placenta inside

The action of oxytocin is to make the uterus contract as well make the cervix
close. If it is given prior to placental delivery, the placenta will be trapped
inside because the action of the drug is almost immediate if given
parenterally.

Option A: Active management of the third stage of labor has been shown to
reduce the risk of postpartum hemorrhage. It usually involves the
administration of uterotonic drugs. Although active management has been
shown to reduce the risk of postpartum hemorrhage, it may have an impact
275
C.BY: HOSSAM HAMDY MATERNITY RNPEDIA – ‫موسوعه التمريض‬

on the well-being of the mother and baby in terms of the amount of blood
that has been transfused to the baby before the separation of the placenta.

Option C: In the United States, oxytocin is the uterotonic most often


administered at birth. It is commonly administered after delivery of the
placenta. However, it does not facilitate placental delivery.

Option D: For the prevention of postpartum hemorrhage, and in conjunction


with the other components of active management of the third stage of labor,
oxytocin can be administered with the delivery of the anterior shoulder or
after the delivery of the placenta.
18. Question

1 point(s)

In a gravido-cardiac mother, the first 2 hours postpartum (4th stage of labor


and delivery) particularly in a cesarean section is a critical period because at
this stage

A. There is a fluid shift from the placental circulation to the maternal


circulation which can overload the compromised heart.

B. The maternal heart is already weak and the mother can die.

C. The delivery process is strenuous to the mother.

D. The mother is tired and weak which can distress the heart.

Answer: A. There is a fluid shift from the placental circulation to the


maternal circulation which can overload the compromised heart.

During the pregnancy, there is an increase in maternal blood volume to


accommodate the need of the fetus. When the baby and placenta have been
delivered, there is a fluid shift back to the maternal circulation as part of
physiologic adaptation during the postpartum period. In a cesarean section,
the fluid shift occurs faster because the placenta is taken out right after the
baby is delivered giving it less time for the fluid shift to gradually occur.

Option B: Heart rate increases in a linear fashion during pregnancy by 10 to


20 bpm over baseline and returns to pre-pregnant levels in 6 weeks
postpartum. There is ventricular remodeling during pregnancy and left
ventricular wall thickness and mass increase by 28% to 52% above pre-
276
C.BY: HOSSAM HAMDY MATERNITY RNPEDIA – ‫موسوعه التمريض‬

pregnancy values. Cardiac contractility and ventricular ejection fraction


don’t undergo any significant change during the entire peripartum period.

Option C: There is generalized physical fatigue immediately after delivery.


The pulse rate may be elevated a few hours after the childbirth, due to
excitement or pain, and usually normalizes on the second day. The blood
pressure could be elevated due to pain or excitement but is generally in the
normal range

Option D: Cardiac output increases throughout pregnancy. However, in the


immediate postpartum period, following delivery, there is an increase in
circulating blood volume from the contraction of the uterus and an increase
in preload from the relief of inferior vena cava obstruction, leading to an
increase in stroke volume and heart rate leading to a 60 to 80% rise in
cardiac output, which rapidly declines to pre-labor values in 1 to 2 hours
following delivery and to pre-pregnancy values in two weeks postpartum.
19. Question

1 point(s)

This drug is usually given parenterally to enhance uterine contraction:

A. Terbutaline

B. Pitocin

C. Magnesium sulfate

D. Lidocaine

Answer: B. Pitocin

The common oxytocin given to enhance uterine contraction is Pitocin. This


is also the drug given to induce labor.

Option A: Terbutaline, sold under the brand name Bricanyl among others, is
a ?2 adrenergic receptor agonist, used as a “reliever” inhaler in the
management of asthma symptoms and as a tocolytic (anti-contraction
medication) to delay preterm labor for up to 48 hours.

Option C: Magnesium sulfate is often quite effective in slowing


contractions, although this effect and how long it lasts varies from woman to
woman. Like all tocolytic medications, however, magnesium sulfate does not
277
C.BY: HOSSAM HAMDY MATERNITY RNPEDIA – ‫موسوعه التمريض‬

consistently prevent or delay preterm delivery for a significant period of


time. The most common explanation is that magnesium lowers calcium
levels in uterine muscle cells. Since calcium is necessary for muscle cells to
contract, this is thought to relax the uterine muscle.

Option D: The local anesthetic is transferred to the fetus slowly, and its
margin of safety is also increased. Considering how local anesthetics have
small direct effects on the fetus even at submaximal doses, lidocaine may be
considered relatively safe for use in pregnant women.
20. Question

1 point(s)

The partograph is a tool used to monitor labor. The maternal parameters


measured/monitored are the following, except?

A. Vital signs

B. Fluid intake and output

C. Uterine contraction

D. Cervical dilatation

Answer: B. Fluid intake and output

Partograph is a monitoring tool designed by the World Health Organization


for use by health workers when attending to mothers in labor, especially the
high risk ones. For maternal parameters all of the above is placed in the
partograph except the fluid intake since this is placed in a separate
monitoring sheet. WHO further modified the partograph for the third time.
This simplified partograph is color-coded. The area to the left of the alert
line is colored green representing the normal progress. The area to the right
of the action line is colored red indicating dangerously slow progress. The
area between the alert and action line is colored amber indicating the need
for greater vigilance

Option A: WHO has recommended use of the partograph, a low-tech paper


form that has been hailed as an effective tool for the early detection of
maternal and fetal complications during childbirth. All the recordings for the
maternal condition are entered at the foot of the partograph below the

278
C.BY: HOSSAM HAMDY MATERNITY RNPEDIA – ‫موسوعه التمريض‬

recording of uterine contraction. Maternal vital signs such as temperature,


pulse, BP, urine output and urine for protein and acetone are monitored.

Option C: Below the cervical dilatation, there is a space for recording


uterine contractions per 10 min and the scale is numbered from 1 to 5. Each
square represents one contraction. So if two contractions are felt in 10 min,
two squares are shaded.

Option D: The central feature of the partogram is a graph where cervical


dilatation is plotted. Along the left side, there are squares from 0 to 10, each
representing 1-cm dilatation. Along the bottom of graph are numbers 0–24
each presenting 1 h. The first stage of labor is divided into latent and active
phases. The latent phase is from 0 to 3 cm, and it lasts up to 8 h. The active
phase is from 3 to 10 cm (full cervical dilatation). The dilatation of the
cervix is plotted with “x.”
21. Question

1 point(s)

The following are natural childbirth procedures, except?

A. Lamaze method

B. Dick-Read method

C. Ritgen’s maneuver

D. Psychoprophylactic method

Answer: C. Ritgen’s maneuver

Ritgen’s method is used to prevent perineal tear/laceration during the


delivery of the fetal head. Lamaze method is also known as
psychoprophylactic method and Dick-Read method are commonly known
natural childbirth procedures which advocate the use of nonpharmacologic
measures to relieve labor pain.

Option A: Lamaze breathing historically is considered the hallmark of


Lamaze preparation for childbirth. Controlled breathing enhances relaxation
and decreases the perception of pain. It is one of many comfort strategies
taught in Lamaze classes. In restricted birthing environments, breathing may
be the only non-pharmacological comfort strategy available to women.
279
C.BY: HOSSAM HAMDY MATERNITY RNPEDIA – ‫موسوعه التمريض‬

Conscious breathing and relaxation, especially in combination with a wide


variety of comfort strategies, can help women avoid unnecessary medical
intervention and have a safe, healthy birth.

Option B: The term ‘natural childbirth’ derives from the title of a short 1933
treatise by Grantly Dick-Read. In this and several other books and articles
published over the next quarter-century, the British-born physician outlined
an alternative to the anesthetized, medically controlled way of birth common
among Western women of privilege, based on the premise that fear lay at the
root of pain in labor. For Dick-Read, whether or not a mother experienced
pain in labor depended not on some property inherent to the physiology of
parturition but on cultural attitudes to childbirth.
Through education and relaxation, women could overcome what he termed
the ‘Fear–Tension–Pain’ cycle and labor in comfort without resorting to
medical intervention. Preparation for labor meant providing pregnant women
with detailed instruction, from their physician, midwife, or qualified
childbirth educator, on the physiology of pregnancy and birth, nutrition,
exercise, hygiene, and infant care.

Option D: In the late 1940s, Soviet scientists invented a new non-


pharmacological method called the ‘psychoprophylactic method of painless
childbirth’ (PPM), which later became well known as the Lamaze method in
the West.1 This gift of Soviet science to the women of the world was based
on the assumption that it was possible to eliminate the sensation of bodily
pain during labor by training the mind of a pregnant woman before she gives
birth.
22. Question

1 point(s)

The following are common causes of dysfunctional labor. Which of these


can a nurse, on her own manage?

A. Pelvic bone contraction

B. Full bladder

C. Extension rather than flexion of the head

D. Cervical rigidity

Answer: B. Full bladder


280
C.BY: HOSSAM HAMDY MATERNITY RNPEDIA – ‫موسوعه التمريض‬

A full bladder can impede the descent of the fetal head. The nurse can
readily manage this problem by doing a simple catheterization of the mother.

Option A: The narrower shape of the android pelvis can make labor difficult
because the baby might move more slowly through the birth canal. Some
pregnant women with an android pelvis may require a C-section.

Option C: Abnormal labor could also be secondary to the passenger, the


size of the infant, and/or the presentation of the infant. In addition to
problems caused by the differential in size between the fetal head and the
maternal bony pelvis, the fetal presentation may include asynclitism or head
extension. Asynclitism is malposition of the fetal head within the pelvis,
which compromises the narrowest diameter through the pelvis.

Option D: According to the most recent evidence, arrest of labor in the first
stage should be defined as more than or equal to 6cm dilation with ruptured
membranes and one of the following: 4 hours or more of adequate
contractions (>200 MVU) or 6 hours or more of inadequate contractions and
no cervical change.
23. Question

1 point(s)

At what stage of labor is the mother advised to bear down?

A. When the mother feels the pressure at the rectal area.

B. During a uterine contraction.

C. In between uterine contraction to prevent uterine rupture.

D. Anytime the mother feels like bearing down.

Answer: B. During a uterine contraction

The primary power of labor and delivery is the uterine contraction. This
should be augmented by the mother’s bearing down during a contraction.

Option A: During the second stage of labor, the fetal presentation comes
down and compression occurs in both the bladder and rectum, generating a
reflex that causes a strong urge to bear down, or ‘push’. Therefore, the
combination of involuntary intrauterine contractions and voluntary expulsive

281
C.BY: HOSSAM HAMDY MATERNITY RNPEDIA – ‫موسوعه التمريض‬

effort, through the abdominal and respiratory muscles, will help fetus
delivery.

Option C: Maternal pushing during the second stage of labor is an important


and indispensable contributor to the involuntary expulsive force developed
by uterine contraction.

Option D: Waiting for the urge to push with an epidural does shorten the
duration of pushing and increases spontaneous vaginal delivery, but
lengthens the second stage and doubles the risk of low umbilical cord pH
(based on data from one study).
24. Question

1 point(s)

The normal dilatation of the cervix during the first stage of labor in a
nullipara is

A. 1.2 cm./hr

B. 1.5 cm./hr.

C. 1.8 cm./hr

D. 2.0 cm./hr

Answer: A. 1.2 cm./hr

For nullipara, the normal cervical dilatation should be 1.2 cm/hr. If it is less
than that, it is considered a protracted active phase of the first stage. For
multipara, the normal cervical dilatation is 1.5 cm/hr.

Option B: For nulliparous women, Friedman (Friedman Studies) reported


that the active phase of labor approximates the time from 2.5 cm cervical
dilatation through complete dilatation, approximated at 10 cm. Use of 2.5 cm
dilatation as the onset of active labor was an aggregate estimate and was,
therefore, not strictly applicable to any individual woman.

Option C: Active phase labor was further divided into three sub-phases, i.e.,
an acceleration phase, a phase of maximum slope, and a deceleration phase.
Friedman described the acceleration phase as a rapid change in the slope of
cervical dilation approximating the time needed for the cervix to dilate from
2.5 cm to 4 cm, and the phase of maximum slope as a period of rapid
282
C.BY: HOSSAM HAMDY MATERNITY RNPEDIA – ‫موسوعه التمريض‬

cervical dilation progressing linearly from approximately 4 cm to 9 cm


cervical dilatation. Friedman reported the mean and slowest-yet-normal (i.e.,
mean – 2 standard deviations) cervical dilation rates in the phase of
maximum slope to be 3.0 and 1.2 cm/hr, respectively.

Option D: The deceleration phase was identified when the rate of dilation
once again slowed as full dilatation was reached. For the aggregate of all
labors, this phase approximated the time needed for the cervix to dilate from
9 cm to 10 cm. Friedman included data from some women without a
spontaneous labor onset and some who were not low-risk by modern
standards.
25. Question

1 point(s)

When the fetal head is at the level of the ischial spine, it is said that the
station of the head is

A. Station –1

B. Station “0”

C. Station +1

D. Station +2

Answer: B. Station “0”

Determining is defined as the relationship of the fetal head and the level of
the ischial spine. At the level of the ischial spine, the station is “0”. Above
the ischial spine it is considered (-) station and below the ischial spine it is
(+) station.

Option A: By 6 cm of dilation, the median station was 0 (95% CI ?2 to 1)


for nulliparous and ?1 (95% CI ?3 to 0) for multiparous women. At 8 cm,
95% of nulliparous women were at ?1 station or lower.

Option C: The fetal head is already engaged in station +1. The difference
between numbers in the score is equivalent to the length in centimeters.
Moving from +1 to +2 is a movement of about 1 centimeter.

Option D: +2 to +3 station is crowning and beginning to emerge from the


birth canal.
283
C.BY: HOSSAM HAMDY MATERNITY RNPEDIA – ‫موسوعه التمريض‬

26. Question

1 point(s)

During an internal examination, the nurse palpated the posterior fontanel to


be at the left side of the mother at the upper quadrant. The interpretation is
that the position of the fetus is:

A. LOA

B. ROP

C. LOP

D. ROA

Answer: A. LOA

The landmark used in determining fetal position is the posterior fontanel


because this is the nearest to the occiput. So if the nurse palpated the occiput
(O) at the left (L) side of the mother and at the upper/anterior (A) quadrant
then the fetal position is LOA.

Option B: In the right occiput posterior position (ROP), the baby is facing
forward and slightly to the right (looking toward the mother’s left thigh).
This presentation may slow labor and cause more pain.

Option C: When facing forward, the baby is in the occiput posterior


position. If the baby is facing forward and slightly to the left (looking toward
the mother’s right thigh) it is in the left occiput posterior (LOP) position.
This presentation can lead to more back pain (sometimes referred to as “back
labor”) and slow progression of labor.

Option D: The right occiput anterior (ROA) presentation is also common in


labor. In this position, the back of the baby is slightly off center in the pelvis
with the back of the head toward the mother’s right thigh. In general, OA
positions do not lead to problems or additional pain during labor or birth.
27. Question

1 point(s)

The following are types of breech presentation, except:

284
C.BY: HOSSAM HAMDY MATERNITY RNPEDIA – ‫موسوعه التمريض‬

A. Footling

B. Frank

C. Complete

D. Incomplete

Answer: D. Incomplete

Breech presentation means the buttocks of the fetus is the presenting part. If
it is only the foot/feet, it is considered footling. If only the buttocks, it is a
frank breech. If both the feet and the buttocks are presenting it is called
complete breech.

Option A: The footling breech can have any combination of one or both
hips extended, also known as footling (one leg extended) breech, or double
footling breech (both legs extended).

Option B: In a frank breech, the fetus has flexion of both hips, and the legs
are straight with the feet near the fetal face, in a pike position.

Option C: The complete breech has the fetus sitting with flexion of both
hips and both legs in a tuck position.
28. Question

1 point(s)

When the nurse palpates the suprapubic area of the mother and found that
the presenting part is still movable, the right term for this observation that
the fetus is

A. Engaged

B. Descended

C. Floating

D. Internal Rotation

Answer: C. Floating
285
C.BY: HOSSAM HAMDY MATERNITY RNPEDIA – ‫موسوعه التمريض‬

The term floating means the fetal presenting part has not entered/descended
into the pelvic inlet. If the fetal head has entered the pelvic inlet, it is said to
be engaged.

Option A: If the fetal head accommodates two fingerbreadths above pelvic


brim, it is said to be engaged.

Option B: Using the rule of fifths, the distance between the base and vertex
of the fetal head is divided into five equal parts. Each fifth corresponds to 2
cm or approximately one transverse fingerbreadth.

Option D: As the head descends, the presenting part, usually in the


transverse position, is rotated about 45° to anteroposterior (AP) position
under the symphysis. Internal rotation brings the AP diameter of the head in
line with the AP diameter of the pelvic outlet.
29. Question

1 point(s)

The placenta should be delivered normally within how many minutes after
the delivery of the baby?

A. 5 minutes

B. 30 minutes

C. 45 minutes

D. 60 minutes

Answer: B. 30 minutes

The placenta is delivered within 30 minutes from the delivery of the baby. If
it takes longer, probably the placenta is abnormally adherent and there is a
need to refer already to the obstetrician.

Option A: The absolute time limit for delivery of the placenta, without
evidence of significant bleeding, remains unclear. Periods ranging from 30-
60 minutes have been suggested.

Option C: Retained placenta can be defined as lack of expulsion of the


placenta within 30 minutes of delivery of the infant. This is a reasonable
definition in the third trimester when the third stage of labor is actively
286
C.BY: HOSSAM HAMDY MATERNITY RNPEDIA – ‫موسوعه التمريض‬

managed (ie, administration of a uterotonic agent before delivery of the


placenta, controlled cord traction) because 98 percent of placentas are
expelled by 30 minutes in this setting.

Option D: Physiologic management of the third stage (ie, delivery of the


placenta without the use of uterotonic agents or cord traction) increases the
frequency of retained placenta: only 80 percent of placentas are expelled by
30 minutes and it takes approximately 60 minutes before 98 percent of
placentas are expelled.
30. Question

1 point(s)

When shaving a woman in preparation for cesarean section, the area to be


shaved should be from:

A. Under breast to mid-thigh including the pubic area.

B. The umbilicus to the mid-thigh.

C. Xiphoid process to the pubic area.

D. Above the umbilicus to the pubic area.

Answer: A. Under breast to mid-thigh including the pubic area.

Shaving is done to prevent infection and the area usually shaved should
sufficiently cover the area for surgery, cesarean section. The pubic hair is
definitely to be included in the shaving.

Option B: Infections of surgical incisions are the third most frequently


reported hospital?acquired infections. Women who give birth by cesarean
section are exposed to infection from germs already present on the mother’s
own skin, or from external sources. The risk of infection following a
cesarean section can be 10 times that of vaginal birth. Therefore, preventing
infection by properly preparing the skin before the incision is made is an
important part of the overall care given to women prior to cesarean birth.

Option C: The xiphoid process is definitely not included in shaving prior to


cesarean section. The Centers for Disease Control and Prevention (CDC)
estimates that 27 million surgical procedures are performed in the United
States each year. The CDC’s National Nosocomial Infections Surveillance
287
C.BY: HOSSAM HAMDY MATERNITY RNPEDIA – ‫موسوعه التمريض‬

system reports that surgical site infections are the third most frequently
reported nosocomial infection, accounting for 14% to 16% of all such
infections (CDC 2005). Preventing infection by properly preparing the skin
before the incision is thus a vital part of the overall care given to women
during a cesarean birth.

Option D: Proper preparation of an incision site involves removing surface


dirt and oil with a soap or detergent scrub plus applying a topical
antimicrobial agent that will reduce the bacterial population to a minimal
level. In surgical patients, the choice of surgical scrub and the duration of
scrubbing have not been shown to make any significant difference in the rate
of surgical site infection in either clean or clean?contaminated wounds (such
as cesarean skin incision).
31. Question

1 point(s)

During the postpartum period, the fundus of the uterus is expected to go


down normally about how many centimeters per day?

A. 1.0 cm

B. 2.0 cm

C. 2.5 cm

D. 3.0 cm

Answer: A. 1.0 cm

The uterus will begin involution right after delivery. It is expected to


regress/go down by 1 cm. per day and becomes no longer palpable about 1
week after delivery.

Option B: During the normal puerperium period, the uterine involution is


defined by the changing indices of the uterine size, the uterine cavity inserts,
and the uterine artery flow. Most of the studies publish the first ultrasound
examination findings on the 1st, 2nd, and 3rd postpartum days, but there is
not a single ultrasound study examining the uterus within the first two hours
after delivery.

288
C.BY: HOSSAM HAMDY MATERNITY RNPEDIA – ‫موسوعه التمريض‬

Option C: The most obvious postpartum change is involution of the uterus


from a 1-kg structure with a 5- to 10-L volume to a 60-g structure holding 3
to 5 mL. This involution begins during the third stage of labor, accelerates
after expulsion of the placenta, and continues over the next 5 to 6 weeks.
Typically, the uterus is at the umbilicus after delivery of the placenta, and it
decreases in height by about a centimeter a day until it again becomes a
pelvic organ at about 12 days postpartum. Slower involution continues over
the next several weeks until prepregnant size is attained. Restoration of the
normal endometrial lining occurs by the 16th day postpartum.

Option D: The most intensive uterine involution period is the first month
after delivery. The trend of involution in primiparous and multiparous
women is similar; however, in multiparous women, it lasts longer than 6–8
weeks.
32. Question

1 point(s)

The lochia on the first few days after delivery is characterized as

A. Pinkish with some blood clots

B. Whitish with some mucus

C. Reddish with some mucus

D. Serous with some brown tinged mucus

Answer: C. Reddish with some mucus

Right after delivery, the vaginal discharge called lochia will be reddish
because there is some blood, endometrial tissue, and mucus. Since it is not
pure blood it is non-clotting. Lochia rubra (or cruenta) is the first discharge,
Composed of blood, shreds of fetal membranes, decidua, vernix caseosa,
lanugo and membranes. It is red in color because of the large amount of
blood it contains. It lasts 1 to 4 days after birth, before easing to light
“spotting”.

Option A: Lochia serosa is the term for lochia that has thinned and turned
brownish or pink in color. It contains serous exudate, erythrocytes,
leukocytes, cervical mucus, and microorganisms. This stage continues until
around the tenth day after delivery. Lochia serosa which persists to some
289
C.BY: HOSSAM HAMDY MATERNITY RNPEDIA – ‫موسوعه التمريض‬

weeks after birth can indicate late postpartum hemorrhaging and should be
reported to a physician.

Option B: Lochia alba (or purulenta) is the name for lochia once it has
turned whitish or yellowish-white. It typically lasts from the second through
the third to sixth weeks after delivery. It contains fewer red blood cells and is
mainly made up of leukocytes, epithelial cells, cholesterol, fat, mucus and
microorganisms. Continuation beyond a few weeks can indicate a genital
lesion, which should be reported to a physician.

Option D: Between days four and seven, the blood should turn a pinkish or
brownish color. Clots should get smaller or disappear. By the end of the first
week, the discharge will likely be white or yellow in color. In three to six
weeks, it should stop.
33. Question

1 point(s)

Lochia normally disappears after how many days postpartum?

A. 5 days

B. 7-10 days

C. 18-21 days

D. 28-30 days

Answer: B. 7-10 days

Normally, lochia disappears after 10 days postpartum. What’s important to


remember is that the color of lochia gets to be lighter (from reddish to
whitish) and scantier every day.

Option A: Lochia for the first 3 days after delivery is dark red in color. A
few small blood clots, no larger than a plum, are normal.

Option C: For the fourth through tenth day after delivery, the lochia will be
more watery and pinkish to brownish in color.

Option D: From about the seventh to the tenth day through the fourteenth
day after delivery, the lochia is creamy or yellowish in color. Moms who
have cesarean sections may have less lochia after 24 hours than moms who
290
C.BY: HOSSAM HAMDY MATERNITY RNPEDIA – ‫موسوعه التمريض‬

had vaginal deliveries. The bleeding generally stops within 4 to 6 weeks


after delivery.
34. Question

1 point(s)

After an Rh(-) mother has delivered her Rh (+) baby, the mother is given
RhoGam. This is done in order to:

A. Prevent the recurrence of Rh(+) babies in future pregnancies.

B. Prevent the mother from producing antibodies against the Rh(+) antigen
that she may have gotten when she delivered to her Rh(+) baby.

C. Ensure those future pregnancies will not lead to maternal illness.

D. To prevent the newborn from having problems of incompatibility when it


breastfeeds.

Answer: B. Prevent the mother from producing antibodies against the


Rh(+) antigen that she may have gotten when she delivered to her Rh(+)
baby

In Rh incompatibility, a Rh(-) mother will produce antibodies against the


fetal Rh (+) antigen which she may have gotten because of the mixing of
maternal and fetal blood during labor and delivery. Giving her RhoGam right
after birth will prevent her immune system from being permanently
sensitized to Rh antigen.

Option A: RhoGAM is a prescription medicine that is used to prevent Rh


immunization, a condition in which an individual with Rh-negative blood
develops antibodies after exposure to Rh-positive blood.

Option C: RhoGAM prevents the Rh-negative mother from making


antibodies directed against her baby’s Rh-positive red blood cells during her
pregnancy.

Option D: Rho(D) immune globulin is immune globulin (IgG) rich in IgG


antibodies against erythrocyte antigen Rho(D). IgG is a normal component
of breastmilk. Rho(D) immune globulin is frequently used in nursing
mothers and no adverse effects have been reported in breastfed infants. No
special precautions are required.
291
C.BY: HOSSAM HAMDY MATERNITY RNPEDIA – ‫موسوعه التمريض‬

35. Question

1 point(s)

To enhance milk production, a lactating mother must do the following


interventions, except:

A. Increase fluid intake including milk.

B. Eat foods that increase lactation which is called galactagogues.

C. Exercise adequately like aerobics.

D. Have adequate nutrition and rest.

Answer: C. Exercise adequately like aerobics.

All the above nursing measures are needed to ensure that the mother is in a
healthy state. However, aerobics does not necessarily enhance lactation.

Option A: It is widely assumed that milk production requires a high fluid


intake on the part of the mother, yet the evidence suggests that lactating
women can tolerate a considerable amount of water restriction and that
supplemental fluids have little effect on milk volume. However, thirst may
sometimes function too slowly to prevent dehydration among women with
high fluid losses resulting from exercise or high ambient temperature
(experienced by many women without air conditioning in the summer).

Option B: A galactagogue or galactogogue (pronounced gah-lak´tah-gog) is


something that can help a breastfeeding mother to increase her breast milk
supply. The word itself is a combination of the Greek terms “galact-”
meaning milk, and “-agogue” meaning leading to or promoting. Herbs are
commonly used to boost low milk supply, but certain actions, foods, and
medications can help a breastfeeding mom make more breast milk as well.

Option D: Maternal anxiety and stress, which may be exacerbated by poor


lactation management, are believed to influence milk production by
inhibiting the milk-ejection reflex. This reflex usually operates well in
women who are relaxed and confident of their ability to breastfeed. In tense
women, however, the reflex may be impaired. Early studies in humans by
Gopalan (1958) and Edozien et al. (1976) suggest the same relationship:
milk output of women in India and Nigeria increased when protein intake
was increased from 50 to 60 g/day to approximately 100 g/day.
292
C.BY: HOSSAM HAMDY MATERNITY RNPEDIA – ‫موسوعه التمريض‬

36. Question

1 point(s)

The nursing intervention to relieve pain in breast engorgement while the


mother continues to breastfeed is

A. Apply cold compress on the engorged breast.

B. Apply warm compress on the engorged breast.

C. Massage the breast.

D. Apply analgesic ointment.

Answer: B. Apply warm compress on the engorged breast

Warm compress is applied if the purpose is to relieve pain but ensure


lactation to continue. If the purpose is to relieve pain as well as suppress
lactation, the compress applied is cold.

Option A: Using cold packs on the affected breast can help reduce swelling
and relieve pain. Use warm packs just before a feed (for up to a few minutes)
to help trigger the let-down reflex to help clear the blockage and may relieve
pain.

Option C: Gentle massage by stroking toward the nipple while the baby
feeds may help in draining the breast of too much milk.

Option D: The doctor may recommend an over-the-counter pain reliever,


such as acetaminophen or ibuprofen. Ointments may interfere with the
infant’s breastfeeding.
37. Question

1 point(s)

A woman who delivered normally per vaginam is expected to void within


how many hours after delivery?

A. 3 hrs

B. 4 hrs

293
C.BY: HOSSAM HAMDY MATERNITY RNPEDIA – ‫موسوعه التمريض‬

C. 6-8 hrs

D. 12-24 hours

Answer: C. 6-8 hrs

A woman who has had normal delivery is expected to void within 6-8 hrs. If
she is unable to do so after 8 hours, the nurse should stimulate the woman to
void. If nursing interventions to stimulate spontaneous voiding don’t work,
the nurse may decide to catheterize the woman.

Option A: The precise pathophysiology of PPUr is still unknown; however,


it is likely to be multifactorial, including physiological, neurological, and
mechanical processes in the postpartum period

Option B: Postpartum urinary retention has been classified into overt and
covert retention by Yip et al. Women who are unable to micturate
spontaneously within 6 h after vaginal delivery are categorized as having
overt (symptomatic) urinary retention. Covert (asymptomatic) urinary
retention is defined as having a postvoid residual bladder volume (PVRBV)
of more than 150 mL, detected by ultrasound or by catheterization, with no
symptoms of urinary retention.

Option D: Postpartum urinary retention (PPUR) is an upsetting condition


that has no standard literature definition. It has been variably defined as the
abrupt onset of aching or inability to completely micturate, requiring urinary
catheterization, over 12 h after giving birth or not to void spontaneously
within 6 h of vaginal delivery.
38. Question

1 point(s)

To ensure adequate lactation the nurse should teach the mother to:

A. Breastfeed the baby on self-demand day and night.

B. Feed primarily during the day and allow the baby to sleep through the
night.

C. Feed the baby every 3-4 hours following a strict schedule.

D. Breastfeed when the breasts are engorged to ensure adequate supply.

294
C.BY: HOSSAM HAMDY MATERNITY RNPEDIA – ‫موسوعه التمريض‬

Answer: A. Breastfeed the baby on self-demand day and night

Feeding on self-demand means the mother feeds the baby according to the
baby’s need. Therefore, this means there will be regular emptying of the
breasts, which is essential to maintain adequate lactation.

Option B: Some newborns wake up and breastfeed every 2 to 3 hours like


clockwork, but that’s not always the case. The baby may want to breastfeed
many times in a short period, and then he might sleep for a little longer. This
type of feeding is called cluster or bunch feeding. Other babies are sleepy,
especially in the very early days, so the mother may have to wake the baby
up to breastfeed. All of these patterns are normal. As long as the child is
getting enough breast milk and growing well, there is nothing to worry.

Option C: On average, a breastfed newborn eats approximately every 2 to 3


hours around the clock. That’s about 8 to 12 times in a 24-hour period.
Newborn have little stomachs and ?breast milk is easily digested, so they
should breastfeed often.

Option D: In the beginning, breastfeed the newborn for as long as she will
stay on the breast. Continue to breastfeed until there are signs that the child
is satisfied. This way, the mother can be sure that the baby is getting enough
breast milk at each feeding. Plus, keeping the baby breastfeeding longer,
stimulates milk production and helps the mother to build up her breast milk
supply.
39. Question

1 point(s)

An appropriate nursing intervention when caring for a postpartum mother


with thrombophlebitis is:

A. Encourage the mother to ambulate to relieve the pain in the leg.

B. Instruct the mother to apply elastic bondage from the foot going towards
the knee to improve venous return flow.

C. Apply warm compress on the affected leg to relieve the pain.

D. Elevate the affected leg and keep the patient on bedrest.

295
C.BY: HOSSAM HAMDY MATERNITY RNPEDIA – ‫موسوعه التمريض‬

Answer: D. Elevate the affected leg and keep the patient on bed rest.

If the mother already has thrombophlebitis, the nursing intervention is bed


rest to prevent the possible dislodging of the thrombus and keeping the
affected leg elevated to help reduce the inflammation.

Option A: During pregnancy, an increase in most procoagulant factors and a


reduction in fibrinolytic activity occurs. Plasma fibrinogen levels gradually
increase after the third month of pregnancy, to double those of the
nonpregnant state. In the second half of pregnancy, levels of factors VII,
VIII, IX, and X also increase. Decreased fibrinolytic activity is probably
related to a decrease in the level of circulating plasminogen activator. In
addition, a 68% reduction in protein S levels is measured during pregnancy
and in the postpartum period. Protein S levels do not return to the reference
range until 12 weeks after delivery. These changes are necessary to prevent
hemorrhage during placental separation.

Option B: The routine use of graduated support stockings (class I or II),


especially when the patient is confined on an airplane or otherwise, is
extremely important.

Option C: A warm water compress is valuable in the treatment of phlebitis,


and could decrease the degree of phlebitis both effectively and
inexpensively.
40. Question

1 point(s)

The nurse should anticipate that hemorrhage related to uterine atony may
occur postnatally if this condition was present during the delivery:

A. Excessive analgesia was given to the mother.

B. Placental delivery occurred within thirty minutes after the baby was born.

C. An episiotomy had to be done to facilitate delivery of the head.

D. The labor and delivery lasted for 12 hours.

Answer: A. Excessive analgesia was given to the mother.

296
C.BY: HOSSAM HAMDY MATERNITY RNPEDIA – ‫موسوعه التمريض‬

Excessive analgesia can lead to uterine relaxation thus lead to hemorrhage


postpartally. Both B and D are normal and C is at the vaginal introitus thus
will not affect the uterus.

Option B: The absolute time limit for delivery of the placenta, without
evidence of significant bleeding, remains unclear. Periods ranging from 30-
60 minutes have been suggested.

Option C: An episiotomy is a minor incision made during childbirth to


widen the opening of the vagina. A perineal tear or laceration often forms on
its own during a vaginal birth. Rarely, this tear will also involve the muscle
around the anus or the rectum. Both episiotomies and perineal lacerations
require stitches to repair and ensure the best healing. Both are similar in
recovery time and discomfort during healing.

Option D: Normal labor usually begins within 2 weeks (before or after) the
estimated delivery date. In a first pregnancy, labor usually lasts 12 to 18
hours on average; subsequent labors are often shorter, averaging 6 to 8
hours.
41. Question

1 point(s)

According to Rubin’s theory of maternal role adaptation, the mother will go


through 3 stages during the postpartum period. These stages are:

A. Going through, adjustment period, adaptation period

B. Taking-in, taking hold and letting-go

C. Attachment phase, adjustment phase, adaptation phase

D. Taking-hold, letting-go, attachment phase

Answer: B. Taking-in, taking-hold and letting-go

Rubin’s theory states that the 3 stages that a mother goes through for
maternal adaptation are: taking-in, taking-hold and letting-go. In the taking-
in stage, the mother is more passive and dependent on others for care. In
taking-hold, the mother begins to assume a more active role in the care of the
child and in letting-go, the mother has become adapted to her maternal role.

297
C.BY: HOSSAM HAMDY MATERNITY RNPEDIA – ‫موسوعه التمريض‬

Option A: The taking-in phase usually sets 1 to 2 days after delivery. This is
the time of reflection for the woman because within the 2 to 3 day period, the
woman is passive. The taking-in phase provides time for the woman to
regain her physical strength and organize her rambling thoughts about her
new role.

Option C: The taking hold phase starts 2 to 4 days after delivery. The
woman starts to initiate actions on her own and makes decisions without
relying on others. She starts to focus on the newborn instead of herself and
begins to actively participate in newborn care. The woman still needs
positive reinforcements despite the independence that she is already showing
because she might still feel insecure about the care of her child.

Option D: During the letting go phase, the woman finally accepts her new
role and gives up her old roles like being a childless woman or just a mother
of one child. This is the phase where postpartum depression may set in.
Readjustment of relationships is needed for an easy transition to this phase.
42. Question

1 point(s)

The neonate of a mother with diabetes mellitus is prone to developing


hypoglycemia because:

A. The pancreas is immature and unable to secrete the needed insulin.

B. There is rapid diminution of glucose level in the baby’s circulating blood


and his pancreas is normally secreting insulin.

C. The baby is reacting to the insulin given to the mother.

D. His kidneys are immature leading to a high tolerance for glucose.

Answer: B. There is rapid diminution of glucose level in the baby’s


circulating blood and his pancreas is normally secreting insulin.

If the mother is diabetic, the fetus while in utero has a high supply of
glucose. When the baby is born and is now separate from the mother, it no
longer receives a high dose of glucose from the mother. In the first few hours
after delivery, the neonate usually does not feed yet thus this can lead to
hypoglycemia.

298
C.BY: HOSSAM HAMDY MATERNITY RNPEDIA – ‫موسوعه التمريض‬

Option A: The primary function of ?-cells is to store and secrete insulin in


response to glucose load. When ?-cells lose the ability to adequately sense
blood glucose concentration, or to release sufficient insulin in response, this
is classified as ?-cell dysfunction. ?-cell dysfunction is thought to be the
result of prolonged, excessive insulin production in response to chronic fuel
excess

Option C: ?-cell dysfunction is exacerbated by insulin resistance. Reduced


insulin-stimulated glucose uptake further contributes to hyperglycemia,
overburdening the ?-cells, which have to produce additional insulin in
response. The direct contribution of glucose to ?-cell failure is described as
glucotoxicity. Thus, once ?-cell dysfunction begins, a vicious cycle of
hyperglycemia, insulin resistance, and further ?-cell dysfunction is set in
motion.

Option D: Insulin resistance occurs when cells no longer adequately respond


to insulin. At the molecular level, insulin resistance is usually a failure of
insulin signaling, resulting in inadequate plasma membrane translocation of
glucose transporter 4 (GLUT4)—the primary transporter that is responsible
for bringing glucose into the cell to use as energy.
43. Question

1 point(s)

Which of the following is an abnormal vital sign in postpartum?

A. Pulse rate between 50-60/min

B. BP diastolic increase from 80 to 95mm Hg

C. BP systolic between 100-120mm Hg

D. Respiratory rate of 16-20/min

Answer: B. BP diastolic increase from 80 to 95mm Hg

All the vital signs given in the choices are within normal range except an
increase of 15mm Hg in the diastolic which is a possible sign of
hypertension in pregnancy.

Option A: The pulse rate may be elevated a few hours after the childbirth,
due to excitement or pain, and usually normalizes on the second day.
299
C.BY: HOSSAM HAMDY MATERNITY RNPEDIA – ‫موسوعه التمريض‬

Option C: The blood pressure could be elevated due to pain or excitement


but is generally in the normal range. A significant decrease (> 20% below
baseline) in blood pressure could be a sign of postpartum hemorrhage or
septic shock.[4] Conversely, high blood pressure could be a sign of pain or
pre-eclampsia.

Option D: The respiratory rate also begins to fall back to the pre-pregnancy
level within 2 to 3 days.
44. Question

1 point(s)

The uterine fundus right after delivery of placenta is palpable at

A. Level of Xiphoid process

B. Level of umbilicus

C. Level of symphysis pubis

D. Midway between umbilicus and symphysis pubis

Answer: B. Level of umbilicus

Immediately after the delivery of the placenta, the fundus of the uterus is
expected to be at the level of the umbilicus because the contents of the
pregnancy have already been expelled. The fundus is expected to recede by 1
fingerbreadth (1cm) every day until it becomes no longer palpable above the
symphysis pubis.

Option A: The fundus continues to descend into the pelvis at the rate of
approximately one centimeter (finger-breadth) per day and should be
nonpalpable by two weeks postpartum.

Option C: By approximately one-hour post-delivery, the fundus is firm and


at the level of the umbilicus. The fundus continues to descend into the pelvis
at the rate of approximate

Option D: Immediately after delivery, the upper portion of the uterus,


known as the fundus, is midline and palpable halfway between the
symphysis pubis and the umbilicus.
45. Question
300
C.BY: HOSSAM HAMDY MATERNITY RNPEDIA – ‫موسوعه التمريض‬

1 point(s)

A nurse is monitoring the amount of lochia drainage in a client who is 2


hours postpartum and notes that the client has a saturated perineal pad in 1
hour. The nurse reports the amount of lochial flow as:

A. Scanty

B. Light

C. Heavy

D. Excessive

Answer: C. Heavy

Heavy lochial discharge is a saturated menstrual pad in 1 hour. The woman


can expect to see fresh red or browny-red blood loss. The flow of blood may
be quite heavy, soaking a maternity pad every few hours. Don’t be alarmed
if there are one or two quite large blood clots, they may even be as large as a
plum, or the woman may pass several smaller ones about the same size as
grapes. They are all just remnants of the placenta coming out of the body as
it’s no longer needed.

Option A: Scanty = less than 2.5 cm on a menstrual pad in 1 hour.16. After


three weeks: Any blood loss at this stage should be a pale, yellowish-white
in colour – or the woman may find there is no blood at all.

Option B: Light = less than 10 cm on a menstrual pad in 1 hour. After one


week, the blood should now have turned a pinky brown colour and the stain
on maternity pads should be getting smaller and lighter. The pad shouldn’t
be soaking at any time and the woman should let her midwife know if she
thinks she might be passing too much blood after one week. She may pass
little blood clots, about the size or a raisin or smaller. This is all totally
normal.

Option D: Excessive = menstrual pad saturated in 15 minutes. If the woman


passes large blood clots after the first 24 hours, or she continues to pass
blood clots after one week, it is important to contact a midwife or doctor
straight away.
46. Question

301
C.BY: HOSSAM HAMDY MATERNITY RNPEDIA – ‫موسوعه التمريض‬

1 point(s)

In a woman who is not breastfeeding, menstruation usually occurs after how


many weeks?

A. 2-4 weeks

B. 6-8 weeks

C. 6 months

D. 12 months

Answer: B. 6-8 weeks

When the mother does not breastfeed, the normal menstruation resumes
about 6-8 weeks after delivery. This is due to the fact that after delivery, the
hormones estrogen and progesterone gradually decrease thus triggering
negative feedback to the anterior pituitary to release the Follicle-Stimulating
Hormone (FSH) which in turn stimulates the ovary to again mature a
Graafian follicle and the menstrual cycle post pregnancy resumes.

Option A: If the mother is nursing, she will typically not have her normal
period for many months, depending on the amount and frequency of nursing
and a number of supplemental feedings, if any. Research suggests only 20%
of breastfeeding mothers will get their period back within the first six
months.

Option C: Once the mother has weaned, her period will usually follow
within a month or two. The woman may also see her period return as her
baby begins to eat more solid foods or begins to supplement with formula or
solids. This is normal as the amount of breastfeeding is less, meaning the
mother is more likely to ovulate.

Option D: Some women use the lactational amenorrhea (LAM) technique as


a means of birth control during this time. This is a very specific method of
birth control with strict rules. Not every breastfeeding mother can use this
for birth control.?
47. Question

1 point(s)

The following are nursing measures to stimulate lactation, except:


302
C.BY: HOSSAM HAMDY MATERNITY RNPEDIA – ‫موسوعه التمريض‬

A. Frequent regular breastfeeding

B. Breast pumping

C. Breast massage

D. Application of cold compress on the breast

Answer: D. Application of cold compress on the breast

To stimulate lactation, a warm compress is applied to the breast. A cold


application will cause vasoconstriction thus reducing the blood supply
consequently the production of milk.

Option A: Make sure that the baby is latching on to the breast ly. Latching
the baby on properly is the most efficient way to increase the supply. A poor
latch is often the main reason a mother’s supply of breast milk isn’t as
abundant as it can be. Without a proper latch, the baby cannot remove the
milk from the breast well.

Option B: Use a breast pump or a hand expression technique to continue to


stimulate the breasts after finishing breastfeeding the baby. The extra
stimulation will tell the woman’s body that she needs more breast milk.

Option C: Starting about two months before the date the baby is expected to
join the family, if time permits, introducing a routine of stimulus and
expression for the breasts can help with milk production. Gently massage the
breasts by hand for a few minutes, then use a hospital-grade (multi-user)
double electric breast pump for about 10 minutes more. Do this after waking,
before going to sleep, and several times throughout the day for the body to
begin reacting to the implied “demand” for breast milk. Drops of milk
usually appear, on average, about a month or so after starting this routine,
and milk supply typically builds over time.
48. Question

1 point(s)

When the uterus is firm and contracted after delivery but there is vaginal
bleeding, the nurse should suspect which of the following?

A. Uterine hypercontractility

B. Uterine atony
303
C.BY: HOSSAM HAMDY MATERNITY RNPEDIA – ‫موسوعه التمريض‬

C. Uterine inversion

D. Laceration of soft tissues of the cervix and vagina

Answer: D. Laceration of soft tissues of the cervix and vagina.

When the uterus is firm and contracted it means that the bleeding is not in
the uterus but other parts of the passageway such as the cervix or the vagina.
The most common complication of a perineal laceration is bleeding. Most
bleeding can be quickly controlled with pressure and surgical repair.
However, hematoma formation can lead to large amounts of blood loss in a
very short time.

Option A: Uterine hyperstimulation or hypercontractility is a serious


complication of labor induction. It is defined as single contractions lasting 2
minutes or more, or five or more contractions in a 10 minute period.

Option B: Uterine atony refers to the corpus uteri myometrial cell’s


inadequate contraction in response to endogenous oxytocin that is released in
the course of delivery. It leads to postpartum hemorrhage as the delivery of
the placenta leaves disrupted spiral arteries which are uniquely void of
musculature and dependent on contractions to mechanically squeeze them
into a hemostatic state. Uterine atony is a principal cause of postpartum
hemorrhage, an obstetric emergency. Globally, this is one of the top 5 causes
of maternal mortality.

Option C: Uterine inversion is one of the most serious complications of


childbirth. Uterine inversion refers to the collapse of the fundus into the
uterine cavity. Although it does not often occur, it carries a high risk of
mortality due to hemorrhage and shock.
49. Question

1 point(s)

The following are interventions to make the fundus contract postpartally,


except:

A. Make the baby suck the breast regularly.

B. Apply ice cap on fundus.

C. Massage the fundus vigorously for 15 minutes until contracted.


304
C.BY: HOSSAM HAMDY MATERNITY RNPEDIA – ‫موسوعه التمريض‬

D. Give oxytocin as ordered.

Answer: C. Massage the fundus vigorously for 15 minutes until


contracted.

Massaging the fundus of the uterus should not be vigorous and should only
be done until the uterus feels firm and contracted. If the massage is vigorous
and prolonged, the uterus will relax due to overstimulation.

Option A: If the woman breastfeeds, the hormone oxytocin is released,


which causes the uterus to contract.

Option B: Cooling the uterus by placing an icepack on the lower abdomen is


one of the standard non-pharmacological prophylactic strategies to prevent
PPH in Japan; the reasoning is that cold compresses may help to contract the
myometrium and decrease blood loss. Cold therapy causes blood vessels
within the smooth muscles to constrict, which subsequently decreases blood
flow. Furthermore, blood vessels in the skin are affected by cold, resulting in
somatovisceral reflex and subsequent vasoconstriction of relevant internal
organs

Option D: Prophylactic administration of oxytocin (Pitocin) reduces rates of


postpartum hemorrhage by 40 percent; this reduction also occurs if oxytocin
is given after placental delivery. Oxytocin is the drug of choice for
preventing postpartum hemorrhage because it is at least as effective as ergot
alkaloids or prostaglandins and has fewer side effects. Misoprostol (Cytotec)
has a role in the prevention of postpartum hemorrhage; this agent has more
side effects but is inexpensive, heat- and light-stable, and requires no
syringes.
50. Question

1 point(s)

The following are nursing interventions to relieve episiotomy wound pain,


except:

A. Giving analgesic as ordered

B. Sitz bath

C. Perineal heat

305
C.BY: HOSSAM HAMDY MATERNITY RNPEDIA – ‫موسوعه التمريض‬

D. Perineal care

Answer: D. Perineal care

Perineal care is primarily done for personal hygiene regardless of whether


there is pain or not; episiotomy wound or not.

Option A: Analgesics such as ibuprofen may be taken as prescribed by the


physician to relieve pain.

Option B: Use sitz baths (sit in water that covers the vulvar area) a few
times a day. Wait until 24 hours after giving birth to take a sitz bath as well.
The woman can buy tubs in any drug store that will fit on the rim of the
toilet. If she prefers, she can sit in this kind of tub instead of climbing into
the bathtub.

Option C: Take warm baths but wait until 24 hours after giving birth. Make
sure that the bathtub is cleaned with a disinfectant before every bath.
51. Question

1 point(s)

Postpartum blues is said to be normal provided that the following


characteristics are present. These are

A. Within 3-10 days only

B. Woman exhibits the following symptoms- episodic tearfulness, fatigue,


oversensitivity, poor appetite

C. Maybe more severe symptoms in primipara

D. All of the above

Answer: D. All of the above

All the symptoms 1-3 are characteristic of postpartal blues. It will resolve by
itself because it is transient and is due to a number of reasons like changes in
hormonal levels and adjustment to motherhood. If symptoms last more than
2 weeks, this could be a sign of abnormality like postpartum depression and
needs treatment.
306
C.BY: HOSSAM HAMDY MATERNITY RNPEDIA – ‫موسوعه التمريض‬

Option A: Postpartum blues, also known as “baby blues,” affect


approximately 50% to 80% of new mothers. Symptoms may include mood
swings with times of feeling anxious, irritable, or tearful interspersed with
times of feeling well. Sleeping difficulties may also occur.

Option B: The symptoms usually begin 3-4 days after delivery, worsen by
days 5-7, and tend to resolve by day 12. For symptoms that last longer than 2
weeks, it is important for the individual to seek medical attention since
approximately 1 in 5 women with postpartum blues develops postpartum
major depression.

Option C: About 60–80% of all new mothers suffer from the PPB which
rarely requires medication and normally subsides with support and
education. It is significant to carry out the follow-up because up to 20% of
these mothers are likely to progress to PPD and an adverse consequence on
children’s cognitive growth.
52. Question

1 point(s)

The neonatal circulation differs from the fetal circulation because

A. The fetal lungs are non-functioning as an organ and most of the blood in
the fetal circulation is mixed blood.

B. The blood at the left atrium of the fetal heart is shunted to the right atrium
to facilitate its passage to the lungs.

C. The blood in the left side of the fetal heart contains oxygenated blood
while the blood on the right side contains unoxygenated blood.

D. None of the above.

Answer: A. The fetal lungs are non-functioning as an organ and most of


the blood in the fetal circulation is mixed blood.

The fetal lungs are fluid-filled while in utero and are still not functioning. It
only begins to function in extrauterine life. Except for the blood as it enters
the fetus immediately from the placenta, most of the fetal blood is mixed
blood.

307
C.BY: HOSSAM HAMDY MATERNITY RNPEDIA – ‫موسوعه التمريض‬

Option B: The hole between the top two heart chambers (right and left
atrium) is called a patent foramen ovale (PFO). This hole allows the oxygen
rich blood to go from the right atrium to the left atrium and then to the left
ventricle and out the aorta. As a result the blood with the most oxygen gets
to the brain.

Option C: The placenta accepts the blood without oxygen from the fetus
through blood vessels that leave the fetus through the umbilical cord
(umbilical arteries, there are two of them). When blood goes through the
placenta it picks up oxygen. The oxygen rich blood then returns to the fetus
via the third vessel in the umbilical cord (umbilical vein). The oxygen rich
blood that enters the fetus passes through the fetal liver and enters the right
side of the heart.
53. Question

1 point(s)

The normal respiration of a newborn immediately after birth is characterized


as:

A. Shallow and irregular with short periods of apnea lasting not longer than
15 seconds, 30-60 breaths per minute.

B. 20-40 breaths per minute, abdominal breathing with active use of


intercostals muscles.

C. 30-60 breaths per minute with apnea lasting more than 15 seconds,
abdominal breathing.

D. 30-50 breaths per minute, active use of abdominal and intercostal


muscles.

Answer: A. Shallow and irregular with short periods of apnea lasting


not longer than 15 seconds, 30-60 breaths per minute.

A newly born baby still is adjusting to extra uterine life and the lungs are just
beginning to function as a respiratory organ. The respiration of the baby at
this time is characterized as usually shallow and irregular with short periods
of apnea, 30-60 breaths per minute. The apneic periods should be brief
lasting, not more than 15 seconds otherwise it will be considered abnormal.

308
C.BY: HOSSAM HAMDY MATERNITY RNPEDIA – ‫موسوعه التمريض‬

Option B: The natural elastic property of the lungs is to deflate. When


balanced by the outward recoil of the chest wall, functional residual capacity
(FRC) occurs at the end of expiration to prevent alveoli from collapsing. The
newborn chest wall, composed primarily of cartilage, is more pliable,
predisposing neonatal lungs to pulmonary atelectasis and decreased FRC.

Option C: Normally, the newborn’s respiratory rate is 30 to 60 breaths per


minute. Tachypnea is defined as a respiratory rate greater than 60 breaths per
minute. Tachypnea is a compensatory mechanism for hypercarbia,
hypoxemia, or acidosis (both metabolic and respiratory), making it a
common but nonspecific finding in a large variety of respiratory,
cardiovascular, metabolic, or systemic diseases.

Option D: A newborn’s breathing rate is normally 40 to 60 breaths per


minute. The Apgar score helps find breathing problems and other health
issues. It is part of the special attention given to a baby in the first few
minutes after birth. The baby is checked at 1 minute and 5 minutes after birth
for heart and respiratory rates, muscle tone, reflexes, and color. A baby who
needs help with any of these issues is getting constant attention during those
first 5 to 10 minutes.
54. Question

1 point(s)

The anterior fontanelle is characterized as:

A. 3-4 cm anteroposterior diameter and 2-3 cm transverse diameter, diamond


shape.

B. 2-3 cm anteroposterior diameter and 3-4 cm transverse diameter and


diamond shape.

C. 2-3 cm in both anteroposterior and transverse diameter and diamond


shape.

D. None of the above.

Answer: A. 3-4 cm anteroposterior diameter and 2-3 cm transverse


diameter, diamond shape.

309
C.BY: HOSSAM HAMDY MATERNITY RNPEDIA – ‫موسوعه التمريض‬

The anterior fontanelle is a diamond shape with the anteroposterior diameter


being longer than the transverse diameter. The posterior fontanelle is a
triangular shape.

Option B: The anterior fontanelle is the largest of the six fontanelles, and it
resembles a diamond-shape ranging in size from 0.6 cm to 3.6 cm with a
mean of 2.1 cm.

Option C: It forms through the juxtaposition of the frontal bones and


parietal bones with the superior sagittal sinus coursing beneath it. Two
frontal bones join to form one-half the anterior fontanelle with the metopic
suture serving as the parallel divider between the paired bones.

Option D: Next, the parietal bones are positioned against each other to
complete the fontanelle. The positioning of the two parietal bones against
each other gives rise to the sagittal suture. Finally, the alignment of the
frontal bones against the parietal bones establishes the coronal suture.
55. Question

1 point(s)

The ideal site for vitamin K injection in the newborn is:

A. Right upper arm

B. Left upper arm

C. Either right or left buttocks

D. Middle third of the thigh

Answer: D. Middle third of the thigh

Neonates do not have well-developed muscles of the arm. Since Vitamin K


is given intramuscular, the site must have sufficient muscles like the middle
third of the thigh.

Option A: The deltoid in infants is not sufficiently bulky to absorb IM


medications adequately. The vastus lateralis muscle avoids the thicker layer
of subcutaneous fat on the anterior thigh.

Option B: The anterolateral thigh is the preferred site for IM injection in


infants under 12 months of age. Medications are injected into the bulkiest
310
C.BY: HOSSAM HAMDY MATERNITY RNPEDIA – ‫موسوعه التمريض‬

part of the vastus lateralis thigh muscle, which is the junction of the upper
and middle thirds of this muscle.

Option C: Using the vastus lateralis muscle avoids the risk of sciatic nerve
damage from gluteal injection. Also, the vastus lateralis muscle has a larger
muscle mass than the gluteal region and therefore has reduced risk of severe
local reactions.
56. Question

1 point(s)

At what APGAR score at 5 minutes after birth should resuscitation be


initiated?

A. 1-3

B. 7-8

C. 9-10

D. 6-7

Answer: A. 1-3

An APGAR of 1-3 is a sign of fetal distress which requires resuscitation. The


baby is alright if the score is 8-10. Apgar is a quick test performed on a baby
at 1 and 5 minutes after birth. The 1-minute score determines how well the
baby tolerated the birthing process. The 5-minute score tells the health care
provider how well the baby is doing outside the mother’s womb.

Option B: A score of 7, 8, or 9 is normal and is a sign that the newborn is in


good health. This test is done to determine whether a newborn needs help
breathing or is having heart trouble.

Option C: The Apgar score is based on a total score of 1 to 10. The higher
the score, the better the baby is doing after birth. A score of 10 is very
unusual, since almost all newborns lose 1 point for blue hands and feet,
which is normal for after birth.

Option D: Any score lower than 7 is a sign that the baby needs medical
attention. The lower the score, the more help the baby needs to adjust outside
the mother’s womb.

311
C.BY: HOSSAM HAMDY MATERNITY RNPEDIA – ‫موسوعه التمريض‬

57. Question

1 point(s)

Right after birth, when the skin of the baby’s trunk is pinkish but the soles of
the feet and palm of the hands are bluish this is called:

A. Syndactyly

B. Acrocyanosis

C. Peripheral cyanosis

D. Cephalo-caudal cyanosis

Answer: B. Acrocyanosis

Acrocyanosis is the term used to describe the baby’s skin color at birth when
the soles and palms are bluish but the trunk is pinkish. This is relatively
common in young infants, and is generally a physiologic finding due to the
large arteriovenous oxygen difference that results during slow flow through
peripheral capillary beds.

Option A: Syndactyly is the most common congenital malformation of the


limbs, with an incidence of 1 in 2000-3000 live births. It is a failure of
differentiation in which the fingers fail to separate into individual
appendages. This separation usually occurs during the sixth and eighth
weeks of embryological development.

Option C: Peripheral cyanosis is the bluish discoloration of the distal


extremities (hands, fingertips, toes), and can sometimes involve circumoral
and periorbital areas. Mucous membranes are generally not involved.
Peripheral cyanosis is rarely a life-threatening medical emergency.

Option D: Cephalocaudal proceeds or occurs in the long axis of the body


especially in the direction from head to tail.
58. Question

1 point(s)

The minimum birth weight for full-term babies to be considered normal is:

A. 2,000 gms
312
C.BY: HOSSAM HAMDY MATERNITY RNPEDIA – ‫موسوعه التمريض‬

B. 1,500 gms

C. 2,500 gms

D. 3,000 gms

Answer: C. 2,500 gms

According to the WHO standard, the minimum normal birth weight of a full-
term baby is 2,500 gms or 2.5 Kg.

Option A: Low birth weight (LBW) are birth weights that are less than 5
pounds, 8 ounces (2,500 grams).

Option B: Very low birth weight (VLBW)refers to birth weights less than
3.4 pounds (1,500 grams).

Option D: High birth weight (HBW) refers to birth weights of more than 8
pounds, 13 ounces (4,000 grams).
59. Question

1 point(s)

This procedure is done to prevent ophthalmia neonatorum is:

A. Marmet’s technique

B. Ophthalmic wash

C. Ritgen’s method

D. Crede’s method

Answer: D. Crede’s method

Crede’s method/prophylaxis is the procedure done to prevent ophthalmia


neonatorum which the baby can acquire as it passes through the birth canal
of the mother. Usually, an ophthalmic ointment is used.

Option A: Developed by a mother who needed to express her milk over a


long period of time for medical reasons, the Marmet technique mimics the
313
C.BY: HOSSAM HAMDY MATERNITY RNPEDIA – ‫موسوعه التمريض‬

actions of a breastfeeding baby and is the most recommended method of


expressing breastmilk by hand.

Option B: Irrigation of the eye with sterile isotonic saline may be done for
neonatal conjunctivitis, and systemic treatment is required for
staphylococcal, gonococcal, Chlamydia, Pseudomonas, and hepatic
conjunctivitis

Option C: Ritgen’s maneuver means that the fetal chin is reached between
the anus and the coccyx and pulled anteriorly, while using the fingers of the
other hand on the fetal occiput to control speed of delivery and keep flexion
of the fetal neck.
.
60. Question

1 point(s)

Which of the following characteristics will distinguish a postmature neonate


at birth?

A. Plenty of lanugo and vernix caseosa.

B. Lanugo mainly on the shoulders and vernix in the skin folds.

C. Pinkish skin with good turgor.

D. Almost leather-like, dry, cracked skin, negligible vernix caseosa.

Answer: D. Almost leather-like, dry, cracked skin, negligible vernix


caseosa

A post mature fetus has the appearance of an old person with dry wrinkled
skin and the vernix caseosa has already diminished.

Option A: Lanugo plays an important role in binding the vernix caseosa to


the skin of fetuses. Vernix caseosa is the viscous white covering on
newborns that protects their skin, prevents water loss, plays an important
role in thermoregulation, and contributes to innate immunity.

Option B: Lanugo is the first type of hair to develop in humans. The


interaction of lanugo with the vernix is also important in controlling the
tempo of the fetal developmental rate during various times in the gestation
cycle. Lanugo arises at about three months into development. Hair growth
314
C.BY: HOSSAM HAMDY MATERNITY RNPEDIA – ‫موسوعه التمريض‬

starts on the scalp around the eyebrow, nose, and forehead area and proceeds
in a cephalocaudal direction from head to toe. It is shed at about 33 to 36
weeks gestation, when it becomes subsequently incorporated into the
amniotic fluid, eventually contributing to the composition of the meconium.

Option C: The skin of a healthy newborn at birth has: Deep red or purple
skin and bluish hands and feet. The skin darkens before the infant takes their
first breath (when they make that first vigorous cry). A thick, waxy
substance called vernix covering the skin.
61. Question

1 point(s)

What would be the appropriate first nursing action when caring for a 20-year
old G1P0 woman at 39 weeks gestation who is in active labor and for whom
an assessment reveals mild variable fetal heart rate deceleration

A. Notify the physician

B. Prepare the client for immediate delivery

C. Readjust the fetal monitor

D. Change the maternal position

Answer: D. Change the maternal position

The cause of variable fetal heart decelerations is umbilical cord compression,


which can usually be ed by changing the maternal position.

Option A: Before informing the physician, the nurse must first intervene.
Common causes of variable decelerations include vagal reflex triggered by
head compression during pushing and cord compression such as that caused
by short cord, nuchal cord, body entanglement, prolapsed cord, decreased
amniotic fluid, and fetal descent.

Option B: Perform a cervical exam to rule out prolapsed cord and funic
presentation and check for imminent delivery only if appropriate.

Option C: Variable decelerations occur when the fetal heart rate decrease is
greater than or equal to 15 beats per minute and last for longer than or equal
to 15 seconds but less than 2 minutes from onset to return to baseline.

315
C.BY: HOSSAM HAMDY MATERNITY RNPEDIA – ‫موسوعه التمريض‬

62. Question

1 point(s)

In basal body temperature (BBT) technique, the sign that ovulation has
occurred is an elevation of body temperature by

A. 1.0-1.4 degrees centigrade

B. 0.2-0.4 degrees centigrade

C. 2.0-4.0 degrees centigrade

D. 1.0-4.0 degrees centigrade

Answer: B. 0.2-0.4 degrees centigrade

The release of the hormone progesterone in the body following ovulation


causes a slight elevation of basal body temperature of about 0.2 – 0.4
degrees centigrade.

Option A: Basal body temperature (BBT) is defined as the lowest natural,


non-pathologic body temperature recorded after a period of rest. Women
have used charting average basal body temperatures over the length of a
menstrual period has been a tool to determine if ovulation has occurred or
not.

Option C: Ideally, charting the daily basal body temperature will show a
noticeable increase of approximately 0.5 to 1 degrees F shortly after
ovulation. This increase then sustains until the start of menstruation in which
BBT then decreases back to baseline.

Option D: For most accurate results, temperatures need to be recorded at the


same time every day, immediately upon waking up. Measurements are also
subject to environmental influences such as a fever secondary to an
infectious process, emotional stressors, alcohol consumption, and the
addition or discontinuation of oral contraception from a daily regimen.
63. Question

1 point(s)

Lactation Amenorrhea Method(LAM) can be an effective method of natural


birth control if
316
C.BY: HOSSAM HAMDY MATERNITY RNPEDIA – ‫موسوعه التمريض‬

A. The mother breastfeeds mainly at night time when ovulation could


possibly occur.

B. The mother breastfeeds exclusively and regularly during the first 6


months without giving supplemental feedings.

C. The mother uses mixed feeding faithfully.

D. The mother breastfeeds regularly until 1 year with no supplemental


feedings.

Answer: B. The mother breastfeeds exclusively and regularly during


the first 6 months without giving supplemental feedings.

A mother who breastfeeds exclusively and regularly during the first 6


months benefits from lactation amenorrhea. There is evidence to support the
observation that the benefits of lactation amenorrhea last for 6 months
provided the woman has not had her first menstruation since delivery of the
baby.

Option A: This method requires breastfeeding of the baby every 4 hours


during the day and every 6 hours at night. Elevated prolactin levels and a
reduction of gonadotropin-releasing hormone from the hypothalamus during
lactation suppress ovulation. This leads to a reduction in luteinizing hormone
(LH) release and inhibition of follicular maturation.

Option C: The duration of this suppression varies and is influenced by the


frequency and duration of breastfeeding and the length of time since birth.
No supplementation of other foods or formula are allowed, only exclusive
breastfeeding.

Option D: The baby must be younger than 6 months for perfect use. The
perfect-use failure rate within the first 6 months is 0.5%. The typical-use
failure rate within the first 6 months is 2%.
64. Question

1 point(s)

The intrauterine device prevents pregnancy by the ffg mechanism, except:

A. Endometrium inflames.

B. Fundus contracts to expel uterine contents.


317
C.BY: HOSSAM HAMDY MATERNITY RNPEDIA – ‫موسوعه التمريض‬

C. Copper embedded in the IUD can kill the sperms.

D. Sperms will be barred from entering the fallopian tubes.

Answer: D. Sperms will be barred from entering the fallopian tubes.

An Intrauterine device is a T-shaped piece of plastic placed inside the uterus.


The piece of plastic contains copper or a synthetic progesterone hormone
that prevents pregnancy. The device releases a constant low dose of a
synthetic hormone continually throughout the day. Usually, IUDs are coated
with copper to serve as spermicide killing the sperms deposited into the
female reproductive tract. But the IUD does not completely fill up the
uterine cavity thus sperms which are microscopic in size can still pass
through.

Option A: An intrauterine device is a foreign body so that if it is inserted


into the uterine cavity, the initial reaction is to produce an inflammatory
process and the uterus will contract in order to try to expel the foreign body.

Option B: The IUD changes the lining of the uterus, preventing implantation
should fertilization occur. It is important to consider the ethical implications
of this third method.

Option C: The released progesterone or copper creates changes in the


cervical mucus and inside the uterus that kills sperm or makes them
immobile.
65. Question

1 point(s)

Oral contraceptive pills are of different types. Which type is most


appropriate for mothers who are breastfeeding?

A. Estrogen-only

B. Progesterone only

C. Mixed type- estrogen and progesterone

D. 21-day pills mixed type

318
C.BY: HOSSAM HAMDY MATERNITY RNPEDIA – ‫موسوعه التمريض‬

Answer: B. Progesterone only

Currently, there are three types of oral contraceptive pills: combined


estrogen-progesterone, progesterone only and the continuous or extended use
pill. If the mother is breastfeeding, the progesterone only type is the best
because estrogen can affect lactation.

Option A: Estrogen has some effect with inhibiting follicular development


because of its negative feedback on the anterior pituitary with slow FSH
secretion; it’s just not as prominent as the progesterone’s effect.

Option C: The most commonly prescribed pill is the combined hormonal


pill with estrogen and progesterone. Progesterone is the hormone that
prevents pregnancy, and the estrogen component will control menstrual
bleeding.

Option D: Each pill has the same amount of hormone in it. One pill is taken
each day for 21 days and then no pills are taken for the next 7 days.
66. Question

1 point(s)

The natural family planning method called Standard Days (SDM), is the
latest type and easy to use method. However, it is a method applicable only
to women with regular menstrual cycles between how many days?

A. 21-26 days

B. 26-32 days

C. 28-30 days

D. 24- 36 days

Answer: B. 26-32 days

Standard Days Method (SDM) requires that the menstrual cycles are regular
between 26-32 days. There is no need to monitor temperature or mucus
secretion. This natural method of family planning is very simple since all
that the woman pays attention to is her cycle. With the aid of CycleBeads,
the woman can easily monitor her cycles.

319
C.BY: HOSSAM HAMDY MATERNITY RNPEDIA – ‫موسوعه التمريض‬

Option A: The Standard Days Method (SDM) is a method of family


planning that assumes ovulation to be close to the midpoint of the menstrual
cycle; fertility falls between days 8 and 19; and is most effective for cycle
lengths between twenty-six and thirty-two days.

Option C: The fixed-day algorithm for the SDM is based on statistical


evidence that ovulation occurs around the middle of the cycle for menstrual
cycles that have lengths between twenty-six and thirty-two days.
Theoretically, the SDM will be most effective when the day of ovulation
falls close to the center of the menstrual cycle.

Option D: For the SDM to be effective, the actual fertile phase of the
menstrual cycle should not fall outside of days 8 through 19 too frequently.
The fertile phase of the menstrual cycle is approximately six days, that is, the
day of ovulation and the five days before. The most fertile of those six days
are the two days before ovulation.
67. Question

1 point(s)

Nurse Lesley is conducting health teachings to a group of first-time mothers.


Which of the following are signs of ovulation? Select all that apply.

A. Mittelschmerz

B. Spinnbarkeit

C. Thin watery cervical mucus

D. Elevated body temperature of 4.0 degrees centigrade

Answer: A, B, & C

Mittelschmerz, spinnbarkeit and thin watery cervical mucus are signs of


ovulation. When ovulation occurs, the hormone progesterone is released
which can cause a slight elevation of temperature between 0.2-0.4 degrees
centigrade and not 4 degrees centigrade. Mittelschmerz is one-sided, lower
abdominal pain associated with ovulation. German for “middle pain,”
mittelschmerz occurs midway through a menstrual cycle — about 14 days
before the next menstrual period. In most cases, mittelschmerz doesn’t
require medical attention. The ‘stretchability’ of cervical mucus, or the
length that strands of cervical mucus reach before breaking–? 6 cm, a
320
C.BY: HOSSAM HAMDY MATERNITY RNPEDIA – ‫موسوعه التمريض‬

reaction that parallels ‘ferning’ reaction, peaking on the 14th day–ovulation


of the menstrual cycle. This kind of cervical mucus stretches further than
creamy cervical mucus, and it appears clearer. While not the “ideal” fertile
cervical mucus, watery cervical mucus is fertile.
68. Question

1 point(s)

The following methods of artificial birth control works as a barrier device,


except:

A. Condom

B. Cervical cap

C. Cervical Diaphragm

D. Intrauterine device (IUD)

Answer: D. Intrauterine device (IUD)

Intrauterine device prevents pregnancy by not allowing the fertilized ovum


from implanting on the endometrium. Some IUDs have copper added to it
which is spermicidal. It is not a barrier since the sperms can readily pass
through and fertilize an ovum at the fallopian tube.

Option A: Condoms are the only type of contraception that can both prevent
pregnancy and protect against sexually transmitted infections (STIs).
Condoms are a “barrier” method of contraception. They are made of very
thin latex (rubber), polyurethane or polyisoprene and are designed to prevent
pregnancy by stopping sperm from meeting an egg.

Option B: The cervical cap is a birth control (contraceptive) device that


prevents sperm from entering the uterus. The cervical cap is a reusable, deep
silicone cup that is inserted into the vagina and fits tightly over the cervix.
The cervical cap is held in place by suction and has a strap to help with
removal. The cervical cap is effective at preventing pregnancy only when
used with spermicide.

Option C: The diaphragm is a birth control (contraceptive) device that


prevents sperm from entering the uterus. The diaphragm is a small, reusable
rubber or silicone cup with a flexible rim that covers the cervix. Before sex,
321
C.BY: HOSSAM HAMDY MATERNITY RNPEDIA – ‫موسوعه التمريض‬

the diaphragm is inserted deep into the vagina so that part of the rim fits
snugly behind the pubic bone. The diaphragm is effective at preventing
pregnancy only when used with spermicide.
69. Question

1 point(s)

Which of the following is a true statement about normal ovulation?

A. It occurs on the 14th day of every cycle.

B. It may occur between 14-16 days before next menstruation.

C. Every menstrual period is always preceded by ovulation.

D. The most fertile period of a woman is 2 days after ovulation.

Answer: B. It may occur between 14-16 days before next menstruation.

Not all menstrual cycles are ovulatory. Normal ovulation in a woman occurs
between the 14th to the 16th day before the next menstruation. A common
misconception is that ovulation occurs on the 14th day of the cycle. This is a
misconception because ovulation is determined not from the first day of the
cycle but rather 14-16 days before the next menstruation.

Option A: Ovulation occurs approximately 10-12 hours after the LH peak.


The LH surge is initiated by a dramatic rise of estradiol produced by the
preovulatory follicle

Option C: The luteal phase is 14 days long in most women. If the corpus
luteum is not rescued by pregnancy, it will undergo atresia. The resultant
progesterone withdrawal results in menses.

Option D: The follicular phase begins from the first day of menses until
ovulation. The development of ovarian follicles characterizes this phase. The
LH surge is initiated by a dramatic rise of estradiol produced by the
preovulatory follicle and results in subsequent ovulation. The LH surge
stimulates luteinization of the granulosa cells and stimulates the synthesis of
progesterone responsible for the midcycle FSH surge. Also, the LH surge
stimulates resumption of meiosis and the completion of reduction division in
the oocyte with the release of the first polar body.
70. Question
322
C.BY: HOSSAM HAMDY MATERNITY RNPEDIA – ‫موسوعه التمريض‬

1 point(s)

If a couple would like to enhance their fertility, the following means can be
done. Select all that apply.

A. Monitor the basal body temperature of the woman every day to determine
peak period of fertility.

B. Have adequate rest and nutrition

C. Have sexual contact only during the dry period of the woman

D. Undergo a complete medical check-up to rule out any debilitating disease

Answer: A, B, & D

All of the above are essential for enhanced fertility except option C because
during the dry period the woman is in her infertile period thus even when
sexual contact is done, there will be no ovulation, thus fertilization is not
possible.

Option A: By measuring the basal body temperature every morning before


getting out of bed, the woman might be able to detect, first, a very slight
decrease then a very slight rise in temperature for three mornings in a row.
The temperature rise may be as little as half of a degree. This can be a signal
that she has ovulated. Keep in mind that an egg only survives about 24 hours
after ovulation so this so-called fertile window may not be a good indicator
of when the couple should have sex.

Option B: The doctor might recommend that the couple make lifestyle
changes before they get pregnant. These might include getting to a healthy
weight; improving diet/exercise habits; eliminating alcohol; quitting
smoking, if they smoke; and cutting back on caffeine.

Option D: Most healthy couples will conceive within a year of actively


trying to get pregnant. If the woman doesn’t get pregnant within a year and
is under age 35, they should see a doctor for a fertility evaluation. If the
woman is over 35, she should only wait six months before seeing a doctor.
71. Question

1 point(s)

323
C.BY: HOSSAM HAMDY MATERNITY RNPEDIA – ‫موسوعه التمريض‬

In the sympto-thermal method, the parameters being monitored to determine


if the woman is fertile or infertile are:

A. Temperature, cervical mucus, cervical consistency

B. Release of ovum, temperature, and vagina

C. Temperature and wetness

D. Temperature, endometrial secretion, mucus

Answer: A. Temperature, cervical mucus, cervical consistency

The 3 parameters measured/monitored which will indicate that the woman


has ovulated are- a temperature increase of about 0.2-0.4 degrees centigrade,
a softness of the cervix and cervical mucus that looks like the white of an
egg which makes the woman feel “wet”.

Option B: The symptothermal method is a combination of methods. The two


most commonly used are the BBT method and the cervical mucus method.
The Marquette method combines BBT and cervical mucus tracking with use
of an electronic hormonal fertility monitor. The monitor detects hormones in
urine to confirm fertile days. It can be purchased online or at a pharmacy.

Option C: The symptothermal method combines calendar calculations, basal


body temperature charting, and cervical mucus monitoring.

Option D: Cervical secretions are the foundation for this method, and the
other techniques provide a “double-check.” Women may use other signs
(e.g., consistency and position of the cervix) or symptoms (e.g., breast
tenderness, ovulatory pain) to aid in the identification of the fertile period.
72. Question

1 point(s)

The following are important considerations to teach the woman who is on a


low dose (mini-pill) oral contraceptive except:

A. The pill must be taken every day at the same time.

B. If the woman fails to take a pill in one day, she must take 2 pills for added
protection.

324
C.BY: HOSSAM HAMDY MATERNITY RNPEDIA – ‫موسوعه التمريض‬

C. If the woman fails to take a pill in one day, she needs to take another
temporary method until she has consumed the whole pack.

D. If she is breastfeeding, she should discontinue using mini-pill and use the
progestin-only type.

Answer: B. If the woman fails to take a pill in one day, she must take 2
pills for added protection.

If the woman fails to take her usual pill for the day, taking a double dose
does not give additional protection. What she needs to do is to continue
taking the pills until the pack is consumed and use at the time another
temporary method to ensure that no pregnancy will occur. When a new pack
is started, she can already discontinue using the second temporary method
she employed.

Option A: Combined oral contraceptive pills are to be taken daily at


approximately the same time each day. Avoid taking them greater than 24
hours apart as this could affect efficacy.

Option C: If you miss a tablet just take the missed tablet as soon as you
remember and the next tablet at the usual time (taking 2 tablets in 1 day). If
you miss 2 tablets in a row in the first or second week then take 2 tablets the
day you remember and 2 tablets the next day, then resume 1 per day. Use
another form of contraception until you begin a new cycle.

Option D: Progestin-only oral contraceptives, or “The Mini-Pill,” contain


only a progestin (a female hormone). The method, when used daily, is highly
effective for breastfeeding women. This method of contraception has a
slightly higher failure rate than oral contraceptives (OCs) containing both
estrogen and progestin.
73. Question

1 point(s)

To determine if the cause of infertility is a blockage of the fallopian tubes,


the test to be done is

A. Huhner’s test

B. Postcoital test

325
C.BY: HOSSAM HAMDY MATERNITY RNPEDIA – ‫موسوعه التمريض‬

C. Rubin’s test

D. None of the above

Answer: C. Rubin’s test

Rubin’s test is a test to determine patency of fallopian tubes. Huhner’s test is


also known as post-coital test to determine the compatibility of the cervical
mucus with sperms of the sexual partner.

Options A and B: The postcoital test or Huhner’s test determines the


adequacy of sperm and the receptivity of cervical mucus. It is the only test
which evaluates the interaction between sperm and the female genital tract
fluids. The Sims-Huhner test should be an integral part of an infertility
investigation, but it must not be used as a substitute for semen analysis.
Since cervical mucus accurately reflects the ovarian cycle, the PC test is a
useful indicator of the endocrine preparation of the female reproductive
system. It is also an important method for the evaluation of a variety of
contraceptive steroids which may act directly or indirectly upon cervical
secretion.
74. Question

1 point(s)

Infertility can be attributed to male causes such as the following except:

A. Cryptorchidism

B. Orchitis

C. Sperm count of about 20 million per milliliter

D. Premature ejaculation

Answer: C. Sperm count of about 20 million per milliliter

Sperm count must be within normal in order for a male to successfully sire a
child. The normal sperm count is 20 million per milliliter of seminal fluid or
50 million per ejaculate.

326
C.BY: HOSSAM HAMDY MATERNITY RNPEDIA – ‫موسوعه التمريض‬

Option A: Cryptorchidism is the most common genital problem encountered


in pediatrics. Cryptorchidism literally means hidden or obscure testis and
generally refers to an undescended or maldescended testis.

Option B: Orchitis is an acute inflammatory reaction of the testis secondary


to infection. Most cases are associated with a viral mumps infection;
however, other viruses and bacteria can cause orchitis. Testicular
examination reveals the following: testicular enlargement, induration of the
testis, tenderness, erythematous scrotal skin, edematous scrotal skin, and
enlarged epididymis associated with epididymo-orchitis.

Option D: Premature ejaculation occurs when a man ejaculates sooner


during sexual intercourse than he or his partner would like. Premature
ejaculation is a common sexual complaint. Estimates vary, but as many as 1
out of 3 men say they experience this problem at some time.
75. Question

1 point(s)

Spinnbarkeit is an indicator of ovulation which is characterized as:

A. Thin watery mucus which can be stretched into a long strand about 10
cm.

B. Thick mucus that is detached from the cervix during ovulation.

C. Thin mucus that is yellowish in color with fishy odor.

D. Thick mucus vaginal discharge is influenced by high levels of estrogen.

Answer: A. Thin watery mucus which can be stretched into a long


strand about 10 cm

At the midpoint of the cycle when the estrogen level is high, the cervical
mucus becomes thin and watery to allow the sperm to easily penetrate and
get to the fallopian tubes to fertilize an ovum. This is called spinnbarkeit.
And the woman feels “wet”. When progesterone is secreted by the ovary, the
mucus becomes thick and the woman will feel “dry”.

Option B: Creamy cervical mucus is considered non-fertile since it greatly


restricts the movement of sperm. It is often pearly white or creamy yellow. It
is thick and feels like lotion when rubbed between the fingers.
327
C.BY: HOSSAM HAMDY MATERNITY RNPEDIA – ‫موسوعه التمريض‬

Option C: Although “yeast” is the name most women know, bacterial


vaginosis (BV) actually is the most common vaginal infection in women of
reproductive age. Bacterial vaginosis often will cause an abnormal smelling
vaginal discharge. The discharge usually is thin and milky, and is described
as having a “fishy” odor.

Option D: Spinnbarkeit mucus is the stringy, stretchy quality of cervical


mucus found especially around the time of ovulation. Usually a result of
high estrogen levels, spinnbarkeit mucus refers to the egg white quality of
cervical mucus that is easier for sperm to penetrate.

328
C.BY: HOSSAM HAMDY MATERNITY RNPEDIA – ‫موسوعه التمريض‬

Maternity Nursing NU.6 (Quiz #1: 25 Questions)

1. Question

A nursing instructor is conducting a lecture and is reviewing the functions of the


female reproductive system. She asks the student nurse to describe the follicle-
stimulating hormone (FSH) and the luteinizing hormone (LH). The student
nurse accurately responds by stating that:

A. FSH and LH are released from the anterior pituitary gland.

B. FSH and LH are secreted by the corpus luteum of the ovary

C. FSH and LH are secreted by the adrenal glands

D. FSH and LH stimulate the formation of milk during pregnancy.

Answer: A. FSH and LH are released from the anterior pituitary gland.

FSH and LH, when stimulated by the gonadotropin-releasing hormone from the
hypothalamus, are released from the anterior pituitary gland to stimulate
follicular growth and development, the growth of the Graafian follicle, and
production of progesterone.

Option B: The primary hormone produced from the corpus luteum is


progesterone, but it also produces inhibin A and estradiol. In the absence of
fertilization, the corpus luteum will regress over time.

Option C: Development of the ovarian follicle is largely under FSH control,


and the secretion of estrogen from this follicle is dependent on FSH and LH.
The granulosa cells of the ovary secrete inhibin, which plays a role in cellular
differentiation.

Option D: In women, LH stimulates estrogen and progesterone production


from the ovary. A surge of LH in the mid menstrual cycle is responsible for
ovulation, and continued LH secretion subsequently stimulates the corpus
luteum to produce progesterone.

2. Question

329
C.BY: HOSSAM HAMDY MATERNITY RNPEDIA – ‫موسوعه التمريض‬

A nurse is describing the process of fetal circulation to a client during a prenatal


visit. The nurse accurately tells the client that fetal circulation consists of:

A. Two umbilical veins and one umbilical artery.

B. Two umbilical arteries and one umbilical vein.

C. Arteries carrying oxygenated blood to the fetus.

D. Veins carrying deoxygenated blood to the fetus.

Answer: B. Two umbilical arteries and one umbilical vein.

Blood pumped by the embryo’s heart leaves the embryo through two umbilical
arteries. Once oxygenated, the blood then is returned by one umbilical vein.
Arteries carry deoxygenated blood and waste products from the fetus, and veins
carry oxygenated blood and provide oxygen and nutrients to the fetus.

Option A: The fetal circulation system is distinctly different from adult


circulation. This intricate system allows the fetus to receive oxygenated blood
and nutrients from the placenta. It comprises the blood vessels in the placenta
and the umbilical cord, which contains two umbilical arteries and one umbilical
vein.

Option C: Oxygenated blood from the mother in the placenta flows through the
umbilical vein and into the inferior vena cava (IVC), bypassing the liver via the
ductus venosus. From the IVC, oxygenated blood travels to the right atrium of
the heart. There is greater pressure in the right atrium compared to the left
atrium in fetal circulation; therefore most of the blood is shunted from the right
atrium to the left atrium through an opening called the foramen ovale. Once in
the left atrium, blood travels through the left ventricle into the aorta and the
systemic circulation.

Option D: The deoxygenated blood travels back to the placenta via the
umbilical arteries to be oxygenated by the mother. Additionally, some
oxygenated blood in the right atrium can also enter the right ventricle and then
the pulmonary artery. Because there is high resistance to blood flow in the
lungs, the blood is shunted from the pulmonary artery into the aorta via the
ductus arteriosus, hence bypassing the lungs. Blood then enters the systemic
circulation, and the deoxygenated blood is recycled back to the mother via the
umbilical arteries.

3. Question
330
C.BY: HOSSAM HAMDY MATERNITY RNPEDIA – ‫موسوعه التمريض‬

During a prenatal visit at 38 weeks, a nurse assesses the fetal heart rate. The
nurse determines that the fetal heart rate is normal if which of the following is
noted?

A. 80 BPM

B. 100 BPM

C. 150 BPM

D. 180 BPM

Answer: C. 150 BPM.

The fetal heart rate depends on gestational age and ranges from 160-170 BPM
in the first trimester but slows with fetal growth to 120-160 BPM near or at
term. At or near term, if the fetal heart rate is less than 120 or more than 160
BPM with the uterus at rest, the fetus may be in distress.

Option A: Data from a recently published study in a different context (Serra et


al., 2009) is compatible with the findings of our exploratory analysis with a
lower limit of 115 or 120 bpm for the gestational ages. Data for the 97th and
99th percentiles are not shown in this study. But shifting the lower limit to 120
will increase the number of false alarms whereas a lower limit of 115 will
inevitably increase the risk to misinterpret maternal heart rates as fetal heart
rate.

Option B: A lower limit of 120 bpm leads only near term to more false alarms
since normal FHR decreases further, and is more appropriate, to avoid
misinterpretation of maternal heartbeat as FHR.

Option D: The upper limit of 160 bpm raised concerns in the FIGO meeting in
1985, as Saling described abnormal findings in 24% of scalp blood analyses if
the baseline was higher than 160 bpm. It could be shown that the current FIGO
guidelines based on computerized analyses of the CTG show a high sensitivity
to detect fetal acidosis in case of a suspect or pathological classification of the
baseline level.

4. Question

A client arrives at a prenatal clinic for the first prenatal assessment. The client
tells a nurse that the first day of her last menstrual period was September 19th,

331
C.BY: HOSSAM HAMDY MATERNITY RNPEDIA – ‫موسوعه التمريض‬

2013. Using Naegele’s rule, the nurse determines the estimated date of
confinement as:

A. July 26, 2013

B. June 12, 2014

C. June 26, 2014

D. July 12, 2014

Answer: C. June 26, 2014.

Accurate use of Naegele’s rule requires that the woman has a regular 28-day
menstrual cycle. Add 7 days to the first day of the last menstrual period,
subtract three months, and then add one year to that date.

Option A: An average pregnancy lasts 280 days from the first day of the last
menstrual period (LMP) or 266 days after conception. Historically, an accurate
LMP is the best estimator to determine the due date.

Option B: Naegele’s rule, derived from a German obstetrician, subtracts 3


months and adds 7 days to calculate the estimated due date (EDD). It is prudent
for the obstetrician to get a detailed menstrual history, including duration, flow,
previous menstrual periods, and hormonal contraceptives. These factors are
used to determine the length of her cycles and ovulation period.

Option D: There are several fallacies with Naegele’s rule. First, a woman may
not accurately recall the first day of her menstrual cycle. Second, this method
assumes a woman’s cycle is exactly 28 days, with ovulation occurring at day
14, however, it does not consider menstrual cycles with shorter or longer
durations. Third, there are small variations in the duration between fertilization
and blastocyst implantation. Last, this method cannot differentiate between
menstrual bleeding and early pregnancy bleeding.

5. Question

A nurse is collecting data during an admission assessment of a client who is


pregnant with twins. The client has a healthy 5-year-old child that was delivered
at 37 weeks and tells the nurse that she doesn’t have any history of abortion or
fetal demise. The nurse would document the GTPAL for this client as:

A. Gravida 3, para 2001


332
C.BY: HOSSAM HAMDY MATERNITY RNPEDIA – ‫موسوعه التمريض‬

B. Gravida 2, para 0101

C. Gravida 1, para 1101

D. Gravida 2, para 1001

Answer: D. Gravida 2, para 1001

Pregnancy outcomes can be described with the acronym GTPAL.

You can learn more about obstetric history with our GTPAL Guide here.

“G” is Gravidity, the number of pregnancies.

“T” is term births, the number of born at term (37 weeks or after).

“P” is preterm births, the number born between 20-36 weeks gestation.

“A” is abortions or miscarriages, losses before 20 weeks.

“L” is live births, the number of births of living children.

6. Question

A nurse is performing an assessment of a primipara who is being evaluated in a


clinic during her second trimester of pregnancy. Which of the following
indicates an abnormal physical finding necessitating further testing?

A. Consistent increase in fundal height

B. Fetal heart rate of 180 BPM

C. Braxton Hicks contractions

D. Quickening

Answer: B. Fetal heart rate of 180 BPM.

The normal range of the fetal heart rate depends on gestational age. The heart
rate is usually 160-170 BPM in the first trimester and slows with fetal growth,
near and at term, the fetal heart rate ranges from 120-160 BPM. The other
options are expected.
333
C.BY: HOSSAM HAMDY MATERNITY RNPEDIA – ‫موسوعه التمريض‬

Option A: A fundal height measurement is typically done to determine if a


baby is small for its gestational age. The measurement is generally defined as
the distance in centimeters from the pubic bone to the top of the uterus. The
expectation is that after week 24 of pregnancy the fundal height for a normally
growing baby will match the number of weeks of pregnancy — plus or minus 2
centimeters.

Option C: Braxton Hicks contractions are sporadic contractions and relaxation


of the uterine muscle. Sometimes, they are referred to as prodromal or “false
labor” pains. It is believed they start around 6 weeks gestation but usually are
not felt until the second or third trimester of the pregnancy.

Option D: Quickening often occurs between the 16th to the 22nd week of
pregnancy. This is called a presumptive sign of pregnancy as the other
movements of the woman’s body can mimic early fetal movements such as
flatus, peristalsis, and abdominal muscle contractions. A multiparous woman
will usually first notice these fluttering movements of the fetus at an earlier
gestation than a primiparous woman.

7. Question

A nurse is reviewing the record of a client who has just been told that a
pregnancy test is positive. The physician has documented the presence of a
Goodell’s sign. The nurse determines this sign indicates:

A. A softening of the cervix.

B. A soft blowing sound that corresponds to the maternal pulse during


auscultation of the uterus.

C. The presence of hCG in the urine.

D. The presence of fetal movement.

Answer: A. A softening of the cervix.

In the early weeks of pregnancy, the cervix becomes softer as a result of


increased vascularity and hyperplasia, which causes the Goodell’s sign.

Option B: Uterine souffle or placental souffle is a soft, blowing sound heard


using a stethoscope, usually in the second trimester of pregnancy (13–28
weeks). This sound is heard most clearly in the lower part of the uterus and is
synchronous with the pulse of the mother.
334
C.BY: HOSSAM HAMDY MATERNITY RNPEDIA – ‫موسوعه التمريض‬

Option C: hCG levels can usually be detected in the urine about 10 days after
conception. If the woman takes a urine pregnancy test fewer than 10 days after
conception, the at-home tests might give a “false negative” response. This
means it will show that she is not pregnant when she actually is.

Option D: Adequate oxygenation of the fetal tissues is central to fetal


wellbeing. The importance of fetal movements as a marker of health has been
demonstrated in sheep models, with fetal behavior being reflective of fetal brain
function.

8. Question

A nursing instructor asks a nursing student who is preparing to assist with the
assessment of a pregnant client to describe the process of quickening. Which of
the following statements if made by the student indicates an understanding of
this term?

A. “It is the irregular, painless contractions that occur throughout pregnancy.”

B. “It is the soft blowing sound that can be heard when the uterus is
auscultated.”

C. “It is the fetal movement that is felt by the mother.”

D. “It is the thinning of the lower uterine segment.”

Answer: C. “It is the fetal movement that is felt by the mother.”

Quickening is fetal movement and may occur as early as the 16th and 18th week
of gestation, and the mother first notices subtle fetal movements that gradually
increase in intensity. A thinning of the lower uterine segment occurs about the
6th week of pregnancy and is called Hegar’s sign.

Option A: Braxton Hicks contractions are irregular, painless contractions that


may occur throughout the pregnancy.

Option B: Uterine souffle or placental souffle is a soft, blowing sound heard


using a stethoscope, usually in the second trimester of pregnancy (13–28
weeks). This sound is heard most clearly in the lower part of the uterus and is
synchronous with the pulse of the mother.

Option D: The lower uterine segment, therefore, is defined as the portion of the
uterine musculature which must undergo circumferential dilatation during labor,
335
C.BY: HOSSAM HAMDY MATERNITY RNPEDIA – ‫موسوعه التمريض‬

its extent being dependent upon the size of the presenting part and its level in
the uterine cavity. The available evidence suggests that brachystasis, with
retraction, occurs in this segment just as it does in the upper, and that thinning
in the first stage of labor is due not to passive elongation, but rather to active
shortening of the cup-shaped lower pole with dilatation as it is pulled up about
the presenting part.

9. Question

A nurse-midwife is performing an assessment of a pregnant client and is


assessing the client for the presence of ballottement. Which of the following
would the nurse implement to test for the presence of ballottement?

A. Auscultating for fetal heart sounds.

B. Palpating the abdomen for fetal movement.

C. Assessing the cervix for thinning.

D. Initiating a gentle upward tap on the cervix.

Answer: D. Initiating a gentle upward tap on the cervix.

Ballottement is a technique of palpating a floating structure by bouncing it


gently and feeling it rebound. In the technique used to palpate the fetus, the
examiner places a finger in the vagina and taps gently upward, causing the fetus
to rise. The fetus then sinks, and the examiner feels a gentle tap on the finger.

Option A: There are two methods of fetal heart rate monitoring in labor.
Auscultation is a method of periodically listening to the fetal heartbeat.
Electronic fetal monitoring is a procedure in which instruments are used to
continuously record the heartbeat of the fetus and the contractions of the
woman’s uterus during labor.

Option B: The Leopold maneuvers, named after the German obstetrician and
gynecologist Christian Gerhard Leopold (1846–1911), are part of the physical
examination of pregnant women. Four classical maneuvers are used to palpate
the gravid uterus systematically. This method of abdominal palpation is of low
cost, easy to perform, and non-invasive. It is used to determine the position,
presentation, and engagement of the fetus in utero.

Option C: Effacement means that the cervix stretches and gets thinner.
Dilatation means that the cervix opens. As labor nears, the cervix may start to
336
C.BY: HOSSAM HAMDY MATERNITY RNPEDIA – ‫موسوعه التمريض‬

thin or stretch (efface) and open (dilate). This prepares the cervix for the baby to
pass through the birth canal (vagina).

10. Question

A nurse is assisting in performing an assessment on a client who suspects that


she is pregnant and is checking the client for probable signs of pregnancy.
Which of the following signs indicates a probable sign of pregnancy? Select all
that apply.

A. Uterine enlargement

B. Fetal heart rate detected by nonelectric device

C. Outline of the fetus via radiography or ultrasound

D. Chadwick’s sign

E. Braxton Hicks contractions

F. Ballottement

Answer: A, D, E, and F.

The probable signs of pregnancy include:

Uterine Enlargement

Hegar’s sign or softening and thinning of the uterine segment that occurs at
week 6.

Goodell’s sign or softening of the cervix that occurs at the beginning of the 2nd
month

Chadwick’s sign or bluish coloration of the mucous membranes of the cervix,


vagina, and vulva. Occurs at week 6.

Ballottement or rebounding of the fetus against the examiner’s fingers of


palpation

Braxton-Hicks contractions

Positive pregnancy test measuring for hCG.

337
C.BY: HOSSAM HAMDY MATERNITY RNPEDIA – ‫موسوعه التمريض‬

Positive signs of pregnancy include:

Fetal Heart Rate detected by electronic device (Doppler) at 10-12 weeks

Fetal Heart rate detected by nonelectronic device (fetoscope) at 20 weeks AOG

Active fetal movement palpable by the examiners

Outline of the fetus via radiography or ultrasound

11. Question

A pregnant client calls the clinic and tells a nurse that she is experiencing leg
cramps and is awakened by the cramps at night. To provide relief from the leg
cramps, the nurse tells the client to:

A. Dorsiflex the foot while extending the knee when the cramps occur.

B. Dorsiflex the foot while flexing the knee when the cramps occur.

C. Plantar flex the foot while flexing the knee when the cramps occur.

D. Plantar flex the foot while extending the knee when the cramps occur.

Answer: A. Dorsiflex the foot while extending the knee when the cramps
occur.

Legs cramps occur when the pregnant woman stretches the leg and plantar
flexes the foot. Dorsiflexion of the foot while extending the knee stretches the
affected muscle, prevents the muscle from contracting, and stops the cramping.

Option B: If the cramps are in the calf, flex the foot to attempt to stretch the
muscle, or walk around on the heels if the pain isn’t unbearable.

Option C: Because they often happen at night when the legs are slightly bent
and the feet are pointed downward, some have suggested that this tightening
triggers a spasm.

Option D: Although the exact cause of muscle cramps is unknown (idiopathic),


some researchers believe inadequate stretching and muscle fatigue leads to
abnormalities in the mechanisms that control muscle contraction. Other factors
may also be involved, including poor conditioning, exercising or working in
intense heat, dehydration, and depletion of salt and minerals (electrolytes).

338
C.BY: HOSSAM HAMDY MATERNITY RNPEDIA – ‫موسوعه التمريض‬

12. Question

A nurse is providing instructions to a client in the first trimester of pregnancy


regarding measures to assist in reducing breast tenderness. The nurse tells the
client to:

A. Avoid wearing a bra.

B. Wash the nipples and areola area daily with soap and massage the breasts
with lotion.

C. Wear tight-fitting blouses or dresses to provide support.

D. Wash the breasts with warm water and keep them dry.

Answer: D. Wash the breasts with warm water and keep them dry.

The pregnant woman should be instructed to wash the breasts with warm water
and keep them dry. Breasts can become sore in early pregnancy for several
reasons, but one of the primary causes is changing hormone levels (such as
estrogen, progesterone, and prolactin).

Option A: Wearing a supportive bra with wide adjustable straps can decrease
breast tenderness. Back closures rather than front closures will give you the
ability to adjust as necessary.

Option B: The woman should be instructed to avoid using soap on the nipples
and areola area to prevent the drying of tissues.

Option C: Tight-fitting blouses or dresses will cause discomfort. The woman


might instinctually do everything she can to avoid allowing anything to touch
her breasts. For example, if the seatbelt is uncomfortable, adjust the strap that
zigzags across the torso so that it runs between the breasts and not across the top
of one of them.

13. Question

A pregnant client in the last trimester has been admitted to the hospital with a
diagnosis of severe preeclampsia. A nurse monitors for complications
associated with the diagnosis and assesses the client for:

A. Any bleeding, such as in the gums, petechiae, and purpura.

339
C.BY: HOSSAM HAMDY MATERNITY RNPEDIA – ‫موسوعه التمريض‬

B. Enlargement of the breasts.

C. Periods of fetal movement followed by quiet periods.

D. Complaints of feeling hot when the room is cool.

Answer: A. Any bleeding, such as in the gums, petechiae, and purpura.

Severe preeclampsia can trigger disseminated intravascular coagulation because


of the widespread damage to vascular integrity. Bleeding is an early sign of DIC
and should be reported to the M.D.

Option B: Estrogen stimulates growth of the breast duct cells and generates the
secretion of prolactin, another hormone. Prolactin stimulates breast enlargement
and milk production. Progesterone supports the formation and growth of milk-
producing cells within the glands of the breasts.

Option C: The first fetal movements which are felt by the mother are called
quickening. One function of these movements is to alert the pregnant woman
that she has a fetus growing in her uterus. Most providers recommend that
pregnant women monitor fetal movements, especially by the third trimester.
This can be accomplished by simply instructing the woman to have a general
awareness of the fetus and determine if the fetus is moving less than normal on
any given day or about the same as other days.

Option D: At the beginning of your pregnancy, new hormones are like little
workers that help keep everything humming along smoothly. These hormonal
changes also raise your body temperature a small amount.

14. Question

A client in the first trimester of pregnancy arrives at a health care clinic and
reports that she has been experiencing vaginal bleeding. A threatened abortion
is suspected, and the nurse instructs the client regarding management of care.
Which statement, if made by the client, indicates a need for further education?

A. “I will maintain strict bedrest throughout the remainder of the pregnancy.”

B. “I will avoid sexual intercourse until the bleeding has stopped, and for 2
weeks following the last evidence of bleeding.”

C. “I will count the number of perineal pads used on a daily basis and note the
amount and color of blood on the pad.”
340
C.BY: HOSSAM HAMDY MATERNITY RNPEDIA – ‫موسوعه التمريض‬

D. “I will watch for the evidence of the passage of tissue.”

Answer: A. “I will maintain strict bedrest throughout the remainder of the


pregnancy.”

Strict bed rest throughout the remainder of pregnancy is not required. Bedrest
and other activity restrictions have not been found to be efficacious in the
prevention of a threatened abortion progressing to spontaneous abortion and
have been shown to increase the risk of other complications including deep vein
thrombosis and/or pulmonary embolism and therefore should not be
recommended

Option B: The woman is advised to curtail sexual activities until the bleeding
has ceased, and for 2 weeks following the last evidence of bleeding or as
recommended by the physician.

Option C: The woman is instructed to count the number of perineal pads used
daily and to note the quantity and color of blood on the pad. Patients with a
threatened abortion should be managed expectantly without any medical or
surgical interventions. However, patients should be given strict return
precautions concerning excessive vaginal bleeding, abdominal pain, or fever
and patients should be educated on the importance of follow-up.

Option D: The woman also should watch for the evidence of the passage of
tissue. A threatened abortion is defined as vaginal bleeding before 20 weeks
gestational age in the setting of positive urine and/or blood pregnancy test with
a closed cervical os, without passage of products of conception, and without
evidence of fetal or embryonic demise.

15. Question

A prenatal nurse is providing instructions to a group of pregnant clients


regarding measures to prevent toxoplasmosis. Which statement if made by one
of the clients indicates a need for further instructions?

A. “I need to cook meat thoroughly.”

B. “I need to avoid touching mucous membranes of the mouth or eyes while


handling raw meat.”

C. “I need to drink unpasteurized milk only.”

341
C.BY: HOSSAM HAMDY MATERNITY RNPEDIA – ‫موسوعه التمريض‬

D. “I need to avoid contact with materials that are possibly contaminated with
cat feces.”

Answer: C. “I need to drink unpasteurized milk only.”

All pregnant women should be advised to do the following to prevent the


development of toxoplasmosis. Everyone, including immunocompetent patients,
should be educated about toxoplasmosis risk factors and ways to minimize the
risks. Preventing toxoplasmosis is particularly important in seronegative
immunocompromised patients and in pregnant women.

Option A: Avoid eating raw meat, unpasteurized milk, and uncooked eggs,
oysters, clams, and mussels. Rarely, infection by tachyzoites occurs from
ingestion of unpasteurized milk or by direct entry into the bloodstream through
a blood transfusion or laboratory accident. Transmission can also occur via
ingestion of tissue cysts (bradyzoites) in undercooked or uncooked meat or
through transplantation of an organ that contains tissue cysts. (Slaughterhouse
workers and butchers may be at increased risk of infection.) In Europe and the
United States, pork is the major source of T gondii infection in humans.

Option B: Women should be instructed to cook meats thoroughly, avoid


touching mucous membranes and eyes while handling raw meat; thoroughly
wash all kitchen surfaces that come into contact with uncooked meat, wash the
hands thoroughly after handling raw meat; avoid uncooked eggs and
unpasteurized milk; wash fruits and vegetables before consumption.

Option D: Avoid contact with materials that possibly are contaminated with cat
feces, such as cat litter boxes, sandboxes, and garden soil. T gondii oocysts are
ingested in material contaminated by feces from infected cats. Oocysts may also
be transported to food by flies and cockroaches. When T gondii is ingested,
bradyzoites are released from cysts or sporozoites are released from oocysts,
and the organisms enter gastrointestinal cells. Host cell receptors consisting of
laminin, lectin, and SAG1 are involved in T gondii tachyzoite attachment and
penetration. Tachyzoites multiply, rupture cells, and infect contiguous cells.
They are transported via the lymphatics and are disseminated hematogenously
throughout the tissues.

16. Question

A homecare nurse visits a pregnant client who has a diagnosis of mild


Preeclampsia and who is being monitored for pregnancy induced hypertension
(PIH). Which assessment finding indicates a worsening of the preeclampsia and
the need to notify the physician?
342
C.BY: HOSSAM HAMDY MATERNITY RNPEDIA – ‫موسوعه التمريض‬

A. Blood pressure reading is at the prenatal baseline.

B. Urinary output has increased.

C. The client complains of a headache and blurred vision.

D. Dependent edema has resolved.

Answer: C. The client complains of a headache and blurred vision.

If the client complains of a headache and blurred vision, the physician should be
notified because these are signs of worsening preeclampsia.

Option A: In normal pregnancy, women’s mean arterial pressure drops 10-15


mm Hg over the first half of pregnancy. Most women with mild chronic
hypertension (ie, SBP 140-160 mm Hg, DBP 90-100 mm Hg) have a similar
decrease in blood pressures and may not require any medication during this
period.

Option B: In addition to rising hormones, the body’s fluid levels start to


increase during pregnancy. This means the kidneys have to work extra hard to
flush the extra fluid. The amount of urine released will increase as well. In the
third trimester, the baby’s growing size means they’re pressing even more on
the bladder.

Option D: During normal pregnancy total body water increases by 6 to 8 liters,


4 to 6 liters of which are extracellular, of which at least 2 to 3 liters are
interstitial. At some stage in pregnancy 8 out of 10 women have demonstrable
clinical edema.

17. Question

A nurse implements a teaching plan for a pregnant client who is newly


diagnosed with gestational diabetes. Which statement if made by the client
indicates a need for further education?

A. “I need to stay on the diabetic diet.”

B. “I will perform glucose monitoring at home.”

C. “I need to avoid exercise because of the negative effects of insulin


production.”

343
C.BY: HOSSAM HAMDY MATERNITY RNPEDIA – ‫موسوعه التمريض‬

D. “I need to be aware of any infections and report signs of infection


immediately to my health care provider.”

Answer: C. “I need to avoid exercise because of the negative effects of


insulin production.”

Exercise is safe for the client with gestational diabetes and is helpful in
lowering the blood glucose level.

Option A: The goal of dietary therapy is to avoid single large meals and foods
with a large percentage of simple carbohydrates. The diet should include foods
with complex carbohydrates and cellulose, such as whole-grain breads and
legumes.

Option B: The best method for screening for gestational diabetes continues to
be controversial. The 2-step system is currently recommended in the United
States. A 50-g, 1-hour glucose challenge test (GCT) is followed by a 100-g, 3-
hour OGTT for those with an abnormal screening result. Alternatively, for high-
risk women, or in areas in which the prevalence of insulin resistance is 5% or
higher (eg, the southwestern and southeastern United States), a 1-step approach
can be used by proceeding directly to the 100-g, 3-hour OGTT.

Option D: Pregnant women with gestational diabetes mellitus (GDM) are


reported to be at increased risk for infections of the genital tract. Bacterial
vaginosis (BV) is known to be a crucial factor for preterm delivery (PTD),
causing up to 40 percent of premature births.

18. Question

A primigravida is receiving magnesium sulfate for the treatment of pregnancy


induced hypertension (PIH). The nurse who is caring for the client is
performing assessments every 30 minutes. Which assessment finding would be
of most concern to the nurse?

A. Urinary output of 20 ml since the previous assessment

B. Deep tendon reflexes of 2+

C. Respiratory rate of 10 BPM

D. Fetal heart rate of 120 BPM

344
C.BY: HOSSAM HAMDY MATERNITY RNPEDIA – ‫موسوعه التمريض‬

Answer: C. Respiratory rate of 10 BPM.

Magnesium sulfate depresses the respiratory rate. If the respiratory rate is less
than 12 breaths per minute, the physician or other health care provider needs to
be notified, and continuation of the medication needs to be reassessed.

Option A: A urinary output of 20 ml in a 30 minute period is adequate; less


than 30 ml in one hour needs to be reported. The kidneys face remarkable
demands during pregnancy, and it is critical that the practicing nephrologist
understands the normal kidney adaptations to pregnancy. GFR rises early to a
peak of 40% to 50% that of prepregnancy levels, resulting in lower levels of
serum creatinine, urea, and uric acid. There is a net gain of sodium and
potassium, but a greater retention of water, with gains of up to 1.6 L.

Option B: Deep tendon reflexes of 2+ are normal. With preeclampsia, a


woman’s reflexes become unusually active. Increasing blood pressure will lead
to increasing hyperreflexia until uncontrollable seizures eventually result.
Testing for this change is difficult in the field setting; in a clinic setting an
overactive patellar response is a good indicator.

Option D: The fetal heart rate is WNL for a resting fetus. Current international
guidelines recommend for the normal fetal heart rate (FHR) baseline different
ranges of 110 to 150 beats per minute (bpm) or 110 to 160 bpm.

19. Question

A nurse is caring for a pregnant client with preeclampsia. The nurse prepares a
plan of care for the client and documents in the plan that if the client progresses
from preeclampsia to eclampsia, the nurse’s first action is to:

A. Administer magnesium sulfate intravenously

B. Assess the blood pressure and fetal heart rate.

C. Clean and maintain an open airway.

D. Administer oxygen by face mask.

Answer: C. Clean and maintain an open airway.

The immediate care during a seizure (eclampsia) is to ensure a patent airway.


The other options are actions that follow or will be implemented after the
seizure has ceased.
345
C.BY: HOSSAM HAMDY MATERNITY RNPEDIA – ‫موسوعه التمريض‬

Option A: In this case, the doctor may prescribe magnesium sulfate as well as
medications to help reduce blood pressure. Magnesium sulfate therapy is used
to prevent seizures in women with preeclampsia. It can also help prolong a
pregnancy for up to two days.

Option B: Preeclampsia is when the blood pressure, or the force of blood


against the walls of the arteries, becomes high enough to damage the arteries
and other blood vessels. Damage to the arteries may restrict blood flow. It can
produce swelling in the blood vessels in the brain and to the growing baby. If
this abnormal blood flow through vessels interferes with the brain’s ability to
function, seizures may occur.

Option D: The initial treatment for eclampsia includes maintaining oxygen


delivery to both mother and fetus, minimizing the risk of aspiration, treating the
seizure, and controlling hypertension.

20. Question

A nurse is monitoring a pregnant client with pregnancy induced hypertension


who is at risk for preeclampsia. The nurse checks the client for which specific
signs of preeclampsia? Select all that apply.

A. Elevated blood pressure

B. Negative urinary protein

C. Facial edema

D. Increased respirations

E. Polydipsia

Answer: A & C. Elevated blood pressure and facial edema.

The three classic signs of preeclampsia are hypertension, generalized edema,


and proteinuria. Increased respirations are not a sign of preeclampsia.

Option A: Preeclampsia is defined as the presence of (1) a systolic blood


pressure (SBP) greater than or equal to 140 mm Hg or a diastolic blood pressure
(DBP) greater than or equal to 90 mm Hg or higher, on two occasions at least 4
hours apart in a previously normotensive patient, OR (2) an SBP greater than or
equal to 160 mm Hg or a DBP greater than or equal to 110 mm Hg or higher.

346
C.BY: HOSSAM HAMDY MATERNITY RNPEDIA – ‫موسوعه التمريض‬

Option B: In addition to the blood pressure criteria, proteinuria of greater than


or equal to 0.3 grams in a 24-hour urine specimen, a protein (mg/dL)/creatinine
(mg/dL) ratio of 0.3 or higher, or a urine dipstick protein of 1+ (if a quantitative
measurement is unavailable) is required to diagnose preeclampsia.

Option C: Edema exists in many pregnant women, but a sudden increase in


edema or facial edema is suggestive of preeclampsia. The edema of
preeclampsia occurs by a distinct mechanism that is similar to that of
angioneurotic edema.

Option D: Shortness of breath, a racing pulse, mental confusion, a heightened


sense of anxiety, and a sense of impending doom can be symptoms of
preeclampsia. If these symptoms are new to you, they could indicate an elevated
blood pressure, or more rarely, fluid collecting in your lungs (pulmonary
edema).

Option E: Primary polydipsia (PP) is a condition where there is excess


consumption of fluids leading to polyuria with diluted urine and, ultimately,
hyponatremia.

21. Question

Rho (D) immune globulin (RhoGAM) is prescribed for a woman following


delivery of a newborn infant and the nurse provides information to the woman
about the purpose of the medication. The nurse determines that the woman
understands the purpose of the medication if the woman states that it will
protect her next baby from which of the following?

A. Being affected by Rh incompatibility.

B. Having Rh-positive blood.

C. Developing a rubella infection.

D. Developing physiological jaundice.

Answer: A. Being affected by Rh incompatibility.

Rh incompatibility can occur when an Rh-negative mom becomes sensitized to


the Rh antigen. Sensitization may develop when an Rh-negative woman
becomes pregnant with a fetus who is Rh-positive. Administration of Rho(D)
immune globulin prevents the woman from developing antibodies against Rh-
positive blood by providing passive antibody protection against the Rh antigen.
347
C.BY: HOSSAM HAMDY MATERNITY RNPEDIA – ‫موسوعه التمريض‬

Option B: During pregnancy and at delivery, some of the baby’s Rh-positive


blood can enter the maternal circulation, causing the woman’s immune system
to form antibodies against Rh-positive blood.

Option C: Rubella can be prevented with MMR vaccine. This protects against
three diseases: measles, mumps, and rubella. CDC recommends children get
two doses of MMR vaccine, starting with the first dose at 12 through 15 months
of age, and the second dose at 4 through 6 years of age. Teens and adults also
should also be up to date on their MMR vaccination.

Option D: The best preventive of infant jaundice is adequate feeding. Breast-


fed infants should have eight to 12 feedings a day for the first several days of
life. Formula-fed infants usually should have 1 to 2 ounces (about 30 to 60
milliliters) of formula every two to three hours for the first week.

22. Question

A pregnant client is receiving magnesium sulfate for the management of


preeclampsia. A nurse determines the client is experiencing toxicity from the
medication if which of the following is noted on assessment?

A. Presence of deep tendon reflexes.

B. Serum magnesium level of 6 mEq/L.

C. Proteinuria of +3.

D. Respirations of 10 per minute.

Answer: D. Respirations of 10 per minute.

Magnesium toxicity can occur from magnesium sulfate therapy. Signs of


toxicity relate to the central nervous system depressant effects of the medication
and include respiratory depression, loss of deep tendon reflexes, and a sudden
drop in the fetal heart rate and maternal heart rate and blood pressure.

Option A: Although deep tendon reflexes are more useful in assessing


magnesium toxicity, the presence of clonus may indicate an increased risk of
convulsions.

Option B: Therapeutic levels of magnesium are 4-7 mEq/L. Magnesium sulfate


is the first-line treatment for the prevention of primary and recurrent eclamptic

348
C.BY: HOSSAM HAMDY MATERNITY RNPEDIA – ‫موسوعه التمريض‬

seizures. For eclamptic seizures that are refractory to magnesium sulfate,


lorazepam and phenytoin may be used as second-line agents.

Option C: Proteinuria of +3 would be noted in a client with preeclampsia.


Proteinuria is defined as the presence of at least 300 mg of protein in a 24-hour
urine collection, a protein (mg/dL)/creatinine (mg/dL) ratio greater than or
equal to 0.3, or a urine dipstick protein of 1+ (if a quantitative measurement is
unavailable). Serial confirmations 6 hours apart increase the predictive value.
Although more convenient, a urine dipstick value of 1+ or more (30 mg/dL) is
not reliable in the diagnosis of proteinuria.

23. Question

A woman with preeclampsia is receiving magnesium sulfate. The nurse


assigned to care for the client determines that the magnesium therapy is
effective if:

A. Ankle clonus is noted.

B. The blood pressure decreases.

C. Seizures do not occur.

D. Scotomas are present.

Answer: C. Seizures do not occur.

For a client with preeclampsia, the goal of care is directed at preventing


eclampsia (seizures). Seizures were a half or a third less likely to recur after
treatment with magnesium. Maternal mortality was also lower in women
allocated magnesium rather than phenytoin or diazepam, although this did not
achieve statistical significance. Recent Cochrane reviews, however, indicated a
significant reduction in maternal mortality with magnesium.

Option A: Ankle clonus indicated hyperreflexia and may precede the onset of
eclampsia. Although brisk or hyperactive reflexes are common during
pregnancy, clonus is a sign of neuromuscular irritability that usually reflects
severe preeclampsia.

Option B: Magnesium sulfate is an anticonvulsant, not an antihypertensive


agent. Although a decrease in blood pressure may be noted initially, this effect
is usually transient.

349
C.BY: HOSSAM HAMDY MATERNITY RNPEDIA – ‫موسوعه التمريض‬

Option D: Scotomas are areas of complete or partial blindness. Visual


disturbances, such as scotomas, often precede an eclamptic seizure.

24. Question

A nurse is caring for a pregnant client with severe preeclampsia who is


receiving IV magnesium sulfate. Select all nursing interventions that apply in
the care for the client.

A. Monitor maternal vital signs every 2 hours.

B. Notify the physician if respirations are less than 18 per minute.

C. Monitor renal function and cardiac function closely.

D. Keep calcium gluconate on hand in case of a magnesium sulfate overdose.

E. Monitor deep tendon reflexes hourly.

F. Monitor I and O’s hourly.

G. Notify the physician if urinary output is less than 30 ml per hour.

Answer: C, D, E, F, and G.

Option A: BP should be assessed with the goal of maintaining the diastolic BP


at less than 110 mm Hg with administration of antihypertensive medications as
needed (eg, hydralazine, labetalol, nifedipine).

Option B: When caring for a client receiving magnesium sulfate therapy, the
nurse would monitor maternal vital signs, especially respirations, every 30-60
minutes and notify the physician if respirations are less than 12, because this
would indicate respiratory depression.

Option C: Cardiac and renal function are monitored closely. Eclampsia-


associated renal abnormalities can include decreases in glomerular filtration
rate, renal plasma flow, and uric acid clearance as well as proteinuria.
Eclampsia is associated with cardiovascular derangements such as generalized
vasospasm, increased peripheral vascular resistance, and increased left
ventricular stroke work index. Pulmonary capillary wedge pressure (PCWP)
may vary from low to elevated. Importantly, central venous pressure (CVP)
may not correlate with PCWP in patients with severe preeclampsia or
eclampsia.
350
C.BY: HOSSAM HAMDY MATERNITY RNPEDIA – ‫موسوعه التمريض‬

Option D: Calcium gluconate is kept on hand in case of magnesium sulfate


overdose because calcium gluconate is the antidote for magnesium sulfate
toxicity.

Option E: Deep tendon reflexes are assessed hourly. Ankle clonus indicated
hyperreflexia and may precede the onset of eclampsia. Although brisk or
hyperactive reflexes are common during pregnancy, clonus is a sign of
neuromuscular irritability that usually reflects severe preeclampsia.

Option F: Monitor fluid intake and urine output, maternal respiratory rate, and
oxygenation, as indicated, and continuously monitor fetal status. Pulmonary
arterial pressure monitoring is rarely indicated but may be helpful in patients
who have evidence of pulmonary edema or oliguria/anuria.

Option G: The urine output should be maintained at 30 ml per hour because the
medication is eliminated through the kidneys.
Source:

25. Question

In the 12th week of gestation, a client completely expels the products of


conception. Because the client is Rh-negative, the nurse must:

A. Administer RhoGAM within 72 hours.

B. Make certain she receives RhoGAM on her first clinic visit.

C. Not give RhoGAM, since it is not used with the birth of a stillborn.

D. Make certain the client does not receive RhoGAM since the gestation only
lasted 12 weeks.

Answer: A. Administer RhoGAM within 72 hours.

RhoGAM is given within 72 hours postpartum if the client has not been
sensitized already. When the blood of an Rh-positive fetus gets into the
bloodstream of an Rh-negative woman, her body will recognize that the Rh-
positive blood is not hers. Her body will try to destroy it by making anti-Rh
antibodies. These antibodies can cross the placenta and attack the fetus’s blood
cells. This can lead to serious health problems, even death, for a fetus or a
newborn.

351
C.BY: HOSSAM HAMDY MATERNITY RNPEDIA – ‫موسوعه التمريض‬

Option B: RhoGAM is a prescription medicine that is used to prevent Rh


immunization, a condition in which an individual with Rh-negative blood
develops antibodies after exposure to Rh-positive blood. RhoGAM is
administered by intramuscular (IM) injection. RhoGAM is purified from human
plasma containing anti-Rh (anti-D).

Option C: The doctor will administer at least one dose of RhoGAM between 26
and 28 weeks of pregnancy. If the baby is found to be Rh-positive at birth, the
mother will receive an additional dose within 72 hours after delivery.

Option D: 1 Rh-negative pregnant woman in 5 will become sensitive to the Rh-


positive factor if she doesn’t receive RhoGAM. That means that her baby can be
born with one or more of the following things: anemia, a lack of healthy red
blood cells. heart failure

352
C.BY: HOSSAM HAMDY MATERNITY RNPEDIA – ‫موسوعه التمريض‬

Maternity Nursing NU.7 (Quiz #1: 25 Questions)

1. Question

In a lecture on sexual functioning, the nurse plans to include the fact that
ovulation occurs when the:

A. Oxytocin is too high.

B. Blood level of LH is too high.

C. Progesterone level is high.

D. Endometrial wall is sloughed off.

Answer: B. Blood level of LH is too high.

It is the surge of LH secretion in mid-cycle that is responsible for ovulation.


LH is responsible for inducing ovulation, preparation for fertilized oocyte
uterine implantation, and the ovarian production of progesterone through
stimulation of theca cells and luteinized granulosa cells.

Option A: Ovulation is the third phase within the larger Uterine Cycle (i.e.
Menstrual Cycle). The follicular release follows the Follicular phase (i.e.
dominant follicle development) and precedes the Luteal phase (i.e.
maintenance of corpus luteum) that progresses to either endometrial
shedding or implantation. Follicular release occurs around 14 days prior to
menstruation in a cyclic pattern if the hypothalamic-pituitary-ovarian axis
function is well regulated.

Option C: Ovulation occurs around day 14 of a typical 28-day cycle.


Estrogen levels rise as a result of increased estrogen production by
hormonally active granulosa cells within the follicle. One of the estrogen
levels reach a critical point and remain at the level for 2 days, estrogen
transitions from a negative feedback modulator of GnRH to a positive
feedback modulator on the hypothalamus.

Option D: FSH and LH stimulate what remains of the mature follicle after
ovulation to become the corpus luteum. The corpus luteum grows and
secretes progesterone and some estrogen, which makes the endometrium
more receptive to implantation. If fertilization does not occur,
353
C.BY: HOSSAM HAMDY MATERNITY RNPEDIA – ‫موسوعه التمريض‬

progesterone/estrogen levels fall, and the corpus luteum dies forming the
corpus Albicans. These falling hormone levels stimulate FSH to begin
recruiting follicles for the next cycle.
2. Question

The chief function of progesterone is the:

A. Development of the female reproductive system.

B. Stimulation of the follicles for ovulation to occur.

C. Preparation of the uterus to receive a fertilized egg.

D. Establishment of secondary male sex characteristics.

Answer: C. Preparation of the uterus to receive a fertilized egg.

Progesterone stimulates differentiation of the endometrium into a secretory


type of tissue. Progesterone is an endogenous steroid hormone that is
commonly produced by the adrenal cortex as well as the gonads, which
consist of the ovaries and the testes.

Option A: The increase of progesterone, specifically in the menstrual cycle,


occurs due to the initiation of a mid-cycle luteinizing hormone (LH) surge
near the end of the follicular phase. This mid-cycle rise of progesterone also
allows for an increase in follicle-stimulating hormone (FSH).

Option B: At the end of the LH and FSH surge, the menstrual cycle begins
the luteal phase, during which progesterone prepares the endometrium in a
woman’s uterus to receive and nourish the fertilized egg, also known as
implantation during the luteal phase.

Option D: Testosterone is the primary male hormone responsible for


regulating sex differentiation, producing male sex characteristics,
spermatogenesis and fertility.
3. Question

The developing cells are called a fetus from the:

A. Time the fetal heart is heard

B. Eighth week to the time of birth.


354
C.BY: HOSSAM HAMDY MATERNITY RNPEDIA – ‫موسوعه التمريض‬

C. Implantation of the fertilized ovum.

D. End of the send week to the onset of labor.

Answer: B. Eighth week to the time of birth.

In the first 7-14 days, the ovum is known as a blastocyst; it is called an


embryo until the eighth week; the developing cells are then called a fetus
until birth.

Option A: The ectoderm forms the epidermis, nails, hair, peripheral nervous
system, brain, and spinal cord. The mesoderm forms the muscle, bone,
connective tissue, notochord, kidney, gonads, and circulatory system. The
endoderm forms the epithelial lining of the digestive tract, stomach, colon,
liver, bladder, and pancreas.

Option C: Once fertilization takes place, there are quick changes at the
cellular level of the zygote. The zygote is a single cell, and it undergoes
mitosis to create many cells. Once the zygote has reached the thirty-two cell
stage, it becomes morula. Day four begins blastulation and cavities begin to
form by first forming a hollow ball. Some studies suggest that the timing of
this process may affect implantation.

Option D: At sixteen weeks the primitive streak forms. The primitive streak
establishes the midline of the body. The next stage in development is
neurulation. At this time the notochord induces the ectoderm to form the
neural plate which eventually forms the neural tube. The neural tube will
become the brain and spinal cord.
4. Question

After the first four months of pregnancy, the chief source of estrogen and
progesterone is the:

A. Placenta

B. Adrenal cortex

C. Corpus luteum

D. Anterior hypophysis

355
C.BY: HOSSAM HAMDY MATERNITY RNPEDIA – ‫موسوعه التمريض‬

Answer: A. Placenta.

When the placental formation is complete, around the 16th week of


pregnancy; it produces estrogen and progesterone.

Option B: The adrenal cortex—the outer part of the gland—produces


hormones that are vital to life, such as cortisol (which helps regulate
metabolism and helps the body respond to stress) and aldosterone (which
helps control blood pressure).

Option C: The primary purpose of the corpus luteum is to pulse out


hormones, including progesterone. Progesterone is required for a viable
pregnancy to occur and to continue. Progesterone helps the uterine lining,
known as the endometrium, to thicken and become spongy.

Option D: Anterior pituitary undergoes two- to three-fold enlargement


during pregnancy, because of hyperplasia and hypertrophy of lactotroph
cells. In contrast to lactotrophs, the size of other anterior pituitary cells
remains unchanged or decreases.
5. Question

The nurse recognizes that an expected change in the hematologic system that
occurs during the 2nd trimester of pregnancy is:

A. A decrease in WBC’s

B. Increase in hematocrit.

C. An increase in blood volume.

D. A decrease in sedimentation rate.

Answer: C. An increase in blood volume.

The blood volume increases by approximately 40-50% during pregnancy.


The peak blood volume occurs between 30 and 34 weeks of gestation. The
hematocrit decreases as a result of the increased blood volume.

Option A: WBC count increases to 6 to 16 million/mL and can be as high as


20 million/mL during and shortly after labor.

356
C.BY: HOSSAM HAMDY MATERNITY RNPEDIA – ‫موسوعه التمريض‬

Option B: In pregnancy, the RBC volume increases by 20% to 30%, while


the plasma volume increases 45 to 55%. This disproportionate volume
increase leads to dilutional anemia with decreased hematocrit.

Option D: Fibrinogen and factors VII – X levels increase, but the clotting
and bleeding times remain unchanged. However, increased venous stasis and
damaged vessel endothelium result in higher rates of thromboembolic events
during pregnancy.
6. Question

The nurse is aware that an adaptation of pregnancy is an increased blood


supply to the pelvic region that results in a purplish discoloration of the
vaginal mucosa, which is known as:

A. Ladin’s sign

B. Hegar’s sign

C. Goodell’s sign

D. Chadwick’s sign

Answer: D. Chadwick’s sign.

A purplish color results from the increased vascularity and blood vessel
engorgement of the vagina. It can be observed as early as 6 to 8 weeks after
conception, and its presence is an early sign of pregnancy.

Option A: Ladin’s sign is a clinical sign of pregnancy in which there is


softening in the midline of the uterus anteriorly at the junction of the uterus
and cervix. It occurs and is detectable with manual examination at about 6
weeks’ gestation.

Option B: Hegar’s sign is a non-sensitive indication of pregnancy in women


— its absence does not exclude pregnancy. It pertains to the features of the
cervix and the uterine isthmus. It is demonstrated as a softening in the
consistency of the uterus, and the uterus and cervix seem to be two separate
regions.

Option C: Goodell sign is an indication of pregnancy. It is a significant


softening of the vaginal portion of the cervix from increased vascularization.
This vascularization is a result of hypertrophy and engorgement of the
357
C.BY: HOSSAM HAMDY MATERNITY RNPEDIA – ‫موسوعه التمريض‬

vessels below the growing uterus. This sign occurs at approximately four
weeks’ gestation.
7. Question

A pregnant client is making her first antepartum visit. She has a 2-year-old
son born at 40 weeks, a 5-year-old daughter born at 38 weeks, and 7-year-
old twin daughters born at 35 weeks. She had a spontaneous abortion 3 years
ago at 10 weeks. Using the GTPAL format, the nurse should identify that the
client is:

A. G4 T3 P2 A1 L4

B. G5 T2 P2 A1 L4

C. G5 T2 P1 A1 L4

D. G4 T3 P1 A1 L4

Answer: C. G5 T2 P1 A1 L4.

5 pregnancies; 2 term births; twins count as 1; one abortion; 4 living


children. A good starting point is to ask about the number of children the
patient has given birth to. Next, sensitively ask about miscarriages,
stillbirths, ectopics and terminations.

Option A: Gravida should be 5, term births should be 3, and parity should


be 1. Gravidity is the total number of pregnancies, regardless of outcome.

Option B: Parity should be 1. Parity is the total number of pregnancies


carried over the threshold of visibility.

Option D: Gravida should be 5, and term births should be 2.


8. Question

An expected cardiopulmonary adaptation experienced by most pregnant


women is:

A. Tachycardia

B. Dyspnea at rest

C. Progression of dependent edema


358
C.BY: HOSSAM HAMDY MATERNITY RNPEDIA – ‫موسوعه التمريض‬

D. Shortness of breath on exertion

Answer: D. Shortness of breath on exertion.

This is an expected cardiopulmonary adaptation during pregnancy; it is


caused by an increased ventricular rate and elevated diaphragm.

Option A: In pregnancy, the cardiac output increases 30 to 60%, with the


majority of the increase occurring during the first trimester. The maximum
output is reached between 20 and 24 weeks and is maintained until delivery.
Initially, the increase in cardiac output is due to an increase in stroke
volume. As the stroke volume decreases towards the end of the third
trimester, an increase in heart rate acts to maintain the increased cardiac
output.

Option B: During pregnancy, the diaphragm elevates, resulting in a 5%


decrease in total lung capacity (TLC). However, the tidal volume (TV)
increases by 30 to 40%, thereby decreasing the expiratory reserve volume by
20%. Minute ventilation is similarly increased by 30 to 40%, owing to the
fact that TV becomes increased while a constant respiratory rate is
maintained.

Option C: During pregnancy, the extra fluid in the body and the pressure
from the growing uterus can cause swelling (edema) in the ankles and feet.
The swelling tends to get worse as a woman’s due date nears, particularly
near the end of the day and during hotter weather.
9. Question

Nutritional planning for a newly pregnant woman of average height and


weighing 145 pounds should include:

A. A decrease of 200 calories a day.

B. An increase of 300 calories a day.

C. An increase of 500 calories a day.

D. A maintenance of her present caloric intake per day.

Answer: B. An increase of 300 calories a day.

359
C.BY: HOSSAM HAMDY MATERNITY RNPEDIA – ‫موسوعه التمريض‬

This is the recommended caloric increase for adult women to meet the
increased metabolic demands of pregnancy. These calories should come
from a balanced diet of protein, fruits, vegetables and whole grains. Sweets
and fats should be kept to a minimum. A healthy, well-balanced diet can also
help to reduce some pregnancy symptoms, such as nausea and constipation.

Option A: Whenever possible, eat complex carbohydrates and limit simple


carbohydrates.

Option C: Protein needs increase considerably during pregnancy and peak


during the third trimester. Add a protein-rich food to every meal and snack to
ensure enough protein throughout the pregnancy.

Option D: Whole grains and legumes, such as dried peas and beans, and
other healthy carbs like fruit and starchy vegetables should make regular
appearances on the plate. They provide B vitamins and trace minerals, such
as zinc selenium, and magnesium. Grains and legumes are full of nutrients,
including iron and the various B vitamins: thiamin (vitamin B-1), riboflavin
(vitamin B-2), folate, and niacin.
10. Question

During a prenatal examination, the nurse draws blood from a young Rh-
negative client and explain that an indirect Coombs test will be performed to
predict whether the fetus is at risk for:

A. Acute hemolytic disease

B. Respiratory distress syndrome

C. Protein metabolic deficiency

D. Physiologic hyperbilirubinemia

Answer: A. Acute hemolytic disease.

When an Rh-negative mother carries an Rh-positive fetus there is a risk for


maternal antibodies against Rh-positive blood; antibodies cross the placenta
and destroy the fetal RBCs.

Option B: Respiratory distress syndrome, also known as hyaline membrane


disease, occurs almost exclusively in premature infants. In premature infants,
respiratory distress syndrome develops because of impaired surfactant
360
C.BY: HOSSAM HAMDY MATERNITY RNPEDIA – ‫موسوعه التمريض‬

synthesis and secretion leading to atelectasis, ventilation-perfusion (V/Q)


inequality, and hypoventilation with resultant hypoxemia and hypercarbia.

Option C: Infants with protein metabolism disorders are unable to


metabolize certain amino acids and require specialized formulas without the
offending amino acid, allowing the baby to receive essential nutrients for
growth.

Option D: Physiologic jaundice is also referred to as non-pathologic


jaundice, and it is mild and transient. This occurs because of differences in
the metabolism of bilirubin in the neonatal period leading to an increased
bilirubin load.
11. Question

When involved in prenatal teaching, the nurse should advise the clients that
an increase in vaginal secretions during pregnancy is called leukorrhea and is
caused by increased:

A. Metabolic rates.

B. Production of estrogen.

C. Functioning of the Bartholin glands.

D. Supply of sodium chloride to the cells of the vagina.

Answer: B. Production of estrogen.

The increase of estrogen during pregnancy causes hyperplasia of the vaginal


mucosa, which leads to increased production of mucus by the endocervical
glands. The mucus contains exfoliated epithelial cells.

Option A: The placenta produces human placental lactogen (hPL), which


acts to supply nutrition to the fetus. It induces lipolysis to increase free fatty
acids, which are preferentially used by the pregnant mother for fuel. It also
acts as an insulin antagonist to induce a diabetogenic state. This activity
prompts hyperplasia of pancreatic beta-cells to create increased insulin levels
and protein synthesis. In early pregnancy, maternal insulin sensitivity
increases, followed by resistance in the second and third trimesters.

Option C: The incidence of Bartholin gland abscesses during pregnancy was


0.13%. Eight (20%) abscesses occurred in the first, 18 (45%) in the second,
361
C.BY: HOSSAM HAMDY MATERNITY RNPEDIA – ‫موسوعه التمريض‬

11 (47.5%) in the third trimester, and 3 (7.5%) in the post-partum course. No


severe perineal and neonatal infections occurred during pregnancy.

Option D: Vagina, during pregnancy there is an increase in the blood supply


to the vagina, its color change from pink to purple, and becomes more elastic
in the second trimester.
12. Question

A 26-year old multigravida is 14 weeks pregnant and is scheduled for an


alpha-fetoprotein test. She asks the nurse, “What does the alpha-fetoprotein
test indicate?” The nurse bases a response on the knowledge that this test can
detect:

A. Kidney defects

B. Cardiac defects

C. Neural tube defects

D. Urinary tract defects

Answer: C. Neural tube defects.

The alpha-fetoprotein test detects neural tube defects and Down syndrome.
Alpha-fetoprotein (AFP) is a plasma protein produced by the embryonic yolk
sac and the fetal liver. AFP levels in serum, amniotic fluid, and urine
functions as a screening test for congenital disabilities, chromosomal
abnormalities, as well as some other adult occurring tumors and pathologies.

Option A: In some cases, one or both kidneys may fail to develop. In other
instances, an abnormality may be present that blocks the outflow of urine.
This blockage may cause urine to back up into the kidney, a condition called
hydronephrosis, which causes the kidney to appear enlarged on the
ultrasound test. Another common abnormality is called reflux. This occurs
when a valve-like mechanism at the point where the ureter joins the bladder
does not work, allowing urine to wash back up into the kidney.

Option B: The baby’s heart begins to form immediately after conception


and is complete by eight week’s gestation. The heart begins as a tube-shaped
structure that twists and divides to form the heart and heart valves. A
congenital heart defect usually occurs because the heart does not twist or
divide normally. Some mothers wonder if drugs, alcohol, or medications
362
C.BY: HOSSAM HAMDY MATERNITY RNPEDIA – ‫موسوعه التمريض‬

contributed to their child’s heart defect. In most cases, we don’t know why
these defects occur. Although, some heart defects can run in families or be
related to a disease the mother has, diabetes mellitus, for example.

Option D: Common birth defects of the urinary system include hypospadias,


obstructive defects of the renal pelvis, and renal agenesis. Hypospadias is
characterized by the location of the urethral opening on the underside of the
penis. Obstructive defects of the renal pelvis prevent urine from entering the
bladder.
13. Question

At a prenatal visit at 36 weeks’ gestation, a client complains of discomfort


with irregularly occurring contractions. The nurse instructs the client to:

A. Lie down until they stop.

B. Walk around until they subside.

C. Time contraction for 30 minutes.

D. Take 10 grains of aspirin for the discomfort.

Answer: B. Walk around until they subside.

Ambulation relieves Braxton Hicks. Braxton Hicks contractions are thought


to play a role in toning the uterine muscle in preparation for the birth
process. Sometimes Braxton Hicks contractions are referred to as “practice
for labor.” Braxton Hicks contractions do not result in dilation of the cervix
but may have a role in cervical softening.

Option A: Braxton Hicks contractions may stop with a change in activity


level or as the woman changes position. If she can sleep through the
contraction, it is a Braxton Hicks contraction. True labor contractions
continue and may even become stronger with movement or position change.

Option C: Braxton Hicks contractions are unpredictable. They may last less
than 30 seconds or up to 2 minutes. True labor contractions last between 30
to less than 90 seconds and become longer over time.

Option D: There is no medical treatment for Braxton Hicks contractions.


However, taking action to change the situation that triggered the Braxton
Hicks contractions is warranted.
363
C.BY: HOSSAM HAMDY MATERNITY RNPEDIA – ‫موسوعه التمريض‬

14. Question

The nurse teaches a pregnant woman to avoid lying on her back. The nurse
has based this statement on the knowledge that the supine position can:

A. Unduly prolonged labor.

B. Cause decreased placental perfusion.

C. Lead to transient episodes of hypotension.

D. Interfere with free movement of the coccyx.

Answer: B. Cause decreased placental perfusion.

This is because of the impedance of venous return by the gravid uterus,


which causes hypotension and decreased systemic perfusion.

Option A: More recently there is emerging evidence that if the woman


sleeps on her back that this puts her at increased risk of stillbirth (Stacey et
al. 2011; Owusu et al. 2013; Gordon et al. 2015). This is biologically
plausible because of what is already known about negative sequelae of the
woman adopting this position during the day.

Option C: Maternity care provider clinicians have been aware for many
decades that maternal supine position and pregnancy are not a good mix.
This is probably because when the woman lies on her back the gravid uterus
is known to compress the inferior vena cava (Kerr et al. 1964). This can
result in a range of negative sequelae such as maternal hypotension and
reduced blood flow to the fetus (Holmes, 1960).

Option D: The supine position may be disadvantageous for fetal wellbeing


and in compromised pregnancies may be a sufficient stressor to contribute to
fetal demise. This fits well with the triple risk model for stillbirth.
15. Question

The pituitary hormone that stimulates the secretion of milk from the
mammary glands is:

A. Prolactin

B. Oxytocin

364
C.BY: HOSSAM HAMDY MATERNITY RNPEDIA – ‫موسوعه التمريض‬

C. Estrogen

D. Progesterone

Answer: A. Prolactin.

Prolactin is the hormone from the anterior pituitary gland that stimulates
mammary gland secretion. Oxytocin, a posterior pituitary hormone,
stimulates the uterine musculature to contract and causes the “let down”
reflex.

Option B: Oxytocin has been best known for its roles in female
reproduction. It is released in large amounts during labor, and after
stimulation of the nipples. It is a facilitator for childbirth and breastfeeding.
One of the oldest applications of oxytocin as a proper drug is as a therapeutic
agent during labor and delivery. It is a stimulant widely employed to induce
or augment labor, especially at term, when adequate oxytocin receptors are
present. It is also one of the principal uterotonic drugs used to prevent
postpartum hemorrhage.

Option C: Estrogen is a steroid hormone associated with the female


reproductive organs and is responsible for the development of female sexual
characteristics. In the uterus, estrogen helps to proliferate endometrial cells
in the follicular phase of the menstrual cycle, thickening the endometrial
lining in preparation for pregnancy.

Option D: Progesterone is an endogenous steroid hormone that is commonly


produced by the adrenal cortex as well as the gonads, which consist of the
ovaries and the testes. Progesterone is also secreted by the ovarian corpus
luteum during the first ten weeks of pregnancy, followed by the placenta in
the later phase of pregnancy. The conversion of progesterone generation
from the corpus luteum to the placenta generally occurs after week ten.
16. Question

Which of the following symptoms occurs with a hydatidiform mole?

A. Heavy, bright red bleeding every 21 days.

B. Fetal cardiac motion after 6 weeks gestation.

C. Benign tumors found in the smooth muscle of the uterus.

365
C.BY: HOSSAM HAMDY MATERNITY RNPEDIA – ‫موسوعه التمريض‬

D. “Snowstorm” pattern on ultrasound with no fetus or gestational sac.

Answer: D. “Snowstorm” pattern on ultrasound with no fetus or


gestational sac.

The chorionic villi of a molar pregnancy resemble a snowstorm pattern on


ultrasound. Bleeding with a hydatidiform mole is often dark brown and may
occur erratically for weeks or months.

Option A: The most common symptom of placenta previa is bright red,


painless bleeding from the vagina. This is most common in the third
trimester of pregnancy.

Option B: The heart rate (HR) increases between the 5th week of gestation
and 9th week of gestation and after the 13th week of gestation reduces.
Cardiovascular development in a human embryo occurs between 3 and 6
weeks after ovulation. Cardiac function is the first sign of independent
cardiac activity that can be explored with non-invasive techniques such as
Doppler ultrasound

Option C: Uterine smooth muscle tumors are neoplasms composed of


smooth muscle; they range from benign leiomyomas to low-grade and high-
grade leiomyosarcomas. Several histologic subtypes exist, including usual
(spindled), epithelioid, and myxoid tumors.
17. Question

Which of the following terms applies to the tiny, blanched, slightly raised
end arterioles found on the face, neck, arms, and chest during pregnancy?

A. Epulis

B. Linea nigra

C. Striae gravidarum

D. Telangiectasias

Answer: D. Telangiectasias.

The dilated arterioles that occur during pregnancy are due to the elevated
level of circulating estrogen. Telangiectasia is a condition in which widened
366
C.BY: HOSSAM HAMDY MATERNITY RNPEDIA – ‫موسوعه التمريض‬

venules (tiny blood vessels) cause threadlike red lines or patterns on the skin.
These patterns, or telangiectasia, form gradually and often in clusters.
They’re sometimes known as “spider veins” because of their fine and
weblike appearance.

Option A: Occasionally, a severe gum swelling, known as a “pregnancy


epulis” or “pregnancy tumor”, may occur as a severe response to debris left
on teeth. This often may subside spontaneously following the birth of the
baby.

Option B: The linea nigra is a pigmented line extending from the symphysis
pubis to the top of the fundus during pregnancy.

Option C: Striae gravidarum (SG) are atrophic linear scars that represent
one of the most common connective tissue changes during pregnancy. SG
can cause emotional and psychological distress for many women.
18. Question

Which of the following conditions is common in pregnant women in the 2nd


trimester of pregnancy?

A. Mastitis

B. Metabolic alkalosis

C. Physiologic anemia

D. Respiratory acidosis

Answer: C. Physiologic anemia.

Hemoglobin and hematocrit levels decrease during pregnancy as the increase


in plasma volume exceeds the increase in red blood cell production.

Option A: Mastitis, which mainly affects breastfeeding women, causes


redness, swelling, and pain in one or both breasts. Mastitis is an
inflammation of breast tissue that sometimes involves an infection. The
inflammation results in breast pain, swelling, warmth, and redness. You
might also have fever and chills.

Option B: Metabolic alkalosis is uncommon in pregnancy and is most often


the result of severe vomiting. If this is present at the time of delivery,

367
C.BY: HOSSAM HAMDY MATERNITY RNPEDIA – ‫موسوعه التمريض‬

transient metabolic derangement in the fetus can occur, potentially requiring


additional organ support.

Option D: Progesterone levels are increased during pregnancy. Progesterone


causes stimulation of the respiratory center, which can lead to respiratory
alkalosis. Chronic respiratory alkalosis is a common finding in pregnant
women.
19. Question

A 21-year old client, 6 weeks pregnant is diagnosed with hyperemesis


gravidarum. This excessive vomiting during pregnancy will often result in
which of the following conditions?

A. Bowel perforation

B. Electrolyte imbalance

C. Miscarriage

D. Pregnancy induced hypertension (PIH)

Answer: B. Electrolyte imbalance.

Excessive vomiting in clients with hyperemesis gravidarum often causes


weight loss and fluid, electrolyte, and acid-base imbalances. Severe
hyperemesis requiring hospital admission occurs in 0.3-2% of pregnancies.

Option A: Gastrointestinal perforation (GP) occurs when a hole forms all


the way through the stomach, large bowel, or small intestine. It can be due to
a number of different diseases, including appendicitis and diverticulitis. It
can also be the result of trauma, such as a knife wound or gunshot wound.

Option C: Most miscarriages occur because the fetus isn’t developing


normally. About 50 percent of miscarriages are associated with extra or
missing chromosomes. Most often, chromosome problems result from errors
that occur by chance as the embryo divides and grows — not problems
inherited from the parents.

Option D: Gestational hypertension refers to hypertension with onset in the


latter part of pregnancy (>20 weeks’ gestation) without any other features of
preeclampsia, and followed by normalization of the blood pressure

368
C.BY: HOSSAM HAMDY MATERNITY RNPEDIA – ‫موسوعه التمريض‬

postpartum. Of women who initially present with apparent gestational


hypertension, about one third develops the syndrome of preeclampsia.
20. Question

Clients with gestational diabetes are usually managed by which of the


following therapies?

A. Diet

B. NPH insulin (long-acting)

C. Oral hypoglycemic drugs

D. Oral hypoglycemic drugs and insulin

Answer: A. Diet.

Clients with gestational diabetes are usually managed by diet alone to


control their glucose intolerance. The initial treatment in these patients with
Type 2 diabetes and gestational diabetes is by diet modification and in the
event of failure of control with diet, the treatment is shifted to insulin to
achieve tight glucose control with no risk of placental transfer.

Option B: Insulin therapy remains the standard of care for type 1 diabetes,
type 2 diabetes, and uncontrolled GDM during pregnancy. Regular insulin,
insulin aspart, insulin lispro, and NPH have the most human pregnancy data.

Option C: There are significant concerns about recommending oral


hypoglycemic agents in pregnancy because of their possible risks of
transplacental passage and consequent fetal teratogenesis, hypoglycemia,
hyperbilirubinemia, and polycythemia.

Option D: The pathogenesis of both gestational diabetes and Type 2


diabetes are insulin resistance and inadequate insulin secretion, and hence
the beneficial role of sulfonylurea is evident.
21. Question

The antagonist for magnesium sulfate should be readily available to any


client receiving IV magnesium. Which of the following drugs is the antidote
for magnesium toxicity?

A. Calcium gluconate
369
C.BY: HOSSAM HAMDY MATERNITY RNPEDIA – ‫موسوعه التمريض‬

B. Hydralazine (Apresoline)

C. Narcan

D. RhoGAM

Answer: A. Calcium gluconate.

Calcium gluconate is the antidote for magnesium toxicity. Ten ml of 10%


calcium gluconate is given IV push over 3-5 minutes. Hydralazine is given
for sustained elevated blood pressures in preeclamptic clients.

Option B: Hydralazine is used to treat high blood pressure. Hydralazine is in


a class of medications called vasodilators. It works by relaxing the blood
vessels so that blood can flow more easily through the body.

Option C: Narcan (naloxone) is an opioid antagonist used for the complete


or partial reversal of opioid overdose, including respiratory depression.
Narcan is also used for diagnosis of suspected or known acute opioid
overdose and also for blood pressure support in septic shock. Narcan is
available in generic form.

Option D: RhoGAM is a prescription medicine that is used to prevent Rh


immunization, a condition in which an individual with Rh-negative blood
develops antibodies after exposure to Rh-positive blood.
22. Question

Which of the following answers best describes the stage of pregnancy in


which maternal and fetal blood are exchanged?

A. Conception

B. 9 weeks’ gestation, when the fetal heart is well developed.

C. 32-34 weeks gestation

D. Maternal and fetal blood are never exchanged.

Answer: D. Maternal and fetal blood are never exchanged.

370
C.BY: HOSSAM HAMDY MATERNITY RNPEDIA – ‫موسوعه التمريض‬

Only nutrients and waste products are transferred across the placenta. Blood
exchange only occurs in complications and some medical procedures
accidentally. The fetal circulation system is distinctly different from adult
circulation. This intricate system allows the fetus to receive oxygenated
blood and nutrients from the placenta. It comprises the blood vessels in the
placenta and the umbilical cord, which contains two umbilical arteries and
one umbilical vein.

Option A: The placenta connects the fetus to the wall of the uterus. It
provides oxygen and nutrients from the mother to the growing fetus and also
removes metabolic wastes and carbon dioxide from the fetus via the blood
vessels in the umbilical cord. The umbilical cord develops from the placenta
and is attached to the fetus.

Option B: Oxygenated blood from the mother in the placenta flows through
the umbilical vein and into the inferior vena cava (IVC), bypassing the liver
via the ductus venosus. From the IVC, oxygenated blood travels to the right
atrium of the heart. There is greater pressure in the right atrium compared to
the left atrium in fetal circulation; therefore most of the blood is shunted
from the right atrium to the left atrium through an opening called the
foramen ovale. Once in the left atrium, blood travels through the left
ventricle into the aorta and the systemic circulation.

Option C: The deoxygenated blood travels back to the placenta via the
umbilical arteries to be oxygenated by the mother. Additionally, some
oxygenated blood in the right atrium can also enter the right ventricle and
then the pulmonary artery. Because there is high resistance to blood flow in
the lungs, the blood is shunted from the pulmonary artery into the aorta via
the ductus arteriosus, hence bypassing the lungs. Blood then enters the
systemic circulation, and the deoxygenated blood is recycled back to the
mother via the umbilical arteries.
23. Question

Gravida refers to which of the following descriptions?

A. A serious pregnancy.

B. Number of times a female has been pregnant.

C. Number of children a female has delivered.

D. Number of term pregnancies a female has had.

371
C.BY: HOSSAM HAMDY MATERNITY RNPEDIA – ‫موسوعه التمريض‬

Answer: B. Number of times a female has been pregnant.

Gravida refers to the number of times a female has been pregnant, regardless
of pregnancy outcome or the number of neonates delivered.

Option A: The term gravida comes from the Latin word gravidus. It is used
to describe a woman who is pregnant and is also a medical term for the total
number of confirmed pregnancies a woman has had, regardless of the
outcome of the pregnancy.

Option C: Parity is defined as the number of times that she has given birth
to a fetus with a gestational age of 24 weeks or more, regardless of whether
the child was born alive or was stillborn.

Option D: ACOG and SMFM use these definitions to describe term


pregnancies: Early term: The baby is born between 37 weeks, 0 days and 38
weeks, 6 days. Full term: The baby is born between 39 weeks, 0 days and 40
weeks, 6 days. Late-term: The baby is born between 41 weeks, 0 days and 41
weeks, 6 days.
24. Question

A pregnant woman at 32 weeks’ gestation complains of feeling dizzy and


lightheaded while her fundal height is being measured. Her skin is pale and
moist. The nurse’s initial response would be to:

A. Assess the woman’s blood pressure and pulse.

B. Have the woman breathe into a paper bag.

C. Raise the woman’s legs.

D. Turn the woman on her left side.

Answer: D. Turn the woman on her left side.

During a fundal height measurement, the woman is placed in a supine


position. This woman is experiencing supine hypotension as a result of
uterine compression of the vena cava and abdominal aorta. Turning her on
her side will remove the compression and restore cardiac output and blood
pressure.

Option A: After turning the client on the side, then vital signs can be
assessed. Because this phenomenon is so well known, standard practice is
372
C.BY: HOSSAM HAMDY MATERNITY RNPEDIA – ‫موسوعه التمريض‬

for clinicians to avoid placing the woman in supine position for routine
examinations and procedures, moving the woman into the left lateral if there
are signs of fetal distress in labor, and also advising her to avoid the supine
position herself, at least during the day.

Option B: Breathing into a paper bag is the solution for dizziness related to
respiratory alkalosis associated with hyperventilation.

Option C: Raising her legs will not solve the problem since pressure will
remain on the major abdominal blood vessels, thereby continuing to impede
cardiac output.
25. Question

A pregnant woman’s last menstrual period began on April 8, 2020, and


ended on April 13. Using Naegele’s rule her estimated date of birth would
be:

A. January 15, 2021

B. January 20, 2021

C. July 1, 2021

D. November 5, 2020

Answer: A. January 15, 2021.

Naegele’s rule requires subtracting 3 months and adding 7 days and 1 year if
appropriate to the first day of a Naegele’s rule requires subtracting 3 months
and adding 7 days and 1 year if appropriate to the first day of a pregnant
woman’s last menstrual period. When this rule was used on April 8, 2020,
the estimated date of birth was January 15, 2021.

Option B: Determining gestational age is one of the most critical aspects of


providing quality prenatal care. Knowing the gestational age allows the
obstetrician to provide care to the mother without compromising maternal or
fetal status. It allows for the timing of management, such as administering
steroids for fetal lung maturity, starting ASA therapy with a history of pre-
eclampsia in previous pregnancies, starting hydroxyprogesterone caproate
(Makena) for previous preterm deliveries.

373
C.BY: HOSSAM HAMDY MATERNITY RNPEDIA – ‫موسوعه التمريض‬

Option C: Naegele’s rule, derived from a German obstetrician, subtracts 3


months and adds 7 days to calculate the estimated due date (EDD).

Option D: It is prudent for the obstetrician to get a detailed menstrual


history, including duration, flow, previous menstrual periods, and hormonal
contraceptives. These factors are used to determine the length of her cycles
and ovulation period.

374
C.BY: HOSSAM HAMDY MATERNITY RNPEDIA – ‫موسوعه التمريض‬

Maternity Nursing NU.8 (Quiz #1: 25 Questions)

1. Question

A nurse is caring for a client in labor who is receiving Pitocin by IV infusion


to stimulate uterine contractions. Which assessment finding would indicate
to the nurse that the infusion needs to be discontinued?

A. Three contractions occurring within a 10-minute period

B. Increased urinary output

C. Adequate resting tone of the uterus palpated between contractions

D. A fetal heart rate of 90 beats per minute

Answer: D. A fetal heart rate of 90 beats per minute

A normal fetal heart rate is 120-160 BPM. Bradycardia or late or variable


decelerations indicate fetal distress and the need to discontinue Pitocin. The
goal of labor augmentation is to achieve three good-quality contractions in a
10-minute period.

Option A: Pitocin (oxytocin injection) is a natural hormone that causes the


uterus to contract used to induce labor, strengthen labor contractions during
childbirth, control bleeding after childbirth, or induce an abortion.

Option B: Oxytocin has an antidiuretic effect and increases the urinary


excretion of AQP2 in humans whose urinary concentration mechanism is
preserved. Urine volume and free water clearance were decreased, and urine
osmolality was increased by the administration of oxytocin or dDAVP in the
normal volunteers and CDI patients.

Option C: In a normal labor, one contraction every two to three minutes or


less than five contractions in a 10 minute period is ideal. A uterus must rest
between contractions, having sufficient uterine resting tone (soft to the
touch), and uterine resting time (about one minute).
2. Question

375
C.BY: HOSSAM HAMDY MATERNITY RNPEDIA – ‫موسوعه التمريض‬

A nurse is beginning to care for a client in labor. The physician has


prescribed an IV infusion of Pitocin. The nurse ensures that which of the
following is implemented before initiating the infusion?

A. Placing the client on complete bed rest

B. Continuous electronic fetal monitoring

C. An IV infusion of antibiotics

D. Placing a code cart at the client’s bedside

Answer: B. Continuous electronic fetal monitoring

Continuous electronic fetal monitoring should be implemented during an IV


infusion of Pitocin. Continuous electronic fetal monitoring should be
performed for a minimum of 20 minutes before starting oxytocin and should
be continued until the baby is delivered.

Option A: Complete bed rest is not a necessity before initiating Pitocin


infusion. Pitocin is indicated for the initiation or improvement of uterine
contractions, where this is desirable and considered suitable for reasons of
fetal or maternal concern, in order to achieve vaginal delivery.

Option C: It is unnecessary to administer IV antibiotics before Pitocin


infusion. Immediately administer broad-spectrum antibiotics only to patients
with severe postabortion infection.

Option D: A code cart may be placed beside the client but in case of
overdose, contact the Poison Control Center.
3. Question

A nurse is monitoring a client in active labor and notes that the client is
having contractions every 3 minutes that last 45 seconds. The nurse notes
that the fetal heart rate between contractions is 100 BPM. Which of the
following nursing actions is most appropriate?

A. Encourage the client’s coach to continue to encourage breathing


exercises.

B. Encourage the client to continue pushing with each contraction.

C. Continue monitoring the fetal heart rate.


376
C.BY: HOSSAM HAMDY MATERNITY RNPEDIA – ‫موسوعه التمريض‬

D. Notify the physician or nurse-midwife.

Answer: D. Notify the physician or nurse-midwife.

A normal fetal heart rate is 120-160 beats per minute. Fetal bradycardia
between contractions may indicate the need for immediate medical
management, and the physician or nurse-midwife needs to be notified.

Option A: Steps can be taken to help the fetus get more oxygen, such as
having the mother change position. If these procedures do not work, or if
further test results suggest the fetus has a problem, the ob-gyn or other health
care professional may decide to deliver right away.

Option B: Uterine contractions also may be monitored with a special tube


called an intrauterine pressure catheter that is inserted through the vagina
into the uterus. Internal monitoring can be used only after the membranes of
the amniotic sac have ruptured.

Option C: Fetal heart rate monitoring may help detect changes in the normal
heart rate pattern during labor. If certain changes are detected, steps can be
taken to help treat the underlying problem. Fetal heart rate monitoring also
can help prevent treatments that are not needed.
4. Question

A nurse is caring for a client in labor and is monitoring the fetal heart rate
patterns. The nurse notes the presence of episodic accelerations on the
electronic fetal monitor tracing. Which of the following actions is most
appropriate?

A. Document the findings and tell the mother that the monitor indicates fetal
well-being.

B. Take the mother’s vital signs and tell the mother that bed rest is required
to conserve oxygen.

C. Notify the physician or nurse-midwife of the findings.

D. Reposition the mother and check the monitor for changes in the fetal
tracing.

377
C.BY: HOSSAM HAMDY MATERNITY RNPEDIA – ‫موسوعه التمريض‬

Answer: A. Document the findings and tell the mother that the monitor
indicates fetal well-being.

Accelerations are transient increases in the fetal heart rate that often
accompany contractions or are caused by fetal movement. Episodic
accelerations are thought to be a sign of fetal-well being and adequate
oxygen reserve.

Option B: Inform the mother that they are usually associated with fetal
movement, vaginal examinations, uterine contractions, umbilical vein
compression, fetal scalp stimulation or even external acoustic stimulation.
The presence of accelerations is considered a reassuring sign of fetal well-
being.

Option C: Accelerations are the basis for the nonstress test (NST). The
presence of at least two accelerations, each lasting for 15 or more seconds
above baseline and peaking at 15 or more bpm, in a 20-minute period is
considered a reactive NST.

Option D: The FHR is controlled by the autonomic nervous system. The


inhibitory influence on the heart rate is conveyed by the vagus nerve,
whereas excitatory influence is conveyed by the sympathetic nervous
system. Progressive vagal dominance occurs as the fetus approaches term
and, after birth, results in a gradual decrease in the baseline FHR.
Stimulation of the peripheral nerves of the fetus by its own activity (such as
movement) or by uterine contractions causes acceleration of the FHR.
5. Question

A nurse is admitting a pregnant client to the labor room and attaches an


external electronic fetal monitor to the client’s abdomen. After the
attachment of the monitor, the initial nursing assessment is which of the
following?

A. Identifying the types of accelerations

B. Assessing the baseline fetal heart rate

C. Determining the frequency of the contractions

D. Determining the intensity of the contractions

Answer: B. Assessing the baseline fetal heart rate


378
C.BY: HOSSAM HAMDY MATERNITY RNPEDIA – ‫موسوعه التمريض‬

Assessing the baseline fetal heart rate is important so that abnormal


variations of the baseline rate will be identified if they occur. Options 1 and
3 are important to assess, but not as the first priority.

Option A: The FHR recordings may be interpreted as reassuring,


nonreassuring or ominous, according to the pattern of the tracing. Reassuring
patterns correlate well with a good fetal outcome, while nonreassuring
patterns do not. Evaluation of fetal well-being using fetal scalp stimulation,
pH measurement, or both, is recommended for use in patients with
nonreassuring patterns.

Option C: Frequency is based on the time between the start of one


contraction and the start of the next contraction. Time the frequency of
contractions by noting the time when one contraction starts and the time
when the next contraction starts.

Option D: The intensity of the contractions can be estimated by touching the


uterus. The relaxed or mildly contracted uterus usually feels about as firm as
a cheek, a moderately contracted uterus feels as firm as the end of the nose,
and a strongly contracted uterus is as firm as the forehead.
6. Question

A nurse is reviewing the record of a client in the labor room and notes that
the nurse-midwife has documented that the fetus is at (-1) station. The nurse
determines that the fetal presenting part is:

A. 1 cm above the ischial spine

B. 1 fingerbreadth below the symphysis pubis

C. 1 inch below the coccyx

D. 1 inch below the iliac crest

Answer: A. 1 cm above the ischial spine

Station is the relationship of the presenting part to an imaginary line drawn


between the ischial spines, is measured in centimeters, and is noted as a
negative number above the line and a positive number below the line. At -1
station, the fetal presenting part is 1 cm above the ischial spines.

379
C.BY: HOSSAM HAMDY MATERNITY RNPEDIA – ‫موسوعه التمريض‬

Option B: The doctor will assign a number from -5 to +5 to describe where


the baby is in relation to the ischial spines. The ischial spines are bony
protrusions located in the narrowest part of the pelvis. During a vaginal
exam, the doctor will feel for the baby’s head. If the head is high and not yet
engaged in the birth canal, it may float away from their fingers.

Option C: When the baby’s head is level with the ischial spines, the fetal
station is zero. Once the baby’s head fills the vaginal opening, just before
birth, the fetal station is +5.

Option D: Usually about two weeks before delivery, the baby will drop into
the birth canal. This is called being “engaged.” At this point, the baby is at
station 0. This drop into the birth canal is called a lightening.
7. Question

A pregnant client is admitted to the labor room. An assessment is performed,


and the nurse notes that the client’s hemoglobin and hematocrit levels are
low, indicating anemia. The nurse determines that the client is at risk for
which of the following?

A. A loud mouth

B. Low self-esteem

C. Hemorrhage

D. Postpartum infections

Answer: D. Postpartum infections

Anemic women have a greater likelihood of cardiac decompensation during


labor, postpartum infection, and poor wound healing. Good nutrition is the
best way to prevent anemia if the woman is pregnant or trying to become
pregnant. Eating foods high in iron content (such as dark green leafy
vegetables, red meat, fortified cereals, eggs, and peanuts) can help ensure
that she maintains the supply of iron her body needs to function properly.
The obstetrician will also prescribe vitamins to ensure that the woman has
enough iron and folic acid. Make sure to get at least 27 mg of iron each day.
If the woman does become anemic during pregnancy, it can usually be
treated by taking iron supplements.

380
C.BY: HOSSAM HAMDY MATERNITY RNPEDIA – ‫موسوعه التمريض‬

Option A: The amount of blood in the body increases by about 20-30


percent, which increases the supply of iron and vitamins that the body needs
to make hemoglobin. Hemoglobin is the protein in red blood cells that
carries oxygen to other cells in the body.

Option B: Mild anemia is normal during pregnancy due to an increase in


blood volume. More severe anemia, however, can put the baby at higher risk
for anemia later in infancy. In addition, if the mother is significantly anemic
during the first two trimesters, she is at greater risk for having a preterm
delivery or low-birth-weight baby. Being anemic also burdens the mother by
increasing the risk of blood loss during labor and making it more difficult to
fight infections.

Option C: Anemia does not specifically present a risk for hemorrhage.


Severe anemia may weaken uterine muscular strength or lower resistance to
infectious diseases, contributing to postpartum hemorrhage and subsequent
maternal mortality. However, the severity of anemia that places a woman at
a greater risk of experiencing postpartum hemorrhage or a debilitating and
clinically relevant blood loss has not been investigated. Indeed, the impact of
anemia on the extent of blood loss at childbirth and postpartum is not well-
understood.
8. Question

A nurse assists in the vaginal delivery of a newborn infant. After the


delivery, the nurse observes the umbilical cord lengthen and a spurt of blood
from the vagina. The nurse documents these observations as signs of:

A. Hematoma

B. Placenta previa

C. Uterine atony

D. Placental separation

Answer: D. Placental separation

As the placenta separates, it settles downward into the lower uterine


segment. The umbilical cord lengthens, and a sudden trickle or spurt of
blood appears. Delivery of the placenta usually happens within 5-10 minutes
after delivery of the fetus, but it is considered normal up to 30 minutes after
delivery of the fetus.
381
C.BY: HOSSAM HAMDY MATERNITY RNPEDIA – ‫موسوعه التمريض‬

Option A: A hematoma is a bad bruise. It happens when an injury causes


blood to collect and pool under the skin. The pooling blood gives the skin a
spongy, rubbery, lumpy feel. A hematoma usually is not a cause for concern.
It is not the same thing as a blood clot in a vein, and it does not cause blood
clots.

Option B: Placenta previa occurs when a baby’s placenta partially or totally


covers the mother’s cervix — the outlet for the uterus. Placenta previa can
cause severe bleeding during pregnancy and delivery. If the woman has
placenta previa, she might bleed throughout her pregnancy and during her
delivery.

Option C: Atony of the uterus, also called uterine atony, is a serious


condition that can occur after childbirth. It occurs when the uterus fails to
contract after the delivery of the baby, and it can lead to a potentially life-
threatening condition known as postpartum hemorrhage.
9. Question

A client arrives at a birthing center in active labor. Her membranes are still
intact, and the nurse-midwife prepares to perform an amniotomy. A nurse
who is assisting the nurse-midwife explains to the client that after this
procedure, she will most likely have:

A. Less pressure on her cervix.

B. Increased efficiency of contractions.

C. Decreased number of contractions.

D. The need for increased maternal blood pressure monitoring.

Answer: B. Increased efficiency of contractions

Amniotomy can be used to induce labor when the condition of the cervix is
favorable (ripe) or to augment labor if the process begins to slow. Rupturing
of membranes allows the fetal head to contact the cervix more directly and
may increase the efficiency of contractions.

Option A: Amniotomy is usually performed for the purpose of inducing or


expediting labor or in anticipation of the placement of internal monitors
(uterine pressure catheters or fetal scalp electrodes). It is typically done at
the bedside in the labor and delivery suite.
382
C.BY: HOSSAM HAMDY MATERNITY RNPEDIA – ‫موسوعه التمريض‬

Option C: It is commonly felt that relieving the amniotic sac of amniotic


fluid induces uterine contraction activity, increases the strength of
contractions, and may augment labor by allowing direct pressure from the
fetal scalp on the uterine cervix which may assist in dilating the cervix.

Option D: The nurse has a very important rule in the assessment and
continuous monitoring of pregnant women in labor. The nurse should be
very vigilant and report any untoward change in the hemodynamic status of
the pregnant woman to the clinician at all times.
10. Question

A nurse is monitoring a client in labor. The nurse suspects umbilical cord


compression if which of the following is noted on the external monitor
tracing during a contraction?

A. Early decelerations

B. Variable decelerations

C. Late decelerations

D. Short-term variability

Answer: B. Variable decelerations

Variable decelerations occur if the umbilical cord becomes compressed, thus


reducing blood flow between the placenta and the fetus. Pressure on the cord
initially occludes the umbilical vein, which results in an acceleration (the
shoulder of the deceleration) and indicates a healthy response. This is
followed by occlusion of the umbilical artery, which results in a sharp
downslope. Finally, the recovery phase is due to the relief of the
compression and the sharp return to the baseline, which may be followed by
another healthy brief acceleration or shoulder

Option A: Early decelerations result from pressure on the fetal head during a
contraction. This type of deceleration has a uniform shape, with a slow onset
that coincides with the start of the contraction and a slow return to the
baseline that coincides with the end of the contraction. Thus, it has the
characteristic mirror image of the contraction

Option C: Late decelerations are an ominous pattern in labor because it


suggests uteroplacental insufficiency during a contraction. A late
383
C.BY: HOSSAM HAMDY MATERNITY RNPEDIA – ‫موسوعه التمريض‬

deceleration is a symmetric fall in the fetal heart rate, beginning at or after


the peak of the uterine contraction and returning to baseline only after the
contraction has ended. The descent and return are gradual and smooth.
Regardless of the depth of the deceleration, all late decelerations are
considered potentially ominous.

Option D: Short-term variability refers to the beat-to-beat range in the fetal


heart rate. The FHR is under constant variation from the baseline. This
variability reflects a healthy nervous system, chemoreceptors, baroreceptors,
and cardiac responsiveness. Prematurity decreases variability; therefore,
there is little rate fluctuation before 28 weeks. Variability should be normal
after 32 weeks.
11. Question

A nurse explains the purpose of effleurage to a client in early labor. The


nurse tells the client that effleurage is:

A. A form of biofeedback to enhance bearing down efforts during delivery.

B. Light stroking of the abdomen to facilitate relaxation during labor and


provide tactile stimulation to the fetus.

C. The application of pressure to the sacrum to relieve a backache.

D. Performed to stimulate uterine activity by contracting a specific muscle


group while other parts of the body rest.

Answer: B. Light stroking of the abdomen to facilitate relaxation


during labor and provide tactile stimulation to the fetus.

Effleurage is a specific type of cutaneous stimulation involving light stroking


of the abdomen and is used before a transition to promote relaxation and
relieve mild to moderate pain. Effleurage provides tactile stimulation to the
fetus.

Option A: Women using biofeedback during childbirth reported


significantly lower pain: from admission to labor and delivery, at delivery,
and 24-hr postpartum. Also, women in the biofeedback group labored an
average of 2 hr less and used 30% fewer medications. The results of a study
suggest that EMG biofeedback may be effective in reducing levels of acute
pain experienced by childbearing women.

384
C.BY: HOSSAM HAMDY MATERNITY RNPEDIA – ‫موسوعه التمريض‬

Option C: Low back pain in pregnancy is generally ascribed to the many


changes in load and body mechanics that occur during the carrying of a
child. It is normal to gain between 20 and 40 pounds during pregnancy. This
clearly shifts the body’s center of gravity anteriorly and increases the
moment arm of forces applied to the lumbar spine.

Option D: The primary hormones involved include estrogen, progesterone,


and oxytocin. Oxytocin is one of the most widely studied hormones involved
in uterine contractions. It decreases Ca2+ efflux, by inhibiting the
Ca2+/ATPase of the myometrial cell membrane which pumps calcium from
the inside to the extracellular space, and increases Ca2+ influx, as well as
causes the release of Ca2+ from the SR via IICR.
12. Question

A nurse is caring for a client in the second stage of labor. The client is
experiencing uterine contractions every 2 minutes and cries out in pain with
each contraction. The nurse recognizes this behavior as:

A. Exhaustion

B. Valsalva’s maneuver

C. Involuntary grunting

D. Fear of losing control

Answer: D. Fear of losing control

Pains, helplessness, panicking, and fear of losing control are possible


behaviors in the 2nd stage of labor. In women who have delivered vaginally
previously, whose bodies have acclimated to delivering a fetus, the second
stage may only require a brief trial, whereas a longer duration may be
required for a nulliparous female.

Option A: Labour as a life event is characterized by tremendous


physiological and psychological changes that require major behavioral
adjustments in a short period of time.

Option B: Exercise involving the Valsalva maneuver (holding one’s breath


during exertion) because it can cause increased intra-abdominal pressure.

385
C.BY: HOSSAM HAMDY MATERNITY RNPEDIA – ‫موسوعه التمريض‬

Option C: Labour presents a physical and psychological challenge for


women. The latter stages of pregnancy can be a difficult time emotionally.
Fear and apprehension are experienced alongside excitement. There are
emotions both positive and negative that will affect the woman’s birth
experience.
13. Question

A nurse is monitoring a client in labor who is receiving Pitocin and notes


that the client is experiencing hypertonic uterine contractions. List in order
of priority the actions that the nurse takes.

View Answers:

Reposition the client.

Administer oxygen by face mask at 8 to 10 L/min

Perform a vaginal examination

Stop Pitocin infusion.

Check the client’s blood pressure and heart rate

The order is shown above.

If uterine hypertonicity occurs, the nurse immediately will intervene to


reduce uterine activity and increase fetal oxygenation. The nurse would stop
the Pitocin infusion and increase the rate of the non-additive solution, check
maternal BP for hyper or hypotension, position the woman in a side-lying
position, and administer oxygen by snug face mask at 8-10 L/min. The nurse
then would attempt to determine the cause of the uterine hypertonicity and
perform a vaginal exam to check for a prolapsed cord.

Oxytocin is a natural hormone, which causes the uterus (womb) to have


regular, painful contractions and labor to start. It is available as an
intravenous (into a vein (IV)) drug and infused slowly to artificially
stimulate labor if doctors or midwives feel that it is necessary to accelerate
the birth of the baby, or if the mother requests it.

386
C.BY: HOSSAM HAMDY MATERNITY RNPEDIA – ‫موسوعه التمريض‬

Risks associated with using IV oxytocin to stimulate uterine contractions


include the woman having contractions that are too long or too frequent
(uterine hyperstimulation), which can lead to changes in the baby’s heart rate
and the need for an emergency cesarean.

Stopping oxytocin infusion once active labor has started could result in more
natural childbirth, particularly if the risk of uterine overstimulation and the
need for immediate cesarean section is reduced.

Also, the overall total dose of oxytocin the mother received would be
reduced, which could lead to fewer adverse effects (e.g. maternal nausea,
vomiting, and headache, or changes to the baby’s heart rate).

Discontinuing IV oxytocin probably reduces the risk of women having


contractions that become too long or too strong resulting in changes to the
baby’s heart rate (three trials, 486 women).

14. Question

A nurse is assigned to care for a client with hypotonic uterine dysfunction


and signs of slowing labor. The nurse is reviewing the physician’s orders and
would expect to note which of the following prescribed treatments for this
condition?

A. Medication that will provide sedation

B. Increased hydration

C. Oxytocin (Pitocin) infusion

D. Administration of a tocolytic medication

Answer: C. Oxytocin (Pitocin) infusion.

Therapeutic management for hypotonic uterine dysfunction includes


oxytocin augmentation and amniotomy to stimulate labor that slows.
Hypotonic labor is an abnormal labor pattern, notable especially during the
active phase of labor, characterized by poor and inadequate uterine
contractions that are ineffective to cause cervical dilation, effacement, and
fetal descent, leading to a prolonged or protracted delivery.

Option A: Provided there are no contraindications. Oxytocin is the


medication of choice for augmenting contractions. The dosage regimen
387
C.BY: HOSSAM HAMDY MATERNITY RNPEDIA – ‫موسوعه التمريض‬

should be titrated to effect for achieving adequate uterine contractions.


However, dosing generally does not exceed 30milliunit/ minute. The usual
protocol is 5 units of oxytocin in 500mls of 5% Dextrose intravenous
infusion, starting with 10 drops/min and gradually titrating the rate to
achieve a contraction rate of at least 3 per minute.

Option B: Maintain adequate hydration. Encourage ambulation and avoid


supine position. Although these are not proven to improve contractions or
prolonged labor due to hypocontractility, they may improve the comfort of
the patient.

Option D: A combination of amniotomy and oxytocin augmentation is more


effective in the management of hypocontractile labor than amniotomy alone
when instituted early in the active phase.

15. Question

A nurse in the labor room is preparing to care for a client with hypertonic
uterine dysfunction. The nurse is told that the client is experiencing
uncoordinated contractions that are erratic in their frequency, duration, and
intensity. The priority nursing intervention would be to:

A. Monitor the Pitocin infusion closely

B. Provide pain relief measures

C. Prepare the client for an amniotomy

D. Promote ambulation every 30 minutes

Answer: B. Provide pain relief measures

Management of hypertonic labor depends on the cause. Relief of pain is the


primary intervention to promote a normal labor pattern. The psychological
preparation of patients before labor appears to improve pain tolerance during
labor. This should begin during routine antenatal visits and the counseling
for labor analgesia. This preparation may serve to reduce the need for
neuraxial analgesia in labor, which is a probable predisposing factor for
hypocontractile labor.

Option A: Provided there are no contraindications, oxytocin is the


medication of choice for augmenting contractions. The dosage regimen

388
C.BY: HOSSAM HAMDY MATERNITY RNPEDIA – ‫موسوعه التمريض‬

should be titrated to effect for achieving adequate uterine contractions.


However, dosing generally does not exceed 30milliunit/ minute.

Option C: Membrane rupture (amniotomy) stimulates contractions by the


release of prostaglandins and reflex stimulation of the uterus when the
presenting part becomes closely applied to the lower uterine segment.
Amniotomy should be attempted when vaginal delivery is probable; where
cervical dilatation > 4 cm, there is adequate fetal descent (station -2 or
lower), and the presenting part is well-applied to the lower uterine segment.

Option D: Encourage ambulation and avoid supine position. Although these


are not proven to improve contractions or prolonged labor due to
hypocontractility, they may improve the comfort of the patient.

16. Question

A nurse is developing a plan of care for a client experiencing dystocia and


includes several nursing interventions in the plan of care. The nurse
prioritizes the plan of care and selects which of the following nursing
interventions as the highest priority?

A. Keeping the significant other informed of the progress of the labor.

B. Providing comfort measures.

C. Monitoring fetal heart rate.

D. Changing the client’s position frequently.

Answer: C. Monitoring fetal heart rate.

The priority is to monitor the fetal heart rate. The continuous monitoring of
the external fetal heart rate provides insight into fetal well-being. The
assessment of the fetal heart rate could be performed utilizing external or
internal fetal heart rate monitoring. An alternative is fetal heart rate
auscultation every 15 minutes in the first stage of labor and after each
contraction during the second stage of labor. In the interpretation of the fetal
heart rate strip millimeters considered are baseline viability, basal heart rate,
cardiac accelerations or decelerations, endocrine activity. Strip abnormalities
are characterized based on consideration of the above parameters.

Option A: At admission to labor and delivery, prenatal records and obstetric


history should be reviewed because these optimally inform the provider to
389
C.BY: HOSSAM HAMDY MATERNITY RNPEDIA – ‫موسوعه التمريض‬

the best intrapartum obstetric care. This care includes the determination of
the static gestational age.

Option B: Most labor and delivery units will have an established protocol
for administration of oxytocin that entails the administration of the proper
medication and dosage, as well as criteria for an incremental increase as
clinically warranted. The protocols also include monitoring maternal and
fetal vital signs, as well as the atria, for discontinuation of the medication in
the event of concern for tachycardia systole all fetal well-being.

Option D: The uterine activity is assessed by external tocometry and


targeted at 3 to 5 contractions in the 10-minute window. The contractions
should last 30 to 40 seconds to be effective. Internal intrauterine pressure
assessment using a catheter could be utilized, in which case marked medial
units are used and targeted at more than 200 Montevideo units in a 10-
minute window. The monitoring of uterine contractions should be continuous
during labor.

17. Question

A maternity nurse is preparing to care for a pregnant client in labor who will
be delivering twins. The nurse monitors the fetal heart rates by placing the
external fetal monitor:

A. Over the fetus that is most anterior to the mother’s abdomen.

B. Over the fetus that is most posterior to the mother’s abdomen.

C. So that each fetal heart rate is monitored separately.

D. So that one fetus is monitored for a 15-minute period followed by a 15


minute fetal monitoring period for the second fetus.

Answer: C. So that each fetal heart rate is monitored separately.

In a client with a multifetal pregnancy, each fetal heart rate is monitored


separately. Simultaneous monitoring of twins is preferable to non
simultaneous monitoring to discriminate between their separate FHRs
(ACOG, 1989). Synchronizing the internal clocks of both monitors will help
produce accurate documentation. Otherwise, time increments should be
documented on both monitor tracings for later comparison, to ensure that
each twin has been monitored. If the monitor strips are synchronous,

390
C.BY: HOSSAM HAMDY MATERNITY RNPEDIA – ‫موسوعه التمريض‬

portable real-time ultrasound can be used to verify that both twins are being
monitored independently

Option A: Among the advantages of simultaneous twin monitoring is the


increased likelihood that both twins are being monitored with potentially less
nursing time.

Option B: The nonstress test (NST) is the most widely used method of
evaluating twins for any of the aforementioned risk factors. Normative data
for simultaneous twin NSTs show synchrony or similarity in the tracings
with incidences of 57.14% and 58% in twins monitored from 27?weeks until
term.

Option D: Synchrony is thought by some to occur because the first twin’s


movement produces a vibration and stimulates movement and FHR
accelerations in the second twin. Recently, this idea was supported in a
limited investigation of twins in which vibratory acoustic stimulation evoked
an immediate transition from asynchronous to synchronous FHR tracings in
all 16 tests carried out in the study (Sherer, Abramowicz, D’Amico, Caverly,
& Woods, 1991).

18. Question

A nurse in the postpartum unit is caring for a client who has just delivered a
newborn infant following a pregnancy with placenta previa. The nurse
reviews the plan of care and prepares to monitor the client for which of the
following risks associated with placenta previa?

A. Disseminated intravascular coagulation

B. Chronic hypertension

C. Infection

D. Hemorrhage

Answer: D. Hemorrhage

Because the placenta is implanted in the lower uterine segment, which does
not contain the same intertwining musculature as the fundus of the uterus,
this site is more prone to bleeding. Vaginal bleeding secondary to placenta
previa can lead to postpartum hemorrhage requiring a blood transfusion,

391
C.BY: HOSSAM HAMDY MATERNITY RNPEDIA – ‫موسوعه التمريض‬

hysterectomy, maternal intensive care admission, septicemia, and maternal


death.

Option A: Out of 4,334 obstetrical admissions, DIC was diagnosed in 40


(0.92%) patients. Risk factors noted were eclampsia 28 (70%), abruptio
placentae 7 (17.5%), septicaemia 3 (7.5%), pancytopenia 1 (2.5%), and 1
(2.5%) patient had DIC secondary to hemorrhagic shock due to placenta
previa.

Option B: Women with chronic hypertension had a relative risk of 1.2 (95%
confidence interval 0.4 to 3.7) for placenta previa compared with
normotensive women. However, the risk of pregnancy-induced hypertension
was reduced by half among those with placenta previa (relative risk 0.5, 95%
confidence interval 0.3 to 0.7).

Option C: Patients with placenta previa presenting with vaginal bleeding


have intra-amniotic infection in 5.7% of the cases, and IAI in 17.9%. IAI in
patients with placenta previa and vaginal bleeding is a risk factor for preterm
delivery within 48 h.

19. Question

A nurse in the delivery room is assisting with the delivery of a newborn


infant. After the delivery of the newborn, the nurse assists in delivering the
placenta. Which observation would indicate that the placenta has separated
from the uterine wall and is ready for delivery?

A. The umbilical cord shortens in length and changes in color

B. A soft and boggy uterus

C. Maternal complaints of severe uterine cramping

D. Changes in the shape of the uterus

Answer: D. Changes in the shape of the uterus

Signs of placental separation include lengthening of the umbilical cord, a


sudden gush of dark blood from the introitus (vaginal), a firmly contracted
uterus, and the uterus changing from a discoid (like a disk) to a globular (like
a globe) shape. The client may experience vaginal fullness, but not severe
uterine cramping.

392
C.BY: HOSSAM HAMDY MATERNITY RNPEDIA – ‫موسوعه التمريض‬

Option A: The most reliable sign is the lengthening of the umbilical cord as
the placenta separates and is pushed into the lower uterine segment by
progressive uterine retraction. Placing a clamp on the cord near the perineum
makes it easier to appreciate this lengthening. Never place traction on the
cord without countertraction on the uterus above the symphysis; otherwise,
one may mistake cord lengthening due to impending prolapse or inversion
for that of uncomplicated placental separation.

Option B: The uterus takes on a more globular shape and becomes firmer.
This occurs as the placenta descends into the lower segment and the body of
the uterus continues to retract. This change may be clinically difficult to
appreciate.

Option C: Following the delivery of the fetus, uterine contractions continue


and the placenta is sheared from the underlying endometrium. This
separation primarily occurs by a reduction in the surface area of the placental
site as the uterus shrinks.

20. Question

A nurse in the labor room is performing a vaginal assessment on a pregnant


client in labor. The nurse notes the presence of the umbilical cord protruding
from the vagina. Which of the following would be the initial nursing action?

A. Place the client in Trendelenburg’s position.

B. Call the delivery room to notify the staff that the client will be transported
immediately.

C. Gently push the cord into the vagina.

D. Find the closest telephone and stat page the physician.

Answer: A. Place the client in Trendelenburg’s position.

When cord prolapse occurs, prompt actions are taken to relieve cord
compression and increase fetal oxygenation. The mother should be
positioned with the hips higher than the head to shift the fetal presenting part
toward the diaphragm. Oxygen at 8 to 10 L/min by face mask is delivered to
the mother to increase fetal oxygenation.

Option B: The definitive management of umbilical cord prolapse is


expedient delivery; this is usually by cesarean section. In rare cases, vaginal
393
C.BY: HOSSAM HAMDY MATERNITY RNPEDIA – ‫موسوعه التمريض‬

delivery or operative vaginal delivery may be faster and, thus, preferable, but
this should only occur under the presence and guidance of an experienced
obstetrician.

Option C: No attempt should be made to replace the cord. The examiner,


however, may place a gloved hand into the vagina and hold the presenting
part off of the umbilical cord. Decompression should be done manually by
the medical provider through the placement of their finger or hand in the
vaginal vault and gentle elevation of the presenting part off the umbilical
cord. The provider should be conscientious not to place any additional
pressure on the cord, as this can cause vasospasm and worsen outcomes.

Option D: The nurse should push the call light to summon help, and other
staff members should call the physician and notify the delivery room. If the
cord is visibly protruding from the introitus, it should remain warm and
moist because the ambient temperature is significantly colder than the
temperature in the uterus and can result in vasospasm of the umbilical
arteries, contributing to fetal hypoxia. One method described preventing this
is the replacement of the cord into the vaginal vault followed by insertion of
a moist tampon to keep it in place.

21. Question

A maternity nurse is caring for a client with abruptio placenta and is


monitoring the client for disseminated intravascular coagulopathy. Which
assessment finding is least likely to be associated with disseminated
intravascular coagulation?

A. Swelling of the calf in one leg

B. Prolonged clotting times

C. Decreased platelet count

D. Petechiae, oozing from injection sites, and hematuria

Answer: A. Swelling of the calf in one leg

DIC is a state of diffuse clotting in which clotting factors are consumed,


leading to widespread bleeding. Swelling and pain in the calf of one leg are
more likely to be associated with thrombophlebitis.

394
C.BY: HOSSAM HAMDY MATERNITY RNPEDIA – ‫موسوعه التمريض‬

Option B: Fibrin plugs may clog the microvasculature diffusely, rather than
in an isolated area. Derangement of the fibrinolytic system further
contributes to intravascular clot formation, but in some cases, accelerated
fibrinolysis may cause severe bleeding. Hence, a patient with DIC can
present with a simultaneously occurring thrombotic and bleeding problem,
which obviously complicates the proper treatment.

Option C: Platelets are decreased because they are consumed by the


process; coagulation studies show no clot formation (and are thus normal to
prolong). Exposure to tissue factor (TF) in the circulation occurs via
endothelial disruption, tissue damage, or inflammatory or tumor cell
expression of procoagulant molecules (including TF). TF activates
coagulation via the extrinsic pathway involving factor VIIa. The TF-VIIa
complex activates thrombin, which cleaves fibrinogen to fibrin while
simultaneously causing platelet aggregation..

Option D: The presence of petechiae, oozing from injection sites, and


hematuria are signs associated with DIC. With acute DIC, the physical
findings are usually those of the underlying or inciting condition; however,
patients with the acute disease (ie, the hemorrhagic variety associated with
excess plasmin formation) have petechiae on the soft palate, trunk, and
extremities from thrombocytopenia and ecchymosis at venipuncture sites.
These patients also manifest ecchymosis in traumatized areas.

22. Question

A nurse is assessing a pregnant client in the 2nd trimester of pregnancy who


was admitted to the maternity unit with a suspected diagnosis of abruptio
placentae. Which of the following assessment findings would the nurse
expect to note if this condition is present?

A. Absence of abdominal pain

B. A soft abdomen

C. Uterine tenderness/pain

D. Painless, bright red vaginal bleeding

Answer: C. Uterine tenderness/pain

In abruptio placentae, acute abdominal pain is present. Uterine tenderness


and pain accompany placental abruption, especially with a central abruption
395
C.BY: HOSSAM HAMDY MATERNITY RNPEDIA – ‫موسوعه التمريض‬

and trapped blood behind the placenta. Observation of the fetal monitoring
often reveals increased uterine resting tone, caused by failure of the uterus to
relax in an attempt to constrict blood vessels and control bleeding.

Option A: Placental abruption occurs when there is a compromise of the


vascular structures supporting the placenta. In other words, the vascular
networks connecting the uterine lining and the maternal side of the placenta
are torn away. These vascular structures deliver oxygen and nutrients to the
fetus.

Option B: The abdomen will feel hard and board like on palpation as the
blood penetrates the myometrium and causes uterine irritability. Disruption
of the vascular network may occur when the vascular structures are
compromised because of hypertension or substance use or by conditions that
cause stretching the uterus. The uterus is a muscle and is elastic whereas the
placenta is less elastic than the uterus. Therefore, when the uterine tissue
stretches suddenly, the placenta remains stable and the vascular structure
connecting the uterine wall to the placenta tears away.

Option D: If bleeding is present, the quantity and characteristic of the blood,


as well as the presence of clots, is evaluated. Remember, the absence of
vaginal bleeding does not eliminate the diagnosis of placental abruption.

23. Question

A maternity nurse is preparing for the admission of a client in the 3rd


trimester of pregnancy that is experiencing vaginal bleeding and has a
suspected diagnosis of placenta previa. The nurse reviews the physician’s
orders and would question which order?

A. Prepare the client for an ultrasound.

B. Obtain equipment for external electronic fetal heart monitoring.

C. Obtain equipment for a manual pelvic examination.

D. Prepare to draw a Hgb and Hct blood sample.

Answer: C. Obtain equipment for a manual pelvic examination.

Manual pelvic examinations are contraindicated when vaginal bleeding is


apparent in the 3rd trimester until a diagnosis is made and placenta previa is

396
C.BY: HOSSAM HAMDY MATERNITY RNPEDIA – ‫موسوعه التمريض‬

ruled out. Digital examination of the cervix can lead to maternal and fetal
hemorrhage.

Option A: A diagnosis of placenta previa is made by ultrasound. A patient


presenting with vaginal bleeding in the second or third trimester should
receive a transabdominal sonogram before a digital examination. If there is a
concern for placenta previa, then a transvaginal sonogram should be
performed to confirm the location of the placenta. Transvaginal sonogram
has been shown to be superior to a transabdominal sonogram and is safe.

Option B: External fetal monitoring is crucial in evaluating the fetus that is


at risk for severe hypoxia. Placental abruption presents with severe
abdominal pain, vaginal bleeding, and electronic fetal monitoring may show
tachysystole and a nonreassuring fetal heart tracing; this too can lead to high
morbidity in mortality to the fetus and mother secondary to hemorrhage.

Option D: The H/H levels are monitored, and external electronic fetal heart
rate monitoring is initiated. A leading cause of third-trimester hemorrhage,
placenta previa presents classically as painless bleeding. Bleeding is thought
to occur in association with the development of the lower uterine segment in
the third trimester. Placental attachment is disrupted as this area gradually
thins in preparation for the onset of labor; this leads to bleeding at the
implantation site, because the uterus is unable to contract adequately and
stop the flow of blood from the open vessels.

24. Question

An ultrasound is performed on a client at term gestation that is experiencing


moderate vaginal bleeding. The results of the ultrasound indicate that an
abruptio placenta is present. Based on these findings, the nurse would
prepare the client for:

A. Complete bed rest for the remainder of the pregnancy.

B. Delivery of the fetus.

C. Strict monitoring of intake and output.

D. The need for weekly monitoring of coagulation studies until the time of
delivery.

Answer: B. Delivery of the fetus.

397
C.BY: HOSSAM HAMDY MATERNITY RNPEDIA – ‫موسوعه التمريض‬

The goal of management in abruptio placentae is to control the hemorrhage


and deliver the fetus as soon as possible. Delivery is the treatment of choice
if the fetus is at term gestation or if the bleeding is moderate to severe and
the mother or fetus is in jeopardy.

Option A: Placental abruption occurs when there is a compromise of the


vascular structures supporting the placenta. In other words, the vascular
networks connecting the uterine lining and the maternal side of the placenta
are torn away. These vascular structures deliver oxygen and nutrients to the
fetus. Disruption of the vascular network may occur when the vascular
structures are compromised because of hypertension or substance use or by
conditions that cause stretching the uterus. The uterus is a muscle and is
elastic whereas the placenta is less elastic than the uterus.

Option C: Evaluation of vital signs to detect tachycardia or hypotension,


which may be indicators of a concealed hemorrhage are taken. Blood
specimens such as a complete blood count (CBC), fibrinogen, clotting
profile, and type and RH may be collected. These laboratory values will not
aid in the diagnosis of placental abruption but will provide baseline data
against which to evaluate the patient’s condition over time.

Option D: Women classified with a class 1 or mild placental abruption and


no signs of maternal or fetal distress and a pregnancy less than 37 weeks
gestation may be managed conservatively. These patients are usually
admitted to the obstetrical unit for close monitoring of maternal and fetus
status. Intravenous access and blood work for type and crossmatch is part of
the plan of care. The maternal-fetal dyad will continue to be monitored until
there is a change in condition or until fetal maturity is reached.

25. Question

A nurse in a labor room is assisting with the vaginal delivery of a newborn


infant. The nurse would monitor the client closely for the risk of uterine
rupture if which of the following occurred?

A. Hypotonic contractions

B. Forceps delivery

C. Schultz delivery

D. Weak bearing down efforts

398
C.BY: HOSSAM HAMDY MATERNITY RNPEDIA – ‫موسوعه التمريض‬

Answer: B. Forceps delivery.

Excessive fundal pressure, forceps delivery, violent bearing down efforts,


tumultuous labor, and shoulder dystocia can place a woman at risk for
traumatic uterine rupture. Hypotonic contractions and weak bearing down
efforts do not alone add to the risk of rupture because they do not add to the
stress on the uterine wall.

Option A: Phelan et al found that abnormal patterns of uterine activity, such


as tetany and hyperstimulation, are often not associated with uterine rupture.
In their study, in which monitoring of uterine activity was limited to external
tocodynamometry, tetany was defined as a contraction lasting longer than 90
seconds, and hyperstimulation was defined as more than 5 contractions in 10
minutes.

Option C: The separation of the placenta from the uterine wall during labor;
it begins at the placental center and leads to an expulsion of the placenta
after delivery of the baby.

Option D: Rodriguez et al found that the usefulness of intrauterine pressure


catheters (IUPCs) for diagnosing uterine rupture was not supported. In 76
cases of uterine rupture, the classic description of decreased uterine tone and
diminished uterine activity was not observed in any patients, 39 of whom
had IUPCs in place. In addition, rates of fetal and maternal morbidity and
mortality associated with uterine rupture did not differ with the use of an
IUPC compared with external tocodynamometry.

399
C.BY: HOSSAM HAMDY MATERNITY RNPEDIA – ‫موسوعه التمريض‬

Maternity Nursing NU.9 (Quiz #1: 25 Questions)

• 1. Question

A client is admitted to the birthing suite in early active labor. The priority
nursing intervention on the admission of this client would be:

• A. Auscultating the fetal heart


• B. Taking an obstetric history
• C. Asking the client when she last ate
• D. Ascertaining whether the membranes were ruptured

Answer: A. Auscultating the fetal heart.

Determining the fetal well-being supersedes all other measures. If the FHR is
absent or persistently decelerating, immediate intervention is required. During
labor, cardiotocographic monitoring is often employed to monitor uterine
contractions and fetal heart rate over time. Clinicians monitor fetal heart
tracings to evaluate for any signs of fetal distress that would warrant
intervention as well as the adequacy or inadequacy of contractions.

• Option B: When women first present to the labor and delivery unit, vital
signs, including temperature, heart rate, oxygen saturation, respiratory
rate, and blood pressure, should be obtained and reviewed for any
abnormalities. The patient should be placed on continuous
cardiotocographic monitoring to ensure fetal wellbeing. The patient’s
prenatal record, including obstetric history, surgical history, medical
history, laboratory, and imaging data, should undergo review. Finally, a
history of present illness, review of systems, and physical exam,
including a sterile speculum exam, will need to take place.
• Option C: Labor is a natural process, but it can suffer interruption by
complicating factors, which at times necessitate clinical intervention. The
management of low-risk labor is a delicate balance between allowing the
natural process to proceed while limiting any potential complications.
• Option D: Cervical exams are usually performed every 2 to 3 hours
unless concerns arise and warrant more frequent exams. Frequent cervical
exams are associated with a higher risk of infection, especially if a
rupture of membranes has occurred. Women should be allowed to
ambulate freely and change positions if desired.

400
C.BY: HOSSAM HAMDY MATERNITY RNPEDIA – ‫موسوعه التمريض‬

• 2. Question

A client who is gravida 1, para 0 is admitted in labor. Her cervix is 100%


effaced, and she is dilated to 3 cm. Her fetus is at +1 station. The nurse is aware
that the fetus’ head is:

• A. Not yet engaged


• B. Entering the pelvic inlet
• C. Below the ischial spines
• D. Visible at the vaginal opening

Answer: C. Below the ischial spines

A station of +1 indicates that the fetal head is 1 cm below the ischial spines.
Positive numbers are used when a baby has descended beyond the ischial
spines. During birth, a baby is at the +4 to +5 station.

• Option A: During a vaginal exam, the doctor will feel for the baby’s
head. If the head is high and not yet engaged in the birth canal, it may
float away from their fingers.
• Option B: When the baby’s head is level with the ischial spines, the fetal
station is zero. Once the baby’s head fills the vaginal opening, just before
birth, the fetal station is +5.
• Option D: Usually about two weeks before delivery, the baby will drop
into the birth canal. This is called being “engaged.” At this point, the
baby is at station 0. This drop into the birth canal is called a lightening.

• 3. Question

After doing Leopold’s maneuvers, the nurse determines that the fetus is in the
ROP position. To best auscultate the fetal heart tones, the Doppler is placed:

• A. Above the umbilicus at the midline


• B. Above the umbilicus on the left side
• C. Below the umbilicus on the right side
• D. Below the umbilicus near the left groin

Answer: C. Below the umbilicus on the right side

401
C.BY: HOSSAM HAMDY MATERNITY RNPEDIA – ‫موسوعه التمريض‬

Fetal heart tones are best auscultated through the fetal back; because the
position is ROP (right occiput presentation), the back would be below the
umbilicus and on the right side.

• Option A: The baby’s heartbeat is loudest in its upper chest or upper


back, depending on which way the baby is facing.
• Option B: If you hear the heartbeat loudest above the mother’s
umbilicus, the baby may be in the breech position
• Option D: If you hear the heartbeat loudest below the mother’s
umbilicus, the baby is probably head down.

• 4. Question

The physician asks the nurse the frequency of a laboring client’s contractions.
The nurse assesses the client’s contractions by timing from the beginning of one
contraction:

• A. Until the time it is completely over


• B. To the end of a second contraction
• C. To the beginning of the next contraction
• D. Until the time that the uterus becomes very firm

Answer: C. To the beginning of the next contraction

This is the way to determine the frequency of the contractions. When timing
contractions, start counting from the beginning of one contraction to the
beginning of the next. The easiest way to time contractions is to write down on
paper the time each contraction starts and its duration, or count the seconds the
actual contraction lasts.

• Option A: Contractions are intermittent, with a valuable rest period for


the mother, the baby, and her uterus following each one. When timing
contractions, start counting from the beginning of one contraction to the
beginning of the next.
• Option B: As the strength of each contraction increases, the peaks will
come sooner and last longer. There should be some regularity or pattern
when timed. Persistent contractions that have no rhythm but are five-to-
seven minutes apart or less should be reported to the physician or
midwife.
• Option D: Contractions are often described as a cramping or tightening
sensation that starts in the back and moves around to the front in a wave-
like manner. Others say the contraction feels like pressure in the back.
During a contraction, the abdomen becomes hard to the touch.
402
C.BY: HOSSAM HAMDY MATERNITY RNPEDIA – ‫موسوعه التمريض‬

• 5. Question

The nurse observes the client’s amniotic fluid and decides that it appears
normal, because it is:

• A. Clear and dark amber in color


• B. Milky, greenish yellow, containing shreds of mucus
• C. Clear, almost colorless, and containing little white specks
• D. Cloudy, greenish-yellow, and containing little white specks

Answer: C. Clear, almost colorless, and containing little white specks.

By 36 weeks gestation, normal amniotic fluid is colorless with small particles of


vernix caseosa present. Amniotic fluid is usually clear to pale yellow in color. It
should be odorless, or slightly sweet in odor—although some say it has a
bleach-like smell.

• Option A: Dark fluid can also be seen with an intrauterine fetal demise
(IUFD) when the fetus has died during pregnancy.
• Option B: In full-term or near-term pregnancies, green or brown fluid
may indicate the baby has had a bowel movement (meconium), which
contributes to the color change. This can be an indication of a baby in
distress or simply that the pregnancy has extended enough for the baby to
pass that first stool in utero.
• Option D: The amniotic fluid may also be blood-tinged, especially
during labor, if the cervix has started dilating, or if there are placental
problems.
• 6. Question

At 38 weeks gestation, a client is having late decelerations. The fetal pulse


oximeter shows 75% to 85%. The nurse should:

• A. Discontinue the catheter if the reading is not above 80%.


• B. Discontinue the catheter if the reading does not go below 30%.
• C. Advance the catheter until the reading is above 90% and continue
monitoring.
• D. Reposition the catheter, recheck the reading, and if it is 55%, keep
monitoring.

403
C.BY: HOSSAM HAMDY MATERNITY RNPEDIA – ‫موسوعه التمريض‬

Answer: D. Reposition the catheter, recheck the reading, and if it is 55%,


keep monitoring.

Adjusting the catheter would be indicated. Normal fetal pulse oximetry should
be between 30% and 70%. 75% to 85% would indicate maternal readings. Fetal
pulse oximetry measures how much oxygen the baby’s blood is carrying. It uses
a probe that sits on the baby’s head whilst in the uterus and vagina during labor.
The probe is said not to interfere with the woman’s mobility during labor.

• Option A: This method has two potential advantages over conventional


fetal heart rate monitoring: (i) it directly measures the proportion of
hemoglobin that is carrying oxygen: thus, oxygenation, the primary
variable underlying the tissue-damaging effects of hypoxia/ischemia is
being monitored; and (ii) it relies on an established, safe, noninvasive,
widely?used technology found in every modern intensive care unit and
operating theatre.
• Option B: A variety of fetal pulse oximetry sensors has been studied.
These are placed during a vaginal examination to attach to the top of the
fetal head by suction (Arikan 2000) or clip (Knitza 2004), lie against the
fetal temple or cheek (Mallinckrodt 2000; Nellcor 2004), or to lie along
the fetal back (Prothia 2014). The sensor remains in situ and fetal pulse
oximetry values are recorded for approximately 81% of the monitoring
time (East 1997).
• Option C: A prospective observational study found a low pulse oximetry
oxygen saturation < 30% for at least 10 minutes correlates highly with
fetal acidosis in cases of nonreassuring fetal heart rate (Nonnenmacher
2010). A novel fetal phantom based on actual fetal parameters showed
that the wireless oximeter was capable of identifying 4% and 2% changes
in diameter between the diastolic and systolic point in arteries of over 0.2
and 0.4 mm inner diameter, respectively (Stubán 2009).
• 7. Question

When examining the fetal monitor strip after the rupture of the membranes in a
laboring client, the nurse notes variable decelerations in the fetal heart rate. The
nurse should:

• A. Stop the oxytocin infusion


• B. Change the client’s position
• C. Prepare for immediate delivery
• D. Take the client’s blood pressure

Answer: B. Change the client’s position.


404
C.BY: HOSSAM HAMDY MATERNITY RNPEDIA – ‫موسوعه التمريض‬

Variable decelerations usually are seen as a result of cord compression; a


change of position will relieve pressure on the cord. Variable decelerations can
be seen resulting from the fetal movement if the fetus is premature. In the term
fetus, variable decelerations result from vagus nerve-mediated parasympathetic
effects on the heart. There are several theories regarding the pathway that leads
to this vagal stimulation.

• Option A: If a patient is having uterine tachysystole, reducing the


number of contractions by decreasing oxytocin or administration of a
beta-agonist may be appropriate.
• Option C: Ultimately, if the fetal heart tracing is persistently abnormal,
facilitating delivery is indicated. In the term, laboring patient, an
operative vaginal delivery may be considered. If the patient is remote
from delivery, it may indicate the need for cesarean delivery. In a patient
with preterm prelabor rupture of membranes, induction or augmentation
of labor may be the next step if the fetus is in the vertex presentation.
Alternatively, cesarean delivery may be indicated if the fetus is in the
breech presentation.
• Option D: Fetal heart rate tracings reflect the response of the fetal central
nervous system to intrauterine hypoxia. Variable decelerations are under
vagal mediation through baroreceptors or chemoreceptors. Possibly,
direct cord compression leads to fetal hypertension, which in turn leads to
baroreceptor response and subsequent vagal-mediated heart rate decrease.
Alternatively, hypoxemia resulting from decreased uteroplacental
perfusion triggers chemoreceptors, which in turn lead to a cascade of
physiologic responses that ultimately result in vagal-mediated heart rate
decrease.
• 8. Question

When monitoring the fetal heart rate of a client in labor, the nurse identifies an
elevation of 15 beats above the baseline rate of 135 beats per minute lasting for
15 seconds. This should be documented as:

• A. An acceleration
• B. An early elevation
• C. A sonographic motion
• D. A tachycardia

Answer: A. An acceleration

An acceleration is an abrupt elevation above the baseline of 15 beats per minute


for 15 seconds; if the acceleration persists for more than 10 minutes it is
405
C.BY: HOSSAM HAMDY MATERNITY RNPEDIA – ‫موسوعه التمريض‬

considered a change in baseline rate. A tachycardic FHR is above 160 beats per
minute.

• Option B: Increased variability in the baseline FHR is present when the


oscillations exceed 25 bpm. This pattern is sometimes called a saltatory
pattern and is usually caused by acute hypoxia or mechanical
compression of the umbilical cord. This pattern is most often seen during
the second stage of labor. The presence of a saltatory pattern, especially
when paired with decelerations, should warn the physician to look for and
try to possible causes of acute hypoxia and to be alert for signs that the
hypoxia is progressing to acidosis. Although it is a nonreassuring pattern,
the saltatory pattern is usually not an indication for immediate delivery.
• Option C: Auscultation of the fetal heart rate (FHR) is performed by
external or internal means. External monitoring is performed using a
hand-held Doppler ultrasound probe to auscultate and count the FHR
during a uterine contraction and for 30 seconds thereafter to identify fetal
response. The transducer uses Doppler ultrasound to detect fetal heart
motion and is connected to an FHR monitor. The monitor calculates and
records the FHR on a continuous strip of paper.
• Option D: Fetal tachycardia is defined as a baseline heart rate greater
than 160 bpm and is considered a nonreassuring pattern. Tachycardia is
considered mild when the heart rate is 160 to 180 bpm and severe when
greater than 180 bpm. Tachycardia greater than 200 bpm is usually due to
fetal tachyarrhythmia or congenital anomalies rather than hypoxia alone.

• 9. Question

A laboring client complains of low back pain. The nurse replies that this pain
occurs most when the position of the fetus is:

• A. Breech
• B. Transverse
• C. Occiput anterior
• D. Occiput posterior

Answer: D. Occiput posterior

A persistent occiput posterior position causes intense back pain because of fetal
compression of the sacral nerves. Occiput anterior is the most common fetal
position and does not cause back pain.

• Option A: Breech presentation is defined as a fetus in a longitudinal lie


with the buttocks or feet closest to the cervix. This occurs in 3-4% of all
406
C.BY: HOSSAM HAMDY MATERNITY RNPEDIA – ‫موسوعه التمريض‬

deliveries. The percentage of breech deliveries decreases with advancing


gestational age from 22-25% of births prior to 28 weeks’ gestation to 7-
15% of births at 32 weeks’ gestation to 3-4% of births at term.
• Option B: The transverse lie position is where the baby’s head is on one
side of the mother’s body and the feet on the other, rather than having the
head close to the cervix or close to the heart. The baby can also be
slightly at an angle, but still more sideways, than up or down.
• Option C: The left occiput anterior (LOA) position is the most common
in labor. In this position, the baby’s head is slightly off-center in the
pelvis with the back of the head toward the mother’s left thigh.

• 10. Question

The breathing technique that the mother should be instructed to use as the fetus’
head is crowning is:

• A. Blowing
• B. Slow chest
• C. Shallow
• D. Accelerated-decelerated

Answer: A. Blowing.

Blowing forcefully through the mouth controls the strong urge to push and
allows for a more controlled birth of the head.

• Option B: Slow breathing may be started when contractions are intense


enough that the woman can no longer walk or talk through them without
pausing. Use slow breathing for as long as it is helpful. Switch to another
pattern if the woman becomes tense and can no longer relax during
contractions.
• Option C: Most women feel the need to switch to light breathing at some
time during the active phase of labor. Let the intensity of the contractions
guide in deciding if and when to use light breathing. Breathe in and out
rapidly through the mouth at about one breath per second. Keep breathing
shallow and light. Inhalations should be quiet, but exhalation should be
clearly audible.
• Option D: This is a variation of light breathing. It is sometimes referred
to as “pant-pant-blow” or “hee-hee-hoo” breathing. Variable breathing
combines light shallow breathing with a periodic longer or more
pronounced exhalation. Variable breathing is used in the first stage if the
woman feels overwhelmed, unable to relax, in despair, or exhausted.

407
C.BY: HOSSAM HAMDY MATERNITY RNPEDIA – ‫موسوعه التمريض‬

• 11. Question

During the period of induction of labor, a client should be observed carefully for
signs of:

• A. Severe pain
• B. Uterine tetany
• C. Hypoglycemia
• D. Umbilical cord prolapse

Answer: B. Uterine tetany.

Uterine tetany could result from the use of oxytocin to induce labor. Because
oxytocin promotes powerful uterine contractions, uterine tetany may occur. The
oxytocin infusion must be stopped to prevent uterine rupture and fetal
compromise.

• Option A: Women being offered induction of labor should be informed


that induced labor is likely to be more painful than spontaneous labor.
During the induction of labor, healthcare professionals should provide
women with the pain relief appropriate for them and their pain.
• Option C: Since people with GDM and their babies are at increased risk
of pregnancy complications, some care providers encourage women with
GDM to plan an early birth (usually elective induction) at or near term
instead of waiting for labor to start on its own.
• Option D: Umbilical cord prolapse is an uncommon but potentially fatal
obstetric emergency. When this occurs during labor or delivery the
prolapsed cord is compressed between the fetal presenting part and the
cervix. This can result in a loss of oxygen to the fetus, and may even
result in a stillbirth.
• 12. Question

A client arrives at the hospital in the second stage of labor. The fetus’ head is
crowning, the client is bearing down, and the birth appears imminent. The nurse
should:

• A. Transfer her immediately by stretcher to the birthing unit.


• B. Tell her to breathe through her mouth and not to bear down.
• C. Instruct the client to pant during contractions and to breathe through
her mouth.
• D. Support the perineum with the hand to prevent tearing and tell the
client to pant.
408
C.BY: HOSSAM HAMDY MATERNITY RNPEDIA – ‫موسوعه التمريض‬

Answer: D. Support the perineum with the hand to prevent tearing and
tell the client to pant.

Gentle pressure is applied to the baby’s head as it emerges so it is not born too
rapidly. The head is never held back, and it should be supported as it emerges so
there will be
no vaginal lacerations. It is impossible to push and pant at the same time.

• Option A: Imminent delivery is when the baby’s head is visible at the


vaginal opening during a contraction (crowning). C. A visual inspection
of the perineal area should only be done when contractions are less than 5
minutes apart, there is bleeding/fluid discharge, and/or the patient feels
the urge to push.
• Option B: A visual inspection of the perineal area should only be done
when contractions are less than 5 minutes apart, there is bleeding/fluid
discharge, and/or the patient feels the urge to push. Do not perform a
digital examination to gauge cervical dilation.
• Option C: Begin each contraction with two deep breaths. Inhale deeply
and exhale slowly through pursed lips. Relax the bottom and push down.
Keep the abdominal muscles tight around the baby as she takes another
breath. The woman may find herself making throaty sounds. Repeat these
steps as long as the contraction lasts.

• 13. Question

A laboring client is to have a pudendal block. The nurse plans to tell the client
that once the block is working, she:

• A. Will not feel the episiotomy


• B. May lose bladder sensation
• C. May lose the ability to push
• D. Will no longer feel contractions

Answer: C. May lose the ability to push

A pudendal block is a local anesthetic injection given as a treatment for pain in


the second stage of labor (pushing) just before the delivery of the baby. It is
administered through the vaginal wall and into the pudendal nerve in the pelvis,
providing anesthesia to the perineum. The numbing effect of the pudendal block
may cause the woman to lose the ability to push that is why it is not given when
the baby’s head is too far down in the vagina.
409
C.BY: HOSSAM HAMDY MATERNITY RNPEDIA – ‫موسوعه التمريض‬

• Option A: A pudendal nerve block targets the pudendal nerve trunk as it


enters the lesser sciatic foramen, about 1 cm inferior and medial to the
attachment of the sacrospinous ligament to the ischial spine. Here, the
nerve is medial to the internal pudendal vessels. This nerve is accessed by
2 approaches, transvaginal and transcutaneous (or perineal).
• Option B: Bladder sensation is not lost. The sensory and motor
innervation of the perineum is derived from the pudendal nerve, which is
composed of the anterior primary divisions of the second, third, and
fourth sacral nerves.
• Option D: Pudendal block does not abolish the pain of uterine
contractions and cervical dilatation; this sensation is transmitted by the
sympathetic fibers derived from the spinal levels of T10-L2.

• 14. Question

Which of the following observations indicates fetal distress?

• A. Fetal scalp pH of 7.14


• B. Fetal heart rate of 144 beats/minute
• C. Acceleration of fetal heart rate with contractions
• D. Presence of long-term variability

Answer: A. Fetal scalp pH of 7.14

A fetal scalp pH below 7.25 indicates acidosis and fetal hypoxia. Fetal response
to oxygen deprivation is regulated by the autonomic nervous system, mediated
by parasympathetic and sympathetic mechanisms. The fetus is equipped with
compensatory mechanisms for transient hypoxia during labor, but prolonged,
uninterrupted fetal hypoxia may lead progressively to acidosis with cell death,
tissue damage, organ failure and potentially death.

• Option B: The fetal heart rate changes markedly in response to


prolonged oxygen deprivation, making fetal heart rate monitoring a
potentially valuable and commonly used tool for assessing fetal
oxygenation status in real-time. Non-reassuring fetal heart rate patterns
are observed in approximately 15% of labors
• Option C: While accelerations are associated with fetal well-being,
decelerations, especially prolonged bradycardia, late decelerations, and
severe variable decelerations are indicative of fetal stress and should
prompt the clinician to evaluate and initiate intrauterine resuscitation with
consideration for delivery of the fetus as indicated.
• Option D: Abnormal fetal heart rate patterns have high sensitivity, but
low specificity and low predictive value to discriminate between neonates
410
C.BY: HOSSAM HAMDY MATERNITY RNPEDIA – ‫موسوعه التمريض‬

with or without metabolic acidosis. While a normal fetal heart rate pattern
usually indicates reassuring fetal status, an abnormal fetal heart rate
pattern does not necessarily equate with hypoxia or acidosis.
• 15. Question

Which of the following fetal positions is most favorable for birth?

• A. Vertex presentation
• B. Transverse lie
• C. Frank breech presentation
• D. Posterior position of the fetal head

Answer: A. Vertex presentation

Vertex presentation (flexion of the fetal head) is the optimal presentation for
passage through the birth canal. Toward the end of pregnancy, the fetus moves
into position for delivery. Normally, the position of a fetus is facing rearward
(toward the woman’s back) with the face and body angled to one side and the
neck flexed, and presentation is head first.

• Option B: Transverse lie is an unacceptable fetal position for vaginal


birth and requires a C-section. In transverse lie, the fetus lies horizontally
across the birth canal and presents shoulder first. A cesarean delivery is
done, unless the fetus is the second in a set of twins. In such a case, the
fetus may be turned to be delivered through the vagina.
• Option C: Frank breech presentation, in which the buttocks present first,
can be a difficult vaginal delivery. When delivered vaginally, babies that
present buttocks first are more likely to be injured than those that present
head first. Such injuries may occur before, during, or after birth. The baby
may even die. Complications are less likely when breech presentation is
detected before labor or delivery.
• Option D: Posterior positioning of the fetal head can make it difficult for
the fetal head to pass under the maternal symphysis pubis. In occiput
posterior presentation (also called sunny-side up), the fetus is head first
but is facing up (toward the mother’s abdomen). It is the most common
abnormal position or presentation. When a fetus faces up, the neck is
often straightened rather than bent, and the head requires more space to
pass through the birth canal. Delivery by a vacuum extractor or forceps or
cesarean delivery may be necessary.
• 16. Question

411
C.BY: HOSSAM HAMDY MATERNITY RNPEDIA – ‫موسوعه التمريض‬

A laboring client has external electronic fetal monitoring in place. Which of the
following assessment data can be determined by examining the fetal heart rate
strip produced by the external electronic fetal monitor?

• A. Gender of the fetus


• B. Fetal position
• C. Labor progress
• D. Oxygenation

Answer: D. Oxygenation

Oxygenation of the fetus may be indirectly assessed through fetal monitoring by


closely examining the fetal heart rate strip. Accelerations in the fetal heart rate
strip indicate good oxygenation, while decelerations in the fetal heart rate
sometimes indicate poor fetal oxygenation.

• Option A: In the second and third trimesters of pregnancy, ultrasound


imaging scans the genital anatomy of the fetus to identify its gender. In
the early studies conducted on the use of ultrasound results for identifying
the fetal gender, a male fetus was demonstrated by the presence of a
scrotum and a penis, and a female fetus by the absence of these organs.
• Option B: Ultrasonography is noninvasive and has been found to be
more accurate for assessing position of the fetal head, during labor.
Recent studies by Sherer et al., Chou et al., Dupuis et al., and Zahalka et
al. have shown that ultrasound scanning is a quick and efficient way of
increasing the accuracy of the assessment of fetal head position during
the second stage of labor.
• Option C: Recently, intrapartum transperineal ultrasound for the
assessment of fetal head descent has been introduced to assess labor
progress in the first stage of labor in a more objective and non-invasive
way.
• 17. Question

A laboring client is in the first stage of labor and has progressed from 4 to 7 cm
in cervical dilation. In which of the following phases of the first stage does
cervical dilation occur most rapidly?

• A. Preparatory phase
• B. Latent phase
• C. Active phase
• D. Transition phase

412
C.BY: HOSSAM HAMDY MATERNITY RNPEDIA – ‫موسوعه التمريض‬

Answer: C. Active phase.

Cervical dilation occurs more rapidly during the active phase than any of the
previous phases. The active phase is characterized by cervical dilation that
progresses from 4 to 7 cm. The active phase of labor should occur much more
rapidly, usually within 4 hours after the rupture of membranes when there are
adequate uterine contractions. If the mother does not fully dilate by these
average estimations, she is said to be having an arrest of the active phase of
Stage 1 labor.

• Option A: The preparatory, or latent, phase begins with the onset of


regular uterine contractions and ends when rapid cervical dilation begins.
In a primigravida, the latent phase should not exceed 20 hours. In a
multigravida, the latent phase should not exceed 14 hours. If a female is
in labor and exceeds these average time frames, she is said to be having a
prolonged latent phase.
• Option B: Stage 1 subdivides into latent and active where 0 to 6 cm is
the latent phase of Stage 1, and 6 cm to complete dilation of the cervix is
recognized as the active phase of Stage 1 labor.
• Option D: Transition is defined as cervical dilation beginning at 8 cm
and lasting until 10 cm or complete dilation. The third phase is called
transition and is the last phase. During transition, the cervix dilates from 8
to 10 centimeters. Contractions are usually very strong, lasting 60 to 90
seconds and occurring every few minutes. Most women feel the urge to
push during this phase.
• 18. Question

A multiparous client who has been in labor for 2 hours states that she feels the
urge to move her bowels. How should the nurse respond?

• A. Let the client get up to use the potty


• B. Allow the client to use a bedpan
• C. Perform a pelvic examination
• D. Check the fetal heart rate

Answer: C. Perform a pelvic examination

A complaint of rectal pressure usually indicates a low presenting fetal part,


signaling imminent delivery. The nurse should perform a pelvic examination to
assess the dilation of the cervix and station of the presenting fetal part.
413
C.BY: HOSSAM HAMDY MATERNITY RNPEDIA – ‫موسوعه التمريض‬

• Option A: During labor a lot of pressure may be exerted on the fallopian


tubes, ovaries, and ligaments. The baby’s presenting part (usually the
head) pressed firmly against the bladder and bowel as he descends
through the pelvis. This can lead to great pain, particularly if the woman
doesn’t empty the bladder frequently. About once an hour is a good rule
to remember.
• Option B: Labor is usually painful for several very good reasons. For
one, the cervix, completely insensitive to burning and cauterization, is
nevertheless extremely sensitive to pressure and stretching–precisely
what it undergoes during labor. Most women feel contractions as
cramping sensations in the groin or back, though some experience more
pain in their sides or thighs. As the contractions get longer, stronger, and
closer together over the course of labor, they will be perceived as more or
less painful by different women.
• Option D: When the woman lies on her back for long periods of time, the
weight of the uterus compresses the descending aorta and inferior vena
cava, blood vessels that supply or drain the lower part of the body. This
interference with the circulation reduces the blood pressure,
compromising blood flow to the baby and causing his heart rate to drop.
When the woman stays upright (or at least off her back), placental
circulation improves and fetal heart rate abnormalities may be alleviated.
• 19. Question

Labor is a series of events affected by the coordination of the five essential


factors. One of these is the passenger (fetus). Which are the other four factors?

• A. Contractions, passageway, placental position and function, pattern of


care.
• B. Contractions, maternal response, placental position, psychological
response.
• C. Passageway, contractions, placental position, and function,
psychological response.
• D. Passageway, placental position, and function, paternal response,
psychological response.

Answer: C. Passageway, contractions, placental position and function,


psychological response.

The five essential factors (5 P’s) are passenger (fetus), passageway (pelvis),
powers (contractions), placental position and function, and psyche
(psychological response of the mother).

414
C.BY: HOSSAM HAMDY MATERNITY RNPEDIA – ‫موسوعه التمريض‬

• Option A: The passage is defined as the bony boundaries of the pelvis.


The shape of the pelvis determines how easily the baby can pass through.
The most common pelvic shaped bone for a woman is called a gynecoid
pelvis. This shaped pelvis is easiest for a baby to pass through.
• Option B: The power factor in labor refers to the ability of the uterine
muscle to contract. The uterus is an involuntary muscle. It has to not only
start contracting, but it must establish a pattern of contractions. Every
time the uterus contracts it pushes the baby towards the cervix. This is
really what labor is all about. The contractions cause the cervix to stretch
open and allow the baby into the birth canal.
• Option D: Position refers to not only the position of the baby but also the
position of the mother. The old adage that what is up must come down
also refers to babies. When a mother is in an upright position this
contributes to abdominal wall relaxation. This helps the fundus (the upper
portion of the uterus) to go forward due to the force of gravity, which
then leads to the straightening of the birth canal and widening of the
pelvic outlet. All of which helps a baby to be born.
• 20. Question

Fetal presentation refers to which of the following descriptions?

• A. Fetal body part that enters the maternal pelvis first.


• B. Relationship of the presenting part to the maternal pelvis.
• C. Relationship of the long axis of the fetus to the long axis of the
mother.
• D. A classification according to the fetal part.

Answer: A. Fetal body part that enters the maternal pelvis first.

Presentation is the fetal body part that enters the pelvis first; it’s classified by
the presenting part; the three main presentations are cephalic/occipital, breech,
and shoulder.

• Option B: The relationship of the presenting fetal part to the maternal


pelvis refers to fetal position.
• Option C: The relationship of the long axis to the fetus to the long axis
of the mother refers to fetal lie; the three possible lies are longitudinal,
transverse, and oblique.
• Option D: Fetal station refers to where the presenting part is in the
pelvis. The presenting part. The presenting part is the part of the baby that
leads the way through the birth canal. Most often, it is the baby’s head,
but it can be a shoulder, the buttocks, or the feet.
415
C.BY: HOSSAM HAMDY MATERNITY RNPEDIA – ‫موسوعه التمريض‬

• 21. Question

A client is admitted to the L & D suite at 36 weeks’ gestation. She has a history
of C-section and complains of severe abdominal pain that started less than 1
hour earlier. When the nurse palpates tetanic contractions, the client again
complains of severe pain. After the client vomits, she states that the pain is
better and then passes out. Which is the probable cause of her signs and
symptoms?

• A. Hysteria compounded by the flu


• B. Placental abruption
• C. Uterine rupture
• D. Dysfunctional labor

Answer: C. Uterine rupture.

Uterine rupture is a medical emergency that may occur before or during labor.
Signs and symptoms typically include abdominal pain that may ease after
uterine rupture, vomiting, vaginal bleeding, hypovolemic shock, and fetal
distress. With placental abruption, the client typically complains of vaginal
bleeding and constant abdominal pain.

• Option A: The woman does not have hysteria. Uterine rupture in


pregnancy is a rare and often catastrophic complication with a high
incidence of fetal and maternal morbidity. Numerous factors are known
to increase the risk of uterine rupture, but even in high-risk subgroups, the
overall incidence of uterine rupture is low.
• Option B: Placental abruption occurs when the maternal vessels tear
away from the placenta and bleeding occurs between the uterine lining
and the maternal side of the placenta. As the blood accumulates, it pushes
the uterine wall and placenta apart. The placenta is the fetus’ source of
oxygen and nutrients as well as the way the fetus excretes waste products.
Diffusion to and from the maternal circulatory system is essential to
maintaining these life-sustaining functions of the placenta. When
accumulating blood causes separation of the placenta from the maternal
vascular network, these vital functions of the placenta are interrupted. If
the fetus does not receive enough oxygen and nutrients, it dies.
• Option D: Dysfunctional or prolonged labor refers to prolongation in the
duration of labor, typically in the first stage of labor. Diagnosis of delay
in labor is dependent on careful monitoring of uterine contraction
intensity, duration and frequency, cervical dilation, and descent of the
fetus through the pelvis.

416
C.BY: HOSSAM HAMDY MATERNITY RNPEDIA – ‫موسوعه التمريض‬

• 22. Question

Upon completion of a vaginal examination on a laboring woman, the nurse


records 50%, 6 cm, -1. Which of the following is a interpretation of the data?

• A. Fetal presenting part is 1 cm above the ischial spines.


• B. Effacement is 4 cm from completion.
• C. Dilation is 50% completed.
• D. Fetus has achieved passage through the ischial spines.

Answer: A. Fetal presenting part is 1 cm above the ischial spines.

Station of – 1 indicates that the fetal presenting part is above the ischial spines
and has not yet passed through the pelvic inlet. A station of zero would indicate
that the presenting part has passed through the inlet and is at the level of the
ischial spines or is engaged.

• Option B: Progress of effacement is referred to by percentages with


100% indicating full effacement. Some women may reach 100%
effacement within a few hours. For others, cervical effacement may occur
slowly over several weeks.
• Option C: Dilation by centimeters (cm) with 10 cm indicating full
dilation. One woman may go from having a closed cervix to giving birth
in a matter of hours, while another is 1–2 cm dilated for days or weeks.
Some women do not experience any dilation until they go into active
labor. This means that the cervix is completely closed initially, but it
widens to 10 cm as labor progresses.
• Option D: Passage through the ischial spines with internal rotation would
be indicated by a plus station, such as + 1.

• 23. Question

Which of the following findings meets the criteria of a reassuring FHR pattern?

• A. FHR does not change as a result of fetal activity.


• B. Average baseline rate ranges between 100 – 140 BPM.
• C. Mild late deceleration patterns occur with some contractions.
• D. Variability averages between 6 – 10 BPM.

Answer: D. Variability averages between 6 – 10 BPM.

417
C.BY: HOSSAM HAMDY MATERNITY RNPEDIA – ‫موسوعه التمريض‬

Variability indicates a well-oxygenated fetus with a functioning autonomic


nervous system. The FHR is under constant variation from the baseline. This
variability reflects a healthy nervous system, chemoreceptors, baroreceptors and
cardiac responsiveness. Prematurity decreases variability; therefore, there is
little rate fluctuation before 28 weeks. Variability should be normal after 32
weeks.

• Option A: FHR should accelerate with fetal movement. The FHR is


controlled by the autonomic nervous system. The inhibitory influence on
the heart rate is conveyed by the vagus nerve, whereas excitatory
influence is conveyed by the sympathetic nervous system. Progressive
vagal dominance occurs as the fetus approaches term and, after birth,
results in a gradual decrease in the baseline FHR. Stimulation of the
peripheral nerves of the fetus by its own activity (such as movement) or
by uterine contractions causes acceleration of the FHR.
• Option B: Baseline range for the FHR is 120 to 160 beats per minute.
The baseline rate is interpreted as changed if the alteration persists for
more than 15 minutes. Prematurity, maternal anxiety, and maternal fever
may increase the baseline rate, while fetal maturity decreases the baseline
rate.
• Option C: Late deceleration patterns are never reassuring, though early
and mild variable decelerations are expected, reassuring findings. Late
decelerations are associated with uteroplacental insufficiency and are
provoked by uterine contractions. Any decrease in uterine blood flow or
placental dysfunction can cause late decelerations. Maternal hypotension
and uterine hyperstimulation may decrease uterine blood flow. Postdate
gestation, preeclampsia, chronic hypertension, and diabetes mellitus are
among the causes of placental dysfunction. Other maternal conditions
such as acidosis and hypovolemia associated with diabetic ketoacidosis
may lead to a decrease in uterine blood flow, late decelerations, and
decreased baseline variability.
• 24. Question

Late deceleration patterns are noted when assessing the monitor tracing of a
woman whose labor is being induced with an infusion of Pitocin. The woman is
in a side-lying position, and her vital signs are stable and fall within a normal
range. Contractions are intense, last 90 seconds, and occur every 1 1/2 to 2
minutes. The nurse’s immediate action would be to:

• A. Change the woman’s position


• B. Stop the Pitocin
• C. Elevate the woman’s legs
• D. Administer oxygen via a tight mask at 8 to 10 liters/minute
418
C.BY: HOSSAM HAMDY MATERNITY RNPEDIA – ‫موسوعه التمريض‬

Answer: B. Stop the Pitocin

Late deceleration patterns noted are most likely related to alteration in


uteroplacental perfusion associated with the strong contractions described. The
immediate action would be to stop the Pitocin infusion since Pitocin is an
oxytocin which stimulates the uterus to contract.

• Option A: The woman is already in an appropriate position for


uteroplacental perfusion. A late deceleration is a symmetric fall in the
fetal heart rate, beginning at or after the peak of the uterine contraction
and returning to baseline only after the contraction has ended
• Option C: Elevation of her legs would be appropriate if hypotension
were present. Regardless of the depth of the deceleration, all late
decelerations are considered potentially ominous. A pattern of persistent
late decelerations is nonreassuring, and further evaluation of the fetal pH
is indicated.
Option D: Oxygen is appropriate but not the immediate action. The
occurrence of a late or worsening variable deceleration pattern in the
presence of normal variability generally means that the fetal stress is
either of a mild degree or of recent origin; however, this pattern is
considered nonreassuring.

• 25. Question

The nurse should realize that the most common and potentially harmful
maternal complication of epidural anesthesia would be:

• A. Severe postpartum headache


• B. Limited perception of bladder fullness
• C. Increase in respiratory rate
• D. Hypotension

Answer: D. Hypotension.

Epidural anesthesia can lead to vasodilation and a drop in blood pressure that
could interfere with adequate placental perfusion. The woman must be well
hydrated before and during epidural anesthesia to prevent this problem and
maintain adequate blood pressure.

419
C.BY: HOSSAM HAMDY MATERNITY RNPEDIA – ‫موسوعه التمريض‬

• Option A: Headache is not a side effect since the spinal fluid is not
disturbed by this anesthetic as it would be with a low spinal (saddle
block) anesthesia;
• Option B: Limited perception of bladder fullness is an effect of epidural
anesthesia but is not the most harmful. After having an epidural, the
woman may not be able to feel when her bladder is full because the
epidural affects the surrounding nerves.
• Option C: Respiratory depression is a potentially serious complication.
Occasionally, some medicines used in an epidural can cause slow
breathing or drowsiness.

420
C.BY: HOSSAM HAMDY MATERNITY RNPEDIA – ‫موسوعه التمريض‬

Maternity Nursing NU.10 (Quiz #1: 25 Questions)\

1. Question

A postpartum nurse is preparing to care for a woman who has just delivered
a healthy newborn infant. In the immediate postpartum period, the nurse
plans to take the woman’s vital signs:

A. Every 30 minutes during the first hour and then every hour for the next
two hours.

B. Every 15 minutes during the first hour and then every 30 minutes for the
next two hours.

C. Every hour for the first 2 hours and then every 4 hours.

D. Every 5 minutes for the first 30 minutes and then every hour for the next
4 hours.

Answer: B. Every 15 minutes during the first hour and then every 30
minutes for the next two hours.

The initial or acute period involves the first 6–12 hours postpartum. This is a
time of rapid change with a potential for immediate crises such as
postpartum hemorrhage, uterine inversion, amniotic fluid embolism, and
eclampsia.

Option A: The second phase is the subacute postpartum period, which lasts
2–6 weeks. During this phase, the body is undergoing major changes in
terms of hemodynamics, genitourinary recovery, metabolism, and emotional
status. Nonetheless, the changes are less rapid than in the acute postpartum
phase and the patient is generally capable of self-identifying problems.
These may run the gamut from ordinary concerns about perineal discomfort
to peripartum cardiomyopathy or severe postpartum depression.

Option C: The third phase is the delayed postpartum period, which can last
up to 6 months. Changes during this phase are extremely gradual, and
pathology is rare. This period is used to make sure the mother is stable and to
educate her in the care of her baby (especially the first-time mother). While
still in the hospital, the mother is monitored for blood loss, signs of infection,
421
C.BY: HOSSAM HAMDY MATERNITY RNPEDIA – ‫موسوعه التمريض‬

abnormal blood pressure, contraction of the uterus, and ability to void. There
is also attention to Rh compatibility, maternal immunization statuses, and
breastfeeding. This is the time of restoration of muscle tone and connective
tissue to the prepregnant state. Although change is subtle during this phase, it
behooves caregivers to remember that a woman?s body is nonetheless not
fully restored to prepregnant physiology until about 6 months post-delivery.

Option D: The immediate postpartum period most often occurs in the


hospital setting, where the majority of women remain for approximately 2
days after a vaginal delivery and 3-4 days after a cesarean delivery. During
this time, women are recovering from their delivery and are beginning to
care for the newborn.

2. Question

A postpartum nurse is taking the vital signs of a woman who delivered a


healthy newborn infant 4 hours ago. The nurse notes that the mother’s
temperature is 100.2°F. Which of the following actions would be most
appropriate?

A. Retake the temperature in 15 minutes.

B. Notify the physician.

C. Document the findings.

D. Increase hydration by encouraging oral fluids

Answer: D. Increase hydration by encouraging oral fluids.

The mother’s temperature may be taken every 4 hours while she is awake.
Temperatures up to 100.4 F (38 C) in the first 24 hours after birth are often
related to the dehydrating effects of labor. The most appropriate action is to
increase hydration by encouraging oral fluids, which should bring the
temperature to a normal reading.

Option A: A focused physical examination is important and should include


vital signs, an examination of the respiratory system, breasts, abdomen,
perineum, and lower extremities. A patient with endometritis typically has a
fever of 38°C or greater, tachycardia, and fundal tenderness.

Option B: The new mother should be given discharge instructions and


expectations/precautions to consider once leaving the hospital. The most
422
C.BY: HOSSAM HAMDY MATERNITY RNPEDIA – ‫موسوعه التمريض‬

important information is who and where to call if she has problems or


questions. She also needs details about resuming her normal activity.
Instructions vary, depending on whether the mother has had a vaginal or a
cesarean delivery and any comorbidities that may have been part of her care.

Option C: Although the nurse would document the findings, the most
appropriate action would be to increase the hydration. The woman who has
had a vaginal delivery may resume all physical activity, including using
stairs, riding or driving in a car, and performing muscle-toning exercises, as
long as she experiences no limiting pain or discomfort. The key counseling
is to progressively resume normal activity while being mindful of the
common fatigue and exhaustion experienced while caring for a newborn.

3. Question

The nurse is assessing a client who is 6 hours PP after delivering a full-term


healthy infant. The client complains to the nurse of feelings of faintness and
dizziness. Which of the following nursing actions would be most
appropriate?

A. Obtain hemoglobin and hematocrit levels.

B. Instruct the mother to request help when getting out of bed.

C. Elevate the mother’s legs.

D. Inform the nursery room nurse to avoid bringing the newborn infant to the
mother until the feelings of lightheadedness and dizziness have subsided.

Answer: B. Instruct the mother to request help when getting out of bed.

Orthostatic hypotension may be evident during the first 8 hours after birth.
Feelings of faintness or dizziness are signs that should caution the nurse to
be aware of the client’s safety. The nurse should advise the mother to get
help the first few times the mother gets out of bed.

Option A: Obtaining an H/H requires a physician’s order. This is a blood


test that checks the percent of the blood (called whole blood) that’s made up
of red blood cells. Bleeding can cause a low hematocrit.

Option C: With PPH, the client can lose much more blood, which is what
makes it a dangerous condition. PPH can cause a severe drop in blood

423
C.BY: HOSSAM HAMDY MATERNITY RNPEDIA – ‫موسوعه التمريض‬

pressure. If not treated quickly, this can lead to shock and death. Shock is
when the body organs don’t get enough blood flow.

Option D: Postpartum hemorrhage (also called PPH) is when a woman has


heavy bleeding after giving birth. It’s a serious but rare condition. It usually
happens within 1 day of giving birth, but it can happen up to 12 weeks after
having a baby. About 1 to 5 in 100 women who have a baby (1 to 5 percent)
have PPH.

4. Question

A nurse is preparing to perform a fundal assessment on a postpartum client.


The initial nursing action in performing this assessment is which of the
following?

A. Ask the client to turn on her side.

B. Ask the client to lie flat on her back with the knees and legs flat and
straight.

C. Ask the mother to urinate and empty her bladder.

D. Massage the fundus gently before determining the level of the fundus.

Answer: C. Ask the mother to urinate and empty her bladder.

Before starting the fundal assessment, the nurse should ask the mother to
empty her bladder so that an accurate assessment can be done. The
postpartum recovery period covers the time period from birth until
approximately six to eight weeks after delivery. This is a time of healing and
rejuvenation as the mother’s body returns to prepregnancy states.

Option A: The nurse may place the woman in a supine position or Semi
Fowlers position to avoid a decrease in her blood pressure for fundal
assessment. Patients or a family member can be taught to assess the firmness
of the fundus and to provide massage in the event of a boggy uterus or
excessive bleeding. Patients are encouraged to void before palpation of the
uterine fundus because a full bladder displaces the uterus and can lead to
excessive bleeding.

Option B: When the nurse is performing a fundal assessment, the nurse asks
the woman to lie flat on her back with the knees flexed.

424
C.BY: HOSSAM HAMDY MATERNITY RNPEDIA – ‫موسوعه التمريض‬

Option D: Massaging the fundus is not appropriate unless the fundus is


boggy and soft, and then it should be massaged gently until firm. By
approximately one hour post-delivery, the fundus is firm and at the level of
the umbilicus.

5. Question

The nurse is assessing the lochia on a 1 day PP patient. The nurse notes that
the lochia is red and has a foul-smelling odor. The nurse determines that this
assessment finding is:

A. Normal.

B. Indicates the presence of infection.

C. Indicates the need for increasing oral fluids.

D. Indicates the need for increasing ambulation.

Answer: B. Indicates the presence of infection.

Lochia, the discharge present after birth, is red for the first 1 to 3 days and
gradually decreases in amount. Foul-smelling or purulent lochia usually
indicates infection, and these findings are not normal. The presence of an
offensive odor or large pieces of tissue or blood clots in lochia or the absence
of lochia might be a sign of infection.

Option A: Normal lochia has a fleshy odor. The lochia is the vaginal
discharge that originates from the uterus, cervix, and vagina. The lochia is
initially red and consists of blood and fragments of decidua, endometrial
tissues, and mucus and lasts 1 to 4 days.

Option C: The lochia then changes color to yellowish or pale brown, lasting
5 to 9 days, and is composed mainly of blood, mucus, and leukocytes.
Finally, the lochia is white and contains mostly mucus, lasting up to 10 to 14
days. The lochia can persist up to 5 weeks postpartum.

Option D: Encouraging the woman to drink fluids or increase ambulation is


not an accurate nursing intervention. The lochia can persist up to 5 weeks
postpartum. The cervix and vagina may be edematous and bruised in the
early postpartum period and gradually heal back to normal.

6. Question
425
C.BY: HOSSAM HAMDY MATERNITY RNPEDIA – ‫موسوعه التمريض‬

When performing a PP assessment on a client, the nurse notes the presence


of clots in the lochia. The nurse examines the clots and notes that they are
larger than 1 cm. Which of the following nursing actions is most
appropriate?

A. Document the findings.

B. Notify the physician.

C. Reassess the client in 2 hours.

D. Encourage increased intake of fluids.

Answer: B. Notify the physician.

Normally, one may find a few small clots in the first 1 to 2 days after birth
from pooling of blood in the vagina. Clots larger than 1 cm are considered
abnormal. The cause of these clots, such as uterine atony or retained
placental fragments, needs to be determined and treated to prevent further
blood loss.

Option A: Although the findings would be documented, the most


appropriate action is to notify the physician. Early postpartum hemorrhage is
described as that occurring within the first 24 hours after delivery. Late
postpartum hemorrhage most frequently occurs 1-2 weeks after delivery but
may occur up to 6 weeks postpartum.

Option C: The persistence of red lochia beyond one week might be an


indicator of uterine subinvolution. The presence of an offensive odor or large
pieces of tissue or blood clots in lochia or the absence of lochia might be a
sign of infection.

Option D: The most common cause of immediate postpartum hemorrhage is


atony; therefore uterotonic agents should be readily available for quick
access and prompt administration in order to control bleeding.

7. Question

A nurse in a PP unit is instructing a mother regarding lochia and the amount


of expected lochia drainage. The nurse instructs the mother that the normal
amount of lochia may vary but should never exceed the need for:

A. One peripad per day.


426
C.BY: HOSSAM HAMDY MATERNITY RNPEDIA – ‫موسوعه التمريض‬

B. Two peripads per day.

C. Three peripads per day.

D. Eight peripads per day.

Answer: D. Eight peripads per day.

The normal amount of lochia may vary with the individual but should never
exceed 4 to 8 peripads per day. The average number of peripads is 6 per day.
Postpartum hemorrhage is defined as excessive blood loss during or after the
third stage of labor. The average blood loss is 500 mL at vaginal delivery
and 1000 mL at cesarean delivery.

Option A: Objectively, postpartum hemorrhage is defined as a 10% change


in hematocrit level between admission and the postpartum period or the need
for transfusion after delivery secondary to blood loss.

Option B: Early postpartum hemorrhage may result from uterine atony,


retained products of conception, uterine rupture, uterine inversion, placenta
accreta, lower genital tract lacerations, coagulopathy, and hematoma. In the
United States, postpartum hemorrhage is responsible for 5% of maternal
deaths. Other morbidities associated with hemorrhage include the need for
blood transfusions and/or subsequent surgical interventions that may lead to
future infertility.

Option C: Causes of late postpartum hemorrhage most commonly include


retained products of conception, infection, subinvolution of placental site,
and coagulopathy. Vaginal delivery is associated with a 3.9% incidence of
postpartum hemorrhage. Cesarean delivery is associated with a 6.4%
incidence of postpartum hemorrhage. Delayed postpartum hemorrhage
occurs in 1-2% of patients.

8. Question

A PP nurse is providing instructions to a woman after delivery of a healthy


newborn infant. The nurse instructs the mother that she should expect normal
bowel elimination to return:

A. One the day of the delivery

B. 3 days PP

427
C.BY: HOSSAM HAMDY MATERNITY RNPEDIA – ‫موسوعه التمريض‬

C. 7 days PP

D. within 2 weeks PP

Answer: B. 3 days PP.

After birth, the nurse should auscultate the woman’s abdomen in all four
quadrants to determine the return of bowel sounds. Normal bowel
elimination usually returns 2 to 3 days PP. Surgery, anesthesia, and the use
of narcotics and pain control agents also contribute to the longer period of
altered bowel function.

Option A: The mother may develop flatulence or constipation due to


intestinal ileus (induced by pain or presence of placental hormone relaxin in
the circulation), loss of body fluids, laxity of abdominal wall, and
hemorrhoids.

Option C: The postpartum constipation is due to the progesterone-induced


decrease in gastrointestinal transit time. The compressive effects of the
gravid uterus on the stomach, a decrease in lower esophageal sphincter tone
due to high progesterone levels, and hypersecretion of acid due to high
gastrin levels cause an increase in the incidence of acid reflux during
pregnancy.

Option D: After delivery, the levels of progesterone and gastrin drop within
24 hours, and the acid reflux and associated symptoms resolve in the next
three to four days

9. Question

The following are the physiological maternal changes that occur during the
PP period. Select all that apply.

A. Cervical involution occurs.

B. Vaginal distention decreases slowly.

C. Fundus begins to descend into the pelvis after 24 hours.

D. Cardiac output decreases with resultant tachycardia in the first 24 hours.

E. Digestive processes slow immediately.

428
C.BY: HOSSAM HAMDY MATERNITY RNPEDIA – ‫موسوعه التمريض‬

Answers: A and C. In the PP period, cervical healing occurs rapidly


and cervical involution occurs.

After 1 week the muscle begins to regenerate and the cervix feels firm and
the external os, is the width of a pencil. The fundus begins to descent into the
pelvic cavity after 24 hours, a process known as involution.

Option B: Although the vaginal mucosa heals and vaginal distention


decreases, it takes the entire PP period for complete involution to occur and
muscle tone is never restored to the pregravid state.

Option D: Despite blood loss that occurs during delivery of the baby, a
transient increase in cardiac output occurs. The increase in cardiac output,
which persists about 48 hours after childbirth, is probably caused by an
increase in stroke volume because Bradycardia is often noted during the PP
period.

Option E: Soon after childbirth, digestion begins to begin to be active, and


the new mother is usually hungry because of the energy expended during
labor.

10. Question

A nurse is caring for a PP woman who has received epidural anesthesia and
is monitoring the woman for the presence of a vulva hematoma. Which of
the following assessment findings would best indicate the presence of a
hematoma?

A. Complaints of a tearing sensation.

B. Complaints of intense pain.

C. Changes in vital signs.

D. Signs of heavy bruising.

Answer: C. Changes in vital signs.

Changes in vitals indicate hypovolemia in the anesthetized PP woman with


vulvar hematoma. There may also be intermittent bleeding. Depending on

429
C.BY: HOSSAM HAMDY MATERNITY RNPEDIA – ‫موسوعه التمريض‬

the size and location of the vulvar hematoma, urological or neurological


signs and symptoms may be present.

Option A: A hematoma is described as a collection of blood beneath an


intact epidermis that presents as a swollen fluctuant lump. It can be
extremely tender on palpation. Due to its rich blood supply, the vulva is
highly vulnerable and prone to hematoma formation. Although venous
bleeding is possible, arterial bleeds mainly originate from one of the
branches of the pudendal artery. Vulvar hematoma, rarely, might be
secondary to operative laparoscopy (especially adnexal surgery),
spontaneous rupture of the internal iliac artery, or spontaneous rupture of a
pseudoaneurysm of the pudendal artery.

Option B: Because the woman has had epidural anesthesia and is


anesthetized, she cannot feel pain, pressure, or a tearing sensation. Due to
mechanical urethral obstruction, patients may present with urinary retention
or micturition difficulties. In severe cases, the patient can be
hemodynamically unstable and will require urgent fluid resuscitation or
blood transfusion. Symptoms usually develop within a few hours to days of
delivery, depending on the severity of the condition.

Option D: Heavy bruising may be visualized, but vital sign changes indicate
hematoma caused by blood collection in the perineal tissues. As bleeding
into the vulva is largely restricted only by the Colles fascia and the
urogenital diaphragm, a hematoma in this area will be visible on physical
examination. This is seen as a tender fluctuant lump of variable size. Since
the Colles fascia exerts little resistance, vulvar hematomas can grow to
become 15cm in diameter or more. The observation of a lump or swelling in
the groin may be offered by the patient if asked during the consultation.
Although there is no anatomical explanation, it is discovered that the right
side appears to be more commonly affected.

11. Question

A nurse is developing a plan of care for a PP woman with a small vulvar


hematoma. The nurse includes which specific intervention in the plan during
the first 12 hours following the delivery of this client?

A. Assess vital signs every 4 hours.

B. Inform health care providers of assessment findings.

C. Measure fundal height every 4 hours.

D. Prepare an ice pack for application to the area.


430
C.BY: HOSSAM HAMDY MATERNITY RNPEDIA – ‫موسوعه التمريض‬

Answer: D. Prepare an ice pack for application to the area.

Option D: Application of ice will reduce swelling caused by hematoma


formation in the vulvar area. During labor, a vulvar hematoma can result
from either direct or indirect injury to the soft tissue. Examples of causes of
direct injuries include episiotomy, vaginal laceration repairs, or instrumental
deliveries, while indirect injury can result from extensive stretching of the
birth canal during vaginal delivery.

12. Question

A new mother received epidural anesthesia during labor and had a forceps
delivery after pushing 2 hours. At 6 hours PP, her systolic blood pressure has
dropped 20 points, her diastolic BP has dropped 10 points, and her pulse is
120 beats per minute. The client is anxious and restless. On further
assessment, a vulvar hematoma is verified. After notifying the health care
provider, the nurse immediately plans to:

A. Monitor fundal height.

B. Apply perineal pressure.

C. Prepare the client for surgery.

D. Reassure the client.

Answer: C. Prepare the client for surgery.

The use of an epidural, prolonged second-stage labor and forceps delivery


are predisposing factors for hematoma formation, and a collection of up to
500 ml of blood can occur in the vaginal area. Although the other options
may be implemented, the immediate action would be to prepare the client for
surgery to stop the bleeding.

Option A: Interestingly, most vulvar hematomas are formed after a normal


delivery instead of complicated deliveries. Risk factors for developing vulvar
hematoma include instrumental delivery, episiotomy, primiparity, prolonged
second stage of labor, macrosomia, use of anticoagulants, coagulopathy,
hypertensive disorders of pregnancy, and vulvovaginal varicosity.

431
C.BY: HOSSAM HAMDY MATERNITY RNPEDIA – ‫موسوعه التمريض‬

Option B: Conservative management usually involves the use of ice packs,


local compressions, bed rest, and analgesics. In the event that conservative
management has not been effective, surgery may be performed. In fact,
conservative management of large hematomas has been found to be
associated with a longer period of hospitalization, greater need for
antibiotics, and blood transfusion.

Option D: Surgical management includes surgical drainage of the


hematoma, evacuation of any clots present, ligation of bleeding points, and
the assessment for signs of pressure necrosis (a complication of vulva
hematoma). These can be done under local anesthesia. As further blood loss
during surgery is anticipated, the necessary investigations such as cross-
matching and preparations for a possible blood transfusion should be done.

13. Question

A nurse is monitoring a new mother in the PP period for signs of


hemorrhage. Which of the following signs, if noted in the mother, would be
an early sign of excessive blood loss?

A. A temperature of 100.4°F.

B. An increase in the pulse from 88 to 102 BPM.

C. An increase in the respiratory rate from 18 to 22 breaths per minute.

D. Blood pressure changes from 130/88 to 124/80 mm Hg.

Answer: B. An increase in the pulse from 88 to 102 BPM.

During the 4th stage of labor, the maternal blood pressure, pulse, and
respiration should be checked every 15 minutes during the first hour. A
rising pulse is an early sign of excessive blood loss because the heart pumps
faster to compensate for reduced blood volume.

Option A: A slight rise in temperature is normal. Patients may also have


signs and symptoms of shock, such as confusion, blurry vision, clammy skin,
and weakness.

Option C: The respiratory rate has increased slightly. The patient may also
have an increased heart rate, an increased respiratory rate, and feeling faint
while standing up.

432
C.BY: HOSSAM HAMDY MATERNITY RNPEDIA – ‫موسوعه التمريض‬

Option D: The blood pressure will fall as the blood volume diminishes, but
a decreased blood pressure would not be the earliest sign of hemorrhage. As
the patient continues to lose blood, they may also feel cold, have decreased
blood pressure, and may lose consciousness.

14. Question

A nurse is preparing to assess the uterine fundus of a client in the immediate


postpartum period. When the nurse locates the fundus, she notes that the
uterus feels soft and boggy. Which of the following nursing interventions
would be most appropriate initially?

A. Massage the fundus until it is firm.

B. Elevate the mother's legs.

C. Push on the uterus to assist in expressing clots.

D. Encourage the mother to void.

Answer: A. Massage the fundus until it is firm.

If the uterus is not contracted firmly, the first intervention is to massage the
fundus until it is firm and to express clots that may have accumulated in the
uterus. Uterine atony refers to the corpus uteri myometrial cells inadequate
contraction in response to endogenous oxytocin that is released in the course
of delivery. Risk factors for uterine atony include prolonged labor,
precipitous labor, uterine distension (multi-fetal gestation, polyhydramnios,
fetal macrosomia), fibroid uterus, chorioamnionitis, indicated magnesium
sulfate infusions, and prolonged use of oxytocin.

Option B: Elevating the mother’s legs will not manage the uterine atony.
Ineffective uterine contraction, either focally or diffusely, is additionally
associated with a diverse range of etiologies including retained placental
tissue, placental disorders (such as morbidly adherent placenta, placenta
previa, and abruptio placentae), coagulopathy (increased fibrin degradation
products) and uterine inversion.

Option C: Pushing on an uncontracted uterus can invert the uterus and cause
massive hemorrhage. It leads to postpartum hemorrhage as delivery of the
placenta leaves disrupted spiral arteries which are uniquely void of
musculature and dependent on contractions to mechanically squeeze them
into a hemostatic state. Uterine atony is a principal cause of postpartum
433
C.BY: HOSSAM HAMDY MATERNITY RNPEDIA – ‫موسوعه التمريض‬

hemorrhage, an obstetric emergency. Globally, this is one of the top 5 causes


of maternal mortality.

Option D: Encouraging the client to void will not assist in managing uterine
atony. If the uterus does not remain contracted as a result of the uterine
massage, the problem may be distended bladder and the nurse should assist
the mother to urinate, but this would not be the initial action. Contraction of
the myometrium that mechanically compresses the blood vessels supplying
the placental bed provides the principal mechanism uterine hemostasis after
delivery of the fetus, and the placenta is concluded. The process is
complemented by local decidual hemostatic factors such as tissue factor
type-1 plasminogen activator inhibitor as well as by systemic coagulation
factors such as platelets, circulating clotting factors.

15. Question

A PP nurse is assessing a mother who delivered a healthy newborn infant by


C-section. The nurse is assessing for signs and symptoms of superficial
venous thrombosis. Which of the following signs or symptoms would the
nurse note if superficial venous thrombosis were present?

A. Paleness of the calf area

B. Enlarged, hardened veins

C. Coolness of the calf area

D. Palpable dorsalis pedis pulses

Answer: B. Enlarged, hardened veins

Thrombosis of the superficial veins is usually accompanied by signs and


symptoms of inflammation. These include swelling of the involved extremity
and redness, tenderness, and warmth. Superficial thrombophlebitis is an
inflammatory disorder of superficial veins with coexistent venous
thrombosis. It usually affects lower limbs, particularly the great saphenous
vein (60% to 80%) or the small/short saphenous vein (10% to 20%).

Option A: Patients with superficial thrombophlebitis typically present with a


reddened, warm, inflamed, tender area overlying the track of a superficial
vein. There is often a palpable cord. Some surrounding edema or associated
pruritus may occur. In pregnancy, the risk of SVT is akin to that of DVT,
most commonly in the postpartum period. Advanced age, exogenous
434
C.BY: HOSSAM HAMDY MATERNITY RNPEDIA – ‫موسوعه التمريض‬

estrogens, autoimmune or infectious diseases, obesity, recent trauma or


surgery, active malignancy, history of venous thromboembolic disease, and
respiratory or cardiac failure also increase the risk of SVT.

Option C: Significant swelling of the limb is more commonly associated


with DVT and should only be attributed to SVT after DVT has been
excluded. Patients may have a history of antecedent trauma, which can
include intravenous cannulation or infusion of irritants, such as recent
sclerotherapy for varicose veins. However, it can occur at other sites (10% to
20%) and may occur bilaterally (5% to 10%). Traditionally, this relatively
common process was considered benign and self-limited.

Option D: Physical examination does not adequately identify the extent of


disease; it has been shown to underestimate it in up to 77% of instances.
Compressive ultrasonography can identify concomitant DVT, evaluate the
extent of the thrombus, and confirm the diagnosis. A careful history is
critical to identify risk factors for venous thromboembolism. Initial
presentation in patients older than 40 years without other risk factors should
prompt consideration of underlying malignancy.

16. Question

A nurse is providing instructions to a mother who has been diagnosed with


mastitis. Which of the following statements, if made by the mother, indicates
a need for further teaching?

A. “I need to take antibiotics, and I should begin to feel better in 24-48


hours.”

B. “I can use analgesics to assist in alleviating some of the discomfort.”

C. “I need to wear a supportive bra to relieve the discomfort.”

D. “I need to stop breastfeeding until this condition resolves.”

Answer: D. “I need to stop breastfeeding until this condition resolves.”

In most cases, the mother can continue to breastfeed with both breasts. If the
affected breast is too sore, the mother can pump the breast gently. Regular
emptying of the breast is important to prevent abscess formation. Continuing
to fully empty the breasts has shown to decrease the duration of symptoms in
patients treated both with and without antibiotics. Patients should be

435
C.BY: HOSSAM HAMDY MATERNITY RNPEDIA – ‫موسوعه التمريض‬

encouraged to continue to breastfeed, pump, or hand express. If the patient


stops draining the milk, further stasis occurs, and the infection will progress.

Option A: Antibiotic therapy assists in resolving the mastitis within 24-48


hours. If the symptoms of lactational mastitis persist beyond 12 to 24 hours,
antibiotics should be administered. Because S. aureus is the most common
cause, antibiotic therapy should be tailored accordingly. In the setting of
mild infection without MRSA risk factors, outpatient treatment can be
initiated with dicloxacillin or cephalexin.

Option B: The doctor may recommend an over-the-counter pain reliever,


such as acetaminophen (Tylenol, others) or ibuprofen (Advil, Motrin IB,
others). Lactational mastitis occurs due to a combination of inadequate
drainage of milk, and the introduction of bacteria. Common scenarios
leading to poor milk drainage include infrequent feeding, an oversupply of
milk, rapid weaning, illness in mother or child, and a clogged duct. The
inadequately drained milk stagnates, and organisms grow, leading to
infection. It is thought that bacteria (usually from the infant’s mouth, or
mother’s skin) gain entry to the milk via cracks in the nipple.

Option C: Additional supportive measures include ice packs, breast


supports, and analgesics. Non-steroidal anti-inflammatory drugs (NSAIDs)
can be used for pain control. Heat applied to the breast just before emptying
can help increase milk letdown and facilitate emptying. Cold packs applied
to the breast after emptying can help reduce edema and pain.

17. Question

A postpartum (PP) client is being treated for DVT. The nurse understands
that the client’s response to treatment will be evaluated by regularly
assessing the client for:

A. Dysuria, ecchymosis, and vertigo

B. Epistaxis, hematuria, and dysuria

C. Hematuria, ecchymosis, and epistaxis

D. Hematuria, ecchymosis, and vertigo

Answer: C. Hematuria, ecchymosis, and epistaxis.

436
C.BY: HOSSAM HAMDY MATERNITY RNPEDIA – ‫موسوعه التمريض‬

The treatment for DVT is anticoagulant therapy. The nurse assesses for
bleeding, which is an adverse effect of anticoagulants. This includes
hematuria, ecchymosis, and epistaxis. Dysuria and vertigo are not associated
specifically with bleeding. The cornerstone of treatment is anticoagulation.
NICE guidelines only recommend treating proximal DVT (not distal) and
those with pulmonary emboli. In each patient, the risks of anticoagulation
need to be weighed against the benefits.

Option A: Low-molecular-weight heparin or fondaparinux for five days or


until INR is greater than 2 for 24 hours (unfractionated heparin for patients
with renal failure and increased risk of bleeding). If platelet count drops to
less than 75,000, switch from heparin to fondaparinux, which is not
associated with heparin-induced thrombocytopenia.

Option B: The use of thrombolytic therapy can result in an intracranial


bleed, and hence, careful patient selection is vital. Recently endovascular
interventions like catheter-directed extraction, stenting, or mechanical
thrombectomy have been tried with moderate success. The duration of
treatment for DVT is for 3-6 months, but recurrent episodes may require at
least 12 months of treatment. Patients with cancer need long term treatment.

Option D: Inferior vena cava filters are not recommended in acute DVT.
There are both permanent and temporary inferior vena cava filters available.
These devices may decrease the rate of recurrent DVT but do not affect
survival. Today, only patients with contraindications to anticoagulation with
an increased risk of bleeding should have these filters inserted.

18. Question

A nurse performs an assessment on a client who is 4 hours PP. The nurse


notes that the client has cool, clammy skin and is restless and excessively
thirsty. The nurse prepares immediately to:

A. Assess for hypovolemia and notify the health care provider.

B. Begin hourly pad counts and reassure the client.

C. Begin fundal massage and start oxygen by mask.

D. Elevate the head of the bed and assess vital signs.

Answer: A. Assess for hypovolemia and notify the health care provider.

437
C.BY: HOSSAM HAMDY MATERNITY RNPEDIA – ‫موسوعه التمريض‬

Symptoms of hypovolemia include cool, clammy, pale skin, sensations of


anxiety or impending doom, restlessness, and thirst. When these symptoms
are present, the nurse should further assess for hypovolemia and notify the
health care provider. Patients with hypovolemic shock have severe
hypovolemia with decreased peripheral perfusion. If left untreated, these
patients can develop ischemic injury of vital organs, leading to multi-system
organ failure.

Option B: The first factor to be considered is whether the hypovolemic


shock has resulted from hemorrhage or fluid losses, as this will dictate
treatment. When etiology of hypovolemic shock has been determined,
replacement of blood or fluid loss should be carried out as soon as possible
to minimize tissue ischemia.

Option C: Medical management with uterotonic and pharmacologic agents


is typically the first step if uterine atony is identified. While oxytocin is
given routinely by most institutions at the time of delivery (see prevention),
additional uterotonic medications may be given with bimanual massage in an
initial response to hemorrhage. Uterotonic agents include oxytocin, ergot
alkaloids, and prostaglandins.

Option D: Initial evaluation of the patient should include a rapid assessment


of the patient’s status and risk factors. In postpartum women, signs or
symptoms of blood loss such as tachycardia and hypotension may be
masked, so if these signs are present, there should be a concern for
considerable blood volume loss (greater than 25% of total blood volume).
Continuous assessment of vital signs and on-going estimation of total blood
loss is an important factor in ensuring safe care of the patient with PPH.

19. Question

A nurse is assessing a client in the 4th stage of labor and notes that the
fundus is firm but that bleeding is excessive. The initial nursing action
would be which of the following?

A. Massage the fundus

B. Place the mother in Trendelenburg's position

C. Notify the physician

D. Record the findings

438
C.BY: HOSSAM HAMDY MATERNITY RNPEDIA – ‫موسوعه التمريض‬

Answer: C. Notify the physician

If the bleeding is excessive, the cause may be laceration of the cervix or birth
canal. Perineal trauma is an extremely common and expected complication
of vaginal birth. Lacerations can occur spontaneously or iatrogenically, as
with an episiotomy, on the perineum, cervix, vagina, and vulva.

Option A: Massaging the fundus if it is firm will not assist in controlling the
bleeding. Perineal massage has been shown to decrease the incidence of
lacerations requiring suture, although the reduction was minor. Additional
studies have shown a decrease in third- and fourth-degree lacerations when
massage was performed during the second stage of labor, however, there is
no consistently proven benefit.

Option B: Trendelenburg’s position is to be avoided because it may


interfere with cardiac function. Delayed or immediate pushing after a woman
reached ten centimeters of dilation showed no difference in the incidence of
perineal lacerations. However, there was a higher incidence of delivery with
intact perineum in women who delivered in the lateral position with delayed
pushing compared to immediate pushing in the lithotomy position.

Option D: The most common complication of a perineal laceration is


bleeding. Most bleeding can be quickly controlled with pressure and surgical
repair. However, hematoma formation can lead to large amounts of blood
loss in a very short time. Perineal support or a “hands-on” approach, can be
protective of the perineum and decrease the severity of perineal lacerations
at the time of delivery. However, studies are conflicting on the significant
benefit to this measure.

20. Question

A nurse is caring for a postpartum (PP) client with a diagnosis of DVT who
is receiving a continuous intravenous infusion of heparin sodium. Which of
the following laboratory results will the nurse specifically review to
determine if an effective and appropriate dose of the heparin is being
delivered?

A. Prothrombin time

B. International normalized ratio

C. Activated partial thromboplastin time

D. Platelet count

439
C.BY: HOSSAM HAMDY MATERNITY RNPEDIA – ‫موسوعه التمريض‬

Answer: C. Activated partial thromboplastin time.

Anticoagulation therapy may be used to prevent the extension of thrombus


by delaying the clotting time of the blood. Activated partial thromboplastin
time should be monitored, and a heparin dose should be adjusted to maintain
a therapeutic level of 1.5 to 2.5 times the control. Anticoagulants derive their
effect by acting at different sites of the coagulation cascade. Some act
directly by enzyme inhibition, while others indirectly, by binding to
antithrombin or by preventing their synthesis from the liver (vitamin K
dependent factors).

Option A: This is the initial test used to identify defects in secondary


hemostasis. It is the time taken for blood to clot and generates thrombin. A
delay in the PT or aPTT indicates the presence of either a deficiency or
inhibitor of the clotting factor, except for the antiphospholipid antibody,
which can result in delayed aPTT. The normal range for PT levels is
approximately 11 to 13 seconds, although levels may vary depending on the
laboratory.

Option B: The INR are used to monitor coagulation time when warfarin
(Coumadin) is used. The clotting time is the time it takes for plasma to clot
after the addition of different substrates in vitro under standard conditions
using the capillary method. The average clotting time is between 8 to 15
minutes. Some studies have disputed the use of clotting time as a screening
test.

Option D: Although thrombocytopenia increases bleeding risk, it has been


shown to predispose patients to venous thromboembolism. Heparin-induced
thrombocytopenia is antibody-mediated with complications that include
pulmonary embolism, acute myocardial infarction, and ischemic limb
necrosis. Therefore, estimation of the bleeding risk before initiation of
anticoagulation is essential. The use of argatroban, lepirudin, or danaparoid
is recommended over other non-heparin anticoagulants.

21. Question

A nurse is preparing a list of self-care instructions for a PP client who was


diagnosed with mastitis. Which of the following instructions would be
included on the list. Select all that apply.

A. Take the prescribed antibiotics until the soreness subsides.

B. Wear a supportive bra.


440
C.BY: HOSSAM HAMDY MATERNITY RNPEDIA – ‫موسوعه التمريض‬

C. Avoid decompression of the breasts by breastfeeding or breast pump.

D. Rest during the acute phase.

E. Continue to breastfeed if the breasts are not too sore.

Answer: B, D, and E.

Mastitis is an infection of the lactating breast. Client instructions include


resting during the acute phase, maintaining a fluid intake of at least 3 L a
day, and taking analgesics to relieve discomfort. Additional supportive
measures include the use of moist heat or ice packs and wearing a supportive
bra. Non-steroidal anti-inflammatory drugs (NSAIDs) can be used for pain
control. Heat applied to the breast just before emptying can help increase
milk letdown and facilitate with emptying. Cold packs applied to the breast
after emptying can help reduce edema and pain. Providers should ensure the
patient that breastfeeding with mastitis is safe and that they should continue
to do so if desired. If the patient does not wish to continue to breastfeed, they
should be counseled on the importance of continuing to empty the breasts
and taught alternative methods such as the use of a breast pump or manual
expression.

Option A: Antibiotics may be prescribed and are taken until the complete
prescribed course is finished. They are not stopped when the soreness
subsides. If the symptoms of lactational mastitis persist beyond 12 to 24
hours, antibiotics should be administered. Because S. aureus is the most
common cause, antibiotic therapy should be tailored accordingly. In the
setting of mild infection without MRSA risk factors, outpatient treatment can
be initiated with dicloxacillin or cephalexin.

Option C: Continued decompression of the breast by breastfeeding or


pumping is important to empty the breast and prevent formation of an
abscess. The initial management of lactational mastitis is symptomatic
treatment. Continuing to fully empty the breasts has shown to decrease the
duration of symptoms in patients treated both with and without antibiotics.
Patients should be encouraged to continue to breastfeed, pump, or hand
express. If the patient stops draining the milk, further stasis occurs, and the
infection will progress.

22. Question

441
C.BY: HOSSAM HAMDY MATERNITY RNPEDIA – ‫موسوعه التمريض‬

Methergine or Pitocin is prescribed for a woman to treat PP hemorrhage.


Before administration of these medications, the priority nursing assessment
is to check the:

A. Amount of lochia

B. Blood pressure

C. Deep tendon reflexes

D. Uterine tone

Answer: B. Blood pressure

Methergine and Pitocin are agents that are used to prevent or control
postpartum hemorrhage by contracting the uterus. They cause continuous
uterine contractions and may elevate blood pressure. A priority nursing
intervention is to check blood pressure. The physician should be notified if
hypertension is present. Methergine is in a group of drugs called ergot
alkaloids. It affects the smooth muscle of a woman’s uterus, improving the
muscle tone as well as the strength and timing of uterine contractions.
Methergine is used just after a baby is born, to help deliver the placenta (also
called the “afterbirth”).

Option A: Methergine is administered in the postpartum period to help


deliver the placenta and to help control bleeding and other uterine problems
after childbirth. It is indicated following delivery of the placenta, for routine
management of uterine atony, hemorrhage, and subinvolution of the uterus.
For control of uterine hemorrhage in the second stage of labor following
delivery of the anterior shoulder.

Option C: Methergine (methylergonovine maleate) acts directly on the


smooth muscle of the uterus and increases the tone, rate, and amplitude of
rhythmic contractions. Thus, it induces a rapid and sustained titanic
uterotonic effect which shortens the third stage of labor and reduces blood
loss.

Option D: Caution should be exercised in the presence of sepsis, obliterative


vascular disease. Also, use caution during the second stage of labor. The
necessity for manual removal of a retained placenta should occur only rarely
with proper technique and adequate allowance of time for its spontaneous
separation.

442
C.BY: HOSSAM HAMDY MATERNITY RNPEDIA – ‫موسوعه التمريض‬

23. Question

Methergine or Pitocin are prescribed for a client with PP hemorrhage. Before


administering the medication(s), the nurse contacts the health provider who
prescribed the medication(s) in which of the following conditions is
documented in the client’s medical history?

A. Peripheral vascular disease

B. Hypothyroidism

C. Hypotension

D. Type 1 diabetes

Answer: A. Peripheral vascular disease.

These medications are avoided in clients with significant cardiovascular


disease, peripheral disease, hypertension, eclampsia, or preeclampsia. These
conditions are worsened by the vasoconstriction effects of these medications.
Patients with coronary artery disease or risk factors for coronary artery
disease (e.g., smoking, obesity, diabetes, high cholesterol) may be more
susceptible to developing myocardial ischemia and infarction associated with
methylergonovine-induced vasospasm.

Option B: Before using this medication, tell the doctor or pharmacist about
the medical history, especially of: kidney disease, liver disease, high blood
pressure, heart disease (such as venoatrial shunts, mitral valve stenosis, chest
pain, recent heart attack), diabetes, high cholesterol, smoking/tobacco use,
blood vessel disease (such as Raynaud’s disease), complications during
pregnancy (such as preeclampsia, eclampsia).

Option C: This drug may make the client dizzy. Do not drive, use
machinery, or do any activity that requires alertness until the client is sure he
can perform such activities safely. Limit alcoholic beverages.

Option D: This medication may decrease the amount of breast milk.


Methylergonovine passes into breast milk in small amounts and may have
undesirable effects on a nursing infant. Consult your doctor before breast-
feeding.

24. Question

443
C.BY: HOSSAM HAMDY MATERNITY RNPEDIA – ‫موسوعه التمريض‬

Which of the following factors might result in a decreased supply of


breastmilk in a postpartum (PP) mother?

A. Supplemental feedings with formula

B. Maternal diet high in vitamin C

C. An alcoholic drink

D. Frequent feedings

Answer: A. Supplemental feedings with formula

Routine formula supplementation may interfere with establishing an


adequate milk volume because decreased stimulation to the mother’s nipples
affects hormonal levels and milk production. Especially in the first couple of
weeks, supplementing with formula tricks the breasts into producing less
milk. “In the early weeks, the breasts’ capacity for milk production is
calibrated in response to the amount of milk that is removed,” says lactation
consultant Diana West. “If less milk is removed, the breasts assume that less
milk is needed, so the capacity is set at a lower point.” When the baby is
given formula supplements, she naturally eats less at the breast, and the
breasts respond by making less milk. If supplementation is necessary,
pumping as well as breastfeeding can help to promote a higher volume of
milk production.

Option B: A high diet in Vit C does not decrease the supply of breastmilk in
the mother. Another cause is the use of birth control. Many mothers who
breastfeed and take birth control pills find their milk production doesn’t
change, but for some, any form of hormonal birth control (the pill, patch or
injections) can cause a significant drop in their milk. This is more likely to
happen if they start using these contraceptives before the baby is four months
old, but it can happen later as well. The first step to increasing the milk
supply again is to stop the medication, but talk to a doctor before doing so
and be prepared to change birth control methods. Some mothers also need
extra help (such as prescription medication, herbal supplements, and/or
pumping) to boost milk production.

Option C: Alcohol levels are usually highest in breast milk 30-60 minutes
after an alcoholic beverage is consumed, and can be generally detected in
breast milk for about 2-3 hours per drink after it is consumed. However, the
length of time alcohol can be detected in breast milk will increase the more
alcohol a mother consumes. For example, alcohol from 1 drink can be
444
C.BY: HOSSAM HAMDY MATERNITY RNPEDIA – ‫موسوعه التمريض‬

detected in breast milk for about 2-3 hours, alcohol from 2 drinks can be
detected for about 4-5 hours, and alcohol from 3 drinks can be detected for
about 6-8 hours, and so on.

Option D: The breasts make milk continuously, but the rate at which milk is
made depends on how empty they are. The woman will make more milk
when her breasts are close to empty and less milk when they are already
filled up. When the baby is feeding infrequently, because the mother has put
him on a three- or four-hour schedule for example, or because she is giving
him a pacifier to stretch out the time between feedings, the breasts are fuller
for longer periods of time. That means milk production slows down. When
babies are breastfed in response to their cues, they tend to have shorter,
frequent feedings and this means the breasts are emptier most of the time and
so they continue to produce plenty of milk.

25. Question

Which of the following interventions would be helpful to a breastfeeding


mother who is experiencing engorged breasts?

A. Applying ice

B. Applying a breast binder

C. Teaching how to express her breasts in a warm shower

D. Administering bromocriptine (Parlodel)

Answer: C. Teaching how to express her breasts in a warm shower.

Teaching the client how to express her breasts in warm shower aid with let-
down and will give temporary relief. Breast engorgement is the result of
increased blood flow in the breasts in the days after the delivery of a baby.
The increased blood flow helps the breasts make ample milk, but it can also
cause pain and discomfort.

Option A: Ice can promote comfort by vasoconstriction, numbing, and


discouraging further letdown of milk. Using a warm compress, or taking a
warm shower would encourage milk let down. The mother may also massage
her breasts while nursing, or feed more regularly, or at least every one to
three hours.

445
C.BY: HOSSAM HAMDY MATERNITY RNPEDIA – ‫موسوعه التمريض‬

Option B: For those who don’t breastfeed, painful engorgement typically


lasts about one day. After that period, the breasts may still feel full and
heavy, but the discomfort and pain should subside. The mother may wear a
supportive bra that prevents her breasts from moving significantly.

Option D: Breast engorgement is swelling and inflammation that occurs in


the breasts because of increased blood flow and milk supply. In the days and
weeks after giving birth, the body will begin to produce milk. The mother
may take pain medication approved by her doctor. Bromocriptine (Parlodel)
is used to treat symptoms of hyperprolactinemia (high levels of a natural
substance called prolactin in the body) including lack of menstrual periods,
discharge from the nipples, infertility (difficulty becoming pregnant), and
hypogonadism (low levels of certain natural substances needed for normal
development and sexual function).

446
C.BY: HOSSAM HAMDY MATERNITY RNPEDIA – ‫موسوعه التمريض‬

Maternity Nursing NU.11 (Quiz #1: 20 Questions)

1. Question

On completing a fundal assessment, the nurse notes the fundus is situated on


the client’s left abdomen. Which of the following actions is appropriate?

A. Ask the client to empty her bladder.

B. Straight catheterize the client immediately.

C. Call the client’s health provider for direction.

D. Straight catheterize the client for half of her uterine volume.

Answer: A. Ask the client to empty her bladder.

A full bladder may displace the uterine fundus to the left or right side of the
abdomen. Massage the fundus every 15 minutes during the first hour, every
30 minutes during the next hour, and then, every hour until the patient is
ready for transfer.

Option B: Catheterization is unnecessary invasive if the woman can void on


her own. Chart fundal height. Evaluate from the umbilicus using
fingerbreadths. This is recorded as two fingers below the umbilicus (U/2),
one finger above the umbilicus (1/U), and so forth. The fundus should
remain in the midline. If it deviates from the middle, identify this and
evaluate for a distended bladder.

Option C: Be able to recognize the difference between a full bladder and a


fundus. Full bladders may actually cause postpartum hemorrhage because it
prevents the uterus from contracting appropriately. Nerve blocks may alter
the sensation of a full bladder to the patient and prevent her from urinating.

Option D: If at all possible, ambulate the patient to the bathroom. Urine


output less than 300cc on initial void after delivery may suggest urinary
retention. Document the fundal height and bladder status before the patient
urinates. Reevaluate and document the fundal height and bladder status after
the patient urinates to accurately document an empty bladder.
447
C.BY: HOSSAM HAMDY MATERNITY RNPEDIA – ‫موسوعه التمريض‬

2. Question

The nurse is about to give a Type 2 diabetic her insulin before breakfast on
her first day postpartum. Which of the following answers best describes
insulin requirements immediately postpartum?

A. Lower than during her pregnancy

B. Higher than during her pregnancy

C. Lower than before she became pregnant

D. Higher than before she became pregnant

Answer: C. Lower than before she became pregnant

PP insulin requirements are usually significantly lower than pre-pregnancy


requirements. Occasionally, clients may require little to no insulin during the
first 24 to 48 hours postpartum. Immediately after delivery, postpartum
insulin requirements decrease dramatically as a result of the rapid decrease
in diabetogenic placental hormone levels and resulting dissipation of
pregnancy-induced insulin resistance.

Option A: The policy specifies that women with type 1 or type 2 diabetes
who require ongoing insulin administration should decrease insulin doses
and undergo monitoring of preprandial blood glucose values while on the
postpartum unit. Glycemic targets approximating nonpregnant targets are
utilized. Among women with type 1 diabetes, insulin requirements typically
return to prepregnancy levels or lower following delivery.

Option B: Women are typically advised to decrease basal and prandial


insulin doses to 50 to 80% of their preconception doses, but
recommendations are individualized. If preconception insulin doses are not
known, one-third to one-half of the term pregnancy dose or weight-based
dosing may be used as a starting point.

Option D: Among women with type 2 diabetes, postpartum medication


requirements vary depending on the severity of hyperglycemia postpartum
and the prepregnancy diabetes therapeutic regimen, ranging from no medical
therapy to resumption of insulin therapy at reduced doses (as above) or
noninsulin therapies following delivery.

3. Question
448
C.BY: HOSSAM HAMDY MATERNITY RNPEDIA – ‫موسوعه التمريض‬

Which of the following findings would be expected when assessing the


postpartum client?

A. Fundus 1 cm above the umbilicus 1 hour postpartum.

B. Fundus 1 cm above the umbilicus on a postpartum day 3.

C. Fundus palpable in the abdomen at 2 weeks postpartum.

D. Fundus slightly to the right; 2 cm above umbilicus on postpartum day 2.

Answer: A. Fundus 1 cm above the umbilicus 1 hour postpartum.

Within the first 12 hours postpartum, the fundus usually is approximately 1


cm above the umbilicus. Immediately postpartum, the uterine fundus is
palpable at or near the level of the maternal umbilicus. If the fundus is found
above the navel, the mother probably needs to pass urine. Call a midwife if
the uterus feels soft or is not descending.

Option B: The fundus should be below the umbilicus by PP day 3. By


approximately one-hour post-delivery, the fundus is firm and at the level of
the umbilicus. The fundus continues to descend into the pelvis at the rate of
approximately 1 cm or finger-breadth per day and should be nonpalpable by
14 days postpartum.

Option C: The fundus shouldn’t be palpated in the abdomen after day 10.
Thereafter, most of the reduction in size and weight occurs in the first 2
weeks, at which time the uterus has shrunk enough to return to the true
pelvis. Over the next several weeks, the uterus slowly returns to its
nonpregnant state, although the overall uterine size remains larger than prior
to gestation.

Option D: The uterus should feel firm and should feel about the size of a
grapefruit for the first few days. The fundus (top portion of the uterus)
should be felt at the level of the belly button or lower. The mother can
attempt to feel her fundus by gently pressing on her abdomen. The uterus
shrinks at about the rate of one cm. per day. By day 10 it can no longer be
felt above the pubic bone.

4. Question

449
C.BY: HOSSAM HAMDY MATERNITY RNPEDIA – ‫موسوعه التمريض‬

A client is complaining of painful contractions, or after pains, on postpartum


day 2. Which of the following conditions would increase the severity of
afterpains?

A. Bottle-feeding

B. Diabetes

C. Multiple gestation

D. Primiparity

Answer: C. Multiple gestation

Multiple gestation, multiparity, and conditions that cause overdistention of


the uterus will increase the intensity of after-pains. Afterpain is a common
phenomenon after vaginal delivery. Any factor that causes a delay in the
process of uterus sub involution and consequently returning its size to pre-
pregnancy status could affect the severity of afterpain.

Option A: There was a positive correlation between the number of


pregnancies and the duration of breastfeeding with mean score of afterpain.
Also, the length of ambulation decreased the afterpain intensity. However,
the intensity of afterpain had no significant relationship with stimulation
with oxytocin in labor, prescription of methylergonovine, and also oxytocin
after delivery. Considering that a longer duration of breastfeeding and
ambulation in the early postpartum period could decrease afterpain, it is
suggested to encourage postpartum mothers to begin breastfeeding and
ambulation as soon as possible after birth.

Option B: Diabetes has no correlation with afterpains. Afterpains


(cramping) are the contractions of the uterus occurring in the days following
childbirth. They are normal but can be uncomfortable. Afterpains are usually
strongest on the second and third days following delivery, when the mother
is breastfeeding or after she takes a uterus-contracting medication prescribed
by her physician or midwife. Cramping is most noticeable after the birth of a
second or third baby.

Option D: After-pains are contractions that occur after labor and delivery.
These contractions signal the process of involution, the process of the uterus
shrinking back down to its pre-pregnancy size and shape. While after-pains
are not a reason to worry, they can cause discomfort and even pain. After-
pains can vary significantly from person to person. If this is not the first
450
C.BY: HOSSAM HAMDY MATERNITY RNPEDIA – ‫موسوعه التمريض‬

baby, the pain may be worse than experienced during previous pregnancies.
For pain, the mother can use comfort measures like warm packs, massage of
the fundus through the abdomen, and certain medications (with a
practitioner’s approval). Over-the-counter medication works well for most
women.

5. Question

On which of the postpartum days can the client expect lochia serosa?

A. Days 3 and 4 PP

B. Days 3 to 10 PP

C. Days 10-14 PP

D. Days 14 to 42 PP

Answer: B. Days 3 to 10 PP.

On the third and fourth PP days, the lochia becomes a pale pink or brown
and contains old blood, serum, leukocytes, and tissue debris. This type of
lochia usually lasts until PP day 10. The mother might notice increased
lochia when she gets up in the morning when she is physically active, or
while breastfeeding. Moms who have cesarean sections may have less lochia
after 24 hours than moms who had vaginal deliveries. The bleeding generally
stops within 4 to 6 weeks after delivery. The mother should wear pads, not
tampons, as nothing should go in the vagina for six weeks.

Option A: The lochia is the vaginal discharge that originates from the
uterus, cervix, and vagina. The lochia is initially red and comprises blood
and fragments of decidua, endometrial tissues, and mucus and lasts 1 to 4
days. Lochia rubra usually lasts for the first 3 to 4 days PP.

Option C: The lochia then changes color to yellowish or pale brown, lasting
5 to 9 days, and is composed mainly of blood, mucus, and leukocytes.
Lochia alba, which contains leukocytes, decidua, epithelial cells, mucus, and
bacteria, may continue for 2 to 6 weeks PP.

Option D: Finally, the lochia is white and contains mostly mucus, lasting up
to 10 to 14 days. The lochia can persist up to 5 weeks postpartum. The
persistence of red lochia beyond one week might be an indicator of uterine

451
C.BY: HOSSAM HAMDY MATERNITY RNPEDIA – ‫موسوعه التمريض‬

subinvolution. The presence of an offensive odor or large pieces of tissue or


blood clots in lochia or the absence of lochia might be a sign of infection.

6. Question

Which of the following behaviors characterizes the PP mother in the taking


in phase?

A. Passive and dependent

B. Striving for independence and autonomy

C. Curious and interested in care of the baby

D. Exhibiting maximum readiness for new learning

Answer: A. Passive and dependent

During the taking in phase, which usually lasts 1-3 days, the mother is
passive and dependent and expresses her own needs rather than the neonate’s
needs. The woman becomes dependent on her healthcare provider or support
person with some of the daily tasks and decision-making. This dependence is
mainly due to her physical discomfort from hemorrhoids or the after pains,
from the uncertainty of how she could care for the newborn, and also from
the extreme tiredness she feels that follows childbirth.

Option B: The taking hold phase usually lasts from days 3-10 PP. During
this stage, the mother strives for independence and autonomy. The woman
starts to initiate actions on her own and makes decisions without relying on
others. She starts to focus on the newborn instead of herself and begins to
actively participate in newborn care. The woman still needs positive
reinforcements despite the independence that she is already showing because
she might still feel insecure about the care of her child.

Option C: During the taking hold phase, demonstrate newborn care to the
mother and watch her do a return demonstration of every procedure. Allow
the woman to settle in gradually into her new role while still at the hospital
or healthcare facility because making decisions about the child’s welfare is a
difficult part of motherhood.

Option D: She also is most ready to learn. She is concerned about her ability
to take care of her newborn. This phase is associated with a great deal of

452
C.BY: HOSSAM HAMDY MATERNITY RNPEDIA – ‫موسوعه التمريض‬

anxiety (especially by a new mother). She may have several mood swings.
The mother might be involved in a lot of activity trying to accomplish tasks.

7. Question

Which of the following complications may be indicated by continuous


seepage of blood from the vagina of a PP client, when palpation of the uterus
reveals a firm uterus 1 cm below the umbilicus?

A. Uterine atony

B. Cervical laceration

C. Urinary tract infection

D. Retained placental fragments

Answer: B. Cervical laceration

Continuous seepage of blood may be due to cervical or vaginal lacerations if


the uterus is firm and contracting. Cervical lacerations (CL) are a known
cause of postpartum hemorrhage. Although CL occurs in more than half of
vaginal deliveries,1 they are less than 0.5 cm in length and rarely require
repair.

Option A: Uterine atony may cause subinvolution of the uterus, making it


soft, boggy, and larger than expected. Atony of the uterus, also called uterine
atony, is a serious condition that can occur after childbirth. It occurs when
the uterus fails to contract after the delivery of the baby, and it can lead to a
potentially life-threatening condition known as postpartum hemorrhage.

Option C: UTI won’t cause vaginal bleeding, although hematuria may be


present. Urinary tract infection (UTI) is a common postpartum infection
occurring in 2%–4% of all deliveries. Although postpartum UTI is usually a
mild infection, it is associated with discomfort, prolonged hospital stay and
readmission and has been associated with an increased risk of discontinued
breastfeeding.

Option D: Postpartum retained placental fragments (RPF) are most often


clinically manifested as delayed postpartum hemorrhage or prolonged
postpartum spotting. This is a rare complication of labor, yet can potentially
cause severe morbidity and discomfort.

453
C.BY: HOSSAM HAMDY MATERNITY RNPEDIA – ‫موسوعه التمريض‬

8. Question

What type of milk is present in the breasts 7 to 10 days PP?

A. Colostrum

B. Transitional milk

C. Mature milk

D. Hind milk

Answer: B. Transitional milk

Transitional milk comes after colostrum and usually lasts until 2 weeks PP.
When breastfeeding mothers talk about their milk coming in, they are
referring to the onset of production of transitional milk, the creamy milk that
immediately follows colostrum. Transitional milk is produced anywhere
from about two to five days after birth until ten to fourteen days after birth.

Option A: Alveolar cells of the breast begin to secrete colostrum in the


twelfth to sixteenth week of pregnancy. This is called lactogenesis I.
Colostrum is a thick, yellowish-white fluid which can be expressed from the
breast by the third trimester. Milk secretion is suppressed during pregnancy
by estrogen and progesterone. Colostrum has more protein and fewer
carbohydrates and fat than mature breast milk. Colostrum is rich in secretory
immunoglobulin A (IgA), which helps to protect the infant from infection.
Colostrum also helps to establish a normal gut microbiome in the infant. The
bowel is considered sterile at birth.

Option C: The breast milk starts becoming mature after around two weeks,
but it won’t be fully mature milk until the baby’s about four weeks old. From
now on its composition will be broadly stable – it certainly won’t go through
dramatic changes like in the first month. Soon after it reaches maturity, the
milk starts to contain higher quantities of some components that protect the
baby against bacterial and viral infections. It’s probably no coincidence that
this stage of breast milk production coincides with the time she starts
grabbing objects and putting them in her mouth.

Option D: Hindmilk is the high-fat, high-calorie breast milk that the baby
gets toward the end of a feeding. It’s richer, thicker, and creamier than
foremilk, the breast milk that the baby gets when they first start to
breastfeed. The color of hindmilk is creamy white. Hindmilk satisfies the
454
C.BY: HOSSAM HAMDY MATERNITY RNPEDIA – ‫موسوعه التمريض‬

baby’s hunger and makes the baby feel full and sleepy. It also helps the baby
feel fuller longer.

9. Question

Which of the following complications is most likely responsible for a


delayed postpartum hemorrhage?

A. Uterine subinvolution

B. Clotting deficiency

C. Perineal laceration

D. Cervical laceration

Answer: A. Uterine subinvolution

Late postpartum bleeding is often the result of subinvolution of the uterus.


Retained products of conception or infection often cause subinvolution.
Delayed postpartum bleeding should always raise the possibility of
subinvolution. Although subinvolution may cause bleeding anytime between
1 week and several months postpartum, the most common reported timeline
for presentation is within the second week after delivery.

Option B: A client with a clotting deficiency may also have an immediate


PP hemorrhage if the deficiency isn’t ed at the time of delivery. Postpartum
hemorrhage (PPH) affects 6% of all deliveries and is the leading cause of
maternal death worldwide (19.7%). The incidence of PPH in women with
inherited bleeding disorders is substantially greater than in unaffected
women; however, estimates of relative risk are highly variable.

Option C: Cervical or perineal lacerations can cause an immediate


postpartum hemorrhage. Genital tract trauma is the second leading cause of
postpartum bleeding, followed by uterine atony. The interventions to control
the bleeding include vaginal packing, surgical repair of the lacerations,
vascular ligation and hysterectomy. To preserve future fertility, transcatheter
arterial embolization might be a useful option for the management of
intractable postpartum bleeding before the consideration of more invasive
methods.

Option D: Cervical lacerations (CL) are a known cause of postpartum


hemorrhage. Although CL occurs in more than half of vaginal deliveries,1
455
C.BY: HOSSAM HAMDY MATERNITY RNPEDIA – ‫موسوعه التمريض‬

they are less than 0.5 cm in length and rarely require repair. Suspect a tear in
cases of postpartum hemorrhage where there are a good uterine retraction
and uterine rupture has been ruled out.

10. Question

Before giving a postpartum (PP) client the rubella vaccine, which of the
following facts should the nurse include in client teaching?

A. The vaccine is safe in clients with egg allergies.

B. Breastfeeding isn’t compatible with the vaccine.

C. Transient arthralgia and rash are common adverse effects.

D. The client should avoid getting pregnant for 3 months after the vaccine
because the vaccine has teratogenic effects.

Answer: D. The client should avoid getting pregnant for 3 months after
the vaccine because the vaccine has teratogenic effects.

The client must understand that she must not become pregnant for 3 months
after the vaccination because of its potential teratogenic effects. Women who
are planning to become pregnant should check with their doctor to make sure
they are vaccinated before they get pregnant. Because MMR vaccine is an
attenuated (weakened) live virus vaccine, pregnant women who are not
vaccinated should wait to get MMR vaccine until after they have given birth.
Adult women of childbearing age should avoid getting pregnant for at least
four weeks after receiving an MMR vaccine.

Option A: The rubella vaccine is made from duck eggs so an allergic


reaction may occur in clients with egg allergies. Vaccines that contain small
quantities of egg protein can cause hypersensitivity reactions in some people
with egg allergy. Adverse reactions are more likely with vaccines, such as
yellow fever and influenza vaccines, that are grown in embryonated eggs.

Option B: The virus is not transmitted into the breast milk, so clients may
continue to breastfeed after the vaccination. Early studies found no
transmission of rubella virus to breastfed infants. None of 18 infants who
were breastfed after maternal vaccination with rubella vaccine (various
strains) had detectable antibodies in one study. A study of mothers
vaccinated with the Cendehill strain of live, attenuated rubella virus found no
transmission of the live virus to their breastfed infants. However, rubella
456
C.BY: HOSSAM HAMDY MATERNITY RNPEDIA – ‫موسوعه التمريض‬

vaccine virus can appear in breastmilk and result in infections in some


infants.

Option C: Transient arthralgia and rash are common adverse effects of the
vaccine. Transient joint manifestations, ranging from mild arthralgia to
severe extensive polyarthritis are a troublesome feature of rubella. These
complaints, described as early as 1906 in Osler’s textbook, have been a
source of considerable concern in the past, since they may simulate the
picture of acute rheumatoid polyarthritis.

11. Question

Which of the following changes A. Increase described the insulin needs of a


client with type 1 diabetes who has just delivered an infant vaginally without
complications?

A. Increase

B. Decrease

C. Remain the same as before pregnancy

D. Remain the same as during pregnancy

Answer: B. Decrease.

The placenta produces the hormone human placental lactogen, an insulin


antagonist. After birth, the placenta, the major source of insulin resistance, is
gone. Insulin needs decrease and women with type 1 diabetes may only need
one-half to two-thirds of the prenatal insulin during the first few PP days.

Option A: After delivery, the requirement of insulin shows a sharp decline


and in GDM it is advisable to continue the monitoring to see if the sugars
have become normal in the postpartum period but in cases with type 1 and
type 2 DM it is not prudent to stop the insulin rather the dose of insulin
needs to be decreased by 20-40% of the pregnancy dose as the requirement
of insulin during lactation is less.

Option C: During breast-feeding, sometimes the requirement of insulin can


fall drastically and these women may develop hypoglycemia, so the dose of
insulin needs to be adjusted accordingly. The monitoring of GDM women
with a fasting and postprandial sugar is a good method to diagnose T2DM,
about 15% of GDM remain glucose intolerant or develop type 2DM in the
457
C.BY: HOSSAM HAMDY MATERNITY RNPEDIA – ‫موسوعه التمريض‬

postpartum period and 10-50% of the women develop diabetes within 5


years of delivery.

Option D: Provision of proper obstetric and neonatal care is of prime


concern after delivery. Immediate postpartum type 2 diabetes is uncommon
in women with GDM. The glucose targets during the postpartum period in
women who have undergone cesarean section are not very clear, but strict
control is needed to prevent any infections. Patients with uncontrolled sugars
have improper control and have high chances of postoperative
complications.

12. Question

Which of the following responses is most appropriate for a mother with


diabetes who wants to breastfeed her infant but is concerned about the
effects of breastfeeding on her health?

A. Mothers with diabetes who breastfeed have a hard time controlling their
insulin needs.

B. Mothers with diabetes shouldn’t breastfeed because of potential


complications.

C. Mothers with diabetes shouldn’t breastfeed; insulin requirements are


doubled.

D. Mothers with diabetes may breastfeed; insulin requirements may decrease


from breastfeeding.

Answer: D. Mothers with diabetes may breastfeed; insulin


requirements may decrease from breastfeeding.

Breastfeeding has an antidiabetogenic effect. Insulin needs are decreased


because carbohydrates are used in milk production. Breastfeeding mothers
are at a higher risk of hypoglycemia in the first PP days after birth because
the glucose levels are lower. Mothers with diabetes should be encouraged to
breastfeed.

Option A: Breastfeeding has a positive effect on a mother’s insulin


response. For Type 1 diabetic mothers this can decrease their need for
insulin during the breastfeeding period. Mothers who have Type 2 diabetes
may find they require less hypoglycemic medication while breastfeeding.
Good control of your insulin levels is important while breastfeeding. The
458
C.BY: HOSSAM HAMDY MATERNITY RNPEDIA – ‫موسوعه التمريض‬

woman may need to do some additional monitoring and be in close contact


with her health practitioner during the early weeks until her hormones and
her milk productions stabilize.

Option B: Planning ahead with the birth team to manage these potential
events can be helpful so everyone is prepared and not making decisions in a
stressful situation. If at all possible plan to breastfeed within the first hour
after birth and at least once an hour until the baby’s blood sugar levels
stabilize. Skin-to-skin contact has been found to decrease the risk of
hypoglycemia in newborns and it helps trigger the hormones that drive
breastfeeding.

Option C: A mother who is diabetic or insulin-resistant may find that it


takes a bit longer for her milk volume to increase after giving birth.
Colostrum is providing all the nutrients, (vitamins, minerals, and fats) that
baby needs through the early days. Frequent effective feedings will speed up
the body’s ability to increase the milk volume. In the event that
supplementation is required during the first few days, human donor milk is
the best option.

13. Question

On the first postpartum (PP) night, a client requests that her baby be sent
back to the nursery so she can get some sleep. The client is most likely in
which of the following phases?

A. Depression phase

B. Letting-go phase

C. Taking-hold phase

D. Taking-in phase

Answer: D. Taking-in phase

The taking-in phase occurs in the first 24 hours after birth. The mother is
concerned with her own needs and requires support from staff and relatives.
The woman becomes dependent on her healthcare provider or support person
with some of the daily tasks and decision-making. The woman prefers to talk
about her experiences during labor and birth and also her pregnancy.
Encouraging the woman to talk about her experiences during labor and birth
would greatly help her adjust and let her incorporate it into her new life.
459
C.BY: HOSSAM HAMDY MATERNITY RNPEDIA – ‫موسوعه التمريض‬

Option A: Depression is not one of the phases of postpartum psychological


changes. The changes that the woman undergoes are crucial within the first
24 hours of postpartum, especially the psychological changes. These changes
might affect the woman permanently if not given the appropriate attention
and care.

Option B: The letting-go phase begins several weeks later when the mother
incorporates the new infant into the family unit. During the letting go phase,
the woman finally accepts her new role and gives up her old roles like being
a childless woman or just a mother of one child. Readjustment of
relationships is needed for an easy transition to this phase.

Option C: The taking-hold phase occurs when the mother is ready to take
responsibility for her care as well as the infant’s care. The taking hold phase
starts 2 to 4 days after delivery. The woman starts to initiate actions on her
own and makes decisions without relying on others. She starts to focus on
the newborn instead of herself and begins to actively participate in newborn
care.

14. Question

Which of the following physiological responses is considered normal in the


early postpartum period?

A. Urinary urgency and dysuria

B. Rapid diuresis

C. Decrease in blood pressure

D. Increase motility of the GI system

Answer: B. Rapid diuresis

In the early PP period, there’s an increase in the glomerular filtration rate


and a drop in the progesterone levels, which result in rapid diuresis. During
pregnancy, the compressive forces of the gravid uterus and the progesterone-
induced decrease in ureteral tone, peristalsis, and contraction pressure lead to
the dilation of the calyceal system, increasing the volume of kidneys by 30%
from the pre-pregnant state. The dilated ureters and renal pelvis usually
return to the pre-pregnant state within four-eight weeks.

460
C.BY: HOSSAM HAMDY MATERNITY RNPEDIA – ‫موسوعه التمريض‬

Options A: There should be no urinary urgency, though a woman may feel


anxious about voiding. Conversely, urinary incontinence, especially urge
incontinence, affects 30% of postpartum females and is attributed most
commonly to psychological stress associated with childbirth. The mother
may complain of painful micturition or dysuria that could be due to tears,
laceration of the cervix or vagina, or episiotomy.

Option C: There’s a minimal change in blood pressure following childbirth.


The blood pressure could be elevated due to pain or excitement but is
generally in the normal range. A significant decrease (> 20% below baseline)
in blood pressure could be a sign of postpartum hemorrhage or septic shock.
Conversely, high blood pressure could be a sign of pain or pre-eclampsia.

Option D: A residual decrease in GI motility occurs postpartum. The


compressive effects of the gravid uterus on the stomach, a decrease in lower
esophageal sphincter tone due to high progesterone levels, and
hypersecretion of acid due to high gastrin levels cause an increase in the
incidence of acid reflux during pregnancy. After delivery, the levels of
progesterone and gastrin drop within 24 hours, and the acid reflux and
associated symptoms resolve in the next three to four days.

15. Question

During the 3rd PP day, which of the following observations about the client
would the nurse be most likely to make?

A. The client appears interested in learning about neonatal care.

B. The client talks a lot about her birth experience.

C. The client sleeps whenever the neonate isn’t present.

D. The client requests help in choosing a name for the neonate.

Answer: A. The client appears interested in learning about neonatal


care.

The third to tenth days of PP care are the “taking-hold” phase, in which the
new mother strives for independence and is eager for her neonate. The
woman starts to initiate actions on her own and makes decisions without
relying on others. She starts to focus on the newborn instead of herself and
begins to actively participate in newborn care. The other options describe the
phase in which the mother relives her birth experience.
461
C.BY: HOSSAM HAMDY MATERNITY RNPEDIA – ‫موسوعه التمريض‬

Option B: The taking-in phase usually sets 1 to 2 days after delivery. The
woman prefers to talk about her experiences during labor and birth and also
her pregnancy. Encouraging the woman to talk about her experiences during
labor and birth would greatly help her adjust and let her incorporate it into
her new life.

Option C: This is the time of reflection for the woman because within the 2
to 3 day period, the woman is passive. The taking-in phase provides time for
the woman to regain her physical strength and organize her rambling
thoughts about her new role.

Option D: In the taking in phase, the woman becomes dependent on her


healthcare provider or support person with some of the daily tasks and
decision-making. This dependence is mainly due to her physical discomfort
from hemorrhoids or the after pains, from the uncertainty of how she could
care for the newborn, and also from the extreme tiredness she feels that
follows childbirth.

16. Question

Which of the following circumstances is most likely to cause uterine atony


and lead to PP hemorrhage?

A. Hypertension

B. Cervical and vaginal tears

C. Urine retention

D. Endometritis

Answer: C. Urine retention.

Urine retention causes a distended bladder to displace the uterus above the
umbilicus and to the side, which prevents the uterus from contracting. The
uterus needs to remain contracted if bleeding is to stay within normal limits.
Cervical and vaginal tears can cause PP hemorrhage but are less common
occurrences in the PP period.

Option A: Women with hypertensive disorders are reported to be at


increased risk of developing postpartum hemorrhage (PPH). Severe PPH can
result in serious morbidities, such as adult respiratory distress syndrome,

462
C.BY: HOSSAM HAMDY MATERNITY RNPEDIA – ‫موسوعه التمريض‬

coagulopathy, shock, loss of fertility, Sheehan syndrome, and ultimately


maternal mortality.

Option B: Genital tract trauma is the second leading cause of postpartum


bleeding, followed by uterine atony. The interventions to control the
bleeding include vaginal packing, surgical repair of the lacerations, vascular
ligation, and hysterectomy. Postpartum bleeding that is encountered within
the first 24 hours following delivery is termed primary postpartum bleeding,
whereas secondary postpartum bleeding is defined as excessive vaginal
bleeding that occurs between the first 24 hours of delivery and the 6th week
of the puerperium.

Option D: Postpartum endometritis is an infection of the lining of the womb


which can occur up to six weeks after childbirth. It is much more common
after cesarean section births. It usually causes a temperature, tummy pain,
and heavier vaginal bleeding. Postpartum endometritis needs prompt
treatment with antibiotics.

17. Question

Which type of lochia should the nurse expect to find in a client 2 days PP?

A. Foul-smelling

B. Lochia serosa

C. Lochia alba

D. Lochia rubra

Answer: D. Lochia rubra

Lochia, also known as postpartum bleeding, is a normal discharge of blood


and mucus from the uterus after childbirth. It begins right after delivery and
can continue for four to six weeks postpartum, with the heaviest flow
occurring for the first 10 to 14 days. Some women may have a shorter period
of discharge, while others may have lochia for slightly longer than four to six
weeks. At first, lochia will look dark red and the flow may be heavy. This is
called lochia rubra. The lochia is initially red and comprised of blood and
fragments of decidua, endometrial tissues, and mucus and lasts 1 to 4 days.

Option A: The presence of an offensive odor or large pieces of tissue or


blood clots in lochia or the absence of lochia might be a sign of infection.
463
C.BY: HOSSAM HAMDY MATERNITY RNPEDIA – ‫موسوعه التمريض‬

The cervix and vagina may be edematous and bruised in the early
postpartum period and gradually heal back to normal.

Option B: The lochia then changes color to yellowish or pale brown, lasting
5 to 9 days, and is comprised mainly of blood, mucus, and leukocytes. This
is called lochia serosa.

Option C: Finally, the lochia is white and contains mostly mucus, lasting up
to 10 to 14 days. This is lochia alba. The lochia can persist up to 5 weeks
postpartum. The persistence of red lochia beyond one week might be an
indicator of uterine subinvolution.

18. Question

After the expulsion of the placenta in a client who has six living children, an
infusion of lactated ringer’s solution with 10 units of Pitocin is ordered. The
nurse understands that this is indicated for this client because:

A. She had a precipitate birth

B. This was an extramural birth

C. Retained placental fragments must be expelled

D. Multigravidae are at increased risk for uterine atony

Answer: D. Multigravidas are at increased risk for uterine atony.

Multiple full-term pregnancies and deliveries result in overstretched uterine


muscles that do not contract efficiently and bleeding may ensue. Risk factors
for uterine atony include prolonged labor, precipitous labor, uterine
distension (multifetal gestation, polyhydramnios, fetal macrosomia), fibroid
uterus, chorioamnionitis, indicated magnesium sulfate infusions, and
prolonged use of oxytocin.

Option A: The woman did not have precipitate labor. Contraction of the
myometrium that mechanically compresses the blood vessels supplying the
placental bed provides the principal mechanism uterine hemostasis after
delivery of the fetus, and the placenta is concluded. The process is
complemented by local decidual hemostatic factors such as tissue factor
type-1 plasminogen activator inhibitors as well as by systemic coagulation
factors such as platelets, circulating clotting factors.

464
C.BY: HOSSAM HAMDY MATERNITY RNPEDIA – ‫موسوعه التمريض‬

Option B: In obstetrical, midwifery, and public health scholarship,


“extramural” can describe any birth outside a childbirth facility. More often,
it means a birth both outside and intended, sanctioned location and without
the supervision of a skilled birth attendant. It is not indicated in the situation
that the woman gave birth outside a healthcare facility.

Option C: An expeditious exclusion of retained gestational products or


obstetric lacerations quickly excludes additional co-concomitant etiologies.
Uterine atony refers to the corpus uteri myometrial cells inadequate
contraction in response to endogenous oxytocin that is released in the course
of delivery. It leads to postpartum hemorrhage as delivery of the placenta
leaves disrupted spiral arteries which are uniquely void of musculature and
dependent on contractions to mechanically squeeze them into a hemostatic
state. Uterine atony is a principal cause of postpartum hemorrhage, an
obstetric emergency. Globally, this is one of the top 5 causes of maternal
mortality.

19. Question

As part of the postpartum assessment, the nurse examines the breasts of a


primiparous breastfeeding woman who is one day postpartum. An expected
finding would be:

A. Soft, non-tender; colostrum is present.

B. Leakage of milk at let down.

C. Swollen, warm, and tender upon palpation.

D. A few blisters and a bruise on each areola.

Answer: A. Soft, non-tender; colostrum is present.

Breasts are essentially unchanged for the first two to three days after birth.
Colostrum is present and may leak from the nipples. There are distinct stages
seen in milk production which start before the birth of the baby. The first
milk is colostrum which is available after labor. Occasionally, there is pre-
colostrum before the postpartum stage. Colostrum is high in protein, sodium,
and immunoglobulins while being low in lactose, and this is the first milk
produced for the baby. After 30 to 40 hours postpartum, the milk
composition changes by an increase of lactose and dilution of other
constituents as the volume increases.

465
C.BY: HOSSAM HAMDY MATERNITY RNPEDIA – ‫موسوعه التمريض‬

Option B: Subsequently, the maintenance of lactation follows an


autonomous pattern wherein the sucking of the baby and emptying of the
breast are the main factors regulating the milk flow. Baby and maternal
factors can influence this bonding. Skin-to-skin contact in the first 2 hours
after birth is essential for successful initiation of lactation. The breast crawl,
soon after birth, while the baby is alert will initiate the bonding, and it will
be an assurance by giving comfort and calm to the mother. The breast crawl
is when the baby is placed on the mother’s abdomen after birth; the baby
finds its way to the breast to suckle.

Option C: Maternal factors like pain, anxiety, emotional instability, among


others should be addressed before and after delivery. Getting a latch on the
nipple is important. If the latch is not due to issues such as lip tie or tongue
tie, the mother can develop cracked nipples or pain while breastfeeding. An
evaluation by a lactation consultant or other health professional comfortable
with diagnosing lip and tongue ties may be helpful.

Option D: Pain and discomfort while breastfeeding is expected. Milk


produced can be classified as foremilk and hindmilk. Foremilk contains
higher levels of lactose while hindmilk contains a higher proportion of
proteins and fats. The baby’s nutritional need controls the milk quantity and
components. The knowledge of high lactose foremilk and high fat of
hindmilk will help the mother to regulate the feed from both breasts.

20. Question

Following the birth of her baby, a woman expresses concern about the
weight she gained during pregnancy and how quickly she can lose it now
that the baby is born. The nurse, in describing the expected pattern of weight
loss, should begin by telling this woman that:

A. Return to pre-pregnant weight is usually achieved by the end of the


postpartum period.

B. Fluid loss from diuresis, diaphoresis, and bleeding accounts for about a 3-
pound weight loss.

C. The expected weight loss immediately after birth averages about 11 to 13


pounds.

D. Lactation will inhibit weight loss since caloric intake must increase to
support milk production.

466
C.BY: HOSSAM HAMDY MATERNITY RNPEDIA – ‫موسوعه التمريض‬

Answer: C. The expected weight loss immediately after birth averages


about 11 to 13 pounds.

The woman should plan to return to pre-pregnancy weight by 6 to 12 months


after delivery. Most women lose half of their baby weight by 6 weeks after
childbirth (postpartum). The rest most often comes off over the next several
months. The body needs time to recover from childbirth. If the woman loses
weight too soon after childbirth, it can take longer for her to recover. She
should give herself until the 6-week checkup before trying to slim down.

Option A: Prepregnant weight is usually achieved by 2 to 3 months after


birth, not within the 6-week postpartum period. Through diet and regular
exercise, it might be reasonable to lose up to 1 pound (0.5 kilogram) a week.
It might take six months to one year to return to pre-pregnancy weight,
whether breast-feeding or not.

Option B: Weight loss from diuresis, diaphoresis, and bleeding is about 9


pounds. Most women lose about 13 pounds (5.9 kilograms) during
childbirth, including the weight of the baby, placenta and amniotic fluid.
During the first week after delivery, the woman will lose additional weight
as she shed retained fluids — but the fat stored during pregnancy won’t
disappear on its own.

Option D: Weight loss continues during breastfeeding since fat stores


developed during pregnancy and extra calories consumed are used as part of
the lactation process. If breastfeeding, wait until the baby is at least 2 months
old and the milk supply has normalized before drastically cutting calories.
Women who are exclusively breastfeeding need about 500 more calories per
day than they did before pregnancy. Get these calories from healthy choices
such as fruits, vegetables, whole grains, and lean protein. Breastfeeding
makes the body burn calories which helps lose weight. If the woman is
patient, she may be surprised at how much weight she may lose naturally
while breastfeeding.

21. Question

Which of the following findings would be a source of concern if noted


during the assessment of a woman who is 12 hours postpartum?

A. Postural hypotension

B. Temperature of 100.4°F

C. Bradycardia — pulse rate of 55 BPM

467
C.BY: HOSSAM HAMDY MATERNITY RNPEDIA – ‫موسوعه التمريض‬

D. Pain in left calf with dorsiflexion of the left foot

Answer: D. Pain in left calf with dorsiflexion of the left foot.

Pain in the left calf with dorsiflexion of the left foot indicates a positive
Homan sign and is suggestive of thrombophlebitis and should be
investigated further. The risk of developing blood clots (thrombophlebitis) is
increased for about 6 to 8 weeks after delivery . Typically, blood clots occur
in the deep veins of the legs or pelvis (a disorder called deep vein
thrombosis). Sometimes one of these clots breaks loose and travels through
the bloodstream into the lungs, where it lodges in a blood vessel in the lung,
blocking blood flow. This blockage is called pulmonary embolism. Blood
clots may also develop in the veins just under the skin in the legs. This
disorder is called superficial venous thrombosis (superficial
thrombophlebitis).

Option A: The postpartum period is the period after delivery of conceptus


when maternal physiological and anatomical changes return to the
nonpregnant state. The blood pressure could be elevated due to pain or
excitement but is generally in the normal range. A significant decrease (>
20% below baseline) in blood pressure could be a sign of postpartum
hemorrhage or septic shock. Conversely, high blood pressure could be a sign
of pain or pre-eclampsia.

Option B: A temperature of 100.4°F in the first 24 hours is most likely


indicative of dehydration which is easily ed by increasing oral fluid intake.
The temperature is slightly elevated up to 37.2C (99F) along with increased
shivering, sweating, or diaphoresis in the first 24 hours and normalizes
within 12 hours. The temperature rise is attributable to the systemic
absorption of metabolites accumulated due to muscle contractions. There
could be a transient temperature rise (by 0.5C) on the third or fourth day due
to breast engorgement.

Option C: Bradycardia is expected to be related to circulatory changes after


birth. There is generalized physical fatigue immediately after delivery. The
pulse rate may be elevated a few hours after childbirth due to excitement or
pain and usually normalizes on the second day. The postpartum period, also
known as puerperium, starts following the expulsion of the placenta until
complete physiological recovery of various organ systems. The postpartum
period divides into three arbitrary phases, i.e., acute phase – the first 24
hours after delivery of the placenta, early – up to 7 days, and late – up to 6

468
C.BY: HOSSAM HAMDY MATERNITY RNPEDIA – ‫موسوعه التمريض‬

weeks to 6 months. Each phase has its unique clinical considerations and
challenges.

22. Question

The nurse examines a woman one hour after birth. The woman’s fundus is
boggy, midline, and 1 cm below the umbilicus. Her lochial flow is profuse,
with two plum-sized clots. The nurse’s initial action would be to:

A. Place her on a bedpan to empty her bladder.

B. Massage her fundus

C. Call the physician

D. Administer Methergine 0.2 mg IM which has been ordered prn.

Answer: B. Massage her fundus

A boggy or soft fundus indicates that uterine atony is present. If uterine


atony occurs, healthcare providers should be ready for initial medical
management which is directed to the use of medications to improve tone and
induce uterine contractions. Massaging the uterus is also effective, as is
ensuring an empty cavity. This is confirmed by the profuse lochia and
passage of clots. The first action would be to massage the fundus until firm,
followed by options C and D, especially if the fundus does not become or
remain firm with massage.

Option A: There is no indication of a distended bladder since the fundus is


midline and below the umbilicus. The diagnosis is made during the physical
exam immediately upon conclusion of an obstetric vaginal or cesarean
delivery. Direct palpation at cesarean delivery (typically after the closure of
the uterine incision) or indirect examination at bimanual examination after a
vaginal delivery reveals a boggy, soft, and an unusually enlarged uterus,
typically with co-existent bleeding from the cervical os (harder to appreciate
at cesarean deliveries).

Option C: Diagnosis of diffuse uterine atony is prompted typically by


finding of more than usual blood loss during examination demonstrating a
flaccid and enlarged uterus, which may contain a significant amount of
blood. With focal localized atony, the fundal region may be well contracted
while the lower uterine segment is dilated and atonic, which may difficult to

469
C.BY: HOSSAM HAMDY MATERNITY RNPEDIA – ‫موسوعه التمريض‬

appreciate on abdominal examination, but may be detected on vaginal


examination.

Option D: Maternal support with intravenous (IV) fluids is commenced


through preferably an u8-gauge, intravenous catheter. A team approach is
initiated with the summoning of the needed personnel through a standardized
built-in alert system. Medications used for postpartum hemorrhage
secondary to Uterine atony include the following: oxytocin (Pitocin) can be
given IV 10 to 40 units per 1000 ml or 10 units intramuscularly (IM). The
rapid undiluted infusion may cause hypotension; methylergonovine
(Methergine) given IM 0.2 mg. Given every 2 to 4 hours. Should be avoided
in patients with hypertension; misoprostol (Cytotec): 800 to 1000 mg placed
rectally. May cause a low-grade fever. It has a delayed action.

23. Question

When performing a postpartum check, the nurse should:

A. Assist the woman into a lateral position with upper leg flexed forward to
facilitate the examination of her perineum.

B. Assist the woman into a supine position with her arms above her head and
her legs extended for the examination of her abdomen.

C. Instruct the woman to avoid urinating just before the examination since a
full bladder will facilitate fundal palpation.

D. Wash hands and put on sterile gloves before beginning the check.

Answer: A. Assist the woman into a lateral position with upper leg
flexed forward to facilitate the examination of her perineum.

While the supine position is best for examining the abdomen, the woman
should keep her arms at her sides and slightly flex her knees in order to relax
abdominal muscles and facilitate palpation of the fundus. The nurse must be
well versed in postpartum assessment and be able to identify subtle changes
that could indicate a woman’s deteriorating condition. Components of care
should be standardized regardless of whether the recovery is done in a post-
anesthesia care unit (PACU), a labor and delivery room, or a postpartum
room.

Option B: According to the 2010 recommendations from the Association of


Women’s Health, Obstetric, and Neonatal Nurses (AWHONN), the nurse
470
C.BY: HOSSAM HAMDY MATERNITY RNPEDIA – ‫موسوعه التمريض‬

caring for the woman should not have any other patient or infant care
responsibilities until an initial assessment is completed and documented, the
repair of the episiotomy or perineal lacerations is complete and the woman is
hemodynamically stable. Assessments during the immediate postpartum
period start from the delivery of the placenta and continue for at least 2 hours
or until stable. Assessments should be orderly and ongoing so that timely
identification can be made of any abnormal changes in the woman’s clinical
condition.

Option C: The bladder should be emptied before the check. A full bladder
alters the position of the fundus and makes the findings inaccurate. Assist the
woman to empty her bladder. Catheterize only if the woman is unable to
void and the bladder is distended. Once the bladder is empty, reevaluate the
fundal height. Note the overall appearance of the woman, including skin
color, motor activity, facial expression, speech, mood, state of awareness,
and interactions with others. Any variation from normal assessment
parameters requires reassessment, communication, and early intervention as
indicated to prevent potentially serious consequences.

Option D: Although hands are washed before starting the check, clean (not
sterile) gloves are put on just before the perineum and pad are assessed to
protect from contact with blood and secretions. Involution is the process of
the uterus returning to its prepregnant state. Uterine tone should be assessed
at least as frequently as vital signs, every 15 minutes in the first 2 hours.4
Amount of blood loss should be assessed on an ongoing basis during this
time. Uterine atony is the most common cause of postpartum hemorrhage,
which remains a major cause of maternal morbidity and mortality.

24. Question

Perineal care is an important infection control measure. When evaluating a


postpartum woman’s perineal care technique, the nurse would recognize the
need for further instruction if the woman:

A. Uses soap and warm water to wash the vulva and perineum.

B. Washes from symphysis pubis back to episiotomy.

C. Changes her perineal pad every 2 – 3 hours.

D. Uses the peri bottle to rinse upward into her vagina.

Answer: D. Uses the peri bottle to rinse upward into her vagina.
471
C.BY: HOSSAM HAMDY MATERNITY RNPEDIA – ‫موسوعه التمريض‬

The peri bottle should be used in a backward direction over the perineum.
The flow should never be directed upward into the vagina since debris would
be forced upward into the uterus through the still-open cervix. Rinse
perineum with water after using the toilet and before the woman puts on a
new peri-pad. Caregivers will show how to use a peri-bottle (hand-held
squirt bottle) to rinse the perineum. Squirting warm tap water on the
perineum will keep it clean and may provide comfort for pain. While sitting
on the toilet, the woman should rinse the perineum. She should aim the
bottle opening at her perineum and spray so the water moves from front to
back.

Option A: Warm water and soap is sufficient to clean the perineal area. The
woman should wash her hands before doing perineal care. She should
remove the soiled peri-pad starting at the front (vaginal area) to the back
(anus). Rinse the perineum with water after using the toilet and before the
woman puts on a new peri-pad.

Option B: Wiping from front to back decreases the transmission of bacteria


from the anus. Postpartum perineal care is cleaning and caring for the
perineum after having a baby. The perineum is the area between the vagina
(birth canal) and the anus (rear end opening). In the first few weeks after
childbirth, the woman will probably have soreness or pain in her perineum.
She will also have discharge coming out of her vagina.

Option C: Peri-pads should be changed every 2-3 hours to prevent infection.


Perineal care will help the perineum heal faster, feel better, and help prevent
infection. The woman should ask her caregiver how long she should keep
doing perineal care. She may need to continue doing perineal care for 1 to 3
weeks after giving birth.

25. Question

Which measure would be least effective in preventing postpartum


hemorrhage?

A. Administer Methergine 0.2 mg every 6 hours for 4 doses as ordered.

B. Encourage the woman to void every 2 hours.

C. Massage the fundus every hour for the first 24 hours following birth.

D. Teach the woman the importance of rest and nutrition to enhance healing.

472
C.BY: HOSSAM HAMDY MATERNITY RNPEDIA – ‫موسوعه التمريض‬

Answer: C. Massage the fundus every hour for the first 24 hours
following birth.

The fundus should be massaged only when boggy or soft. Massaging a firm
fundus could cause it to relax. Uterine atony refers to the corpus uteri
myometrial cells inadequate contraction in response to endogenous oxytocin
that is released in the course of delivery. It leads to postpartum hemorrhage
as delivery of the placenta leaves disrupted spiral arteries which are uniquely
void of musculature and dependent on contractions to mechanically squeeze
them into a hemostatic state.

Option A: Active management of the third stage includes uterine massage


with concomitant sustained low-level traction on the umbilical cord.
Simultaneous oxytocin infusion is helpful, although it is reasonable to defer
it to after delivery of the placenta. Medications used for postpartum
hemorrhage secondary to Uterine atony include the following: oxytocin
(Pitocin) can be given IV 10 to 40 units per 1000 ml or 10 units
intramuscularly (IM). The rapid undiluted infusion may cause hypotension;
methylergonovine (Methergine) given IM 0.2 mg. Given every 2 to 4 hours.
Should be avoided in patients with hypertension: misoprostol (Cytotec): 800
to 1000 mg placed rectally. May cause a low-grade fever. It has a delayed
action.

Option B: A fundus that is higher than 2 cm above the umbilicus may


indicate a distended bladder or a uterus that is filled with blood. After
delivery of a large infant, the fundal height can be slightly elevated, and this
may be a normal finding. Assist the woman to empty her bladder.
Catheterize only if the woman is unable to void and the bladder is distended.
Once the bladder is empty, reevaluate the fundal height. Bladder distention,
incomplete emptying, urine retention, and/or the inability to void may occur
during the first few days postpartum. Within 12 hours of birth, changes in
hormone levels (decreased estrogen and oxytocin) occur resulting in diuresis.
Measure and record urine output in the first 24 hours post-birth. A bladder
scan can also be used at this time to assess for post-void residual.

Option D: This is an effective measure to enhance and maintain contraction


of the uterus and to facilitate healing. The nurse must be well versed in
postpartum assessment and be able to identify subtle changes that could
indicate a woman’s deteriorating condition. Components of care should be
standardized regardless of whether the recovery is done in a post-anesthesia
care unit (PACU), a labor and delivery room, or a postpartum room.

473
C.BY: HOSSAM HAMDY MATERNITY RNPEDIA – ‫موسوعه التمريض‬

Maternity Nursing NU.12 (Quiz #1: 20 Questions)

1. Question

A nurse is caring for a client in labor. The nurse determines that the client is
beginning in the second stage of labor when which of the following
assessments is noted?

A. The client begins to expel clear vaginal fluid.

B. The contractions are regular.

C. The membranes have ruptured

D. The cervix is dilated completely.

Answer: D. The cervix is dilated completely.

The second stage of labor begins when the cervix is dilated completely and
ends with the birth of the neonate. After cervical dilation is complete, the
fetus descends into the vaginal canal with or without maternal pushing
efforts. The fetus passes through the birth canal via 7 movements known as
the cardinal movements.

Option A: The first stage of labor is further subdivided into two phases,
which are defined by the degree of cervical dilation. The latent phase is
commonly defined as the 0 to 6 cm, while the active phase commences from
6 cm to full cervical dilation.

Option B: Although precisely determining when labor starts may be inexact,


labor is generally defined as beginning when contractions become strong and
regularly spaced at approximately 3 to 5 minutes apart

Option C: Rupture of membranes results from a variety of factors that


ultimately lead to accelerated membrane weakening. This is caused by an
increase in local cytokines, an imbalance in the interaction between matrix
metalloproteinases and tissue inhibitors of matrix metalloproteinases,
increased collagenase and protease activity, and other factors that can cause
increased intrauterine pressure.
474
C.BY: HOSSAM HAMDY MATERNITY RNPEDIA – ‫موسوعه التمريض‬

2. Question

A nurse in the labor room is caring for a client in the active phases of labor.
The nurse is assessing the fetal patterns and notes a late deceleration on the
monitor strip. The most appropriate nursing action is to:

A. Place the mother in the supine position.

B. Document the findings and continue to monitor the fetal patterns.

C. Administer oxygen via face mask.

D. Increase the rate of Pitocin IV infusion.

Answer: C. Administer oxygen via face mask.

Late decelerations are due to uteroplacental insufficiency as the result of


decreased blood flow and oxygen to the fetus during the uterine contractions.
This causes hypoxemia; therefore oxygen is necessary.

Option A: The supine position is avoided because it decreases uterine blood


flow to the fetus. The client should be turned to her side to displace pressure
of the gravid uterus on the inferior vena cava.

Option B: The findings should be documented after an intervention has been


done.

Option D: An intravenous Pitocin infusion is discontinued when a late


deceleration is noted. Most Pitocin related birth injury cases involve variable
and late decelerations. When the fetal monitor shows late decelerations, it is
often a sign that the baby is in distress because the contractions are
preventing oxygen from adequately transferring between the uterus and
placenta.

3. Question

A nurse is performing an assessment of a client who is scheduled for


cesarean delivery. Which assessment finding would indicate a need to
contact the physician?

A. Fetal heart rate of 180 beats per minute.

B. White blood cell count of 12,000.

475
C.BY: HOSSAM HAMDY MATERNITY RNPEDIA – ‫موسوعه التمريض‬

C. Maternal pulse rate of 85 beats per minute.

D. Hemoglobin of 11.0 g/dL.

Answer: A. Fetal heart rate of 180 beats per minute.

A normal fetal heart rate is 120-160 beats per minute. A count of 180 beats
per minute could indicate fetal distress and would warrant physician
notification.

Option B: WBC count increases to 6 to 16 million/mL and can be as high as


20 million/mL during and shortly after labor.

Option C: Initially, the increase in cardiac output is due to an increase in


stroke volume. As the stroke volume decreases towards the end of the third
trimester, an increase in heart rate acts to maintain the increased cardiac
output.

Option D: By full-term, a normal maternal hemoglobin range is 11-13 g/dL


as a result of the hemodilution caused by an increase in plasma volume
during pregnancy.

4. Question

A client in labor is transported to the delivery room and is prepared for


cesarean delivery. The client is transferred to the delivery room table, and
the nurse places the client in the:

A. Trendelenburg’s position with the legs in stirrups.

B. Semi-Fowler position with a pillow under the knees.

C. Prone position with the legs separated and elevated.

D. Supine position with a wedge under the right hip.

Answer: D. Supine position with a wedge under the right hip.

Vena cava and descending aorta compression by the pregnant uterus impedes
blood return from the lower trunk and extremities. This leads to decreasing
cardiac return, cardiac output, and blood flow to the uterus and the fetus. The
best position to prevent this would be side-lying with the uterus displaced off
476
C.BY: HOSSAM HAMDY MATERNITY RNPEDIA – ‫موسوعه التمريض‬

of abdominal vessels. Positioning for abdominal surgery necessitates a


supine position; however, a wedge placed under the right hip provides
displacement of the uterus.

Option A: Use of Trendelenburg position during prolonged active labor


does not decrease the incidence of cesarean delivery and may be associated
with poor fetal outcomes. Prospective, randomized data on the topic of
maternal positioning in labor would further elucidate the role, if any, for
Trendelenburg position in labor.

Option B: When maternal position in left lateral tilt was compared with the
horizontal position, there was no influence on the incidence of hypotension,
and there were no changes in systolic and diastolic blood pressure.

Option C: Prone position is inappropriate during cesarean delivery. When


the full left lateral tilt was compared with left lateral tilt, maternal position
did not increase the risk of hypotension and there were no changes in
systolic and diastolic blood pressure.

5. Question

A nurse is caring for a client in labor and prepares to auscultate the fetal
heart rate by using a Doppler ultrasound device. The nurse most accurately
determines that the fetal heart sounds are heard by:

A. Noting if the heart rate is greater than 140 BPM.

B. Placing the diaphragm of the Doppler on the mother's abdomen.

C. Performing Leopold’s maneuvers first to determine the location of the


fetal heart.

D. Palpating the maternal radial pulse while listening to the fetal heart rate.

Answer: D. Palpating the maternal radial pulse while listening to the


fetal heart rate.

The nurse simultaneously should palpate the maternal radial or carotid pulse
and auscultate the fetal heart rate to differentiate the two. If the fetal and
maternal heart rates are similar, the nurse may mistake the maternal heart
rate for the fetal heart rate.

477
C.BY: HOSSAM HAMDY MATERNITY RNPEDIA – ‫موسوعه التمريض‬

Option A: As cardiac output increases, the heart rate at rest speeds up from
a normal prepregnancy rate of about 70 beats per minute to 80 or 90 beats
per minute. During exercise, cardiac output and heart rate increase more
when a woman is pregnant than when she is not.

Option B: One type of monitor is a Doppler ultrasound device. It’s often


used during prenatal visits to count the baby’s heart rate. It may also be used
to check the fetal heart rate during labor. The healthcare provider may also
check the baby’s heart rate continuously during labor and birth. To do this,
the ultrasound probe (transducer) is fastened to the belly. It sends the sounds
of the baby’s heart to a computer.

Option C: Leopold’s maneuvers may help the examiner locate the position
of the fetus but will not ensure a distinction between the two rates.

6. Question

Perineal care is an important infection control measure. When evaluating a


postpartum woman’s perineal care technique, the nurse would recognize the
need for further instruction if the woman:

A. Uses soap and warm water to wash the vulva and perineum.

B. Washes from symphysis pubis back to episiotomy.

C. Changes her perineal pad every 2 – 3 hours.

D. Uses the peri bottle to rinse upward into her vagina.

Answer: D. Uses the peri bottle to rinse upward into her vagina.

The peri bottle should be used in a backward direction over the perineum.
The flow should never be directed upward into the vagina since debris would
be forced upward into the uterus through the still-open cervix.

Option A: Wash the vulva and perineum after every visit to the toilet. The
woman may also use just plain warm water to clean the perineum.

Option B: Washing from symphysis pubis to the episiotomy reduces the risk
of infection, especially of Group A Streptococcus infection.

Option C: Change sanitary pads frequently to reduce the risk of infection. It


is normal to bleed for 2-4 weeks after birth.
478
C.BY: HOSSAM HAMDY MATERNITY RNPEDIA – ‫موسوعه التمريض‬

7. Question

Which measure would be least effective in preventing postpartum


hemorrhage?

A. Administer Methergine 0.2 mg every 6 hours for 4 doses as ordered.

B. Encourage the woman to void every 2 hours.

C. Massage the fundus every hour for the first 24 hours following birth.

D. Teach the woman the importance of rest and nutrition to enhance healing.

Answer: C. Massage the fundus every hour for the first 24 hours
following birth.

The fundus should be massaged only when boggy or soft. Massaging a firm
fundus could cause it to relax. Uterine atony is the most common cause of
postpartum hemorrhage. Brisk blood flow after delivery of the placenta
unresponsive to transabdominal massage should prompt immediate action
including bimanual compression of the uterus and use of uterotonic
medications. Massage is performed by placing one hand in the vagina and
pushing against the body of the uterus while the other hand compresses the
fundus from above through the abdominal wall.

Option A: The choice of a second-line uterotonic should be based on


patient-specific factors such as hypertension, asthma, or use of protease
inhibitors. Although it is not a uterotonic, tranexamic acid (Cyklokapron)
may reduce mortality due to bleeding from postpartum hemorrhage (but not
overall mortality) when given within the first three hours and may be
considered as adjuvant therapy.

Option B: Draining the bladder with a Foley catheter may improve uterine
atony and will allow monitoring of urine output.

Option D: Follow-up of postpartum hemorrhage includes monitoring for


ongoing blood loss and vital signs, assessing for signs of anemia (fatigue,
shortness of breath, chest pain, or lactation problems), and debriefing with
patients and staff.

8. Question

479
C.BY: HOSSAM HAMDY MATERNITY RNPEDIA – ‫موسوعه التمريض‬

When making a visit to the home of a postpartum woman one week after
birth, the nurse should recognize that the woman would characteristically:

A. Express a strong need to review events and her behavior during the
process of labor and birth.

B. Exhibit a reduced attention span, limiting readiness to learn.

C. Vacillate between the desire to have her own nurturing needs met and the
need to take charge of her own care and that of her newborn.

D. Have reestablished her role as a spouse/partner.

Answer: C. Vacillate between the desire to have her own nurturing


needs met and the need to take charge of her own care and that of her
newborn.

One week after birth the woman should exhibit behaviors characteristic of
the taking-hold stage as described in option C. This stage lasts for as long as
4 to 5 weeks after birth.

Option A: The taking-in phase provides time for the woman to regain her
physical strength and organize her rambling thoughts about her new role.

Option B: This is also a characteristic of the taking-in stage, which lasts for
the first few days after birth. This dependence is mainly due to her physical
discomfort from hemorrhoids or the after pains, from the uncertainty of how
she could care for the newborn, and also from the extreme tiredness she feels
that follows childbirth.

Option D: This reflects the letting-go stage, which indicates that


psychosocial recovery is complete. During the letting go phase, the woman
finally accepts her new role and gives up her old roles like being a childless
woman or just a mother of one child.

9. Question

Four hours after a difficult labor and birth, a primiparous woman refuses to
feed her baby, stating that she is too tired and just wants to sleep. The nurse
should:

A. Tell the woman she can rest after she feeds her baby.

480
C.BY: HOSSAM HAMDY MATERNITY RNPEDIA – ‫موسوعه التمريض‬

B. Recognize this as a behavior of the taking-hold stage.

C. Record the behavior as ineffective maternal-newborn attachment.

D. Take the baby back to the nursery, reassuring the woman that her rest is a
priority at this time.

Answer: D. Take the baby back to the nursery, reassuring the woman
that her rest is a priority at this time.

The behavior described is typical of this stage and not a reflection of


ineffective attachment unless the behavior persists. Mothers need to
reestablish their own well-being in order to effectively care for their baby.

Option A: Response 1 does not take into consideration the need for the new
mother to be nurtured and have her needs met during the taking-in stage.

Option B: During the taking hold phase, the woman starts to initiate actions
on her own and makes decisions without relying on others. Demonstrate
newborn care to the mother and watch her do a return demonstration of
every procedure.

Option C: This dependence is mainly due to her physical discomfort from


hemorrhoids or the after pains, from the uncertainty of how she could care
for the newborn, and also from the extreme tiredness she feels that follows
childbirth and not a reflection of ineffective attachment.

10. Question

Parents can facilitate the adjustment of their other children to a new baby by:

A. Having the children choose or make a gift to give to the new baby upon
its arrival home.

B. Emphasizing activities that keep the new baby and other children
together.

C. Having the mother carry the new baby into the home so she can show the
other children the new baby.

D. Reducing stress on others by limiting their involvement in the care of the


new baby.

481
C.BY: HOSSAM HAMDY MATERNITY RNPEDIA – ‫موسوعه التمريض‬

Answer: A. Having the children choose or make a gift to give to the new
baby upon its arrival home.

Special time should be set aside just for the other children without
interruption from the newborn. Someone other than the mother should carry
the baby into the home so she can give full attention to greeting her other
children. Children should be actively involved in the care of the baby
according to their ability without overwhelming them.

Option B: When the new baby arrives, have a family member or friend
bring the child to the hospital or birth center for a brief visit. Allow another
loved one to hold the baby for a while so that both parents can give the older
child plenty of cuddles.

Option C: When the baby is home, take the older child to a special place —
such as a favorite playground — to celebrate the new baby’s arrival.

Option D: Sometimes older children — stressed by the changes happening


around them — take out their frustration on a new baby. If the older child
tries to harm the baby, it’s time for a talk about appropriate behavior. Also,
give the older child extra attention and include him or her in activities that
involve the baby, such as singing, bathing, or changing diapers. Praise the
older child when he or she acts lovingly toward the new baby.

11. Question

When making a visit to the home of a postpartum woman one week after
birth, the nurse should recognize that the woman would characteristically:

A. Express a strong need to review events and her behavior during the
process of labor and birth.

B. Exhibit a reduced attention span, limiting readiness to learn.

C. Vacillate between the desire to have her own nurturing needs met and the
need to take charge of her own care and that of her newborn.

D. Have reestablished her role as a spouse/partner.

482
C.BY: HOSSAM HAMDY MATERNITY RNPEDIA – ‫موسوعه التمريض‬

Answer: C. Vacillate between the desire to have her own nurturing


needs met and the need to take charge of her own care and that of her
newborn.

One week after birth the woman should exhibit behaviors characteristic of
the taking-hold stage as described in response C. This stage lasts for as long
as 4 to 5 weeks after birth. The woman starts to initiate actions on her own
and makes decisions without relying on others. Allow the woman to settle in
gradually into her new role while still at the hospital or healthcare facility
because making decisions about the child’s welfare is a difficult part of
motherhood.

Option A: The taking-in phase usually sets 1 to 2 days after delivery. The
woman prefers to talk about her experiences during labor and birth and also
her pregnancy. Encouraging the woman to talk about her experiences during
labor and birth would greatly help her adjust and let her incorporate it into
her new life.

Option B: This is a characteristic of the taking-in stage, which lasts for the
first few days after birth. The woman becomes dependent on her healthcare
provider or support person with some of the daily tasks and decision-making.
This dependence is mainly due to her physical discomfort from hemorrhoids
or the after pains, from the uncertainty of how she could care for the
newborn, and also from the extreme tiredness she feels that follows
childbirth.

Option D: This reflects the letting-go stage, which indicates that


psychosocial recovery is complete. During the letting go phase, the woman
finally accepts her new role and gives up her old roles like being a childless
woman or just a mother of one child. Readjustment of relationships is
needed for an easy transition to this phase.

12. Question

Four hours after a difficult labor and birth, a primiparous woman refuses to
feed her baby, stating that she is too tired and just wants to sleep. The nurse
should: Select all that apply.

A. Tell the woman she can rest after she feeds her baby.

B. Recognize this as a behavior of the taking-in stage.

C. Record the behavior as ineffective maternal-newborn attachment.

483
C.BY: HOSSAM HAMDY MATERNITY RNPEDIA – ‫موسوعه التمريض‬

D. Take the baby back to the nursery, reassuring the woman that her rest is a
priority at this time.

E. Acknowledge this as a behavior of the letting go stage.

Answer: B and D.

The behavior described is typical of this stage and not a reflection of


ineffective attachment unless the behavior persists. Mothers need to
reestablish their own well-being in order to effectively care for their baby.
The taking-in phase usually sets 1 to 2 days after delivery. This is the time of
reflection for the woman because within the 2 to 3 day period, the woman is
passive. The woman prefers to talk about her experiences during labor and
birth and also her pregnancy. The woman becomes dependent on her
healthcare provider or support person with some of the daily tasks and
decision-making. The changes that the woman undergoes are crucial within
the first 24 hours of postpartum, especially the psychological changes. These
changes might affect the woman permanently if not given the appropriate
attention and care.

Option A: This does not take into consideration the need for the new mother
to be nurtured and have her needs met during the taking-in stage. The taking-
in phase provides time for the woman to regain her physical strength and
organize her rambling thoughts about her new role. Encouraging the woman
to talk about her experiences during labor and birth would greatly help her
adjust and let her incorporate it into her new life.

Option C: This dependence is mainly due to her physical discomfort from


hemorrhoids or the after pains, from the uncertainty of how she could care
for the newborn, and also from the extreme tiredness she feels that follows
childbirth. The taking-in phase provides time for the woman to regain her
physical strength and organize her rambling thoughts about her new role.

Option E. During the letting go phase, the woman finally accepts her new
role and gives up her old roles like being a childless woman or just a mother
of one child.

13. Question

Parents can facilitate the adjustment of their other children to a new baby by:

A. Having the children choose or make a gift to give to the new baby upon
its arrival home.
484
C.BY: HOSSAM HAMDY MATERNITY RNPEDIA – ‫موسوعه التمريض‬

B. Emphasizing activities that keep the new baby and other children
together.

C. Having the mother carry the new baby into the home so she can show the
other children the new baby.

D. Reducing stress on other children by limiting their involvement in the


care of the new baby.

Answer: A. Having the children choose or make a gift to give to the new
baby upon its arrival home.

Regardless of the older child’s age, make sure that he or she gets individual
attention when the new baby arrives. If you’re taking pictures or videos,
include the older child. Take pictures or videos of him or her alone, too.
Consider having a few small gifts on hand to give to your older child in case
friends visit with gifts for the new baby.

Option B: Special time should be set aside just for the other children
without interruption from the newborn. Spend regular one-on-one time
together. Try to give the toddler a bit of undivided attention, even if it’s just
10 to 20 minutes a day. One way to accomplish this more easily is to wear
the newborn in a sling, which gives the mother two free hands to play a
game with the older child. And have the older child cuddle while you’re
nursing.

Option C: Someone other than the mother should carry the baby into the
home so she can give full attention to greeting her other children.
Acknowledge the child’s feelings. Know that the little one may express
negative feelings or act out, and don’t scold. Instead say, “Being a big
sibling can be hard. Sometimes you will feel sad or mad or do things you
don’t mean to do and that’s OK. We will always love you and want to help
you feel better.”

Option D: Children should be actively involved in the care of the baby


according to their ability without overwhelming them. Ask for help with
baby-related tasks. Ask the child to put diapers on the shelf next to the
changing table or fetch blankets or bottles for the baby. Once the mother
feels he’s ready, the older child can even help burp, bathe and dress the new
baby. No doubt he will feel proud to be given some new responsibilities.

14. Question

485
C.BY: HOSSAM HAMDY MATERNITY RNPEDIA – ‫موسوعه التمريض‬

A primiparous woman is in the taking-in stage of psychosocial recovery and


adjustment following birth. The nurse, recognizing the needs of women
during this stage, should:

A. Foster an active role in the baby’s care.

B. Provide time for the mother to reflect on the events of and her behavior
during childbirth.

C. Recognize the woman’s limited attention span by giving her written


materials to read when she gets home rather than doing a teaching session
now.

D. Promote maternal independence by encouraging her to meet her own


hygiene and comfort needs.

Answer: B. Provide time for the mother to reflect on the events of and
her behavior during childbirth.

The focus of the taking-in stage is nurturing the new mother by meeting her
dependency needs for rest, comfort, hygiene, and nutrition. Women express
a need to review their childbirth experience and evaluate their performance.
This dependence is mainly due to her physical discomfort from hemorrhoids
or the after pains, from the uncertainty of how she could care for the
newborn, and also from the extreme tiredness she feels that follows
childbirth. Encouraging the woman to talk about her experiences during
labor and birth would greatly help her adjust and let her incorporate it into
her new life.

Option A: Once they are met, she is more able to take an active role, not
only in her own care but also the care of her newborn. The taking hold phase
starts 2 to 4 days after delivery. The woman starts to initiate actions on her
own and makes decisions without relying on others. Demonstrate newborn
care to the mother and watch her do a return demonstration of every
procedure.

Option C: Short teaching sessions, using written materials to reinforce the


content presented, are a more effective approach. The woman still needs
positive reinforcements despite the independence that she is already showing
because she might still feel insecure about the care of her child. Allow the
woman to settle in gradually into her new role while still at the hospital or
healthcare facility because making decisions about the child’s welfare is a
difficult part of motherhood.
486
C.BY: HOSSAM HAMDY MATERNITY RNPEDIA – ‫موسوعه التمريض‬

Option D: This is the time of reflection for the woman because, within the 2
to 3 day period, the woman is passive. The woman becomes dependent on
her healthcare provider or support person with some of the daily tasks and
decision-making. The taking-in phase provides time for the woman to regain
her physical strength and organize her rambling thoughts about her new role.

15. Question

All of the following are important in the immediate care of the premature
neonate. Which nursing activity should have the highest priority?

A. Neurological assessment to determine gestational age.

B. Placement in a warm environment.

C. Identification by bracelet and footprints.

D. Instillation of antibiotics in the eyes.

Answer: B. Placement in a warm environment.

Babies can’t adjust to temperature changes as well as adults. Babies can lose
heat rapidly, nearly 4 times faster than an adult. Premature and low-
birthweight babies don’t have much body fat. Their bodies may not be ready
to control their own temperature, even in a warm environment. Even full-
term and healthy newborns may not be able to keep their body warm if the
environment is too cold. Wet skin can cause the baby to lose heat quickly by
evaporation. He or she can quickly lose 2° to 3°F. It is important to warm
and dry the baby right away using warm blankets and skin-to-skin contact.
Another source of warmth such as a heat lamp or over-bed warmer may also
be used.

Option A: Health assessments of the new baby start right away. One of the
first checks is the Apgar test. The Apgar test is a scoring system to evaluate
the condition of the newborn at 1 minute and 5 minutes after birth. The
healthcare provider or midwife and nurses will evaluate these signs and give
a point value. A score of 7 to 10 is considered normal. A score of 4 to 6 may
mean that the baby needs some rescue breathing measures (oxygen) and
careful monitoring. A score of 3 or below means that the baby needs rescue
breathing and lifesaving techniques.

Option C: Footprints are often taken and recorded in the medical record.
Before a baby leaves the delivery area, ID bracelets with matching numbers
487
C.BY: HOSSAM HAMDY MATERNITY RNPEDIA – ‫موسوعه التمريض‬

are placed on the baby and on you. Babies often have 2, on the wrist and
ankle. These should be checked each time the baby comes or goes from your
room.

Option D: The baby’s eyes start to grow around 16 weeks. The most rapid
growth happens in the last 12 weeks of pregnancy. Experts think premature
birth interrupts this later growth, leading to ROP. Other risk factors include
anemia, breathing problems, blood transfusions, and poor health. ROP
causes blood vessels in the eye to grow abnormally and spread through the
retina. These new blood vessels are fragile, and they leak blood into the eye.
Scar tissue can form and pull the retina away from the back of the eye,
causing vision loss.

16. Question

Vasectomy is a procedure done on a male for sterilization. The organ


involved in this procedure is

A. Prostate gland

B. Seminal vesicle

C. Testes

D. Vas deferens

Answer: D. Vas deferens

Vasectomy is a procedure wherein the vas deferens of the male is ligated and
cut to prevent the passage of the sperms from the testes to the penis during
ejaculation.

Option A: The prostate is a gland about the size of a chestnut and weighs
about 30 grams (about 1 ounce). It is part of the male reproductive system
and is located inside the body. The prostate’s most important function is the
production of a fluid that, together with sperm cells from the testicles and
fluids from other glands, makes up semen. The muscles of the prostate also
ensure that the semen is forcefully pressed into the urethra and then expelled
outwards during ejaculation.

Option B: The seminal vesicles are a pair of glands that also include the
prostate gland and the bulbourethral glands. The seminal vesicles are located
in the pelvis superior to the rectum, inferior to the fundus of the bladder and
488
C.BY: HOSSAM HAMDY MATERNITY RNPEDIA – ‫موسوعه التمريض‬

posterior to the prostate. The seminal vesicles contribute around 70% of the
fluid that will eventually become semen. The fluid that they secrete has a
number of properties and components that are important for semen function
and sperm survival.

Option C: The testes are male sex glands that have both an endocrine and
exocrine function. The testes are oval-shaped reproductive structures that are
found in the scrotum and separated by the scrotal septum. The testis is the
male reproductive gland that is responsible for producing sperm and making
androgens, primarily.

17. Question

Breast self-examination is best done by the woman on herself every month


during

A. The middle of her cycle to ensure that she is ovulating.

B. During the menstrual period.

C. Right after the menstrual period so that the breast is not being affected by
the increase in hormones particularly estrogen.

D. Just before the menstrual period to determine if ovulation has occurred.

Answer: C. Right after the menstrual period so that the breast is not
being affected by the increase in hormones particularly estrogen.

The best time to do self-breast examination is right after the menstrual period
is over so that the hormonal level is low thus the breasts are not tender.

Option A: The best time to examine the breasts is usually 1 week after the
menstrual period starts, when the breasts are least likely to be swollen or
tender. Examining the breasts at other times in the menstrual cycle may
make it hard to compare results of one exam with another.

Option B: The hormone levels fluctuate each month during the menstrual
cycle, which causes changes in breast tissue. Swelling begins to decrease
when the woman’s period starts. The best time to perform a self-exam for
breast awareness is usually the week after the period ends.

Option D: The best time to do a monthly self-breast exam is about 3 to 5


days after the woman’s period starts. Do it at the same time every month.
489
C.BY: HOSSAM HAMDY MATERNITY RNPEDIA – ‫موسوعه التمريض‬

The breasts are not as tender or lumpy at this time in the monthly cycle. If
the woman has gone through menopause, she should do the exam on the
same day every month.

18. Question

A woman is considered to be menopause if she has experienced cessation of


her menses for a period of

A. 6 months

B. 12 months

C. 18 months

D. 24 months

Answer: B. 12 months

If a woman has not had her menstrual period for 12 consecutive months, she
is considered to be in her menopausal stage. Menopause is the time in a
woman’s life when her period stops. It usually occurs naturally, most often
after age 45. Menopause happens because the woman’s ovaries stop
producing the hormones estrogen and progesterone.

Option A: A woman has reached menopause when she has not had a period
for one year. Changes and symptoms can start several years earlier. They
include changes in periods; hot flashes and/or night sweats; trouble sleeping;
vaginal dryness; mood swings; trouble focusing; and less hair on the head,
more on the face.

Option C: Skipping periods during perimenopause is common and expected.


Often, menstrual periods will skip a month and return, or skip several
months and then start monthly cycles again for a few months. Periods also
tend to happen on shorter cycles, so they are closer together. Despite
irregular periods, pregnancy is possible.

Option D: Menopause is a natural biological process. But the physical


symptoms, such as hot flashes, and emotional symptoms of menopause may
disrupt sleep, lower energy or affect emotional health. There are many
effective treatments available, from lifestyle adjustments to hormone
therapy.

490
C.BY: HOSSAM HAMDY MATERNITY RNPEDIA – ‫موسوعه التمريض‬

19. Question

Which of the following is the practice of self-breast examination in a


menopausal woman?

A. She should do it at the usual time that she experiences her menstrual
period in the past to ensure that her hormones are not at its peak.

B. Any day of the month as long it is regularly observed on the same day
every month.

C. Anytime she feels like doing it ideally every day.

D. Menopausal women do not need regular self-breast exams as long as they


do it at least once every 6 months.

Answer: B. Any day of the month as long it is regularly observed on the


same day every month

Menopausal women still need to do self-examination of the breast regularly.


Any day of the month is alright provided that she practices it monthly on the
same day that she has chosen. The hormones estrogen and progesterone are
already diminished during menopause so there is no need to consider the
time to do it in relation to the menstrual cycle.

Option A: After menopause, the breast undergoes involution, with the


replacement of the pre-existing breast parenchyma with adipose and
connective tissue.

Option C: Because of the normal hormonal fluctuations in a woman’s body


that affect breast tissue, it is important to select the same time every month
so you will be able to distinguish between a normal change and something
that feels different.

Option D: Choose a day of the month (e.g., the 1st or 15th of the month)
and consistently perform the breast self-exam on that same day every month.
Perform the exam again that same day but lying down – This way the
woman will develop a feeling for her breasts in a different position, allowing
for greater knowledge of the way her breasts feel. Forty percent of diagnosed
breast cancers are detected by women who feel a lump, so establishing a
regular breast self-exam is very important.

20. Question
491
C.BY: HOSSAM HAMDY MATERNITY RNPEDIA – ‫موسوعه التمريض‬

In assisted reproductive technology (ART), there is a need to stimulate the


ovaries to produce more than one mature ova. The drug commonly used for
this purpose is:

A. Bromocriptine

B. Provera

C. Clomiphene

D. Estrogen

Answer: C. Clomiphene

Clomiphene or Clomid acts as an ovarian stimulant to promote ovulation.


The mature ova are retrieved and fertilized outside the fallopian tube (in-
vitro fertilization) and after 48 hours the fertilized ovum is inserted into the
uterus for implantation.

Option A: Bromocriptine (Parlodel) is used to treat symptoms of


hyperprolactinemia (high levels of a natural substance called prolactin in the
body) including lack of menstrual periods, discharge from the nipples,
infertility (difficulty becoming pregnant) and hypogonadism (low levels of
certain natural substances needed for normal development and sexual
function). Bromocriptine is in a class of medications called dopamine
receptor agonists. It treats hyperprolactinemia by decreasing the amount of
prolactin in the body.

Option B: Provera is a prescription medicine used to treat the symptoms of


heavy menstrual bleeding, absent or irregular menstrual bleeding and as
contraception. Provera may be used alone or with other medications. Provera
belongs to a class of drugs called Antineoplastics, Hormones, Progestins.

Option D: Estrogen, or oestrogen, is a category of sex hormone responsible


for the development and regulation of the female reproductive system and
secondary sex characteristics.

492

You might also like